You are on page 1of 242

A patient with massive burns developed acute renal insufficiency characterized by a significant and rapid

deceleration of glomerular filtration. What is the mechanism of its development?


A-Damage of glomerular filter
B-Reduction of functioning nephron number
C-Renal artery embolism
D-Reduction of renal blood flow
E-Rise of pressure of tubular fluid
A girl has been diagnosed with adrenogenital syndrome (pseudohermaphroditism). This pathology is caused
by hypersecretion of the following adrenal hormone:
A-Estrogens
B-Mineralocorticoids
C-Glucocorticoids
D-Catecholamines
E-Androgens
A 12-year-old adolescent suffering from bronchial asthma has a severe attack of asthma: he presents with
marked expiratory dyspnea, skin pallor. What type of alveolar ventilation disorder is observed?
A-Central
B-Thoracodiaphragmatic
C-Obstructive
D-Neuromuscular
E-Restrictive
A patient has been given high doses of hydrocortisone for a long tim e. This caused atrophy of one of the
adrenal cortex zones. Which zone is it?
A-Glomerular and reticular
B)C-Fascial
D-Reticular
E-Glomerular
Examination of a patient revealed overgrowth of facial bones and soft tissues, tongue enlargement, wide
interdental spaces in the enlarged dental arch. What changes of the hormonal secretion are the most likely?
A-Hyposecretion of the somatotropic hormone
B-Hyposecretion of thyroxin
C-Hyposecretion of insulin
D-Hypersecretion of insulin
E-Hypersecretion of the somatotropic hormone
A 58-year-old patient suffers from the cerebral atherosclerosis. Examination revealed hyperlipoidemi a. What
class of lipoproteins will most probably show increase in concentration in this patients blood serum?
A-Chylomicrons
B-Cholesterol
C-Fatty acid complexes with albumins
D-Low-density lipoproteins
E-High-density lipoproteins
Nappies of a neworn have dark spots being the evidence of homogentisic acid formation. This is caused by
the metabolic disorder of the following substance:
A-Tryptophan
B-Cholesterol
C-Galactose
D-Methionine
E-Tyrosine

In patients with the biliary tract obstruction the blood coagulation is inhibited; the patients have frequent
haemorrhages caused by the subnormal assimilation of the following vitamin:
A-A
B-E
C-C
D-D
E-K
Vagus nerves of an experimental animal have been cut on the both sides. What respiratory changes will result
from this?
A-Respiration will become shallow and infrequent
B-Respiration will become deep and infrequent
C-There will be no respiratory changes
D-Respiration will become shallow and frequent
E-Respiration will become deep and frequent
As a result of a road accident a driver has gotten a traum a. Now he is in shock condition and presents with a
decrease in daily diuresis down to 300 ml. What is the main pathogenetic factor of such alteration in the
diuresis?
A-Secondary hyperaldosteronism
B-Oncotic blood pressure drop
C-Decrease in number of the functioning glomerules
D-Increase in vascular permeability
E-Arterial pressure drop
While determining power inputs of a patients organism it was established that the respiratory coefficient
equaled 1,0. This means that in the cells of the patient the following substances are mainly oxidized:
A-Proteins and carbohydrates
B-Proteins
C-Fats
D-Carbohydrates
E-Carbohydrates and fats
After a serious psycho-emotional stress a 45-year-old patient suddenly felt constricting heart pain irradiating
to the left arm, neck and left scapul a. His face turned pale, the cold sweat stood out on it. The pain attack
was stopped with nitroglycerin e. What process has developed in this patient?
A-Myocardial infarction
B-Stroke
C-Stomach ulcer perforation
D-Stenocardia
E-Psychogenic shock
Following exposure to radiation a lot of mutant cells appeared in a patient. Some time later most of them
were detected and destroyed by the following cells of the immune system:
A-T-lymphocytes-supressors
B-B-lymphocyte
C-T-lymphocytes-killers
D-Plasmoblasts
E-Stem cells
Two weeks after lacunar tonsillitis a 20-year-old man started complaining about general weakness, lower
eyelid edemat a. After examination the patient was diagnosed with acute glomerulonephritis. What are the
most likely pathological changes in the urine formula?
A-Proteinuria

B-Presence of fresh erythrocytes


C-Pyuria
D-Natriuria
E-Cylindruria
While playing volleyball a sportsman jumped and then landed across the external edge of his foot. This
caused acute pain in the talocrural articulation, active movements became limited, passive movements
remained unlimited but painful. In the region of the external ankle a swelling appeared, the skin turned red
and became warmer to the touch. What type of peripheral circulation disorder has developed in this case?
A-Embolism
B-Arterial hyperaemia
C-Stasis
D-Thrombosis
E-Venous hyperaemia
After the prior sensibilization an experimental animal was given a subcutaneous injection of an antigen. The
place of injection exhibited a fibrinous inflammation with alteration of the vessel walls, basal substance and
fibrous structures of the connective tissue in form of mucoid and fibrinoid swelling and necrosis. What
immunological reaction is it?
A-Immediate hypersensitivity
B-Granulomatosis
C-Reaction of transplantation immunity
D-Normergic reaction
E-Delayed-type hypersensitivity
A 12-year-old teenager has significantly put off weight within 3 months; glucose concentration rose up to 50
millimolel. He fell into a com a. What is the main mechanism of its development?
A-Hypoglycemic
B-Hyperosmolar
C-Lactacidemic
D-Hypoxic
E-Ketonemic
After a disease a 16-year-old boy is presenting with decreased function of protein synthesis in the liver as a
result of vitamin K deficiency. This may cause disorder of:
A-Osmotic blood pressure
B-Anticoagulant production
C-Erythropoietin production
D-Blood coagulation
E-Erythrocyte sedimentation rate
After taking poor-quality food a patient developed repeated episodes of diarrhe a. On the next day he
presented with decreased arterial pressure, tachycardia, extrasystol e. Blood pH is 7,18. These abnormalities
were caused by the development of:
A-Nongaseous acidosis
B-Metabolic alkalosis
C-Gaseous acidosis
D-Gaseous alkalosis
E-Nongaseous alkalosis
A patient underwent a surgery for excision of a cyst on pancreas. After this he developed haemorrhagic
syndrome with apparent disorder of blood coagulation. Development of this complication can be explained
by:
A-Activation of fibrinolytic system
B-Activation of anticoagulation system

C-Activation of Christmas factor


D-Insufficient fibrin production
E-Reduced number of thrombocytes
A patient with clinical presentations of immunodeficiency has undergone immunological tests. They revealed
significant decrease in number of cells that form rosettes with sheep erythrocytes. What conclusion can be
drown on the ground of the analysis data?
A-Decrease in natural killer level (NK-cells)
B-Decrease in B-lymphocyte level
C-Lack of effector cells of the humoral immunity
D-Decrease in T-lymphocyte level
E-Decrease in complement system level
After a surgery a 36-year-old woman was given an intravenous injection of concentrated albumin solution.
This has induced intensified water movement in the following direction:
A-From the capillaries to the intercellular fluid
B-From the intercellular fluid to the cells
C-No changes of water movement will be observed
D-From the intercellular fluid to the capillaries
A patient has pellagr a. Interrogation revealed that he had lived mostly on maize for a long time and eaten
little meat. This disease had been caused by the deficit of the following substance in the maize:
A-Alanine
B-Histidine
C-Tryptophan
D-Proline
E-Tyrosine
A 1,5-year-old child presents with both mental and physical lag, decolorizing of skin and hair, decrease in
catecholamine concentration in bloo d. When a few drops of 5% solution of trichloroacetic iron had been
added to the childs urine it turned olive green. Such alteration are typical for the following pathology of the
amino acid metabolism:
A-Alkaptonuria
B-Albinism
C-Xanthinuria
D-Tyrosinosis
E-Phenylketonuria
A patient complains about dyspnea provoked by the physical activity. Clinical examination revealed anaemia
and presence of the paraprotein in the zone of gamma-globulins. To confirm the myeloma diagnosis it is
necessary to determine the following index in the patients urine:
A-Antitrypsin
B-Bilirubin
C-Bence Jones protein
D-Haemoglobin
E-Ceruloplasmin
An 8-year-old child was admitted to the infectious department with fever (up to 38C) and punctuate brightred skin rash. The child was diagnosed as having scarlet fever. Objectively: mucous membrane of pharynx is
apparently hyperaemic and edematic, the tonsils are enlarged and have dull yellowish-grey foci with some
black areas. What inflammation is the reason for the pharynx alterations?
A-Serous
B-Catarrhal
C-Fibrinous
D-Haemorrhagic

E-Purulent necrotic
A 32-year-old patient consulted a doctor about the absence of lactation after parturition. Such disorder might
be explained by the deficit of the following hormone:
A-Thyrocalcitonin
B-Glucagon
C-Vasopressin
D-Prolactin
E-Somatotropin
Examination of a patient admitted to the surgical department with symptoms of acute appendicitis revealed
the following changes in the white blood cells: the total count of leukocytes is 16109/l. Leukocyte formula:
basophils - 0, eosinophils - 2%, juvenile forms - 2%, stabnuclear - 8%, segmentonuclear - 59%, lymphocytes 25%, monocytes- 4%. The described changes can be classified as:
A-Neutrophilia with regenerative left shift
B-Neutrophilia with degenerative left shift
C-Neutrophilia with right shift
D-Neutrophil\ic leukemoid reaction
E-Neutrophilia with hyperregenerative left shift
ECG of a patient shows such alterations: P-wave is normal, P-Q-interval is short, ventricular QRST complex
is wide, R-wave is double-peak or two-phas e. What form of arrhythmia is it?
A-Atrioventricular block
B-Fredericks syndrome (atrial flutter)
C-Ciliary arrhythmia
D-Ventricular fibrillation
E-WPW syndrome (Wolff-Parkinson-White)
Examination of a patient 24 hours after appendectomy revealed neutrophilic leukocytosis with a regenerative
shift. What is the most likely mechanism of leukocytosis development?
A-Deceleration of leukocyte breakdown
B-Deceleration of leukocyte migration to the tissues
C-Intensification of leukopoiesis and deceleration of leukocyte migration to the tissues
D-Intensification of leucopoiesis
E-Redistribution of the leukocytes in the organism
In response to a change in body position from horizontal to vertical blood circulation system develops
reflectory pressor reaction. Which of the following is its compulsory component?
A-Increase in the heart rate
B-Systemic dilatation of the arterial resistive vessels
C-Systemic constriction of the venous vessels
D-Weakening of the pumbing ability of heart
E-Decrease in the circulating blood volume
A 50-year-old patient complains about general weakness, appetite loss and cardiac arrhythmi a. The patient
presents with muscle hypotonia, flaccid paralyses, weakened peristaltic activity of the bowels. Such condition
might be caused by:
A-Hyperkaliemia
B-Hypokaliemia
CHypoproteinemia
D-Hyponatremia
E-Hypophosphatemia
A 46-year-old patient suffering from the diffuse toxic goiter underwent resection of the thyroid glan d. After
the surgery the patient presents with appetite loss, dyspepsia, increased neuromuscular excitement. The body

weight remained unchange d. Body temperature is normal. Which of the following has caused such a
condition in this patient?
A-Reduced production of parathormone
B-Reduced production of thyroxin
C-Increased production of thyroxin
D-Increased production of calcitonin
E-Increased production of thyroliberin
70-year-old patient suffers from atherosclerosis complicated by the lower limb thrombosis that has caused
gangrene on his left toes. What is the most likely cause of the thrombosis origin?
A-Transformation of fibrinogen into fibrin
B-Thrombocyte adhesion
C-Prothrombinase activation
D-Transformation of prothrombin into thrombin
E-Impaired heparin synthesis
A 38-year-old patient with an uterine haemorrhage lasting for 2 days was delivered to the admission war d.
Which of the following will be revealed in the patients blood?
A-Eosinophilia
B-Leukocytosis
C-Decrease in the haematocrite index
D-Increase in the colour index
E-Deceleration in ESR
A 48-year-old patient was admitted to the hospital with complaints about weakness, irritability, sleep
disturbanc e. Objectively: skin and scleras are of yellow colour. In blood: increased concentration of total
bilirubin with prevailing direct bilirubin. The feces are acholi c. The urine is dark (contains bile pigments).
What type of jaundice is it?
A-Parenchymatous
B-Haemolytic
C-Crigler-Najjar syndrome
D-Mechanic
E-Gilberts syndrome
A female patient suffering from bronchial asthma had got a viral infection that provoked status asthmaticus
with fatal outcom e. Histological examination of lungs revealed spasm and edema of bronchioles, apparent
infiltration of their walls with lymphocytes, eosinophils and other leukocytes; labrocyte degranulation. What
mechanism of hypersensitivity underlies the described alterations?
A-Immune complex
B-Autoimmune
C-Inflammatory
D-Immune cytolysis
E-Reagin reaction
Medical examination at the military registration and enlistment office revealed that a 15-year-old boy was
high, with eunuchoid body proportions, gynecomastia, female pattern of pubic hair distribution. The boy had
also fat deposits on the thighs, no facial hair, high voice, subnormal intelligence quotient. Which karyotype
corresponds with this disease?
A-47, XXX
B-45, XO
C-46, XX
D-47, XXY
E-46, XY

A 10-year-old child had the mantoux tuberculin test administere d. 48 hours later a papule up to 8 mm in
diameter appeared on the site of the injection. What type of hypersensitivity reaction developed after the
tuberculin injection?
A-Type IV hypersensitivity reaction
B-Type II hypersensitivity reaction
C-Seroreaction
D-Arthus phenomenon
E-Atopic reaction
A 49-year-old patient consulted a doctor about increased fatigability and dyspnea provoked by physical
activity. ECG results: heart rate - 50/min, PQ-interval is prolonged, QRS- complex is unchanged, the number
of P-waves exceeds the number of QRS-complexes. What type of arrhythmia is it?
A-Sinoatrial block
B-Extrasystole
C-Sinus bradycardia
D-Atrioventricular block
E-Ciliary arrhythmia
A patient presents with icteritiousness of skin, scleras and mucous membranes. Blood plasma the total
bilirubin is increased, stercobilin is increased in feces, urobilin is increased in urin e. What type of jaundice is
it?
A-Obturational
B-Parenchymatous
C-Haemolytic
D-Gilberts disease
E-Cholestatic
Examination of a patient with frequent haemorrhages from the internal organs and mucous membranes
revealed proline and lysine within the collagen fibers. Disorder of their hydroxylation is caused by lack of the
following vitamin:
A-Vitamin K
B-Vitamin A
C-Vitamin E
D-Vitamin B1
E-Vitamin C
A patient suffers from the haemorrhagic syndrome that shows itself in frequent nasal bleedings,
posttraumatic and spontaneous intracutaneous and intra-articular haemorrhages. After a laboratory study a
patient was diagnosed with the type B haemophili a. This disease is provoked by the deficit of the following
factor of blood coagulation:
A-V
B-IX
C-VIII
D-VII
E-XI
After transfusion of 200 ml of blood a patient presented with body temperature rise up to 37,9C. Which of
the following substances is the most likely cause of temperature rise?
AInterleukin-4
B-Tumour necrosis factor
C-Interleukin-1
D-Interleukin-3
E-Interleukin-2

Examination of a patient suffering from chronic hepatitis revealed a significant decrease in the synthesis and
secretion of bile acids. What process will be mainly disturbed in the patients bowels?
A-Carbohydrate digestion
B-Amino acid absorption
C-Fat emulsification
D-Protein digestion
E-Glycerin absorption
A 28-year-old female patient consulted a gynecologist about sterility. Examination revealed underdeveloped
ovaries and uterus, irregular menstrual cycl e. Analysis of the sex chromatin revealed 2 Barrs bodies in most
somatic cells. What chromosome disease is most likely?
A-Turners syndrome
B-Triple X syndrome
C-Edwards syndrome
D-Klinefelters syndrome
E-Pataus syndrome
A 49-year-old driver complains about unbearable constricting pain behind the breastbone irradiating to the
neck. The pain arose 2 hours ago. Objectively: the patients condition is grave, he is pale, heart tones are
decrease d. Laboratory studies revealed high activity of creatine kinase and LDH1. What disease are these
symptoms typical for?
A-B. Diabetes mellitus
B-Cholelithiasis
C-Stenocardia
D-Acute myocardial infarction
E-Acute pancreatitis
As a result of a trauma a patient has developed traumatic shock that led to the following disorders: AP is
140/90 mm Hg, Ps is 120 bpm. The patient is fussy, talkative, pal e. Such state relates to the following shock
phase:
A-Latent period
B-Torpid
C)D-Terminal
E-Erectile
ECG of a 44-year-old patient shows signs of hypertrophy of both ventricles and the right atrium. The patient
was diagnosed with the tricuspid valve insufficiency. What pathogenetic variant of cardiac dysfunction is
usually observed in case of such insufficiency?
A-Coronary insufficiency
B-Heart overload by volume
C-Cardiac tamponade
D-Heart overload by resistance
E-Primary myocardial insufficiency
To prevent the transplant rejection after organ transplantation it is required to administer hormonotherapy
for the purpose of immunosuppression. What hormones are used for this purpose?
A-Catecholamines
B-Glucocorticoids
C-Sexual hormones
D-Mineralocorticoids
E-Thyroid
Lung ventilation in a person is increased as a result of physical activity. Which of the following indices of the
external respiration is much higher than in a state of rest?

A-Inspiratory reserve volume


B-Expiratory reserve volume
C-Vital capacity of lungs
D-Respiratory volume
E-Total lung capacity
A middle-aged man went to a foreign country because he had been offered a job ther e. However he had been
unemployed for quite a long tim e. What endocrine glands were exhausted most of all in this man?
A-Parathyroid glands
B-Adrenal glands
C-Thyroid gland
D-Seminal glands
E-Substernal gland
After a sprint an untrained person develops muscle hypoxi a. This leads to the accumulation of the following
metabolite in muscles:
A-Glucose 6-phosphate
B-Ketone bodies
C-Acetyl CoA
D-Oxaloacetate
E-Lactate
Cooling of the human body in water is much more faster than in the air. What way of heat emission in water
is much more effective?
A)B-Convection
C-Heat conduction
D-Heat radiation
E-Sweat evaporation
While eating a child choked on food and aspirated it. The child has severe cough, cyanotic skin and mucous
membranes, rapid pulse, infrequent respiration, prolonged expiration. The child has developed the following
disorder of the external respiration:
A-Stenotic respiration
B-Inspiratory dyspnea under asphyxia
C-Expiratory dyspnea under asphyxia
D-Biots respiration
E-Alternating respiration
As a result of continuous starvation the glomerular filtration rate has increased by 20%. The most probable
cause of the glomerular filtration alteration under the mentioned conditions is:
A-Increase in the permeability of the renal filter
B-Decrease in the oncotic pressure of blood plasma
C-Increase of the filtartion quotient
D-Increase in the systemic arterial pressure
E-Increase of the renal blood flow
After a hypertonic crisis a patient presents with lacking spontaneous movements in his right arm and leg,
muscle tone of these extremities is increase d. What type of motor dysfunction has developed in this case?
A-Peripheral paresis
B-Reflectrory paresis
C-Central paresis
D-Central paralysis
E-Peripheral paralysis

A patient with android-type obesity had been suffering from arterial hypertension, hyperglycemia, glycosuria
for a long time and died from the cerebral haemorrhag e. Pathologic examination revealed pituitary basophil
adenoma, adrenal cortex hyperplasi a. What is the most likely diagnosis?
A-Acromegalia
B-Diabetes mellitus
C-Pituitary nanism
D-Itsenko-Cushings syndrome
E-Adiposogenital dystrophy
A patient has osmotic pressure of blood plasma at the rate of 350 mOsmol/l (norm is 300 mOsmol/l). This will
cause hypersecretion of the following hormone:
A-Natriuretic
B-Aldosterone
C-Adrenocorticotropin
D-Vasopressin
E-Cortisol
A newborn develops dyspepsia after the milk feeding. When the milk is substituted by the glucose solution the
dyspepsia symptoms disappear. The newborn has the subnormal activity of the following enzyme:
A-Amylase
B-Invertase
C-Lactase
D-Isomaltase
E-Maltase
*** *** ***
Two hours after an exam a student had a blood count done and it was revealed that he had leukocytosis
without significant leukogram modifications. What is the most probable mechanism of leukocytosis
development?
A-Leukopoiesis intensification and deceleration of leukocyte lysis
B-Deceleration of leukocyte migration to the tissues
C-Leukopoiesis intensification
D-Deceleration of leukocyte lysis
E-Redistribution of leukocytes in the organism
A patient was stung by a bee. Examination revealed that his left hand was hot, pink, edematic, there was a big
red blister on the site of sting. What is the leading mechanism of edema development?
A-Drop of oncotic pressure in tissue
B-Reduced vessel filling
C-Increased vessel permeability
D-Drop of osmotic pressure in tissue
E-Injury of vessels caused by the sting
A patient has a decreased vasopressin synthesis that causes polyuria and as a result of it evident organism
dehydratation. What is the mechanism of polyuria development?
A-Acceleration of glomerular filtration
B-Reduced tubular reabsorption of water
C-Reduced glucose reabsorption
D-Reduced tubular reabsorption of protein
E-Reduced tubular reabsorption of Na ions
A 62 year old patient who previously worked as stoker was admitted to a hospital with complaints about
general weakness, abrupt weight loss, hoarse voice, dyspnea, dry cough. Laryngoscopy revealed a tumour in
the pharynx that invaded vocal cords and epiglottis. What is the most probable cause of tumour
development?

A-Polycyclic aromatic carbohydrates


B-Ionizing radiation
C-Aromatic amines and amides
D-Nitrosamines
E-Retroviruses
Patients who suffer from severe diabetes and dont receive insulin have metabolic acidosis. This is caused by
increased concentration of the following metabolites:
A-Cholesterol
B-Unsaturated fatty acids
C-Triacylglycerols
D-Fatty acids
E-Ketone bodies
People adapted to high external temperatures have such pecularity: profuse sweating isnt accompanied by
loss of large volumes of sodium chlorid E. This is caused by the effect of the following hormone upon the
perspiratory glands:
A-Natriuretic
B-Tgyroxin
C-Vasopressin
D-Aldosterone
E-Cortisol
A man weighs 80 kg, after long physical activity his circulating blood volume is reduced down to 5,4 l,
hematocrit makes up 50%, whole blood protein is 80 g/l. These blood characteristics are determined first of
all by:
A-Increased circulating blood volume
B-Increased protein concentration in plasm
C-Increased diuresis
D-Increased number of erythrocytes
E-Water loss with sweat
A 42 year old woman with neuralgia of trifacial nerve complains about periodical reddening of the right part
of her face and neck, sense of warmth gush, increased skin sensitivity. These effects can be explained by the
following type of arterial hyperemia:
A-Neuroparalytic
B-Neurotonic
C-Functional
D-Metabolic
E-Reactive
A patient suffering from pheochromocytoma complains of thirst, dry mouth, hunger. Blood test for sugar
revealed hyperglycemi A. What type of hyperglycemia is it?
A-Hypoinsulinemic
B-Adrenal
C-Hypercorticoid
D-Alimentary
E-Somatotropic
A 15 year old girl has pale skin, glossitis, gingivitis. Blood count: erythrocytes - 3,31012/l, hemoglobin - 70 g/l,
colour index - 0,5. Examination of blood smear revealed hypochromia, microcytosis, poikilocytosis. What type
of anemia is it?
A-Sickle-cell
B-B12-folic acid-deficient
C-Iron-deficient

D-Hemolytic
E-Thalassemia
A female patient underwent liver transplantation. 1,5 month after it her condition became worse because of
reaction of transplant rejection. What factor of immune system plays the leading part in this reaction?
A-Natural killers
B-T-killers
C-T-helpers
D-B-lymphocytes
E-Interleukin-1
Examination of a child who frequently suffers from infectious diseases revealed that IgG concentration in
blood serum was 10 times less than normal, IgA and IgM concentration was also significantly reduce D.
Analysis showed also lack of B-lymphocytes and plasmocytes. What disease are these symptoms typical for?
A-Louis-Bar syndrome
B-Di George syndrome
C-Brutons disease
D-Swiss-type agammaglobulinemia
E-Dysimmunoglobulinemia
A patient is 44 years ol D. Laboratory examination of his blood revealed that content of proteins in plasma
was 40 g/l. What influence will be exerted on the transcapillary water exchange?
A-Both filtration and reabsorption will be increased
B-Both filtration and reabsorption will be decreased
C-Filtration will be decreased, reabsorption - increased
D-Exchange will stay unchanged
E-Filtration will be increased, reabsorption decreased
Lungs of a preterm infant have areas of atelectasis (pulmonary collapse). The main cause is:
A-Underdeveloped inspiration muscles
B-Increased viscous resistance
C-Surfactant deficiency
D-Surfactant excess
E-Diminished force of surface tension of lungs
A child is pale, pastose, muscular tissue is bad developed, lymph nodes are enlarge D. He often suffers from
angina and pharyngitis, blood has signs of lymphocytosis. The child is also predisposed to autoallergic
diseases. What type of diathesis can be presumed in this case?
A-Asthenic
B-Gouty
C-Exudative
D-Hemorrhagic
E-Lymphohypoplastic
A patient with skin mycosis has disorder of cellular immunity. The most typical characteristic of it is
reduction of the following index:
A-B-lymphocytes
B-Plasmocytes
C-Immunoglobulin E
D-Immunoglobulin G
E-T-lymphocytes
Examination of a patient suffering from frequent haemorrhages in the inner organs and mucous membranes
revealed proline and lysine being included in collagen fibers. Impairment of their hydroxylation is caused by
lack of the following vitamin:
A-D

B-K
C-C
D-E
E-A
Examination of a man who had been working hard under higher temperature of the environment revealed
abnormal quantity of blood plasma proteins. What phenomenon is the case?
A-Dysproteinemia
B-Absolute hypoproteinemia
C-Absolute hyperproteinemia
D-Relative hyperproteinemia
E-Paraproteinemia
A patient consumed a lot of reach in proteins food that caused increase of rate of proteolytic enzymes of
pancreatic juic E. It is also accompanied by increase of rate of the following enzyme:
A-Gastricsin
B-Tripsin
C-Enterokinase
D-Pepsin
E-Renin
An unconscious young man with signs of morphine poisoning entered admission offic E. His respiration is
shallow and infrequent which is caused by inhibition of respiratory centr E. What type of respiratory failure
is it?
A-Perfusive
B-Diffusive
C-Ventilative dysregulatory
D-Ventilative obstructive
E-Ventilative restrictive
A patient has delayed conduction of excitement through the atrioventricular nod E. What changes of ECG
will be observed?
A-Negative T wave
B-Prolongation of Q-S interval
C-S-T-segment displacement
D-Prolongation of P-Q interval
E-Prolongation of Q-T interval
Roentgenological examination of skull base bones revealed enlargement of sellar cavity, thinning of anterior
clinoid processes, destruction of different parts, destruction of different parts of sella turcic A. Such bone
destruction might be caused by a tumour of the following wndocrinous gland:
A-Thymus gland
B-Thyroid gland
C-Adrenal glands
D-Hypophysis
E-Epiphysis
A patient was admitted to the infectious department. His symptoms: dry skin, decreased skin turgor, ricewater stool. The patient was diagnosed with choler A. What disorder of water-electrolytic balance is most
often observed in this disease?
A-Hypoosmotic hypohydration
B-Hyperosmotic hypohydration
C-Hyperosmotic hyperhydration
D-Hypoosmotic hyperhydration
E-Isoosmotic hypohydration

Inflammatory processes cause synthesis of protein of acute phase in an organism. What substances stumulate
their synthesis?
A-Interferons
B-Angiotensin
C-Interleukin-1
D-Immunoglobulins
E-Biogenic amins
An alcoholic woman has born a girl with mental and physical developmental lag. Doctors diagnosed the girl
with fetal alcohol syndrom E. What effect is the cause of the girls state?
A-Mutagenic
B-Malignization
C-Mechanic
D-Carcinogenic
E-Teratogenic
A patient complains of frequent diarrheas, especially after consumption of rich food, weight loss. Laboratory
examination revealed steatorrhea; his feces were hypocholi C. What might have caused such condition?
A-Obturation of biliary tracts
B-Unbalanced diet
C-Lack of pancreatic lipase
D-Lack of pancreatic phospholipase
E-Inflammation of mucous membrane of small intestine
A 30 year old woman has face edemat A. Examination revealed proteinuria (5,87 g/l), hypoproteinemia,
dysproteinemia, hyperlipidemi A. What condition is the set of these symptoms typical for?
A-Chronic renal failure
B-Nephritic syndrome
C-Nephrotic syndrome
D-Chronic pyelonephritis
E-Acute renal failure
A 44 year old woman complains of general weakness, heart pain, significant increase of body weight.
Objectively: moon face, hirsutism, AP is 165/100 mm Hg, height - 164 cm, weight - 103 kg; the fat is mostly
accumulated on her neck, thoracic girdle, belly. What is the main pathogenetic mechanism of obesity?
A-Increased production of glucocorticoids *
B-Increased insulin production
C-Reduced glucagon production
D-Increased mineralocorticoid production
E-Reduced production of thyroid hormones
A 64 year old woman has impairment of twilight vision (hemeralopy). What vitamin should be recommended
in the first place?
A-Vitamin B6
B-Vitamin B2
C-Vitamin A
D-Vitamin C
E-Vitamin E
Hepatitis has led to the development of hepatic failur E. Mechanism of edemata formation is activated by the
impairment of the following liver function:
A-Antitoxic
B-Glycogen-synthetic
C-Barrier

D-Protein-synthetic
E-Chologenetic
A patient with obliterating atherosclerosis underwent sympathectomy of femoral artery in the region of
femoral trigon E. What type of arterial hyperemia was induced by the operation?
A-Reactive
B-Functional
C-Metabolic
D-Neurotonic
E-Neuroparalytic
A 47 year old man with myocardium infarction was admitted to the cardiological department. What changes
of cellular composition of peripheral blood are induced by necrotic changes in the myocardium?
A-Monocytosis
B-Neutrophilic leukocytosis
C-Thrombocytopenia
D-Lymphopenia
E-Eosinophilic leukocytosis
A 46 year old woman suffering from chololithiasis developed jaundic E. Her urine became dark-yellow and
feces became colourless. Blood serum will have the highest concentration of the following substance:
A-Unconjugated bilirubin
B-Mesobilirubin
C-Biliverdin
D-Urobilinogen
E-Conjugated bilirubin
Examination of a 42 year old patient revealed a tumour of adenohypophysis. Objectively: the patients weight
is 117 kg, he has moon-like hyperemic face, red-blue striae of skin distension on his belly. Osteoporosis and
muscle dystrophy are present. AP is 210/140 mm Hg. What is the most probable diagnosis?
A-Cushings disease
B-Conns disease
C-Cushings syndrome
D-Essential hypertension
E-Diabetes mellitus
A patient is ill with diabetes mellitus accompanied by hyperglycemia on an empty stomach (7,2 millimole/l).
The hyperglycemia rate can be retrospectively estimated (over the last 4-8 weeks before the examination) on
the ground of the rate of the following blood plasma protein:
A-Albumin
B-Glycated hemoglobin
C-Fibrinogen
D-C-reactive protein
E-Ceruloplasmin
A 4 year old child complained of pain during deglutition, indisposition. Objectively: palatine arches and
tonsils are moderately edematic and hyperemic, there are greyish-white films up to 1 mm thick closely
adhering to the subjacent tissues. What pathological process are these changes typical for?
A-Inflammation
B-Organization
C-Necrosis
D-Dystrophy
E-Metaplasia

Atria of an experimental animal were superdistended by blood that resulted in decreased reabsorption of Na+
and water in renal tubules. This can be explained by the influence of the following factor upon kidneys:
A-Renin
B-Angiotensin
C-Natriuretic hormone
D-Aldosterone
E-Vasopressin
A 56 year old patient suffering from cardiac insufficiency has edema of feet and shins, edematous skin is pale
and col D. What is the leding mechanism of edema pathogenesis?
A-Disorder of lymph outflow
B-Positive water balance
C-Rise of hydrostatic pressure in venules
D-Drop of oncotic pessure in capillaries
E-Increase of capillary permeability
A 50 year old man who was referred to the hospital for treatment of cervical lymphadenitis underwent test
for induvidual sensitivity to penicillin. 30 seconds after he went hot all over, AP dropped down to 0 mm Hg
that led to cardiac arrest. Resuscitation was unsuccessful. Autopsy results: acute venous plethora of internal
organs; histological examination of skin (from the site of injection) revealed degranulation of mast cells
(tissue basophils). Degranulation was also revealed in myocardium and lungs. What type of hypersensitivity
reaction is it?
A)B-Immunocomplex-mediated
C-Complement-mediated cytotoxic
D-Anaphylactic
E-Delayed-type hypersensitivity
A 25 year old Palestinian woman complains of weakness, dizziness, dyspne A. In anamnesis: periodically
exacerbating anemi A. In blood: Hb - 60 g/l, erythrocytes - 2,51012/l, reticulocytes - 35o/oo, anisocytosis and
poikilocytosis of erythrocytes, a lot of target cells and polychromatophils. What type of anemia is it?
A-Addison-Biermer disease
B-Minkowsky-Shauffard disease
C-Thalassemia
D-Sickle-cell anemia
E-Glucose 6-phosphate dehydrogenase-deficient anemia
A clinic observes a 49 year old patient with significant prolongation of coagulation time, gastrointestinal
haemorrhages, subcutaneous hematomas. These symptoms might be explained by the deficiency of the
following vitamin:
A-K
B-E
C-B6
D-H
E-B1
A patient suffering from periodical attacks caused by inhalation of different flavoring substances was
diagnosed with atopic bronchial asthm A. IgE level was increase D. This is typical for the following type of
reactions:
A-Cytotoxic reactions
B-delayed-type hypersensitivity
C-Anaphylactic reactions
D-Immunocomplex reactions
E-Autoimmune reactions

48 hours after tuberculine test (Mantoux test) a child had a papule 10 mm in diameter on the spot of
tuberculine injection. What hypersensitivity mechanism underlies these changes?
A-Antibody-dependent cytotoxicity
B-Anaphylaxy
C-Cellular cytotoxicity
D-Granulomatosis
E-Immunocomplex cytotoxicity
A 48 year old patient complained about intense pain, slight swelling and reddening of skin over the joints,
temperature rise up to 38oC. Blood analysis revealed high concentration of urates. This condition might be
caused by disturbed metabolism of:
A-Cholesterol
B-Pyrimidines
C-Collagen
D-Carbohydrates
E-Purines

A concentrated solution of sodium chloride was intravenously injected to an animal. This caused decreased
reabsorption of sodium ions in the renal tubules. It is the result of the following changes of hormonal
secretion:
A-Vasopressin increase
B-Vasopressin reduction
C-Aldosterone increase
D-Aldosterone reduction
E-Reduction of atrial natriuretic factor
A 17 year old boy fell seriously ill, the body temperature rose up to 38,5oC, there appeared cough, rhinitis,
lacrimation, nasal discharges. What inflammation is it?
A-Catarrhal
B-Hemorrhagic
C-Fibrinous
D-Purulent
E-Serous
A 45 year old patient was admitted to the cardiological department. ECG data: negative P wave overlaps
QRS complex, diastolic interval is prolonged after extrasystol E. What type of extrasystole is it?
A-Ventricular
B-Atrioventricular
C-Bundle-branch
D-Sinus
E-Atrial
A 5 year old child is ill with measles. Blood analysis revealed increase of total number of leukocytes up to
13109/l. Leukogram: basophils - 0, eosinophils - 1, myelocytes - 0, juvenile neutrophils - 0, band neutrophils 2, segmented neutrophils - 41, lymphocytes - 28, monocytes - 28. Name this phenomenon:
A-Lymphocytosis
B-Eosinopenia
C-Neutropenia
D-Monocytosis
E-Agranulocytosis
Voluntary breath-holding caused increase of respiration depth and frequency. The main factor stimulating
these changes of external respiration is:
A-Decreased tension of CO2 in blood

B-Increased tension of CO2 in blood


C-Decreased concentration of H+ in blood
D-Decreased tension of O2 in blood
E-Increased tension of O2 in blood
A patient staying in the pulmonological department was diagnosed with pulmonary emphysema accompanied
by reduced elasticity of pulmonary tissu E. What type of respiration is observed?
A-Inspiratory dyspnea
B-Superficial respiration
C-Expiratory dyspnea
D-Infrequent respiration
E-Periodic respiration
A patient ill with neurodermatitis has been taking prednisolone for a long tim E. Examination revealed high
rate of sugar in his bloo D. This complication is caused by the drug influence upon the following link of
carbohydrate metabolism:
A-Glycogenogenesis activation
B-Activation of insulin decomposition
C-Inhibition of glycogen synthesis
D-Gluconeogenesis activation
E-Intensification of glucose absorption in the bowels
A 34 year old woman was diagnosed with hereditary microspherocytic hemolytic anemia (MinkowskyShauffard disease). What mechanism caused haemolysis of erythrocytes?
A-Hemoglobinopathy
B-Autoimmune disorder
C-Enzymopathy
D-Bone marrow hypoploasia
E-Membranopathy
A 25 year old man has spent a long time in the sun under high air humidity. As a result of it his body
temperature rose up to 39oC. What pathological process is it?
A-Noninfectious fever
B-Burn disease
C-Infectious fever
D-Hypothermia
E-Hyperthermia
*** *** ***
A 2-year-old child with mental and physical retardation has been delivered to a hospital. He presents with
frequent vomiting after having meals. There is phenylpyruvic acid in urine. Which metabolism abnormality is
the reason for this pathology?
A-Amino-acid metabolism
B-Water-salt metabolism
C-Phosphoric calcium metabolism
D-Carbohydrate metabolism
E-Lipidic metabolism
A student failed to answer all the questions of examination paper correctly. As a result he blushed, felt hot
and lost confidence. What type of arterial hyperemia has developed in this case?
A-Metabolic hyperemia
B-Neuroparalytic hyperemia
C-Neurotonic hyperemia
D-Pathologic hyperemia
E-Postishemic hyperemia

A 45-year-old patient was admitted to the cardiological department. ECG data: negative P wave overlaps
QRS complex, diastolic interval is prolonged after extrasystole. What type of extrasystole is it?
A-Atrioventricular
B-Ventricular
C-Sinus
D-Atrial
E-Bundle-branch
A 25-year-old man has spent a long time in the sun under high air humidity. As a result of it his body
temperature rose up to 39C. What pathological process is it?
A-Noninfectious fever
B-Hyperthermia
C-Hypothermia
D-Burn disease
E-Infectious fever
12 hours after an accute attack of retrosternal pain a patient presented a jump of aspartate aminotransferase
activity in blood serum. What pathology is this deviation typical for?
A-Viral hepatitis
B-Myocardium infarction
C-Collagenosis
D-Diabetes insipidus
E-Diabetes mellitus
After an immunoassay a child was diagnosed with immunodeficiency of humoral immunity. What is the
reason for the primary immunodeficiency development in the child?
A-Immune responsiveness and resistance disorders
B-Toxic damage of B-lymphocytes
C-Pathometabolism in mothers organism
D-Hereditary abnormality of immune system
E-Embryonal development abnormalities
An infant has pylorospasm, weakness, hypodynamia, convulsions as a result of frequent vomiting. What kind
of acid-base disbalance is it?
A-Metabolic acidosis
B-Gaseous alkalosis
C-Exogenous nongaseous acidosis
D-Excretory acidosis
E-Excretory alkalosis
A 26-year-old man is in the torpid shock phase as a result of a car accident. In blood: 3,2109/l. What is the
leading mechanism of leukopenia development?
A-Intensified elimination of leukocytes from the organism
B-Redistribution of leukocytes in bloodstream
C-Leikopoiesis inhibition
D-Disturbed going out of mature leukocytes from the marrow into the blood
E-Lysis of leukocytes in the blood-forming organs
Blood analysis of a patient with jaundice reveals conjugated bilirubinemia, increased concentration of bile
acids. There is no stercobilinogen in urine. What type of jaundice is it?
A-Hemolytic jaundice
B-Obstructive jaundice
C-Parenchymatous jaundice
D-Cythemolytic jaundice
E-Hepatocellular jaundice

ECG of a 44-year-old patient shows signs of hypertrophy of both ventricles and the right atrium. The patient
was diagnosed with the tricuspid valve insufficiency. What pathogenetic variant of cardiac dysfunction is
usually observed in case of such insufficiency?
A-Primary myocardial insufficiency
B-Cardiac tamponade
C-Coronary insufficiency
D-Heart overload by resistance
E-Heart overload by volume
A 30-year-old male patient with acute pancreatitis has been found to have a disorder of cavitary protein
digestion. The reason for such condition can be the hyposynthesis and hyposecretion of the following enzyme:
A-Pepsin
B-Amylase
C-Tripsin
D-Lipase
E-Dipeptidase
Blood count of an athlete is as follows: erythrocytes - 5,51012/l, Hb- 180 g/l, leukocytes 7109/l, neutrophils
- 64%, basophils - 0,5%, eosinophils - 0,5%, monocytes - 8%, lymphocytes - 27%. First of all, such results
indicate the stimulation of:
A-Immunogenesis
B-Lymphopoiesis
C-Leukopoiesis
D-Erythropoiesis
E-Granulocytopoiesis
A 19-year-old female patient has had low haemoglobin rate of 90-95 g/l since childhood. Blood count results
obtained after hospitalisation are as follows: erythrocytes - 3,21012/l, Hb- 85 g/l, colour index - 0,78;
leukocytes - 5,6109/l, platelets 210109/l. Smear examination revealed anisocytosis, poikilocytosis and
target cells. Reticulocyte rate is 6%. Iron therapy was ineffective. What blood pathology corresponds with the
described clinical presentations?
A-Favism
B-Enzymopathy
C-Thalassemia
D-Membranopathy
E-Sickle-cell anemi
A patient is 44 years old. Laboratory examination of his blood revealed that content of proteins in plasma was
40 g/l. What influence will be exerted on the transcapillary water metabolism?
A-Filtration will be decreased, reabsorption - increased
B-Both filtration and reabsorption will be decreased
C-Filtration will be increased, reabsorption decreased
D-Both filtration and reabsorption will be increased
E-Metabolism will stay unchanged
A disaster fighter at a nuclear power plant developed hemorrhagic syndrome on the background of acute
radiation disease. What is the most important factor of syndrome pathogenesis?
A-Decreased activity of coagulative factors
B-Increased activity of fibrinolysis factors
C-Vascular wall damage
D-Thrombocytopenia
E-Increased activity of anticoagulative system factors

A patient with enteritis accompanied by massive diarrhea has low water rate in the extracellular space, high
water rate inside the cells and low blood osmolarity. What is such disturbance of water-electrolytic
metabolism called?
A-Osmolar hypohydration
B-Hypo-osmolar hyperhydration
C-Hyperosmolar hyperhydration
D-Hypo-osmolar hypohydration
E-Hyperosmolar hypohydration
A tooth extraction in a patient with chronic persistent hepatitis was complicated with prolonged hemorrhage.
What is the reason for the haemorrhagic syndrome?
A-Decrease in thrombin production
B-Increase in thromboplastin production
C-Decrease in fibrin production
D-Increase in fibrinogen synthesis
E-Fibrinolysis intensification
A patient complains of hydruria (7 liters per day) and polydipsia. Examination reveals no disorders of
carbohydrate metabolism. These abnormalities might be caused by the dysfunction of the following endocrine
gland:
A-Islets of Langerhans (pancreatic islets)
B-Adrenal cortex
C-Adenohypophysis
D-Neurohypophysis
E-Adrenal medulla
A patient has been diagnosed with alkaptonuria. Choose an enzyme whose deficiency can be the reason for
this pathology:
A-Phenylalanine hydroxylase
B-Homogentisic acid oxidase
C-Dioxyphenylalanine decarboxylase
D-Glutamate dehydrogenase
E-Pyruvate dehydrogenase
A patient has pellagra. Interrogation revealed that he had lived mostly on maize for a long time and eaten
little meat. This disease had been caused by the deficit of the following substance in the maize:
A-Alanine
B-Tryptophan
C-Tyrosine
D-Proline
E-Histidine
A child with a history of frequent angine and pharyngitis has been diagnosed with lymphadenopathy and
splenomegaly. His appearance is characterised by pastosity and paleness, muscular tissue is poorly developed.
Lymphocytosis is present. What kind of diathesis is it?
A-Lymphohypoplastic diathesis
B-Hemorrhagic diathesis
C-Asthenic diathesis
D-Exudative diathesis
E-Gouty diathesis
After transfusion of 200 ml of blood a patient presented with body temperature rise up to 37,9C. Which of
the following substances is the most likely cause of temperature rise?
A-Interleukin-1
B-Interleukin-2
C-Interleukin-4

D-Interleukin-3
E-Tumour necrosis factor
A patient with massive burns developed acute renal insufficiency characterized by a significant and rapid
deceleration of glomerular filtration. What is the mechanism of its development?
A-Rise of pressure of tubular fluid
B-Renal artery embolism
C-Damage of glomerular filter
D-Reduction of functioning nephron number
E-Reduction of renal blood flow
During surgical manipulations a patient has been given novocaine injection for anesthesia. 10 minutes later
the patient developed paleness, dyspnea, hypotension. What type of allergic reaction is it?
A-Cell-mediated immune reaction
B-Anaphylactic immune reaction
C-Aggregate immune reaction
D-Cellulotoxic immune reaction
E-Stimulating immune reaction
A female patient presents with endocrine dysfunction of follicular cells of the ovarian follicles resulting from
an inflammation. The synthesis of the following hormone will be inhibited:
A-Progesterone
B-Lutropin
C-Estrogen
D-Follistatine
E-Follicle stimulating hormone
The minute blood volume in a patient with transplanted heart has increased as a result of physical activity.
What regulative mechanism is responsible for these changes?
A-Parasympathetic unconditioned reflexes
B-Sympathetic conditioned reflexes
C-Sympathetic unconditioned reflexes
D-Parasympathetic conditioned reflexes
E-Catecholamines
A 45-year-old woman has breast cancer. Her left arm has symptoms of lymphatic system insufficiency - limb
edema, lymph node enlargement. What form of lymphatic circulation insufficiency is it?
A-Dynamic insufficiency
B-Mechanic insufficiency
C-Resorption insufficiency
D-Combined insufficiency
E)During starvation muscle proteins break up into free amino acids. These compounds will be the most
probably involved into the following process:
A-Gluconeogenesis in muscles
B-Synthesis of higher fatty acids
C-Decarboxylation
D-Glycogenolysis
E-Gluconeogenesis in liver
A patient presents with icteritiousness of skin, scleras and mucous membranes. Blood plasma the total
bilirubin is increased, stercobilin is increased in feces, urobilin is increased in urine. What type of jaundice is
it?
A-Gilberts disease

B-Cholestatic
C-Obturational
D-Haemolytic
E-Parenchymatous
The patient with complaints of permanent thirst applied to the doctor. Hyperglycemia, polyuria and
increased concentration of 17-ketosteroids in the urine were revealed. What disease is the most likely?
A-Addisons disease
B-Type I glycogenosis
C-Myxoedema
D-Steroid diabetes
- Addisons syndrome
Sex chromosomes of a woman didnt separate and move to the opposite poles of a cell during gametogenesis
(meiosis). The ovum was impregnated with a normal spermatozoon. Which chromosomal disease can be
found in her child?
A-Pataus syndrome
B-Turners syndrome
C-Edwards syndrome
D-Cat cry syndrome
E-Downs syndrome
A concentrated solution of sodium chloride was
intravenously injected to an animal. This caused decreased reabsorption of sodium ions in the renal tubules.
It is the result of the following changes of hormonal secretion:
A-Vasopressin increase
B-Vasopressin reduction
C-Reduction of atrial natriuretic factor
D-Aldosterone reduction
E-Aldosterone increase
A newborn child with pylorostenosis has often repeating vomiting accompanied by apathy, weakness,
hypertonicity, sometimes convulsions. What disorder form of acid-base balance is it?
A-Gaseous alkalosis
B-Metabolic acidosis
C-Nongaseous alkalosis
D-Excretory acidosis
E-Gaseous acidosis
A patient has been diagnosed with influenza. His condition became drastically worse after taking antipyretic
drugs. His consciousness is confused, AP is 80/50 mm Hg, Ps is 140/m, body temperature droped down to
35,8C. What complication developed in this patient?
A-Acidosis
B-Hypovolemia
C-Collapse
D-Hyperthermia
E-Alkalosis
Medical examination at the military registration and enlistment office revealed that a 15-year-old boy was
high, with eunuchoid body proportions, gynecomastia, female pattern of pubic hair distribution. The boy had
also fat deposits on the thighs, no facial hair, high voice, subnormal intelligence quotient. Which karyotype
corresponds with this disease?
A-46, XX
B-47, XXY
C-45, XO

D-47, XXX
E-46, XY
A patient with marked pneumofibrosis that developed after infiltrating pulmonary tuberculosis has been
diagnosed with respiratory failure. What is its pathogenetic type?
A-Dysregulatory
B-Reflex
C-Restrictive
D-Apneistic
E-Obstructiv
As a result of continuous starvation the glomerular filtration rate has increased by 20%. The most probable
cause of the glomerular filtration alteration under the mentioned conditions is:
A-Increase in the systemic arterial pressure
B-Decrease in the oncotic pressure of blood plasma
C-Increase in the permeability of the renal filter
D-Increase of the filtartion quotient
E-Increase of the renal blood flow
A 28-year-old female patient consulted a gynecologist about sterility. Examination revealed underdeveloped
ovaries and uterus, irregular menstrual cycle. Analysis of the sex chromatin revealed 2 Barrs bodies in most
somatic cells. What chromosome disease is most likely?
A-Edwards syndrome
B-Turners syndrome
C-Klinefelters syndrome
D-Pataus syndrome
E-Triple X syndrome
Blood plasma of a healthy man contains several dozens of proteins. During an illness new proteins can
originate, namely the protein of "acute phase". Select such protein from the listed below:
A-A immunoglobulin
B-C-reactive protein
C-G immunoglobulin
D-Prothrombin
E-Fibrinogen
A 46-year-old patient suffering from the diffuse toxic goiter underwent resection of the thyroid gland. After
the surgery the patient presents with appetite loss, dyspepsia, increased neuromuscular excitement. The body
weight remained unchanged. Body temperature is normal. Which of the following has caused such a
condition in this patient?
A-Increased production of calcitonin
B-Increased production of thyroxin
C-Reduced production of parathormone
D-Increased production of thyroliberin
E-Reduced production of thyroxin
Pulmonary examination of a patient who has worked as a stone grinder for 9 years revealed small dense
roundish nodules consisting of connective tissue. The nodules were found to have peripheral macrophages.
Such pulmonary alterations are indicative of the following disease:
A-Silicosis
B-Bronchial asthma
C-Acute pneumonia
D-Chronic bronchitis
E-Multiple bronchiectasis

A 32-year-old patient consulted a doctor about the absence of lactation after parturition. Such disorder might
be explained by the deficit of the following hormone:
A-Prolactin
B-Glucagon
C-Thyrocalcitonin
D-Somatotropin
E-Vasopressin
A 10-year-old child had the mantoux tuberculin test administered. 48 hours later a papule up to 8 mm in
diameter appeared on the site of the injection. What type of hypersensitivity reaction developed after the
tuberculin injection?
A-Seroreaction
B-Type II hypersensitivity reaction
C-Arthus phenomenon
D-Atopic reaction
E-Type IV hypersensitivity reaction
Lung ventilation in a person is increased as a result of physical activity. Which of the following indices of the
external respiration is much higher than in a state of rest?
A-Respiratory volume
B-Vital capacity of lungs
C-Total lung capacity
D-Inspiratory reserve volume
E-Expiratory reserve volume
A patient has been given high doses of hydrocortisone for a long time. This caused atrophy of one of the
adrenal cortex zones. Which zone is it?
A)B-Reticular
C-Glomerular and reticular
D-Fascial
E-Glomerular
A coprological survey revealed light-colored feces containing drops of neutral fat. The most likely reason for
this condition is the disorder of:
A-Intestinal absorption
B-Gastric juice acidity
C-Pancreatic juice secretion
D-Intestinal juice secretion
E-Bile inflow into the bowel
A 10-year-old girl has a history of repeated acute respiratory viral infection. After recovering she presents
with multiple petechial hemorrhages on the sites of friction from clothing rubbing the skin. What kind of
hypovitaminosis has this girl?
A-C
B-B1
C-B2
D-B6
E-A
A 38-year-old male patient has been ill with systemic lupus erythematosus for three years. He was diagnosed
with diffuse renal affection accompanied by massive edemata and expressive proteinuria. What is the most
likely cause of proteinuria development?
A-Urinary bladder inflammation
B-Autoimmune renal affection

C-Ischemic renal affection


D-Aseptic renal affection
E-Urinary tracts inflammation
One of the parents is suspected of having phenylketonuria recessive gene. What is the risk of giving birth to a
child with inborn phenylketonuria?
A-75%
B-0%
C-50%
D-25%
E-100%
A newborn develops dyspepsia after the milk feeding. When the milk is substituted by the glucose solution the
dyspepsia symptoms disappear. The newborn has the subnormal activity of the following enzyme:
A-Maltase
B-Isomaltase
C-Lactase
D-Amylase
E-Invertase
A 12-year-old teenager has significantly put off weight within 3 months; glucose concentration rose up to 50
millimole/l. He fell into a coma. What is the main mechanism of its development?
A-Lactacidemic
B-Hyperosmolar
C-Ketonemic
D-Hypoxic
E-Hypoglycemic
*** *** ***
Examination of patients with periodontitis revealed the interdependence between the rate of affection of
periodontal tissues and the amount of lysozymes in saliva and gingival liquid. These results can be obtained
during studying the following protection system of an organism:
A-Non-specific resistance
B-Humoral immunity
C-Tolerance
D-Cellular immunity
E-Autoresponsiveness
According to the phenotypic diagnosis a female patient has been provisionally diagnosed with X-chromosome
polysomia. This diagnosis can be confirmed by a cytogenetic method. What karyotype will allow to confirm
the diagnosis?
A-48(XXXY)
B-47(XXY)
C-46(XX)
D-47()
E-48(XXYY)
parodontitis is treated with calcium preparations and a hormone that stimulates tooth mineralization and
inhibits tissue resorption. What hormone is it?
A-Adrenalin
B-Aldosterone
C-Calcitonin
D-Parathormone
E-Thyroxine

A 60 year old patient was found to have a dysfunction of main digestive enzyme of saliva. This causes the
disturbance of primary hydrolysis of:
A-Cellulose
B-Fats
C-Lactose
D-Proteins
E-Carbohydrates
A 26 year old pregnant woman is under treatment at an in-patient hospital. After a continuous attack of
vomiting she was found to have reduced volume of circulating blood. What kind of change in general blood
volume is the case?
A-Polycythemic hypervolemia
B-Oligocythemic hypovolemia
C-Polycythemic hypovolemia
D-Oligocythemic hypervolemia
E-Simple hypovolemia
A man has a considerable decrease in diuresis as a result of 1,5 l blood loss. The primary cause of such
diuresis disorder is the hypersecretion of the following hormone:
A-Corticotropin
B-Vasopressin
C-Natriuretic
D-Parathormone
E-Cortisol
A 20 year old patient complains of morbid thirst and huperdiuresis (up to 10 l daily). Glucose concentration
in blood is normal but it is absent in urine. The patient has been diagnosed with diabetes insipidus. What
hormonal drug is the most appropriate for management of this disorder?
A-Thyroxin
B-Insulin
C-Oxytocin
D-Vasopressin
E-Cortisol
A patient has been diagnosed with alkaptonuria. Choose an enzyme whose deficiency can be the reason for
this pathology:
A-Dioxyphenylalanine decarboxylase
B-Homogentisic acid oxidase
C-Phenylalanine hydroxylase
D-Pyruvate dehydrogenase
E-Glutamate dehydrogenase
A patient suffering from chronic myeloleukemia has got the following symptoms of anemia: decreased
number of erythrocytes and low haemoglobin concentration, oxyphilic and polychromatophilic normocytes,
microcytes. What is the leading pathogenetic mechanism of anemia development?
A-Substitution of haemoblast
B-Intravascular hemolysis of erythrocytes
C-Reduced synthesis of erythropoietin
D-Deficiency of vitamin B12
E-Chronic haemorrhage
From the group of children who were eating sweet sappy watermelon two kids developed the signs of
poisoning: rapid weakness, dizziness, headache, vomiting, edema, tachycardia, cyanosis of mouth, ears, tips of
the fingers cyanosis. High concentration of nitrates was detected. What is the leading mechanism of the
pathogenesis of the poisoning in the two children?

A-Insufficiency of superoxiddismutase
B-Insufficiency of catalase
C-Block cytochrome oxidase
D-Insufficiency of met-Hb-reductase
E-Insufficiency glutathione pyroxidase
A 44 year old woman complains of general weakness, heart pain, significant increase of body weight.
Objectively: moon face, hirsutism, AP is 165/100 mm Hg, height - 164 cm, weight - 103 kg; the fat is mostly
accumulated on her neck, thoracic girdle, belly. What is the main pathogenetic mechanism of obesity?
A-Increased production of glucocorticoids
B-Reduced glucagon production
C-Increased mineralocorticoid production
D-Increased insulin production
E-Reduced production of thyroid hormones
The patient with acute miocardial infarction was given intravenously different solutions during 8 hours with
medical dropper 1500 ml and oxygen intranasally. He died because of pulmonary edema. What caused the
pulmonary edema?
A-Neurogenic reaction
B-Decreased oncotic pressure due to hemodilution
C-Inhalation of the oxygen
D-Allergic reaction
E-Volume overload of the left ventricular
A patient with a history of chronic glomerulonephritis presents with azotemia, oliguria, hypo- and
isosthenuria, proteinuria. What is the leading factor in the pathogenesis of these symptoms development
under chronic renal failure?
A-Disturbed permeability of glomerular membranes
B-Mass decrease of active nephrons
C-Tubular hyposecretion
D-Intensification of sodium reabsorption
E-Intensification of glomerular filtration
A 67 year old patient complains of periodic heart ache, dyspnea during light physical activities. ECG reveals
extraordinary contractions of heart ventricles. Such arrhythmia is called:
A-Extrasystole
B-Tachycardia
C-Fibrillation
D-Flutter
E-Bradycardia
A 5-month-old boy was hospitalized for tonic convulsions. He has a life-time history of this disease.
Examination revealed coarse hair, thinned and fragile nails, pale and dry skin. In blood: calcium - 1,5
millimole/l, phosphor - 1,9 millimole/l. These changes are associated with:
A-Hypothyroidism
B-Hyperaldosteronism
C-Hypoparathyroidism
D-Hyperparathyroidism
E-Hypoaldosteronism
Having helped to eliminate consequences of a failure at a nuclear power plant, a worker got an irradiation
doze of 500 roentgen. He complains of headache, nausea, dizziness. What changes in leukocytes quantity can
be expected 10 hours after irradiation?
A-Neutrophilic leukocytosis
B-Agranulocytosis

C-Leukopenia
D-Leukemia
E-Lymphocytosis
A child has abnormal formation of tooth enamel and dentin as a result of low concentration of calcium ions in
blood. Such abnormalities might be caused by deficiency of the following hormone:
A-Somatotropic hormone
B-Thyrocalcitonin
C-Parathormone
D-Triiodothyronine
E-Thyroxin
Examination of a child who frequently suffers from infectious diseases revealed that IgG concentration in
blood serum was 10 times less than normal, IgA and IgM concentration was also significantly reduced.
Analysis showed also lack of B-lymphocytes and plasmocytes. What disease are these symptoms typical for?
A-Di George syndrome
B-Dysimmunoglobulinemia
C-Louis-Bar syndrome
D-Swiss-type agammaglobulinemia
E-Brutons disease
A patient has been diagnosed with acute glomerulonephritis that developed after he had had streptococcal
infection. It is most likely that the affection of basal glomerular membrane is caused by an allergic reaction of
the following type:
A-Delayed
B-Anaphylactic
C-Cytotoxic
D-Stimulating
E-Immune complex
Blood test of a patient suffering from atrophic gastritis gave the following results: RBCs - 2,01012/l, Hb- 87
g/l, colour index - 1,3, WBCs - 4,0109/l, thrombocytes 180109/l. Anaemia migh have been caused by the
following substance deficiency:
A-Iron
B-Vitamin K
C-Zinc
D-Vitamin B12
E-Vitamin A
Atria of an experimental animal were superdistended by blood that resulted in decreased reabsorption of Na+
and water in renal tubules. This can be explained by the influence of the following factor upon kidneys:
A-Natriuretic hormone
B-Angiotensin
C-Vasopressin
D-Renin
E-Aldosterone
As a result of a road accident a 26-year-old man is in the torpid phase of shock. Blood count: leukocytes 3,2109/l. What is the leading mechanism of leukopenia development?
A-Leukocyte redistribution in the bloodstream
B-Leukocyte destruction in the hematopietic organs
C-Leukopoiesis inhibition
D-Increased excretion of the leukocytes from the organism
E-Faulty release of mature leukocytes from the bone marrow into the blood

After taking poor-quality food a patient developed repeated episodes of diarrhea. On the next day he
presented with decreased arterial pressure, tachycardia, extrasystole. Blood pH is 7,18. These abnormalities
were caused by the development of:
A-Metabolic alkalosis
B-Nongaseous acidosis
C-Nongaseous alkalosis
D-Gaseous alkalosis
E-Gaseous acidosis
After a serious psychoemotional stress a 48 year old patient suddenly developed acute heart ache irradiating
to the left arm. Nitroglycerine relieved pain after 10 minutes. What is the leading pathogenetic mechanism of
this process development?
A-Compression of coronary vessels
B-Spasm of coronary arteries
C-Obstruction of coronary vessels
D-Dilatation of peripheral vessels
E-Increase in myocardial oxygen consumption
After an attack of bronchial asthma a patient had his peripheral blood tested. What changes can be expected?
A-Lymphocytosis
B-Eosinophilia
C-Thrombocytopenia
D-Leukopenia
E-Erythrocytosis
An infant has pylorospasm, weakness, hypodynamia, convulsions as a result of frequent vomiting. What kind
of acid-base disbalance is it?
A-Gaseous alkalosis
B-Excretory alkalosis
C-Exogenous nongaseous acidosis
D-Excretory acidosis
E-Metabolic acidosis
A 42 year old patient complains of pain in the epigastral area, vomiting; vomit masses have the colour of
"coffee-grounds", the patient has also melena. Anamnesis records gastric ulcer. Blood formula: erythrocytes 2,81012/l, leukocytes 8109/l, Hb- 90 g/l. What complication is it?
A-Perforation
B-Haemorrhage
C-Penetration
D-Canceration
E-Pyloric stenosis
HIV has gp41 and gp120 on its surface interacts with target cells of an organism. Which of the following
human lymphocyte antigens is gp120 complementary bound with?
A-CD 28
B-CD 4
C-CD 19
D-CD 8
E-CD 3
Blood analysis of a patient with jaundice reveals conjugated bilirubinemia, increased concentration of bile
acids. There is no stercobilinogen in urine. What type of jaundice is it?
A-Hepatocellular jaundice
B-Obstructive jaundice
C-Cythemolytic jaundice

D-Parenchymatous jaundice
E-Hemolytic jaundice
After a tourniquet application a patient was found to have petechial haemorrhages. The reason for it is the
dysfunction of the following cells:
A-Platelets
B-Lymphocytes
C-Monocytes
D-Neutrophils
E-Eosinophils
A 2 year old child with mental and physical retardation has been delivered to a hospital. He presents with
frequent vomiting after having meals. There is phenylpyruvic acid in urine. Which metabolism abnormality is
the reason for this pathology?
A-Lipidic metabolism
B-Water-salt metabolism
C-Phosphoric calcium metabolism
D-Amino-acid metabolism
E-Carbohydrate metabolism
A month after surgical constriction of rabbits renal artery the considerable increase of systematic arterial
pressure was observed. What of the following regulation mechanisms caused the animals pressure change?
A-Serotonin
B-Noradrenaline
C-Vasopressin
D-Adrenaline
E-Angiotensin-II
Sex chromosomes of a woman didnt separate and move to the opposite poles of a cell during gametogenesis
(meiosis). The ovum was impregnated with a normal spermatozoon. Which chromosomal disease can be
found in her child?
A-Pataus syndrome
B-Turners syndrome
C-Cat cry syndrome
D-Edwards syndrome
E-Downs syndrome
A 49 year old woman spent a lot of time standing. As a result of it she got leg edema. What is the most likely
cause of the edema?
A-Decrease in hydrostatic pressure of blood in veins
B-Increase in hydrostatic pressure of blood in veins
C-Decrease in hydrostatic pressure of blood in arteries
D-Increase in oncotic pressure of blood plasma
E-Increase in systemic arterial pressure
After severe viral hepatitis a 4 year old boy presents with vommiting, occasional loss of consciousness,
convulsions. Blood test revealed hyperammoniemia. Such condition is caused by a disorder of the following
biochemical hepatic process:
A-Disorder of biogenic amines neutralization
B-Disorder of ammonia neutralization
C-Inhibition of transamination enzymes
D-Protein synthesis inhibition
E-Activation of amino acid decarboxylation

Before the cells can utilize the glucoze, it is first transported from the extracellular space through the
plasmatic membrane inside theml. This process is stimulated by the following hormone:
A-Adrenalin
B-Insulin
C-Aldosterone
D-Glucagon
E-Thyroxin
A patient complains about dyspnea provoked by the physical activity. Clinical examination revealed anaemia
and presence of the paraprotein in the zone of gamma-globulins. To confirm the myeloma diagnosis it is
necessary to determine the following index in the patients urine:
A-Antitrypsin
B-Ceruloplasmin
C-Bence Jones protein
D-Bilirubin
E-Haemoglobin
A 50 year old patient suffers from essential hypertension. After a physical stress he experienced muscle
weakness, breathlessness, cyanosis of lips, skin and face. Respiration was accompanied by distinctly heard
bubbling rales. What mechanism underlies the development of this syndrome?
A-Chronic left-ventricular failure
B-Chronic right-ventricular failure
C-Acute left-ventricular failure
D-Cardiac tamponade
E-Collapse
An infectious disease unit admitted a patient with signs of jaundice caused by hepatitis virus. Select an
indicator that is specific only for parenchymatous jaundice:
A-Cholaemia
B-Bilirubinuria
C-Increase in ALT and AST rate
D-Hyperbilirubinemia
E-Urobilinuria
A disaster fighter at a nuclear power plant developed hemorrhagic syndrome on the background of acute
radiation disease. What is the most important factor of syndrome pathogenesis?
A-Decreased activity of coagulative factors
B-Vascular wall damage
C-Increased activity of fibrinolysis factors
D-Thrombocytopenia
E-Increased activity of anticoagulative system factors
Examination of a 42 year old patient revealed a tumour of adenohypophysis. Objectively: the patients weight
is 117 kg, he has moon-like hyperemic face, red-blue striae of skin distension on his belly. Osteoporosis and
muscle dystrophy are present. AP is 210/140 mm Hg. What is the most probable diagnosis?
A-Cushings disease
B-Essential hypertension
C-Conns disease
D-Cushings syndrome
E-Diabetes mellitus
A child is pale, pastose, muscular tissue is bad developed, lymph nodes are enlarged. He often suffers from
angina and pharyngitis, blood has signs of lymphocytosis. The child is also predisposed to autoallergic
diseases. What type of diathesis can be presumed in this case?
A-Lymphohypoplastic
B-Hemorrhagic

C-Asthenic
D-Exudative
E-Gouty
A 56 year old patient suffering from cardiac insufficiency has edema of feet and shins, edematous skin is pale
and cold. What is the leding mechanism of edema pathogenesis?
A-Positive water balance
B-Rise of hydrostatic pressure in venules
C-Increase of capillary permeability
D-Drop of oncotic pessure in capillaries
E-Disorder of lymph outflow
A 64 year old woman has impairment of twilight vision (hemeralopy). What vitamin should be recommended
in the first place?
A-B2
B-A
C-C
D-E
E-B6
Toxic affection of liver results in dysfunction of protein synthesis. It is usually accompanied by the following
kind of dysproteinemia:
A-Absolute hypoproteinemia
B-Relative hyperproteinemia
C-Relative hypoproteinemia
D-Paraproteinemia
E-Absolute hyperproteinemia
A patient with obliterating endarteritis underwent ganglionic sympathectomy. What type of arterial
hyperaemia should have developed as a result of the surgery?
A-Neuroparalytic
B-Metabolic
C-Neurotonic
D-Reactive
E-Functional
After the traumatic tooth extraction a patient is complaining of acute, dull, poorly-localized pain in gingiva,
body temperature rise up to 37,5oC. The patient has been diagnosed with alveolitis. Specify the kind of pain
in this patient:
A-Heterotopic
B-Protopathic
C-Visceral
D-Epicritic
E-Phantom
A 10 year old child had the mantoux tuberculin test administered. 48 hours later a papule up to 8 mm in
diameter appeared on the site of the injection. What type of hypersensitivity reaction developed after the
tuberculin injection?
A-Atopic reaction
B-Type IV hypersensitivity reaction
C-Type II hypersensitivity reaction
D-Seroreaction
E-Arthus phenomenon

A patient has been diagnosed with influenza. His condition became drastically worse after taking antipyretic
drugs. His consciousness is confused, AP is 80/50 mm Hg, Ps is 140/m, body temperature droped down to
35,8oC. What complication developed in this patient?
A-Hypovolemia
B-Acidosis
C-Alkalosis
D-Collapse
E-Hyperthermia
As a result of a trauma a patient has developed traumatic shock that led to the following disorders: AP is
140/90 mm Hg, Ps is 120 bpm. The patient is fussy, talkative, pale. Such state relates to the following shock
phase:
A-)B-Terminal

C-Latent period
D-Erectile
E-Torpid

1. Insufficiency of the mitral valve was founded at man who 12 years ago suffered from rheumatic
myocardites. Investigations have shown that the inflammatory process is absent, minute volume of circulation
is sufficient. Name the nition of general nosology.
A. Pathological state
B. Pathological reaction
C. Disease process
D. Typical pathological process
E. Compensating reaction
2. Atrophy of bone marked in the patient after the removal of teeth. This is an example of:
A. Pathological condition
B. Pathological reaction
C. Pathological process
D. Structural adaptation
E. Disease
1. A 50-year old man has a stomach ulcer. After the treatment digestion become to normal, pain dissapiared,
and his mood improved. However, in a couple of weeks the patient developed pain in epigastric region,
heartburn and acd belching. How this situation might be interpreted?
A. Disease relaps
B. Remission period
C. Terminal status
D. Premonitory period
E. Latent period
2. A 12-year-old boy after coming home from the school began to complaint of headache, nausea, chill,
periodic muscle pain, loss of appetite, and fatigue. What disease period can show such signs?
A. Prodromal
B. Latent
C. Incybation
D. Height of the disease
E. End of the disease
3. A student fell ill with acute respiratory disease at the end of winter following long period of mental
overloading. What is the cause of the disease?

A.
B.
C.
D.
E.

Pathogenic microorganism
Irrational diet
Mental overloading
Supercooling
Hypovitaminosis

4. Pain in aleft half of the chest and dyspnea appear in a patient during walking. This pain increases during
respiration. The attack of the pain was stopped by using narcotics. At examination of the patient following
data were revealed: severe condition, respiratory rate is 28 per min., respiration is shallow and is dull in the
left axillary region. Blood pressure is 140/80 mm Hg. Patient has sputum with blood. This patient has varicose
dilation of veins. In purpose to correct main pathogenic link it is necessary to use:
A. Anticoagulants
B. Antibiotics
C. Antihistamin preparation
D. Spasmolytics
E. Coronarilytics
5. A 39-years-old patient has been suffering from gastric ulcer for last 4 years. Pain in epigastric region,
heartburn, nausea, and constipation appear mainly in autumn and spring. Name this condition.
A. Remission
B. Acute period
C. Complication
D. Pathologic condition
E. Relapse
6. Gasping respiration appears in a patient with severe lung pathology. What terminal condition is this
characteristic for?
A. Agony
B. Pre-agony
C. Clinical death
D. Biological death
E. Terminal pause
7. A 10-yeaes-old child endured several rheumatic attacks. At examination of him it was established that he
had inflammatory process in his joints and signs of mitral valve insufficiency. What pathological appearance
in this patient may be attributed
to "disease"?
A. Mitral valve disease
B. Mitral valve insufficiency
C. Rheumatism
D. Inflammation of joints
E. Arthritis
8. Which scientist emphasized senescence of connective tissue cells cytoplasm?
A. Bogomolets
B. Mechnikov
C. Dilman
D. Frolkis
E. Berdichev
9. A 28-year-old patient with symptoms of acute appendicitis was admitted to the surgical department. Acute
pains in his right epigastric area and near the umbilicus were registered during palpation on examination.
Schetkin-Blumberg symptom was positive. Which period of disease was observed in that patient?
A. Period of manifestation
B. Latent period
C. Prodromal period
D. Final period
E. Period of functional disorder

10. Increase in pulse rate, respiratory rate, and increase of BP were noticed on the height of 1000 m above the
sea in a 25-year-old woman, coming for holydaj in the mountainous resort. In some time all those symptoms
disappeared. Which process was noticed in a woman?
A. Adaptation
B. Compensation
C. Decompensation
D. Stress
E. Parabiosis
11. A man with the complex of deviation of his health is considered sick. What is the more typically for disease?
A. Disorder of physiological regulation of functions
B. Decrease of ability to work
C. Disorder of immunity
D. Psychological deviations
E. Decrease of adaptation

1. A driver who got a trauma in a road accident and is shocked has reduction of daily urinary

output down to 300

ml. What is the main pathogenetic factor of such diuresis change?


A Drop of arterial pressure
B Drop of oncotic blood pressure
C Increased vascular permeability
D Decreased number of functioning glomerules
E Secondary hyperaldosteronism

2. A 26 year old man is in the torpid shock phase as a result of a car accident. In blood: 3,2*10 9/l. What is the
leading mechanism of leukopenia development?
A Redistribution of leukocytes in bloodstream
B Leikopoiesis inhibition
C Disturbed going out of mature leukocytes from the marrow into the blood
D Lysis of leukocytes in the blood-forming organs
E Intensified elimination of leukocytes from the organism

3. As a result of a trauma a patient has developed traumatic shock that led to the following disorders: AP is 140/90
mm Hg, Ps is 120 bpm. The patient is fussy, talkative, pale. Such state
A Erectile
B Latent period
C Terminal
D Torpid
E -

relates to the following shock phase:

4.

In result of the damage of one of the Atomic Power Plant reacttor the run-out of radioelements happened.
People in the increased radiation zone were radiated with approximately 250-300 r. They were immediately
hospitalized. What changes in the blood count would be typical?
A Lymphopenia
B Leukopenia
C Anemia
D Thrombopenia
E Neutropenia
*** *** ***

1. A driver who got a trauma in a road accident and is shocked has reduction of daily urinary

output down to 300

ml. What is the main pathogenetic factor of such diuresis change?


A Drop of arterial pressure
B Drop of oncotic blood pressure
C Increased vascular permeability
D Decreased number of functioning glomerules
E Secondary hyperaldosteronism

2. A 26 year old man is in the torpid shock phase as a result of a car accident. In blood: 3,2*10 9/l. What is the

leading mechanism of leukopenia development?


A Redistribution of leukocytes in bloodstream
B Leikopoiesis inhibition
C Disturbed going out of mature leukocytes from the marrow into the blood
D Lysis of leukocytes in the blood-forming organs
E Intensified elimination of leukocytes from the organism

3. As a result of a trauma a patient has developed traumatic shock that led to the following disorders: AP is 140/90
mm Hg, Ps is 120 bpm. The patient is fussy, talkative, pale. Such state
A Erectile
B Latent period
C Terminal
D Torpid
E -

relates to the following shock phase:

4.

In result of the damage of one of the Atomic Power Plant reacttor the run-out of radioelements happened.
People in the increased radiation zone were radiated with approximately 250-300 r. They were immediately
hospitalized. What changes in the blood count would be typical?
A Lymphopenia
B Leukopenia
C Anemia
D Thrombopenia
E Neutropenia

1. In result of the damage of one of the Atomic Power Plant reactor the run-out of radioelements happened.
People in the increased radiation zone were radiated with approximately 250-300 r. They were immediately
hospitalized. What changes in the blood count would be typical?
A Lymphopenia
B Leukopenia
C Anemia
D Thrombopenia
E Neutropenia
1. Liquidator at Chernobyl Atomic Electric Station underwent total irradiation. In what organ or tissues is
most possible development of tumor?
A. Hemopoetic tissue
B. Lungs
C. Stomach
D. Skin
E. Bone tissue
2. A patient suffering of osseo-cerebral form of radiation sickness. What sprouts of hematogenesis re injured
at this disease?
A. All.
B. Erythrocytic.
C. Megacaryocytic
D. Granulocytic.
E. Agranulocytic
3. A man, aged 35 years, in 15 min after a motor-car accident was diagnosed massive trauma of the lower
extremities without profuse external bleeding. The victim is in the excited state. What component of
pathogenesis of traumatic shock is a leading one and needs immediate correction?
A. Pain.
B. Sharp hepatic insufficiency.
C. Intoxication.
D. Cardiac function disorders.
E. Internal loss of plasma
4. As a the result of the damage of the reactors at AES running out of radio-active products took place.
People that were in the area of high radiation got about 250-300R. They were immediately hospitalized. What
main symptom will the victims have?

A.
B.
C.
D.
E.

Lymphopenia
Leucopenia
Anemia
Thrombocytopenia
Neutropenia

5. Accident at Chernobyl AES resulted in development of radiation sikness in the liquidators. Disorders
develops as the result of initial radio-chemical effect of ionizing radiation mainly on molecules of:
A. Water
B. Albumens
C. Lipids
D. Enzymes
E. Carbonhydrates
6. A patient immediatly after the irradiation (single dose is 80 Gr) instantly developed such symptoms as:
collapse, loss of consciousness, anuria, cramps and paralysis. What form of radiation sickness in this case?
A. Acute cerebral
B. Osseo-cranial
C. Toxemic
D. Gastrointestinal
E. Chronic radiation sickness
7. Formation of lipids and quinone radiotoxins is one of molecular mechanisms of indirect action of ionizing
irradiation. What is mechanism of radiotoxin effect on the cells?
A. Depression of synthesis of nucleic acids
B. Activating is POL
C. Stimulation of oxydation of pyrimidine
D. Depression of the activity of atioxydant system
E. Deamination of purine bases
8. As a result of violation of rules of accident prevention the worker of the radiological department of
regional hospital got a single radiation. In 8 days the ulcero-necrotic changes in the oral cavity were observed.
In the blood was: Er-3,2 x 1012/L, reticulocytes - 0,01% Hb-60g/L, L - 2,3x 10 9/L, thrombocytes- 50 G/L What
period of radiation sickness is characterizes by such changes?
A.
B.
C.
D.
E.

Period of clinical manifestations


Period of primary reactions
Latent period
Period of imagined safety
Period of outcomes

9. Which of nervous tissues features makes it high radio resistance?


A. Lack of mitotic activity in neurons
B. High content of lipids
C. Decrease of permeability of plasmatic membrane of neurons for ionizing radiation
D. Permanent high functional activity
E. Great intensity of oxygen consumption
10. Pathogenic action of ionizing radiation was studied during the experiment. Name the symptomocomplex
that dosnt develop at radiation of the organism by the median doses of ionizing radiation.
A Rough structural changes and death of nervous cells
B. Hemorrhagic syndrome
C. Intestinal autointoxication
D. Nervous-reflex disorders
E. Depression of hemopoesis
11. Victim during an accident on a submarine atomic boat got the dose of irradiation 6 Gr. It expected
essential disorders of functions and structure of cells:
A. Bone marrow
B. Intestinal epithelium
C. Epithelium of skin
D. Pulp of spleen
E. Thyroid gland

12. A driver was admitted to the resuscitation department. He does not reacts to the questions, is indifferent
to everything, pale, his breathing is superficial, infrequent. Arterial pressure -75/50 mm. Name the main link
of pathogenesis of abovementioned pathology.
A. Central nervous system inhibition
B. Excitation of CNS
C. Loss of blood
D. Toxemia
E. Redistribution of blood
13. How can one liquidate the main link that appears at traumatic shock?
A. By anesthesia
B. By excitation of vasomotor centre
C. By hypertensive drugs
D. By activating of blood sedimentation
E. By antioxidants
14. The worker of AES was admitted to the a clinic after a single radiation with the complaints of a headache,
loss of consciousness, high of temperature, weakness, vomiting, diarrhea. Blood analyses showed leukocytosis
with lymphopenia. What period of radiation sickness is the most possible in a patient?
A. Primary reactions
B. Prodromal
C. Lethal
D. Developed clinical picture
E. Period of imagined sufety
15. A patient that is in the clinic for acute radiation sickness complaints of a headache, insomnia, marked
weakness, difficulties of eating. The gums inflammation, necrotic angina, skin hemorrhagis, blood in urine
and fecus were founded in the physical examination. Damage of what tissue is the main in pathogenesis of
disorders in this case?
A. Hemopoietic
B. Bone tissue
C. Glandular epithelium
D. Nervous
E. Lymphoid
16. On the peak of acute radiation sickness a patient had acute weakness, his skin was pail, tachycardia, pain
in the different parts of the intestine, vomiting, fecus with the tinge of blood. Arterial pressure 70/40 mm
Hg. Two weeks before the patient got a radiation dose 30 Gr. What form of radiation sickness is characterized
by such symptoms?
A. Intestinal
B. Toxemic
C. Osseo-cranial
D. Hemorrhagic
E. Cerebral
17. White rat got the X-ray radiation in a dose 10 Gr. Give the most possible change of the peripheral blood
picture that is observed in 0 hours after the radiation.
A. Neutrophyic leukocytosis
B. Leukopenia, anemia
C. Anemia
D. Anemia, leukopenia, thrombocytopenia
E. Afranulocytosis
18. A rabbit was radiated with a dose 0,5 Gr. In what system are the most marked disorders observed in 10
hours after radiation?
A. Hemopoietic
B. Nervous
C. Cardio-vascular
D. Sexual
E. Lymphoid

19. During the work for liquidation of the consequences of AES liquidation of the consequences a worker got
radiation in a dose of 10 Gr. He complains of a headache, nausea, loss of consciousness. What changes of
leukocytes can one expect in a patient 10 hours after later after radiation?
A. Lymphocytosis
B. Leucopenia
C. Agranulocytosis
D. Neutrophilic leucocytosis
E. Leukemia
20. In a peak period of acute radiation sickness in a patient had leukopenia, thrombocytopenia, autoinfection,
autointoxication, bleeding, high temperature were observed. What form of radiation sickness is characteristic
of such picture?
A. Intestinal
B. Osseo-cerebral
C. Toxemic
D. Cerebral
E. Hmorragie
21. A 35-year-old man following 30 min after a car incident presented massive trauma of the low extremities
without significant external bleeding. The patient it agitated. What component of the traumatic shock is
prominent in this case and needs to be corrected immediately.
A. Pain
B. Internal haemorrhage
C. Internal loss of plasma
D. Intoxication
E. Disorder of the internal organs
22. At the height of the acute radiation sickness a patient had leukopenia, thrombocytopenia, autoinfection,
autointoxication, hemorrhagic diathesis, and fever. What kind of the radiation sickness might be
accompanied by these symptoms?
A. Bone marrow
B. Intestinal
C. Toxemic
D. Cerebral
E. Haemorrhagic
23. A primary pathogenic mechanisms of the radiation cell lesion is bound with:
A. Water radiolosos
B. Activation of the cell membrane phospholipase
C. Accumulation of the Ca++ in the cells
D. Increasing of the Na+ diffusioan in the cell
E. Intracellular acidosis
24. A man was brought to the hospital in 3 days after the influence of ionizing radiation in a dose # Gr. What
physiological system might be primary involved into the pathological process?
A. Blood
B. Digestive
C. Cardiovascular
D. Immune
E. Endocrine
25. A Chernobil liquidator complaints of fatigue, skin hemorrhages, and diarrhea. Blood test results:
erythrocytes sedimentation rate 25 mm/h. erythroxytes 2,4x10 12/l, leukocytes 2,2x109/l, platelits
70x109/l. what phase of radiation sickness takes place?
A. Height of the disease
B. Primary acute reactions
C. Imaginary clinical well-being
D. Restoration
E. Disease outcome
26. A diver that for a long time was on the depth 40 m at decompression developed in cassion disease. The
main of pathogenic link was embolism:
A. Gas

B.
C.
D.
E.

Air
Fatty
Paradoxal
Tissue

27. At the decompression of the cabin of airplane at the altitude of 19km the immediate death of the pilots
came. What is its cause?
A. Boiling of blood
B. Cerebral hemorrhages
C. Gas embolism of cardiac vessels
D. Bleeding
E. Paralises of respiratory center
28. The state Of passengers of the airplane with partial decompression became sharply worse. Pain in t he
ears, frontal sinus, meteorism, bleeding from the nose. What peripheral blood disorder caused this bad state?
A. Gas embolism
B. Ischemia
C. Venous hyperemia
D. Thromboembolism
E. Stasis
29. The patient with burns of 40% surface of body burn shock developed. What is the primary mechanism of
its development?
A. Pain
B. Dehydration
C. Disorder of mineral metabolism
D. Autoimmunization
E. Disorder of protein metabolism
30. After the forced quick getting a diver up to the surface from the depth he developed the signs of caisson
disease: pain in joints, itching of the skin, winkiig in the eyes, loss of consciousness. What type of embolism
caused them?
A Gas
B. Air
C. Fatty
D. Tissue
E. Thromboembolism
31. After immersion of a diver on a depth of 60 km he developed the symptoms of the disorder of the central
nervous system - excitement, euphoria, loss of attention, professional errors. These symptoms are associated
with toxic effect on neurons if:
A. Nitrogen
B. Oxygen
C. Carbon dioxide
D. Ammonia
E. Lactate
32. Decompression of a pilot's cabin happened on height of 14000 m. What tyje of metabolism did he develop?
A Gas embolism
B. Tissue
C. Thromboembolism
D. Air embolism
E. Fat
33. At the emergency getting up from the depth the diver developed cramps with the loss of consciousness.
What pathogenic mechanism is the main one in the development of this disorder?
A. Gas embolism
B. Hypoxia
C. Toxic effect of oxygen
D. Toxic effect of nitrogen
E. Toxic effect of C02.
34. At the development of mountain sickness subsequently develop below mentioned changes of indices of the
state of vital functions of the organism. What of them are of pathologic character?

A.
B.
C.
D.
E.

Increase of respiratory rate


Increase of breathing depth
Increase of pulse rate
Hypoxemia
Hypodynamia and apathy

35. For the first time the character of breathing of the alpinist who was at the altitude of 5000 m above a see
level changed during sleep: after some respiratory movements there begins respiratory standstill than deep
respiratory movements were restored again and so on. What is the most possible cause of changes of
respiration?
A. Decrease of air temperature
B. Decrease of partial pressure of C02 in air
C. Decrease of partial pressure of 02 in air
D. Increase of blood flow rate
E. Increase of oxygen blood volume
36. An alpinist during climbing the mountain at the altitude of 6 000 m above a sea level developed euphoria,
inadequate estimation of situation, hallucinations. What IS the main cause of the development of these
symptoms of mountain sickness?
A. Physical overstrain
B. Decrease of atmospheric pressure
C. Snow ophthalmia
D. Decrease of partial pressure of air oxygen
E. Expansion of air in frontal sinus
37. A man lived in high -mountain conditions for a long time. What change in the system of blood is
observed?
A. Increase of the diameter of the blood vessels
B. Decrease of the number of leucocytes
C. Decrease of the number of erythrocytes
D. Increase of the number of erythrocytes
E. Decrease of the number of thrombocytes
38. An alpinist was climbing the mountains for several days. At the altitude of 5 000 m tachypnoe,
tachycardia, pain in the ears, headache of expansive character developed. Indicate possible cause of these
symptoms.
A. Increase of partial pressure of air oxygen
B. Insufficient ventilation of lungs
C. Gas embolism
D. Decrease of air temperature
E. Decrease of barometric pressure
39. At the experimental study of the influence of low barometric pressure effect on white rats in barocamera
at the altitude that corresponds to 2 000 m above the sea level the animals developed tachypnoe, increase of
the number of erythrocytes and hemoglobin concentration. At what altitude above a sea level the
phenomenon of self boiling of blood is observed?
A. 14 000 m
B. 7 500 m
C. 8 500 m
D. 19 000 m
E. 10 000 m
40. At climbing the mountains in the group of students there were a tachypnoe, euphoria, tachycardia,
increase of erythrocytes and hemoglobin in blood. What itate from the side of blood accompanies the above
described symptoms?
A. True erythremia
B. Erythrocytisis
C. Megaloblastic anaemia
D. Leucopenia
E. Thrombocytopenia
41. The member of high -mountain expedition at the height of 6 km devebped dizziness, weakness. He lost his
consciousness. This was respiration standstill. These disorders developed due to:
A. Insufficient content of 02 in the organism

B.
C.
D.
E.

Excessive removing of C02 from the organism


Insufficient formation of C02 in tissues
Insufficient utilization of 02 by tissues
Insufficient liberation of 02 by hemoglobin

42. Acute general disorders are observed at long-term action of environmental high temperature on the
organism. What is the base of this phenomenon?
A. Cardiac insufficiency
B. Increase of body temperature
C. Dehydration
D. Intoxication
E. Hypoxia
4 5 . On admitting to the hospital the man with the closed craniocerebral trauma had a headache, nausea,
vomiting, high tC. Pathology from the side of inner organs was not r e v e aled. What form of thermoregulation
disorder is observed in a patient?
A. Exogenic hyperthermia
B. Fever
C. Endogenic hyperthermia
D. Centrogenic hyperthermia
E. All answers are correct
46. A man that had suffered from the explosion in the mine was admitted to the reception department of the
hospital. His general state was satisfactory. He was conscious. There were areas of hyperemia of skin with
blisters on the face, hands and trunk. What factor mainly had the effect on the miner?
A. Chemical
B. Thermal
C.
Airwave
D.
Psychological
E. Toxic
47. A diver after submergence in depth of 60 m presented symptoms of central nervous system dysfunction excitement, euphoria, decreasing of attention, professional mistakes. These symptoms take place due to
influence on neurons of
A. Nitrogen
B. Oxygen
C. Carbonic acid
D. Ammonia
E. Lactate
48. At what directions of passing of the electric current through the organism there is the greatest threat to
the life of the patient?
A. Back of the head -lower jaw.
B. Left leg -right leg.
C. Right leg - right hand.
D. Left leg-left hand.
E. back-abdomen
49. During a rain storm a ragged electric wire fell down on the head of the victim whereupon he perished.
Death occurred as a result of:
A. Paralysis of respiratory center
B. Disorders of cerebral blood circulation
C. Oppression of the cortex
D. Paralysis of the motor -vascular center
E. Irritation of nuclei of vagus nerve
50. An electrician working with the violation of rules of accident preventioi by chance touched bare electric
wire with both hands and perished. Death was at the result of:
A. Fibrillation of auricles and ventricles
B. Complete atrio-ventricular block
C. Oppression of automatism of sinoarterial node
D. Decrease of myocardial contractive ability
E. Disorder of the vagus cardiac regulation

51. Electric current with the voltage of 1000 V was passed through the brain of an experimental animal. The
animal perished. The skin was cyanotic. Determine the course of death:
A. Respiration standstill
B. Cardiac arrest
C. Collapse
D. Psychogenic shock
E. Cardiogenic shock
52. As a the result of a contact with the bare electric wire a small damage of grayish color by sizes appeared 2
x 1,5 s-m appeared on the skin. It was separated Irom healthy tissues by the demarcation line. There was no
exudate and erythema. Define the described damage:
A. Electric burn of II degree
B. Electric burn of a I degree
C. Inflammation
D. Necrosis
E. All answers are correct
53. In 2 weeks after electric trauma necrosis of the skin and surrounding tissues developed in the peace of a
contact with the electric wire. Define the leading factor of the necrosis development in this case:
A. Discharge of a greatest amount of heat
B. Coagulation of proteins
C. Polarization of cell membranes
D. Damage of blood vessels
E. Injury of the skin and muscles with the products of POL
54. A direct electric current passed through the organism of a man that was in the state of easy alcoholic
intoxication (I = 20 ma, U = 220 V, t = 0,03 sec) at the position of electrodes head - leg whereupon the
paralysis of respiratory center was observed. What factor contribute to the above mentioned complication?
A CNS disorder
B. The pass way of electric current
C. State of alcohol intoxication
D. Types of electric current
E. The time of current action
55. A man who ignored the rules of accident prevention got an electric trauma with electric current passing
through the cardiac muscle. What dangerous disorder in the work of the heart requires immediate
interference may develop in this situation?
A Tachycardia
B. Extrasystolia
C. Fibrillation
D. Arrhythmia
E. Bradycardia
56. The doctor of the first-aid station determined respiratory standstill with the signs of the work of the heart
in the victim from the influence of electric current. What was the most possible pass way of the electric
current passing in this case?
A. A right hand - head
B. A left leg - right hand
C. A right leg - right hand
D. A left leg - right hand
E. Abdomen - right leg
57. A man had an electric trauma with the electric current passing through ihmuscle of the heat. What
dangerous disorders in the work of the heart that require immediate interference may develop in this
situation?
A. Bradycardia
B. Extrasystolia
C. Fibrillation of die auricle
D. Fibrillation of ventricles
E. Tachycardia
58. An electrician, working with violation of safety measures, touched an electrical cable and died. He died
due to

A. Atrial and ventricular fibrillation


B. Complete atrioventricular heart block
C. Depression of the sinoatrial automatism
D. Decreasing of contractive capacity of the heart muscle
E. Disorder of the vagal heart regulation
59. As a result of a trauma a patient has developed traumatic shock that led to the following disorders: AP is
140/90 mm Hg, Ps is 120 bpm. The patient is fussy, talkative, psie. Such state relates to the following shock
phase:
A. Terminal
B. Latent period
C. Erectile
D. Torpid
E.-Desorder of the vagal heart regulation
*** ****
1. A woman who was infected with toxoplasmosis during the pregnancy has a child with multiple congenital
defects.This is a result of:
A Teratogenesis
B Cancerogenesis
C Biological mutogenesis
D Chemical mutogenesis
E Recombination
1. A galvanizer addressed to a doctor because of appearing of pink itching spots and vesicles on his arms skin.
He worked a lot with nickel compounds recently. What is themechanism of these lesions development?
A. Modification of skin proteins due to action of nickel ions with immunopathological reaction
development
B. Irritant action of nickel compounds on skin
C. Vegetative disorders related to nickel compounds received by respiratory tract
D. Infection of skin due to microtraumas
E. Nickel is a complete antigen
2. What is the reason for pneumonia development in children after cooling?
A. Microorganisms
B. General cooling
C. Decrease in reactivity due to previous diseases
D. Insufficiency of nutrition
E. All these factors
3. A patient very often has diseases of nasopharynx, which appear under the influence of different factors and
in the most cases manifest by inflammation. Which of listed factors is the most probable reason for these
diseases?
A. Microorganisms
B. General cooling of organism
C. Overstrain
D. Immunodeficiency
E. Insufficiency of nutrition

*** ***
1. A child was born with cleft palate. Examination revealed aorta defects and reduced number of T-lymphocytes in
blood. What immunodeficient syndrome is it?
A DiGeorge
B Wiskott-Aldrich
C Chediak-Higashi
D Louis-Bar
E Swiss-type
2. Examination of a child who frequently suffers from infectious diseases revealed that IgG

concentration

in

blood serum was 10 times less than normal, IgA and IgM concentration was also significantly reduced. Analysis
showed also lack of B-lymphocytes and plasmocytes. What disease are these symptoms typical for?
A Bruton's disease
B Swiss-type agammaglobulinemia
C Dysimmunoglobulinemia
D Louis-Bar syndrome
E Di George syndrome
3. A child is pale, pastose, muscular tissue is bad developed, lymph nodes are enlarged. He often suffers from angina
and pharyngitis, blood has signs of lymphocytosis. The child is also predisposed to autoallergic diseases. What
type of diathesis can be presumed in this case?
A Lymphohypoplastic
B Exudative
C Gouty
D Asthenic
E Hemorrhagic
1. Macroph ages surrounded wit h al i en er yt hro c yt es w ere found at m i croscopi c ex ami nat i on of
sm e ar of ex udat i on t ook from Wat sufferi n g from asept i c peri t oni t is wi t h addi t i on of bi rd
er yt hro c yt es. What stage of pha go c yt osi s does t hi s oc curr enc e cor respond to?
A. Incomplete phagocytosis
B. Adhesi on
C. Hemotaxis
D. Engulfment
E. Intracellular digestion
2. Aft er 4 t h subcut aneous i nt rodu ct i on of horse serum i n rabbi t s sharp infl am m at i on t yp e
Art hus phenom enon devel oped. Whi ch t yp e of al t er ed re act i vi t y conc erns such pat hol og y?
A. H yp er ergi a
B. Positive hypoergia
C. Negative hypoergia
D. Disergia
E. Anergia
3. A 10-year-ol d gi rl suf f ers f rom vi ral and my coti c d i seases very often. S h e h as con gen i tal
h eart d iseas e an d i nsu ff i ci en cy of th yroi d gl an d . T-l ymp h ocy tes i s ab sen t from h er
i mmu n ol ogi cal test Wh at di sord e r of i mmu n e syste m tak es p l ace in th i s case?
A. Bruton's hypogammaglobulinemia
B. Mixed immunodeficiency
C. Terner's syndrome
D. H ypopl asi a of t h ym us
E. Inherited defect of complement system
4 De crease i n l evel s of IgG an d p arti cu l arl y IgA, IgM w as f ou nd du ri n g exa mi n ati on of
i mmu n e statu s in 5-yea r-ol d b oy. B -l ymp h ocyf es_an d pl asma ce jtea re absen t from hi s
bl ood an d l ymp h n od es. Reacti on s of T-ly mp h ocytes are normal. Th i s is i nh e ri ted sex-l in k ed
di sease. Wh at di agn osi s i s mo re p ossi b l e?
A. B rut on's di sease
B. Luis-Barr syndrome\
C. Wiskott-Aldrich syndrome
D. Swiss type of immunodeficiency
E. Early hypogammaglobulinemia
1. A ch i ld had recu r ren t can di d osi s ( my coti c sto mati ti s). In ju ry has a sev ere cou rse th at
is d u e to h erp es vi ru s, co mp l i cati on s at th e vacci n ati on of B CG an d vacci n e agai n st a
pox. Lymp h ocy top eni a i s i n bl ood . Th e nu mb ers of oth er l eu k ocytes i s w i th i n th e n orm.
Immu n ogl obu l in s are a li ttl e low er b el ow th e n or m. Rea cti on of H DT is ab sen t Th e
d ecrease of fu n cti on i n g of wh at c el l s is th e base of th e synd rome?
A. T-l ym po c yt es
B. Neutrophils
C. Macrophage
D. B- lympocytes
E. Endotheliocytes

2. A b oy, aged 1,5, is il l wi th ch ron i c pyod er mi a and th ree ti mes was il l w i th pn eu mon i a.
Th e n u mb e r of i mmu n ogl obu l in s G an d A i s d ecreased i n bl ood . Wh at typ e of
i mmu n od efi ci en cy has a ch i ld ?
A. Brut on 's h ypo gam m a gl obul i nem i a
B. Swiss type
C. Hypoplasia of gland
D.Wiskott-Oldrich syndrome
E. Luis-Barr syndrome
3. Th e d i agn osi s of th y mu s h yp erp l asi a was mad e i n a chi l d of tw o years ol d . Wh at
in di c es of th e state of th e i mmu n e syste m i s th e most ch aract eri sti c for thi s
i mmu n od efi ci en cy?
A. Dec re ase of T-l ym pho c yt es
B. Decrease of B- lymphocytes
C. Deficiency of T and B - lymphocytes
D. Absence of plasmatic cells
E . Decrease of immunoglobulins M
4. A girl w as b orn i n t er m. Up to 1 yea r ol d was b reast fed . At th e en d of th e fi rst year of l if e
sh e suf f ered f rom li n geri n g p n eu mon i a. A gi rl lat el y b egan to w al k ; th e gai t w as
un stab l e, w i th th e di sord er of move men t coord i n ati on . Th ere w as teleangiectasia on th e sk in
an d con ju n cti va. De creases of T-cel l l ev el (32%), ab sen c e of Ig; were rev eal ed at exa mi n ati on .
Wh at syn d rome d evel op ed in a ch i ld ?
A. Lui s- Ba rr s ynd rom e
B. Di George syndrome
C. Bmlon's disease
D. Chcdiak-Higashi syndrome
E . I mmunodeficiency of Swiss type
5. Decrease of IgG an d esp eci al l y IgA an d IgM w as reveal ed i n a boy 5 mon th s aged the
exami n ati on of i mmu n e statu s B -ly mp h ocytes an d p las mati c cel l s w ere absen t in Mood
an d l ymp h ati c nod es. Rea cti on s of T-cel l are p reserv ed . Th e di sease is Inhered as in ocu l ated
wi th sex. Wh at di agn osis i s th e most p ossi bl e?
A. Bruton's di se ase
B. Luis-Barr syndrome
C. Wiskott-Aldrich syndrome
D. Swiss type of immune deficiency
E . Early hypogammaglobulinemia
6. Sh arp d ecrease of n u mb er of B - l y mp h ocytes an d i mmu n ogl obu l in s M an d G i s observed
in a b oy aged 2 w i th ch ron i c pn eu mon i a. Wh at mi c roorgan i sms can cau se development of th i s
di sease?
A. St aph yl oco ccus
B. Adenovirus
C. Herpesvirus
D. Mycobacterium of tuberculosis
E . Candidas
7. Di sord er of cel l i mmu n i ty syste m was rev eal ed at th e exami n ati on of pati en t i mmu n e
syste m stat e wi th th e ch ron i c cand i d osi s of sk in . De crease of wh at i nd i ces i s the mos t
ch aract eri sti c of th i s case?
A. T-l ym pho c yt es
B. Immunoglobulins G
C. Immunoglobulins E
D. B-lymphocytes
E . Plasmocytes
8. Th e d ecrease of i mmu n e syste m acti vi ty is obse rved in AIDS -i nf ec ted pati en t. In ju ry of
wh at cel l s i s th e most resp on si b l e f or th e sta te of i mmu n od efi ci en cy in th i s pati en t?
A. T-hel pers
B. T-suppressors

C. macrophages
D. B-lymphocytes
E. Neutrophils
9 . Vomi tin g, d i arrh ea wi th steator rh ea, h ep atosp l en omegal i a were ob serv ed in a ch i ld i n
th e f i rst w eek s af te r b i rth . It i s di agn osed th e in h eri t ed au toso mal - recessi on si ckn ess of
Vol man ' s Wh at w as th e cause of th i s state?
A. Absenc e of aci d l i pase
B. Absence of glycocerebrolidase
C. Absence of sphingomielinase
D. Deficiency of glucose-6 -phosphate
E. Deficiency of amilo-1,6 glucosidase
2
Pn eu mon i as an d p u ru l en t in ju ri es of th e sk i n are of ten in a b oy aged 5, as w ell as
in hi s gran d -d ad . Th e ab sen ce of B -l y mp h ocytes i s reveal ed at i mmu n ol ogi cal
exami n ati on . Wh at i mmu n e syste m d isord e r i s obse rved i n thi s case?
A. H yp ogam m a gl obul i nem i a of B rut on
B. Hypoplasia of thymus
C. Combined immunodeficiency
D. Syndrome of Shereshevskii-Turner
E. Inherited deficiency of the complement system
11. Move men t coo rdi n ati on di sord er, in ju ry of smal l vessel s of sk i n (tel ean gi e ctasi a),
d ecreased of i mmu n ol ogi cal reacti vi ty are ob serv ed i n a ch il d aged 10 th at was ad mi tted
to th e cl i n i c in a seve re stat e. Wh at typ e of i mmu n od efi ci en cy may th i s stat e is associ ated
wi th ?
A. Lui s- Ba rr s ynd rom e
B. Di George syndrome
C. Agammaglobulinemia of Bruton's
D. Disorders in B-lymphocytes system
E. Wiskott-Aldrich syndrome
12. Th y mu s h yp e rp l asi a was reveal ed i n a n ew -b orn ch i ld wi th cra mp synd rome and
d ef ec t of in te r-ven tri cu l ar sep tu m of th e h eart at th e X- rays exa mi n ati on of th e ch est
Wh at i mmu n od ef i ci en cy may b e su p p osed in th is p ati en t?
A. Di Geo rge s ynd rom e
B. Bruton syndrome
C. Wiskott-Aldrich syndrome
D. Syndrome of Guda
E. Luis-Barr syndrome
13. Mi c e w i th ou t h ai rs an d eel ! rea cti on s of sl ow typ e w ere b rou gh t to th e l ab orator y. I 01
thi s p ath ol ogy th e mo st p ossib l e i s?
A. Absenc e t h ym us gl and
B. Absence of gamma globulins in blood
C. Disorder of hemopoiesis
D. Disorder of phagocytosis
E . Insufficiency of complement factors
14. At i mmu n od ef i ci en cy acco rd in g th e syst em of B -ly mp h ocytes d i seases d evel op . The
mai n rol e i n p ath ogen esi s of w i tch b el on gs to:
A. Di sorder of s ynt h esi s of ant i bodi es
B. The disorder of immunological reactions of cellular type
C. To the loss of ability to rejection of transplant
D. To the decrease of slow hypersensitivity
E. To the decrease of antitumor immunity
15. Viral an d bac teri al i nf ec ti ons, e cz ematou s i n ju ri es of th e sk in of ten d evel op in a b oy,
aged 12. Dec reas e of T~l y mp h ocytes an d IgM at n or mal con t en t of IgE and IgG w ere
rev eal ed at th e exa mi n ati on . Wh at typ e of p ath ol ogy of th e i mmu n e syste m i s ob serv ed i n
a boy?
A. Com bi ned i m m unodefi ci en c y
B. Hypoplasia of thymus
C. Hypogammaglobulinemia of Bruton
D. Syndrome of Shereshevskii-Turner

E. Inherited deficiency of the complement system


16. T h ere are of t en fu ngi an d vi ral di seases in a gi rl aged 20 wh o su ff er in tes tin al
pol ip osi s. Th e l ack of wh at l in k of th e i mmu n e syst em i s th e most possi bl e in th i s case?
A. T-I ym p hoc yt es
B. B-lymphocytes
C. Natural killers
D. Complements
E. Phagocytes
17. Viral an d fu ngi di seases are of ten in a gi rl aged 10 w i th h ered i tary d ef ects of th e
h eart an d th yroi d . Ab sen ce of T-cel l i s rev eal ed at th e i mmu n ol ogi cal an al ysi s. Wh at
di sord er of th e i mmu n e syst em i s ob served i n thi s cas e?
A. H yp opl asi a of t h ym us
B. Hypogammaglobulinemia of Bruton
C. Combined immunodeficiency
D. Syndrome of Shereshevskii-Turner
E. Inherited deficiency of the complement system
18.
B ru ton ' s d iseas e i s di agn osed in a ch il d aged 5. It con si st of sev ere cou rse of
bac teri al in f ecti on s, ab sen ce of B - ly mp h ocyt es an d p las mati c cel l s. Wh at ch an ges of
i mmu n ogl obu l in con ten t wi ll b e ob serv ed in b lood seru m in th is ch i l d ?
A. Decrease number of IgD, IgE
B. Increase number of IgD, IgE
C. De cr ease num ber of IgA , IgM
D. Increase number of IgA, IgM
E. Without change
19.
Th e nu mb er an d f un cti on al T an d B - ly mp h ocytes ac ti vi ty i s not ch an ged i n a
pati en t wi th cl in i cal si gn s of i mmu n od ef i ci en cy. Th e d ef ect of fu n cti on of an ti gen
p resen ti n g of i mmu n ocomp et en t cel l s tak es pl ace. Def ect of w hat c el l s is most possi bl e?
A. Fibroblasts, B- lymphocytes, T- lymphocytes
B. B- lymphocytes, T- lymphocytes
C. Macroph ages, monoc yt e s
D. NK-cells
E. 0- lymphocytes
20.

Wh at is synd rome of Di - G eorg e?


A. Immunodepression after the spleen removal
B. Insufficiency of the IgM
C. Insufficiency of the plasmatic cells
D. H yp opl asi a of t h ym us gl and
E. Inherited insufficiency of C4

21.
Th e p ati en t w as mad e tran spl an tati on of d on ors h eart. Wh at cond i ti on s mu st on e
fol l ow to p rev en t regec ti on of organ tran sp l an tat e?
A. Transplantation of bone marrow
B. Transfusion of donors blood
C. Choosi n g t he donor aft er t he ant i gens of H LA
D. removal of spleen
E. Administration of immunomodulators
22.
A boy aged 3 years i s il l wi th ch ron i c p n eu mon i a. H e h as th e l ow in di c es of B l ymp h ocyt es syste m. Th e di agn osi s i s hyp oga mmagl obu l i n emi a of B ru ton . In d i cate th e
comp l i cati on of B -l ymp h ocyt es syste m i nsuf fi ci en c y.
A. Decreased resistance of organism to the viruses
B. Decreased resistance of organism to the mycotic and tubercular infection
C . Dec re ased r esi st an ce of organi sm t o t he p yro ge ni c co cci fl or a
D. Absence of reaction of transplant regection
E. Increased of risk of development of tumors in an organism
23. A child with i nh e ri ted h eat d ef ect f ace d ef ect ab sen ce of th yroi d and th ymu s inland and
bl ood T-ly mp h ocyt es was ad mi tted to th e ch i ld ren hosp i tal . Wh at inherited p ath ol ogy is i t
associ ated w ith ?

A Luis-Barr syndrome
B. Syndrom of Terner
C. Bruton's disease
D. Down's disease
E. Di George s yndrom e

24. An HIV-p osi ti ve man exp eri en c ed i mmu n od efi ci en c y. Wh at typ e of i mmu n e cel l s i s
involved in th e i mmu n od ef i ci en cy d evel op i n g?
A. T-hel pers
B. T-suppressors
C. Macrophages
D. B-lymphocytes
E. T-killers
25 . In th e con di ti on s of ge tti n g u p in mou n tai n s a man h ad th e tach ypn oe, tachycardia,
eup h ori a. Ch an ges th at are ob served are th e resu l t of rea cti vi ty:
A. Specific
B. Nonspe ci fi c
C. Group
D. Disergic
E. Hyperergic
26. T h e p ati en t wi th rad i cu l i ti s w as mad e a n ovocain e b l ock ad e af ter w hi ch h e grew dark in
eyes, hi s arte ri al p ressu re d ecreased to 90/50 mmH g, b reath in g b eca me more frequent u p to 24
p er mi n u te. Th e stat e of p ati en t i s th e resu l t of reac ti vi ty:
A. Physiology
B. Hypoergic
C. Normoergic
D. Nonspecific
E. H yp er ergi c
27. Af te r th e trau ma of on e eye p ati en t th e si gh t of th e se cond eye b egan grad u all y to
rai l . Wh at me ch an i sm can on e exp l ai n s thi s p h en omen on ?
A. By the decrease of organism resistance
B. By an infection
C . B y t he dam a ge of hi st ohem at i c barri e r
D. By immunodeficiency
E. By immune depression
28. It i s kn ow n th at at so me p ath ol ogi cal p roc esses reac ti vi ty of organ i sm ch an ges
op p osi tel y to th e ch an ge of i ts resi stan ce. At what path ol ogi cal p rocess es or di seases su ch
si tu ati on i s possib l e?
A. Shock
B. Fever
C. Inflammation
D. Posthaemorragic anemia
E. Arterial hypertension
29. Non sp eci fi c p rot ecti v e facto rs of th e oral cavi ty f rom p en et rati n g of p ath ogeni c
mi croo rgan is ms p lay an i mp ortan t rol e i n th e gen eral syste m of ph ysi ol ogy resi stan ce of
th e organi s m. Whi ch of ab ove men ti on ed co mp on en ts in th e oral cavi ty i s th e most
i mp or tan t f actor of n on sp eci fi c p rote cti on ?
A. Complement
B. Phagocytosis
C. B-lysin
D. Properdine
E. Lyso ci m e
30. Wh at is th e basi c d if f eren c e of p ath ol ogi cal rea cti vi ty f rom p h ysi ol ogi cal on e?
A. Participation of immunological mechanisms

B.
C.
D.
E.

Change of permeability of histohaematic barriers


Change of nervous regulation
Change of the endocrine regulation
R est ri ct i on of ad apt at i on possi bi l it i es

31. Wh at is co rrel ati on b etw een reac ti vi ty and resi stan ce du ri n g hi b ern ati on ?
A. Decreased reactivity and resistance
B. Decreased resistance and increased reactivity
C. De cr eased re act i vit y and i ncre ased resi st anc e
D. Increased reactivity and resistance
E. Reactivity and resistance does not change
32. T h e exa mp l e of non sp e cif i c reac ti vi ty i s:
A. Gen er al adapt at i on s yn drom e;
B. Transplantation immunity;
C. Allergic reactions of immediate type;
D. Allergic reactions of slow type;
E. Immunodeficiency state;
33 . Q ui t e of ten th e cau se
of se cond ary i mmu n od ef i ci en cy is an in f ecti on in vol vemen t,
wh en th e cau sati ve agen ts p rop agat e d i rec tl y in th e cel l s of i mmu n e syste m an d d estroy i t
th e f ol l ow i n g di seases are ch ara ct eri zed by:
A. In fe ct i ous m ononucl eosi s, AIDS
B. Poliomyelitis, type A hepatitis
C. Q-febris. epidemic typhus
D. Dysentery, cholera
E.T uberculosis, mycobacteriosis

***************
1. Mac roph ages su r roun d ed wi th ali en e ryth rocytes were f ou n d at mi cros cop i c
exami n ati on of smear of exu d ati on took f rom Wat suf f eri n g f rom asep ti c p eri ton i ti s w i th
ad d i ti on of bi rd ery th rocy tes. Wh at stage of p hagocy tosi s does thi s oc cu rren ce corresp on d
to?
A.
Incomplete phagocytosis
B.
Adh esi on
C.
Hemotaxis
D.
Engulfment
E.
Intracellular digestion
2.
Af t er 4 th su b cu tan eou s in trod u cti on of h orse seru m i n rabb i ts sh arp
in fl a mma ti on typ e Ar thu s p h en o men on d evel op ed . Whi ch typ e of al te red reacti vi ty
con ce rn s su ch p ath ol ogy?
A. H yp e rergi a

B. Positive hypoergia
C. Negative hypoergia
D.
E.

Disergia
Anergia

3. A 10-year-ol d gi rl suf f ers f rom vi ral an d my coti c di seases very often. S h e has con gen i tal
h eart di sease an d in suf fi ci en cy of th yroi d gl an d . T-ly mp h ocytes i s ab sen t from h er
i mmu n ol ogi cal test Wh at di sord e r of i mmu n e syste m tak es p l ace in th i s case?
A.
Bruton's hypogammaglobulinemia
B.
Mixed immunodeficiency
C.
Terner's syndrome
D.
H yp opl asi a of th y mu s
E.
Inherited defect of complement system
4 Dec reas e i n l evel s of IgG an d p arti cu l arl y IgA, IgM w as foun d du ri n g exami n ati on of
i mmu n e statu s in 5-yea r-ol d b oy. B -l ymp h ocyf es_an d pl asma ce jtea re absen t from hi s
bl ood an d l ymp h n od es. Reacti on s of T-ly mp h ocytes are normal. Th i s is i nh e ri ted sex-l in k ed

di sease. Wh at di agn osi s i s mo re p ossi b l e?


A.
B ru ton ' s d iseas e
B.
Luis-Barr syndrome\
C.
Wiskott-Aldrich syndrome
D.
Swiss type of immunodeficiency
E.
Early hypogammaglobulinemia
1.
A chi l d had recu rren t can di d osi s (mycoti c sto mati ti s). In ju ry has a sev ere cou rse
th at i s du e to h erp es vi ru s, comp l i ca ti on s at th e vacci n ati on of B CG an d vacci n e agai n st
a pox. Lymp h ocytop en i a i s in bl ood . Th e nu mb ers of oth er l eu k ocyt es i s wi th in th e n or m.
Immu n ogl obu l in s are a l i ttl e low er b el ow th e n orm. Rea cti on of H DT i s ab sen t Th e
d ecrease of fu n cti on i n g of wh at c el l s is th e base of th e synd rome?
T-ly mp ocyt es
Neutrophils
Macrophage
B- lympocytes
Endotheliocytes
2.
A boy, aged 1,5, is i l l wi th ch roni c pyod er mi a an d th ree ti mes was i ll w i th
pn eu mon i a. Th e n u mb e r of i mmu n ogl ob u l in s G an d A i s d ecreased i n bl ood . Wh at typ e of
i mmu n od efi ci en cy has a ch i ld ?
B ru ton ' s h yp oga mmagl ob ul i n emi a
Swiss type
Hypoplasia of gland
Wiskott-Oldrich syndrome
Luis-Barr syndrome
3.
Th e d i agn osi s of th y mu s h yp erp l asi a was mad e i n a chi l d of tw o years ol d . Wh at
in di c es of th e state of th e i mmu n e syste m i s th e most ch aract eri sti c for thi s
i mmu n od efi ci en cy?
Dec reas e of T-ly mp h ocytes
Decrease of B- lymphocytes
Deficiency of T and B - lymphocytes
Absence of plasmatic cells
Decrease of immunoglobulins M
4.
A girl was b orn in te rm. Up to 1 yea r ol d was b reast fed . At th e en d of th e fi rst year
of l if e sh e suf f ered from li n geri n g p n eu mon i a. A gi rl l atel y b egan to w al k ; th e gai t was
un stab l e, wi th th e d isord e r of mo ve men t coord i n ati on . Th ere w as teleangiectasia on th e sk in
an d con jun c ti va. De creases of T-cel l l evel (32%), ab sen c e of Ig; were rev eal ed at
exami n ati on . Wh at synd rome d evel op ed i n a ch il d ?
Lu i s-B arr syn d rome
Di George syndrome
Bmlon's disease
Chcdiak-Higashi syndrome
I mmunodeficiency of Swiss type
5.
Decrease of IgG and esp e ci all y IgA and IgM was rev eal ed in a b oy 5 mon th s aged the
exami n ati on of i mmu n e statu s B -ly mp h ocytes an d p las mati c cel l s w ere absen t in Mood
an d l ymp h ati c nod es. Rea cti on s of T-cel l are p reserv ed . Th e di sease is Inhered as
in ocu l ated w i th sex. Wh at d i agn osi s i s th e mo st p ossib l e?
Bruton' s di sease
Luis-Barr syndrome
Wiskott-Aldrich syndrome
Swiss type of immune deficiency
Early hypogammaglobulinemia
6. Sh arp d ecrease of n u mb er of B - l y mp h ocytes an d i mmu n ogl obu l in s M an d G i s observed
in a b oy aged 2 wi th ch ron i c p n eu mon i a. Wh at mi croo rgan is ms can cau se development of thi s
di sease?
S tap h yl ococcu s
Adenovirus

Herpesvirus
Mycobacterium of tuberculosis
Candidas
7. Di sord er of cel l i mmu n i ty syste m was rev eal ed at th e exami n ati on of pati en t i mmu n e
syste m state wi th th e ch ron i c can d id osi s of ski n . De crease of wh at in d i ces i s the mos t
ch aract eri sti c of th i s case?
T-ly mp h ocytes
Immunoglobulins G
Immunoglobulins E
B-lymphocytes
Plasmocytes
8.
Th e d ec reas e of i mmu n e syste m acti vi ty is ob serv ed in AI DS -in f ect ed pati en t.
In ju ry of wh at cel l s i s th e most resp on sib l e f or th e state of i mmu n od efi ci en cy i n th i s
pati en t?
T-h el p ers
T-suppressors
macrophages
B-lymphocytes
Neutrophils
9.
Vomi tin g, d i arrh ea wi th steator rh ea, h ep atosp l en omegal i a were ob serv ed in a
ch il d in th e fi rst w eek s af t er bi rth . It is d i agn osed th e i nh e ri ted au tosomal -rec essi on
si ck n ess of Vol man ' s Wh at w as th e cau se of th i s state?
Ab sen ce of acid l i p ase
Absence of glycocerebrolidase
Absence of sphingomielinase
Deficiency of glucose-6 -phosphate
Deficiency of amilo-1,6 glucosidase
Pn eu mon i as and p u ru l en t i n ju ri es of th e sk i n are of ten i n a boy aged 5, as w ell as i n hi s
grand - dad . Th e ab sen c e of B -l y mp h ocytes is reveal ed at i mmu n ol ogi cal exami n ati on .
Wh at i mmu n e syst em d i sord er is ob served in th is cas e?
H yp oga mmagl ob ul i n emi a of B ru ton
Hypoplasia of thymus
Combined immunodeficiency
Syndrome of Shereshevskii-Turner
Inherited deficiency of the complement system
11.
Move men t coo rdi n ati on di sord er, in ju ry of smal l vessel s of sk i n (tel ean gi e ctasi a),
d ecreased of i mmu n ol ogi cal reac ti vi ty are ob served i n a ch il d aged 10 th at w as ad mi tted
to th e cl i n i c i n a sev ere stat e. Wh at typ e of i mmu n od efi ci en cy ma y th i s state is associ at ed
wi th ?
Lu i s-B arr syn d rome
Di George syndrome
Agammaglobulinemia of Bruton's
Disorders in B-lymphocytes system
Wiskott-Aldrich syndrome
12.
Th y mu s hyp erp l asi a was rev eal ed i n a n ew -b orn chi l d wi th cra mp synd rome an d
d ef ec t of in te r-ven tri cu l ar sep tu m of th e h eart at th e X-rays exa mi n ati on of th e ch est
Wh at i mmu n od ef i ci en cy may b e su p p osed in th is p ati en t?
Di G eorg e synd rome
Bruton syndrome
Wiskott-Aldrich syndrome
Syndrome of Guda
Luis-Barr syndrome
13.
Mi ce wi th ou t hai rs and e el ! reac ti on s of sl ow typ e w ere b rou gh t to th e l ab ora tory.
I 01 thi s p ath ol ogy th e mo st p ossib l e i s?
Ab sen ce th y mu s gl an d
Absence of gamma globulins in blood
Disorder of hemopoiesis

Disorder of phagocytosis
Insufficiency of complement factors
14.
At i mmu n od ef i ci en cy ac cord i n g th e syste m of B -l ymp h ocy tes d i seases d evel op . The
mai n rol e i n p ath ogen esi s of w i tch b el on gs to:
Di sord er of syn th esi s of an ti b odi es
The disorder of immunological reactions of cellular type
To the loss of ability to rejection of transplant
To the decrease of slow hypersensitivity
To the decrease of antitumor immunity
15.
Viral and b acte ri al in f ecti on s, ec ze ma tou s in ju ri es of th e ski n of t en d evel op i n a
boy, aged 12. Dec reas e of T~l y mp h ocytes an d IgM at n or mal con t en t of IgE and IgG w ere
rev eal ed at th e exami n ati on . Wh at typ e of path ol ogy of th e i mmu n e syst em i s ob served i n
a boy?
Comb i n ed i mmu n od ef i ci en cy
Hypoplasia of thymus
Hypogammaglobulinemia of Bruton
Syndrome of Shereshevskii-Turner
Inherited deficiency of the complement system
16.
Th ere are of ten f un gi an d vi ral d i seases i n a gi rl aged 20 wh o su ff er in tes tin al
pol ip osi s. Th e l ack of wh at l in k of th e i mmu n e syst em i s th e most possi bl e in th i s case?
T-Iymp h ocyt es
B-lymphocytes
Natural killers
Complements
Phagocytes
17.
Viral and f un gi d i seases are of ten i n a gi rl aged 10 w i th h ered i tary d ef ects of th e
h eart an d th yroi d . Ab sen ce of T-cel l i s rev eal ed at th e i mmu n ol ogi cal an al ysi s. Wh at
di sord er of th e i mmu n e syst em i s ob served i n thi s cas e?
H yp opl asi a of th y mu s
Hypogammaglobulinemia of Bruton
Combined immunodeficiency
Syndrome of Shereshevskii-Turner
Inherited deficiency of the complement system
18.
B ru ton ' s d iseas e i s di agn osed in a ch il d aged 5. It con si st of sev ere cou rse of
bac teri al i nf ec ti on s, ab sen ce of B - l ymp h ocy tes an d pl asmati c cel l s. Wh at ch an ges of
i mmu n ogl obu l in con ten t wi ll b e ob serv ed in b lood seru m in th is ch i l d ?
A.
Decrease number of IgD, IgE
B.
Increase number of IgD, IgE
C.
Dec reas e n u mb e r of IgA, IgM
D. Increase number of IgA, IgM
E. Without change
19.
Th e n u mb e r an d fu n cti on al T and B -l ymp h ocy tes ac ti vi ty i s not ch an ged i n a
pati en t wi th cl in i cal si gn s of i mmu n od ef i ci en cy. Th e d ef ect of fu n cti on of an ti gen
p resen ti n g of i mmu n ocomp et en t cel l s tak es pl ace. Def ect of w hat c el l s is most possi bl e?
A.
Fibroblasts, B- lymphocytes, T- lymphocytes
B.
B- lymphocytes, T- lymphocytes
C.
Mac roph ages, mon ocy tes
D.
NK-cells
E.
0- lymphocytes
20.
A.
B.
C.
D.
E.

Wh at is synd rome of Di - G eorg e?


Immunodepression after the spleen removal
Insufficiency of the IgM
Insufficiency of the plasmatic cells
H yp opl asi a of th y mu s gl an d
Inherited insufficiency of C4

21.
Th e p ati en t w as mad e tran spl an tati on of d on ors h eart. Wh at cond i ti on s mu st on e
fol l ow to p rev en t regec ti on of organ tran sp l an tat e?

A.
B.
C.
D.
E.

Transplantation of bone marrow


Transfusion of donors blood
Ch oosi n g th e d on or af t er th e an ti gen s of HL A
removal of spleen
Administration of immunomodulators

22.
A boy aged 3 years i s il l wi th ch ron i c p n eu mon i a. H e h as th e l ow in di c es of B l ymp h ocyt es syste m. Th e di agn osi s i s hyp oga mmagl obu l i n emi a of B ru ton . In d i cate th e
comp l i cati on of B -l ymp h ocyt es syste m i nsuf fi ci en c y.
A.
Decreased resistance of organism to the viruses
B.
Decreased resistance of organism to the mycotic and tubercular infection
C.
Dec reas ed resi stan c e of organ i s m to th e p yrogen i c co cci f lora
D.
Absence of reaction of transplant regection
E.
Increased of risk of development of tumors in an organism
23. A child with i nh e ri ted h eat d ef ect fac e d ef ect ab sen ce of th yroi d and th y mu s inland and
bl ood T-ly mp h ocyt es was ad mi tted to th e ch i ld ren hosp i tal . Wh at inherited path ol ogy i s i t
associ ated w ith ?
A Luis-Barr syndrome
B.
Syndrom of Terner
C.
Bruton's disease
D.
Down's disease
E. Di G eo rge syn d rome

24. An H IV-posi ti ve man exp e ri en ced i mmu n od ef i ci en cy. Wh at typ e of i mmu n e c el l s is


involved in th e i mmu n od ef i ci en cy d evel op i n g?
A.
T-h el p ers
B.
T-suppressors
C.
Macrophages
D.
B-lymphocytes
E.
T-killers
25. In th e cond i ti on s of getti n g u p i n mou n tai n s a man had th e tach yp n oe, tachycardia,
eup h ori a. Ch an ges th at are ob served are th e resu l t of rea cti vi ty:
A.
Specific
B.
Non sp eci f i c
C.
Group
D. Disergic
E. Hyperergic
26. Th e p ati en t w i th rad i cu l i ti s was mad e a n ovocain e bl ock ad e af ter wh i ch h e grew dark
in eyes, hi s art eri al p ressu re d ec reased to 90/50 mmH g, b reath i n g b eca me mo re frequent up
to 24 p er mi n u te. Th e state of p ati en t i s th e resu l t of rea cti vi ty:
A.
Physiology
B.
Hypoergic
C.
Normoergic
D.
Nonspecific
E. H yp erergi c
27. Af te r th e trau ma of on e ey e p ati en t th e si gh t of th e secon d eye b egan grad u all y to
rai l . Wh at me ch an i sm can on e exp l ai n s thi s p h en omen on ?
A.
By the decrease of organism resistance
B.
By an infection
C.
By th e da mag e of h i stoh emati c bar ri er
D.
By immunodeficiency
E.
By immune depression
28. It i s kn ow n th at at so me p ath ol ogi cal p roc esses reac ti vi ty of organ i sm ch an ges
op p osi tel y to th e ch an ge of i ts resi stan ce. At what path ol ogi cal p rocess es or di seases su ch
si tu ati on i s possib l e?

A.
B.
C.
D.
E.

Sh ock
Fever
Inflammation
Posthaemorragic anemia
Arterial hypertension

29. Non sp eci f i c p rot ecti v e f actors of th e oral cavi ty f rom p en etra tin g of path ogen i c
mi croo rgan is ms pl ay an i mp ortan t rol e in th e gen e ral syste m of ph ysi ol ogy resi stan ce of
th e organ i sm. Whi ch of ab ove men ti on ed comp on en ts in th e oral cavi ty i s th e most
i mp or tan t f actor of n on sp eci fi c p rote cti on ?
A.
Complement
B.
Phagocytosis
C.
B-lysin
D.
Properdine
E.
Lysoci me
30. Wh at is th e basi c d if f eren c e of p ath ol ogi cal rea cti vi ty f rom p h ysi ol ogi cal on e?
A.
Participation of immunological mechanisms
B.
Change of permeability of histohaematic barriers
C.
Change of nervous regulation
D.
Change of the endocrine regulation
E.
Rest ri cti on of ad ap ta ti on p ossib i li ti es
31.
A.
B.
C.
D.
E.

Wh at is co rrel ati on b etw een reac ti vi ty and resi stan ce du ri n g hi b ern ati on ?
Decreased reactivity and resistance
Decreased resistance and increased reactivity
Dec reas ed rea cti vi ty an d in creased resi stan ce
Increased reactivity and resistance
Reactivity and resistance does not change

32.
A.
B.
C.
D.
E.

Th e exa mp l e of n on sp eci fi c rea cti vi ty i s:


Gen e ral ad ap tati on synd rome;
Transplantation immunity;
Allergic reactions of immediate type;
Allergic reactions of slow type;
Immunodeficiency state;

33. Qu i te of ten th e cau se of se cond ary i mmu n od ef i ci en cy is an in f ecti on i nvol ve men t,


wh en th e cau sati ve agen ts p rop agat e d i rec tl y in th e cel l s of i mmu n e syste m an d d estroy i t
th e f ol l ow i n g di seases are ch ara ct eri zed by:
A. Inf ec ti ou s mon onu cl eosi s, AIDS
B.
Poliomyelitis, type A hepatitis
C.
Q-febris. epidemic typhus
D.
Dysentery, cholera
E.T uberculosis, mycobacteriosis

*** *** ***


1. A healthy woman has three sons affected by color blindness who were born after her two marriages.
Children both of her husbands are healthy. What is the most possible pattern of inheritance of this disease?
A X-linked recessive
B Y-linked
C Autosomal recessive
D Autosomal dominant
E X-linked dominant
2. A couple came for medical genetic counseling. The man has hemophilia, the woman is healthy and there
were no cases of hemophilia in her family. What is the risk of having a sick child in this family?
A 0*
B 100%
C 75%
D 50%

E 25%
3. A woman who was sick with rubella during the pregnancy gave birth to a deaf child with hare lip and cleft palate.
This congenital defect is an example of:
A Phenocopy
B Edwards syndrome
C Genocopy
D Pataus syndrome
E Downs syndrome
4. A woman who was infected with toxoplasmosis during the pregnancy has a child with multiple congenital
defects.This is a result of:
A Teratogenesis
B Cancerogenesis
C Biological mutogenesis
D Chemical mutogenesis
E Recombination

1
During a prophylactic medical examination a 7-year-old boy was diagnosed with daltonism. His parents are
healthy and have normal colour vision, but his grandfather on his mothers side has the same abnormality. What is
the type of the abnormality inheritance?
A Recessive, sex-linked
B Dominant, sex-linked
C Semidominance
D Autosomal recessive
E Autosomal dominant
2
A 32 y.o. man is tall, he has gynecomastia, adult woman pattern of hair distribution, high voice, mental
deficiency, sterility. Provisional diagnosis is Klinefelter's syndrome. In order to specify diagnosis it is necessary to
analize:
A Caryotype
B Leukogram
C Spermatogenesis
D Blood group
E Genealogy

3
Daltonism was diagnosed in a 7-year-old boy while prophylactic medical examination. Parents are healthy,
color vision is normal. Grandfather from the mother's side has the same disorder. What is the type of inheriting of
this anomaly?
A Recessive, connected with sex
B Dominant, connected with sex
C Incomplete domination
D Autosomal-recessive
E Autosomal-dominant
4
A married couple came to the genetic counseling. The husband suffers from the
insulin-dependant diabetes, the wife is healthy. What is the probability that this couple will have
dependant child?
A Higher than throughout the population
B The same as throughout the population
C Lower than throughout the population
D 100\%
E 50\%

an insulin-

5
A couple had a child with Down's disease. Mother is 42 years old. This disease is most probably caused by
the following impairment of prenatal development:
A Gametopathy
B Blastopathy

C Embryopathy
D Non-specific fetopathy
E Specific fetopathy
6
Examination of a 12 year old boy with developmental lag revealed achondroplasia:
disproportional
constitution with evident shortening of upper and lower limbs as a result of growth
disorder of epiphyseal
cartilages of long tubal bones. This disease is:
A Inherited, dominant
B Inherited, recessive
C Inherited, sex-linked
D Congenital
E Acquired
7
Amniocentesis revealed two sex chromatin bodies (Barr bodies) in each cell of the sample. What disease is
this character typical for?
A Trisomy X
B Klinefelter syndrome
C Turner's syndrome
D Down's syndrome
E Patau syndrome
8
A 35-year-old male patient has been referred by an andrologist for the genetic counselling for the
deviations of physical and mental development. Objectively: the patient is tall, has asthenic
constitution,
gynecomastia, mental retardation. Microscopy of the oral mucosa cells revealed
sex chromatin (single Barr
body) in 30% of cells. What is the most likely diagnosis?
A Klinefelter syndrome
B DiGeorge syndrome
C Down syndrome
D Recklinghausen's disease
E Cushing pituitary basophilism
1. A child is pale, pastose, muscular tissue is bad developed, lymph nodes are enlarged. He often suffers from
angina and pharyngitis, blood has signs of lymphocytosis. The child is also predisposed to autoallergic
diseases. What type of diathesis can be presumed in this case?
A Lymphohypoplastic
B Exudative
C Gouty
D Asthenic
E Hemorrhagic
2. Examination of a 12 year old boy with developmental lag revealed achondroplasia: disproportional
constitution with evident shortening of upper and lower limbs as a result of growth disorder of epiphyseal
cartilages of long tubal bones. This disease is:
A Inherited, dominant
B Inherited, recessive
C Inherited, sex-linked
D Congenital
E Acquired
3. A couple had a child with Down's disease. Mother is 42 years old. This disease is most probably caused by
the following impairment of prenatal development:
A Gametopathy
B Blastopathy
C Embryopathy
D Non-specific fetopathy
E Specific fetopathy
4. A married couple came to the genetic counseling. The husband suffers from the insulin-dependant diabetes,
the wife is healthy. What is the probability that this couple will have an insulin-dependant child?
A Higher than throughout the population
B The same as throughout the population
C Lower than throughout the population

D 100%
E 50%
5. Daltonism was diagnosed in a 7-year-old boy while prophylactic medical examination. Parents are healthy,
color vision is normal. Grandfather from the mother's side has the same disorder. What is the type of
inheriting of this anomaly?
A Recessive, connected with sex
B Dominant, connected with sex
C Incomplete domination
D Autosomal-recessive
E Autosomal-dominant
6. A 32 y.o. man is tall, he has gynecomastia, adult woman pattern of hair distribution, high voice, mental
deficiency, sterility. Provisional diagnosis is Klinefelter's syndrome. In order to specify diagnosis it is
necessary to analize:
A Caryotype
B Leukogram
C Spermatogenesis
D Blood group
E Genealogy
7. During a prophylactic medical examination a 7-year-old boy was diagnosed with daltonism. His parents are
healthy and have normal colour vision, but his grandfather on his mothers side has the same abnormality.
What is the type of the abnormality inheritance?
A Recessive, sex-linked
B Dominant, sex-linked
C Semidominance
D Autosomal recessive
E Autosomal dominant
8. A woman who was sick with rubella during the pregnancy gave birth to a deaf child with hare lip and cleft
palate. This congenital defect is an example of:
A Phenocopy
B Edwards syndrome
C Genocopy
D Pataus syndrome
E Downs syndrome
9. A couple came for medical genetic counseling. The man has hemophilia, the woman is healthy and there
were no cases of hemophilia in her family. What is the risk of having a sick child in this family?
A 0%
B 100%
C 75%
D 50%

25%

10. A healthy woman has three sons affected by color blindness who were born after her
two marriages. Children both of her husbands are healthy. What is the most possible
pattern of inheritance of this disease?
A
X-linked recessive
B
Y-linked
C
Autosomal recessive
D
Autosomal dominant
E
X-linked dominant
1. The father of pregnant woman suffers from hemeralopia, that is inherited by i recession
sign coupled with X-chromosome. Among the relatives of husband this disease there was no
such disease. What possibility that a born child will suffer by a in mei alopia if it is known
that fetus is mail sex?
A. 50%
B 0%

C. 25%
D. 75%
E. 100%
2. Syndrome of Down is characterized by:
A. Trisomy for 21 chromosome
B. Trisomy for X chromosome
C. Deficiency for 15 chromosome
D. Pathology coupled with X -chromosome
E. High growth of patients with this pathology
3. Neutrophilic leukocytes with a 1 "drumstick" were revealed in sick boy's blood at
examination in medico-genetic consultation. The presence of what syndrome is possible in a
boy?
A. Klinefelter's syndrome
B. Down' s syndrome
C. Syndrome of Shereshevskii-Turner
D. Edward's syndrome
E. Trisomy X-syndrome
4. Ataxia, coordination disorder, teleangiectasia of skin and conjunctiva, adrenal and sevual
glands, disorders mental deficiency, recurrent sinuspulmonic infections, thymus
hypoplasia, lack of Ig and Ig E were revealed in a girl aged 5. In future lymphosarcoma
developed and child died. What type of inheritance is characteristic Of this illness?
A. Autosomal-recessive
B. Partial dominance
C. Dominant
D. Coupled with a sexl chromosome
E. Polygenic
5. Vomiting and diarrhea, general dystrophy, hepato- and splenomegaly are observed in a new-born. Brest
feeding at stopping and the symptoms decrease. What basic Inherited defect will be observed in
pathogenesis?
A.Galactose exchange disorder
B.Hypersecretion of external secretion glands
C.Tyrozine exchange disorder
D.Fenilalanine exchange disorder
E.Insufficiency of glucose - 6-phosphatedegidrogenase
6. At amn i oc en tesi s th ree lu mp s of sexu al ch roma ti n e w ere foun d i n n u cl ei of cel l s of It mai l sex. Wh at kari otyp e d oes th is d i sord er cor resp ond to?
A. 49,XXXX Y
B. 47, XXX
C. 48,XXXY
D. 47,XXY
E. 48,XXXX
7. H et erozygou s moth e r car ri er p assed a mu tan t gen e to th e hal f of son s (w h i ch w ere i l l )
an d a h al f of d au gh ters wh i ch th at were p h en otyp i cal h eal th , bu t are th e car ri es and may
pass a rec essi on gen e wi th X-ch romoso me to th e n ext gen e rati on . Th e gen es of w hat
ab ove men ti on ed di seases may b e passed to dau gh t ers?
A. Polygraphic
B. Thalassemia
C. Phenylketonuria
D. H emoph i l i a
E. Hypertrichosis

8. Mo th er an d on e of d au gh te rs h ave art eri al h yp e rten si on . Moth er and d au gh te r th at i s


il l w ork on con veyor. An oth er dau gh ter al so someti mes has h i gh ar teri al p ressu re. Wh at
grou p of gen eti c di seases does arte ri al hyp er ten si on b el on g to in th i s fa mi l y?
A. Monogenes
B. Chromosomal
C. Mu l tif acto ri al
D. Genomic
E. Acquare
9. A gi rl , aged 16, h as so me d evi ati on s f rom n orm: h er h i gh t i s l ow e r th an in ch i l d ren of
th e sa me age, si gn s of th e sexu al ma tu rety are ab sen t, sh e h as a sh ort neck) wi d e sh ou ld e rs.
In t el l ect is i n a nor m. Wh at of ab ove men ti on ed synd romes may b e supposed?
A. Patau's syndrome
B. Down's syndrome
C. Edward's syndrome
D, Klinefelter's syndrome
E. S yn d rome of Sh eresh evsk ii - Turn er
10. A w omen has d evi ati on f rom nor m in p hysi cal and sexu al d evel op men t. At th e
mi cros copi cal exa mi n ati on of mu cos cel l of oral cavi ty sexu al ch romati n was n ot found.
Wh at di agn osi s i s th e most possib l e?
A. S yn d rome of S h eresh evsk i i -Turn e r
B. Klinefelter's syndrome
C. Down's syndrome
D. Rcclingauzen's disease
E. Trisomy for X chromosome
11. An eld e rl y man has h i gh sen si ti vi ty to i nf ec ti on s, esp e ci all y ac ti vati on of ch ron i c H ral
in f ecti on d i sease of vessel s, parti cul ar of ath e roscl e roti c typ e, p ern i ci ou s i n emi a. Ch an ges
in wh at syste m of organ i s m agi n g are acco mp an i ed by ab ove men ti on ed p h en o men on ?
A. Endocrine
B. Nervous
C. Immu n e
D. Connecting tissue
E. Circulation of blood
12. A man has a h i gh statu re, ast eni c stru ctu re of bod y, gyn eco masti a, men tal
in suf fi ci en cy. At th e mi croscop i c exami n ati on of mu cu s cel l s of th e oral cavi ty th ere was
foun d i n 30% cel l s sexu al ch romati n e (on e Bar ras bod y). Wh at th e mo st p ossi bl e
di agn osi s i n thi s case?
A. Icenko-Cushing syndrome
B. Reclingauzen's disease
C. Syndrome Di George
D. Down's syndrome
E . Kl i n ef el t er' s synd rome
13. A pati en t aged 60 con sul t ed a docto r. Sh e comp l ain ed of pai n i n th e smal l joi n ts of l egs
an d h and s. Joi n ts are i nl arg ed , l ook l ik e as bu l ged n od s. Th ere i s i n creas ed con t en t of
u rates i n th e b lood seru m. Th e d i sord er of wh at met ab oli s m cau sed th e d evel op men t of
thi s stat e?
A. Metadiazines
B. Amino acid
C. Pu ri n e
D. Lipids
E. Carbohydrates
14. A man aged 65 con su l ted a d octo r. H e co mp l ai n ed of acu t e pain in a l arge toys of th e
l eg. H e l ik es and of ten d rin k s b ee r. Gou t was su sp e cti d e. Con ten t of what of b el l ow
me n ti on ed su b stan ces i s i t n ecessa ry to d eter mi n e in b lood to con fi r m a d iagn ose?
A Uri n ary aci d
B. Bilirubin
C . Ketonic bodies
D. Urina

E. Lactate
15. A chi l d aged 6 mon th s i s ob serv ed h as d ec el era ti on of moto r and psych i cal
d evel op men t, p al l or of sk in i ri s of ey e, p osi ti ve test of Fel l i n g (w i th a 5% sol u ti on of
tri ch l orac etat e i ron ). Wh at of ab ove men ti on ed i nh eri ted d iseas es i s di agn osed i n a ch il d ?
A. Urine acid
B. Alkaptonuria
C. Down s di se ase
D. Albinism
E. Ph en yl k etonu ri a
16.
H emol yti c cri si s an d d evel op men t of si cl e- sh ap e an ae mi a are ob serv ed i n a patient
wh o is a ca rri e r of si cl e -sh ap e an o mal y of erh ytrocyt es d u e to th e development of
atal ec tase of th e lu ngs. Wh at i s th e me ch ani s m of th e d evel op men t of th i s f orm of
an ae mi a?
A. Neurohumoral
B. Ncuro-reflectory
C. Humoral
D. Corticovisceral
E. G en eti c
17.
A ch il d was b orn in a mo th e r wh o su ff ers f rom al coh oli s m. Th e si gns of me n tal and
ph ysi cal d efi ci en cy i n th e p roc ess of grow th an d d evel op men t w ere ob served . O f what
ch aract er i s thi s p ath ol ogy?
A. Inherited
B. Chromosomal
C . Acquired
D. In b orn
E. Prenatal
18.
Th e p h ysi cal exa mi n ati on of a p ati en t reveal ed th i n statu re, l arge sk ul l , stron gl y
developed f ron tal p art of f ace, sh ort ex tremi ti es. Wh at statu re typ e i s i t ch aract eri sti c of?
A. Respirator
B. Muscle
C . Digestive
D. Cereb ral
E. Mixed
19. Wh at di seases is a man w i th asten i c typ e of statu re ac cord i n g Crech mer' s
cl assif i cati on mo re of t en il l wi th ?
A. Acute respiratory infections
B. Tuberculosis of lungs
C. Hypoacidic gastritis
D. Hysteria
E. S ch i zop h ren i a
20.

Wh at is d i ath esi s?
A. Diseases of an early child's age
B. Anomaly intrauterine development
C. An omal i c statu re wi th th e i n ad eq u ate reac ti on of organ i sm to i rri ta ti on
D. Inherited enzymopathy
E. Tendency of the people to definite diseases

21.
Ph ysi cal exami n ati on of th e a p ati en t reveal ed : th i n statu re, l arge sku l l , stron gl y
d evel op ed f ron tal p art of face, sh ort ext remi ti es. Wh at con sti tu ti on al typ e i s i t
ch aract eri sti c of ?
A. Mixed
B. Digestive
C. Muscles
D. Cereb ral
E. Respiratory

22.
Karyo typ e 47 XXY was reveal ed in a youn g 20-yea r-ol d tal l asth eni c man wi th
si gn s of h yp ogon adi s m, gyn eco masti a and d ec reas ed sp e rm p rod u cti on (azoosp er mi a ).
Wh at h ered i tary syn d rome mi gh t b e di agn osed in th i s case?
A. Kl i n ef el te r'
B. Wiskott-Aldrich
C. Turner's syndrome
D. Louis-Bar
E. Down's
23. Three l um ps of sex ual chrom at i n w ere rev eal ed in t he cel l nucl ei of the wom an fetus. What
kar yot ype woul d be app ropri at e i n thi s cas e?
A. XXX XY
B. XXX
C. XXXY
D. XXY
E. XY
24. A 28 -yca r-ol d f emal e p ati en t con su l ted a gyn ecol ogi st ab ou t ste ri l i ty. E xami n ati on
revealed u nd erd ev el op ed ovari es and u teru s, i rregu l ar men st ru al cycl e. An al ysi s of the sex
ch romati n reveal ed 2 B arr' s bod i es in most soma ti c c el l s. Wh at ch romoso me disease i s mo st
li k el y?
A. Klinefeltier's syndrome
B. Turner's syndrome
C. Patau's syndrome
D. Triple X syndrome
E. Edwards' syndrome

*** ***
1. A 27- year-old woman has dropped penicillin containing eye drops. In few minutes there appeared feeling
of itching, burning of the skin, lips and eyelids edema, whistling cough, decreasing of BP. What antibodies
take part in the development of this allergic reaction?
A IgE and IgG
B IgM and IgG
C IgA and IgM
D IgM and IgD
E IgG and IgD

2. A 27 y.o. patient put eye drops that contain penicillin. After a few minutes she felt itching and

burning
of her body, there appeared lip and eye-lid edemata; arterial pressure began to drop. What immunoglobulins
took part in the development of this allergic reaction?
A lgE and lgG
B IgM and IgG
C IgA and IgM
D IgM and IgD
E IgG and IgD
1. A 27 y.o. patient put eye drops that contain penicillin. After a few minutes she felt itching and burning of
her body, there appeared lip and eye-lid edemata; arterial pressure began to drop. What immunoglobulins
took part in the development of this allergic reaction?
A lgE and lgG
B IgM and IgG
C IgA and IgM
D IgM and IgD
E IgG and IgD
2. A 27- year-old woman has dropped penicillin containing eye drops. In few minutes there appeared feeling
of itching, burning of the skin, lips and eyelids edema, whistling cough, decreasing of BP. What antibodies
take part in the development of this allergic reaction?
A IgE and IgG
B IgM and IgG
C IgA and IgM
D IgM and IgD

E IgG and IgD


3. A woman has been applying a new cosmetic preparation for a week that resulted in eye-lid inflammation
accompanied by hyperemia, infiltration and painfulness. What type of allergic reaction was developed?
A V
B I
C II
D III
E V
1. Pai n in th e h ear t and joi n ts and p n eu mon i a app ea red i n a pati en t th ree w eek s lat er
acu te myo card i al in farcti on . Wh at i s th e mai n me ch an i sm of d evel op men t of post
in farcti on Dressl er' s syn d rome?
A. Ischemia of myocardium
B. Resorption of enzymes from necrotized area of myocardium
C. Secondary infection
D. Thrombosis of vessels
E. Autoimmune inflammation
2. A p ati en t add ressed to a d en ti st w i th co mp l ai n ts of redn ess an d ed ema of mu cou s
me mb ran e of h is mou th a mon th l ater d en tal p rosth esi s. Al l e rgi c stomati ti s was
di agn osed in th i s pati en t. Wh at typ e of al l ergi c rea cti on by G el l and Cu mb s un d erl i es
thi s d i sease?
A. Cytotoxic
B. Delayed-type hypersensitivity
C. Immune complex-mediated
D. Anaphylactic
E. Stimulating
3. An aph yl acti c sh ock d evel op ed i n a pati en t wi th botu l is m af te r se cond i n jec ti on of
an ti toxi c an ti b otu l in u s seru m mi xtu re. Wh at i s th e mai n me ch an i sm of an ap h yl axi s.'
A. Interaction of T-lymphocytes with mediators
B. Interaction of antigen with IgM
C. Interaction of macrophages with antigens
D. Interaction of antigen with IgE
E. Interaction of T-lymphocytes with tissue basophils
4. In a 27- years- oId man tu b ercu l i n test was car ri ed ou t. Fol l owi n g was observed 24 hou rs
l ater : i nf il tra ti on wi th si ze of 40x35 mm at th e si te of i n jecti on and h yp eremi a of sk in
ab ove i t. Wh at grou p of bi ol ogi c acti v e sub stan ces cau ses d evel op me n I ol al l e rgi c
in fl a mma ti on i n thi s p ati en t?
A. Lymphokines
B. Biogenic amines
C. Prostaglandins
D. Leukotriens
E. Kinins
5. A 24-years- old p ati en t h as ed ema of face and i n creas e i n B P, wh i ch ap p eared 1 .5 week s
l ater sev ere strep to coc cu s ton sil l i ti s. Th e pati en t h as h e matu ri a and p rot ei nu ri a of 1.2
g/L . An ti -st rep to coccu s an ti b odi es and d ecrease i n con ten t of co mp l i men t syste m
comp on en ts w ere reveal ed i n p ati en t' s b l ood . Wh i ch microvessels do d ep osi ts of i mmu n e
comp l exes l ocal i ze in and cau se n ep h rop ath y?
A Proximal tubules
B. Glomerule
C. Dcscendent tubules
D. Loop of Henle
E. Pyramids
6. Nau sea, fati gu e, stomach ach e, p alp i tati on , d if fi cu l t resp i rati on , an d sk in bl i sters
developed i n a pati en t 25 mi n u tes l ater in je cti on of an tib i oti cs. Wh at stag e of al l ergi c
reaction is ob serv ed in th i s pati en t?
A. Pathochemical
B. Biochemical
C. Pathophysiological
D. Immunological

E. Sensibilization
7. S ki n tu b ercu li n t est was carri ed ou t in a pati en t w i th ch ron i c lu n g tu b ercu l osi s. L ocal
hyp eremi a an d ed ema ap p eared i n th e si te of in racu tan eou s in t rodu c ti on of tub e rcu l in
p rep arati on w ith in 24- 48 hou rs. Wh at cel l s are p ri mary eff ec tors i n me ch an i sm of th i s
rea cti on ?
A. Neutrophils
B. T-lymphocytes
C. B-lymphocytes
D. Endotheliocytes
E. Smooth muscle cells of microvessels
8. H yp eremi a, sw el l in g an d th en n ecrosi s of ti ssu e, th ei r reje cti on an d u l cer (Ar thu s
ph en omen on ) d evel op at th e rabb i t in th e p l ace of secon d ary in tra cu tan eu s i n jecti on of a
su b stan ce wi th st ron gl y p ron ou n ced an ti geni c p rop erti es (f or exa mp l e h orse seru m).
Wh at facto rs p lay th e mai n rol e i n p ath ogen esi s of th i s ph en omen on ?
A. Antibodies presented by IgE
B. Antibodies presented by IgD
C. Antibodies presented by IgA
D. Antibodies presented by IgG and/or IgM
E. Specific T-lymphocytes-effectors
9. S ki n rash , i tch in g, sw ell i n g an d pai n in joi n ts, in c reas e i n b od y te mp era tu re, an d
p rotei n u ri a ap p eared in a p ati en t i n 5-8 d ays af te r u se l ots of med i cal seru m. S eru m
si ck n ess w as di agn osed . Wh at i s th e mai n fac tor in p ath ogen esi s of thi s synd rome?
A. Primary systemic accumulation of circulating immune complexes in the blood
B. Primary systemic degranulation of mast cells in the organism
C. Primary systemic activation of T-killers
D. Primary systemic activation of endoteliocytes
E. Primary systemic cytolysis of blood cells
10. Al l ergi c d iagn osti c t ests are u sed for th e d iagn osi s of man y in f ecti ou s di seases
(tu b ercul osi s, b ru cel osi s, tul aremi a e tc). Di agn osi s i s conf i rmed i f papu l a and redn ess
ap p ear in th e pl ace of th e al l erg en in je cti on . An ti gen s i n tera cti on rea cti on i s cond i ti on ed
by:
A. IgE and lymphokines
B. IgM and macrophages
C. T-lymphocytes and lymphokines
D. IgE and T-lymphocytes
E. IgM and tissue basophiles
11. An 18-yea r-old man wi th sh ou l d er p hl eg mon got in tra mu s cul ar i n jec ti on of p eni ci l l in .
Tach yca rdi a, th read - li k e pu l se; d ecrease i n B P down to 80/ 60 mmH g occu r al te r th at
Wh at ki nd of ph armacol ogi c rea cti on d evel ops?
A. Potentiation
B. Reflex action
C. Central action
D. Anaphylaxis
E. Peripheral action
12.
Th yrotoxi cosi s w as di agn osed i n a p ati en t. An ti th yroi d an ti b odi es w ere f ou nd in
hi s b lood . Whi ch typ e of al l ergi c reac ti on is ob serv ed at d evel op men t of thi s d iseas e?
A. Immune complex-mediated
B. Stimulating
C. Anaphylactic
D. Cytotoxic
E. Delayed type hypersensitivity
13.
Hi ves, i tchi n g of th e sk i n , sw el li n g of th e sk i n an d mu cou s me mb ran es, sw ell i n g of
l ymp h ati c n od es d evel op i n th e pati en t i n 9 days af te r in je cti on of med i ci n al se ru m.
Wh at di sease d evel op s?
A. Pollinosis
B. Serum sickness

C. Shwartzman's phenomenon
D. Overy phenomenon
E. Quincke's edema
14.
Dressl er' s synd rome w as di agn osed at th e p ati en t 1.5 mon th l ater my oca rdi u m
in farcti on . It i s ch aract eri z ed b y p eri ca rd i ti s, p l eu ri sy, an d p n eu mon i a. Wh at i s th e
reason f or th i s syn d rome ?
A. Sensitization of the organism by myocardium antigens
B. Decrease in resistance to microorganisms
C. Activation of saprophytic microflora
D. Intoxication of organism by products of necrosis
E. Release of myocardial enzymes to the blood

15. A 20-year- ol d man has in ju ry of th e ri gh t tes ti cl e. Wh at dan ger d oes i t b ri n gs for the
l ef t (healthy) t esti cl e?
A. Mimicry of antigens and development of antibody-mediated damage
B. Development of infectious process
C' Development of atrophy
D. Development of hypertrophy
E. No danger
16. Guinea-p i g' s n eph rocyto toxi c seru m was in j ect ed to th e rab b i t u nd er th e experiment.
Wh at hu man di sease is mod el ed i n thi s case?
A. Nephrotic syndrome
B. Acute pyelonephritis
C. Chronic renal insufficiency
D. Acute diffuse glomerulonephritis
E. Chronic pyelonephritis
17. Acu te gl omeru l on eph ri ti s app ea red i n th e p ati en t 2 w eek s l ater pu ru l en t tonsillitis.
An tib od i es again st mi c roorgan i s m an ti gen s w ere foun d at th e p ati en t . Which
mi croo rgan is m are th ese an ti b od i es agai n st?
A. Hemolytic streptococcus
B. Staphylococcus
C. Pneumococcus
D. Mycobacterium tuberculosis
E. Meningococcus
18. S evere ed e ma of sof t ti ssu es of up p er an d l ow er jaw s, rash on th e sk in of f ace,
red n ess, an d i tch i n g app ea r in th e pati en t i n resp on se to usi n g an esth eti c d rag at tooth
extra cti on . Whi ch p ath ol ogi cal p rocess u nd erl i es th e reacti on to an esth e ti c?
A. Inflammation
B. Drag toxic action
C.Allergy
D. Insufficiency of blood circulation
E. Disorder of lymph outflow
19. Novocai n was i n jec ted by d en ti st f or an esth esi a at tooth extra cti on . S ymp toms of
ph vl acti c sh ock app ea red at th e pati en t f ew mi n u tes l ater. Pati en t h as d rop of B P,
dys p n ea, l oss of con sci ou sn ess and con vu l si on s. Wh at typ e of reac ti on is i t?
A. Immediate type hypersensitivity
B. Cytolytic or cytotoxic reactions
C. Arthus phenomenon reactions
D. Delayed type hypersensitivity
E. Stimulating allergic reaction
20. Man wi th th e cari es i s su b jec ted to constan t sen si ti zati on b y st rep to coccu s an ti gen .
Wh at di sease can ap p ear du e to thi s e ti ol ogi cal f actor?
A. Glomerulonephritis
B. Pancreatitis
C. Myocarditis

D. Pulpits
E. Periodontitis
21. An ti toxi c di ph th eri a seru m w as i n trod u ced to a chi l d su ff eri n g f rom d ip h th eri a. S ki n
eru p ti on acco mp an i ed b y i tchi n g, i n crease i n bod y te mp era tu re to 38C, and pai n i n
join ts occu r red i n p ati en t 10 d ays l ater. Wh at is th e reason f or th ese symp to ms?
A. Contact allergy
B. Serum sickness
C. Atopy
D. Anaphylactic reaction
E. Delayed type hypersensitivity
22. A d en ti st in j ect ed ul tra cai n to a p ati en t b ef ore tooth extra cti on for th e pu rp ose of
an es th esi a. S en si ti vi ty tes t was n ot mad e. An ap h yl acti c sh ook d evel op ed i n th e pati en t i n
f ew mi nu t es af te r d rag i n jec ti on . Wh at c el l s p rod u ce reagi n s, whi ch tak e part i n
d evel op men t of an aph yl acti c rea cti on ?
A. Plasma cells
B. B-lymphocytes
C. T-lymphocytes
D. Mast cells
E. Eozinophiles
23. Too th w as extra ct ed i n a teen age r un d er Novocai n an esth esi a. Pal en ess of sk in ,
dyspn ea an d h yp oten si on occu r red i n th e pati en t 10 mi n u tes lat er. Wh at typ e of al l ergi c
rea cti on i s i t?
A.Anaphylactic
B. Cytotoxic
C. Arthus phenomenon type
D. Delayed type hypersensitivity
E. Stimulating
24.
Tooth was ex trac ted in a te en ager u nd er Novocai n an esth esi a. Pal en ess of sk i n ,
dyspn ea an d h yp oten si on oc cu rred i n th e pati en t 10 mi n u tes l ate r. Wh at su b stan ce d oes
all e rgen react wi th on th e su rf ace of mast cel l s?
A. IgE
B. T-lymphocytes
C IgA
D. IgD
E. IgM
25.
A 43-yea r-ol d wo man i s suf f eri n g f rom pn eu mon i a. S h e b egan co mp l ai ni n g of
weak n ess, f ace and arms bu rn in g pai n i n 10 mi n u tes af t er a mp i ci ll i nu m i n jecti on . Cou gh ,
dyspn ea, p ain in th e ch es t d evel op ed in h er. At cl i ni cal exa mi n ati on p ati en t has cyan osi s,
eyel i d s sw el l i n g, f ace red rash es, h eart rate - 120 p er mi n u te, B P 120 mm H g. muf f l ed
h eart sou nd s, h yp opn oe an d tach yp n ea, resp i rati on w i th d if f eren t moi st ral es. Wh at i s
th e reason for worsen in g of pati en t' s con d i ti on ?
A. Anaphylactic shook
B. Urticaria
C. Quincke's edema
D. Attack of asthma
E. Pulmonary thromboembolism
26.
0.1 ml of h orse se ru m w as i n jec ted to th e gu i n ea-p i g for sensi ti zati on . Wh at are
exte rn al si gn s of sen si ti zati on ?
A. No external signs
B. Skin rashes
C. Joints swelling
D. Increase in body temperature
E. Pain
27.
A 15-yea r-ol d gi rl i s suf f erin g f rom b ron ch i al asth ma. S evere attack of exp i ratory
dyspn ea d evel op s in h er d u rin g th e sp ri n g bl osso mi n g p eri od . Wh at b i ol ogi cal ac ti ve
su b stan ce causes sp as m of b ron chi al s mooth mu s cl e i n thi s cas e?
A. Leukotriene

B. Thromboxane A2
C. Prostacyclin
D. Bradykinin
E. Serotonin
28.
Pa ti en t add ressed to a docto r wi th co mp l ai n ts of h ead ach e, rh i ni ti s, w eakn ess, an d
in c rease i n b od y te mp e ratu re d evel op i n g ev ery sp rin g in bl ossomi n g p eri od . Wh at typ e of
all e rgi c reac ti on b y Gel l an d Cu mb s u nd erl i es thi s d i sease?
A. Anaphylactic
B. Cytotoxic
C. Antibody-dependent cell-mediated cytotoxicity
D. Immune complex-mediated
E. Delayed type hypersensitivity
29. Weak n ess, i tch i n g of ski n , acu t e sp as mod i c pai n i n th e ab d omen , hyp eremi a an d ru sh
on sk in , ta ch ycard i a, an d d ecrease in B P d ow n to 70/ 40 mm H g sud d enl y d evel l op ed in a
pati en t f ew mi n u tes l ate r Novocai n i n jec ti on by th e d en ti st. Wh i ch typ e of all e rgi c
rea cti on s does thi s p ath ol ogy b el on g to?
A.Anaphylactic
B. Cytotoxic
C. Stimulating
D. Cell-mediated
E. Immune complex-mediated
30. Con tact d ermati ti s of up p er extremi ti es d evel op ed i n th e n u rse, wh o has b een
work in g i n man i p ul ati on roo m for 20 years. Wh i ch typ e of al l ergi c reacti on s d ocs th i s
path ol ogy b el on g to?
A.Delayed-type hypersensitivity
B. Primary immunodeficiency
C. Immediate type hypersensitivity
D. B-cells immunodeficiency
E. T-cells immunodeficiency
31. Pai n i n th e joi n ts and l oi n s, h e mo rrh agi c eru p ti on s on th e sk i n , and in c rease i n b od y
te mp e ratu re occu r i n th e pati en t wi th toxe mi c stage of bu rn di sease in 2 h ou rs af te r
all ogen i c p l asma tran sfu si on . Wh at al l ergi c rea cti on tak es p l ace i n thi s cas e?
A. Serum sickness
B. Urticaria
C. Quincke's edema
D. Anaphylactic
E. Autoimmune vasculitis
32. H yp erergi c i nf l ammati on f orm of up p er resp i rato ry trac t (l aryn x, tra ch ea, b ron chi )
d evel ops at 6-yea r-ol d ch i ld . Th reat of resp i rato ry i mp ai r men t d evel op s an d th en
n ecessi ty of usi n g an ti -i nf l ammatory h or mon es occu rs. Wh i ch h ormon e h as an ti in fl a mma tory p rop e rty?
A. Cortisol
B. Adrenaline
C. Growth hormone
D. Testosterone
E. Insulin
33.
Sw el li n g, in c reased B P, p rot ein u ri a, h e matu ri a, d ec reas ed u rin e excreti on w ere
rev eal ed in th e pati en t, whi ch suf f ered sev ere ton sil l i ti s b ef ore. Th ese sy mp to ms are
typi cal of acu te gl omeru l on ep h ri ti s, resul ti n g from d amage of gl omeru l ar b asemen t
me mb ran e. Wh at is me ch ani s m of th is d i sease?
A. Anaphylactic allergic reactions
B. Cytotoxic allergic reactions
C.Immune complex-mediated allergic reaction
D. Delayed-type hypersensitivity
E. Stimulating allergic reaction

34.
An 18-year-ol d pati en t has i nsu li n -d ep en d en t di ab etes mel l i tu s I typ e. Wh at typ e
of al l ergi c rea cti on s un d erl i es b eta c el l s da mage?
A. II type, cytotoxic
B. I type, anaphylactic
C. Ill type, immune complex-mediated
D. IV type, delayed-type hypersensitivity
E. Pseudoallergic reaction
35.
Wh i ch on e a mon g men ti on ed b el ow path ol ogi c p roc esses can b e d esc rib ed as a
rea cti on of i mmed i ate typ e?
A. Pol yn osi s
B. Autoallergy
C. Contact allergy
D. Graft-versus-host reaction
E. Bacterial allergy
36.
In t ramu s cu l ar p eni ci l l in w as ad mi n ist rated to a pati en t. Fol l owi n g th e i n jec ti on
th e cond i ti on of th e pati en t w orsen ed ab ru p tl y: b reath l essn ess and d eath -d amp
ap p eared . Pu l se -140 and w eak , bl ood p ressu re - 90/ 40. Wh at comp l i ca ti on i s th e most
possi bl e in th i s case?
A.An ap h yl acti c sh ock
B. Pulmonary embolism
C. Cardiogenic shock
D. Infectious toxic shock
E. Infectious allergic shock
37. It w as d eci d ed to i n jec t an ti tetan u s seru m to a pati en t, bu t al l ergi c test ap p eared to
b e posi ti ve. Wh at is th e ap p rop ri ate way to con d u ct d esen si ti zati on ?
A. In j ecti n g of s mal l d oses of an ti te tanu s seru m
B. Injecting of glucocorticoids
C. Injecting of a full dose of antitetanus serum
D. Prescribing of antihistamine medications
E. Prescribing of immunosuppressants
38. A 10-yea r ol d chi l d had th e man tou x tub e rcu l i n test ad mi ni st ered . 48 hou rs l ate r a
pap ul e up to 8 mm in d ia me te r ap p eared on th e si te of th e in je cti on . Wh at typ e of
hyp ers en si ti vi ty reac ti on d evel op ed af ter th e tu b ercu li n in j ecti on ?
A. Atopic reaction
B. Seroreaction
C. Arthus phenomenon
D.Typ e IV h yp ers en si ti vi ty rea cti on
E. Type II hypersensitivity reaction
39. Af te r th e p ri or sen si b i li zati on an exp eri men tal an i mal w as gi ven a su b cu tan eou s
in je cti on of an an ti gen . Th e p l ace of in je cti on exh ib i ted a f ib ri n ou s i nf l ammati on wi th
al tera ti on of th e vessel w al l s, basal sub stan c e an d f ib rou s stru ctu res of th e con n ecti v e
ti ssu e in f orm of mu coi d and f ib ri n oi d sw el l in g and n ec rosi s. Wh at i mmu n ol ogi cal
rea cti on i s i t?
A. Granulomatosis
B. Normergic reaction
C. R eact i on of transplantation immunity
D.Del ayed - typ e h yp e rsen si ti vi ty
E. Immediate hypersensitivity

***

***

***

1. While playing volleyball a sportsman made a jump and landed on the outside edge of his foot. He
felt acute pain in the talocrural joint, active movements are limited, passive movements are unlimited but
painful. A bit later there appeared a swelling in the area of external ankle, the skin became red and warm.
What type of peripheral circulation disturbance is the case?

A
B
C
D
E

Arterial hyperemia *
Stasis
Embolism
Venous hyperemia
Thrombosis

2. A patient with obliterating endarteritis underwent ganglionic sympathectomy. What type of arterial
hyperaemia should have developed as a result of the surgery?
A Neuroparalytic *
B Neurotonic
C Metabolic
D Functional
E Reactive
3. A 42 year old woman with neuralgia of trifacial nerve complains about periodical reddening of the right
part of her face and neck, sense of warmth gush, increased skin sensitivity. These effects can be explained by
the following type of arterial hyperemia:
A Neurotonic *
B Neuroparalytic
C Metabolic
D Functional
E Reactive
4. A rabbit's nerve that innervates the right ear was cut and its right superior cervical ganglion was removed.
Immediately after operation the temperature of ear skin was measured. It was
revealed
that
the
temperature of the rabbit's ear skin on the side of denervation was by 1,5 0C higher than on the opposite intact
side. What of the following is the most probable
explanation of the above-mentioned effects?
A Arterial neuroparalytic hyperemia *
B Arterial neurotopical hyperemia
C Atrerial hyperemia induced by metabolic factors
D Reactive arterial hyperemia
E Physiological arterial hyperemia
5. A patient with obliterating atherosclerosis underwent sympathectomy of femoral artery in the region of
femoral trigone. What type of arterial hyperemia was induced by the operation?
A Neuroparalytic *
B Reactive
C Metabolic
D Neurotonic
E Functional
6. Upper neck node of sympathetic trunk was removed from the rabbit on experiment. Reddening and
increased temperature of the skin of head is observed. What form of peripheral circulation of the blood
developed in the rabbit?
A Neuroparalytic arterial hyperemia*
B Neurotonic arterial hyperemia
C Metabolic arterial hyperemia
D Venous hyperemia
E Stasis
1. Increase of blood viscosity, - impression of microvessels walls at capillaroscopy were founded in a patient
with chronic heart failure. What of these violations is possible in this case?
A. Sludge-phenomenon
B. Thrombosis
C. Embolism
D. Arterial hyperemia
E. Venous hyperemia

2. While playing volleyball a sportsman jumped and then landed across the external edge of his foot. This
caused acute pain in the talocrural articulation, active movements became limited, passive movements
remained unlimited but painful. In the region of the external ankle a swelling appeared, the skin turned red
and became warmer to the touch. What type of peripheral circulation disorder has developed in this case?
A Embolism
B Venous hyperaemia
C Arterial hyperaemia
D Thrombosis
E Stasis
3. Postganglionic sympathectomy was made at a patient with obliterating endarteritis. What type of arterial
hyperemia will arise up as a result of the conducted operation?
A. Neuroparalitic
. Neurotonic
. Metabolic
D. Working
. Reactive

4. Woman, 42 age old, with trigeminal nerve neuralgia complain of periodic redness of face and neck right
half, sensation of hot flushes and increase of the skin sensitiveness. Development of what kind of arterial
hyperemia is able to explain these phenomena?
A. Neuroparalitic
B. Neurotonic
. Metabolic
D. Working
. Reactive
5. Nerve which innervate the rabbit right ear was cut. Measuring of ears skin temperature conducted after
operation. Temperature of rabbit ear skin at the side of denervation higher than at the opposite intact side on
1,50. What form of peripheral blood circulation disorders was developed at rabbit?
A. Neuroparalitic arterial hyperemia
B. Neurotonic arterial hyperemia
C. Physiological arterial hyperemia
D. Reactive arterial hyperemia
E. Metabolic arterial hyperemia
6. Student wasnt able to give the right answer on exam. Redness of skin, feeling of heat and uncertainty of
behavior developed at the student. What type of arterial hyperemia will arise up in this case?
A. Neurotonic
B. Neuroparalitic
C. Metabolic
D. Pathological
E. Reactive

7. Desympathisation of the femoral artery in the area of the femoral triangle was made in a patient with
atherosclerosis. What type of arterial hyperemia will arise up as a result of the conducted operation?
A. Neuroparalitic
. Neurotonic
. Metabolic
D. Working
. Reactive

8. Puncture of abdominal cavity for the extraction of fluid was performed to a 45-year-old patient with
diagnosis cirrhosis of liver, ascites. State of unconsciousness suddenly developed in the patient as a result of
decrease in blood pressure after extraction of 5L of fluid. That was considered as a manifestation of brain
blood circulation insufficiency. Which disorder of microcirculation occurs in this case?
A. Ischemia
B. Arterial hyperemia
C. Venous hyperemia
D. Thrombosis
E. Embolism

9. Course of atherosclerosis at a patient, 70 years old, was complicated by thrombosis of lower extremities, a
gangrenous fingers of the left foot. Beginning of thrombosis development is associated with:
A. Transformation of fibrinogen into fibrin
B. Activation prothromboplastin
C. Transformation of prothrombin into thrombin
D. Adhesion, aggregation and agglutination of platelets

E. Reduced synthesis of heparin


10. Thrombocytopenia, reduction of fibrinogen in the blood, appearance of fibrin degradation products,
appearance of petechial hemorrhages are marked at a patients, 43 years, on the base ofseptic. Specify the
cause of these changes:
A. DIC-syndrome
B. Autoimmune thrombocytopenia
C. Hemorrhagic diathesis
D. Violation of platelet formation
E. Exogenous intoxication
11. Injury of sympathetic fibers of the sciatic nerve developed in a patient due to trauma. What kind of
peripheral blood flow disorder takes place in the patient?
A. Neurotonic arterial hyperemia
B. Venous hyperemia
C. Angiospastic ischemia
D. Neuroparalitic arterial hyperemia
E. Obturative ischemia
12. At which from processes does arterial hyperemia take place?
. Arterial hypertension
. Tumor growth
. Inflammation
D. Necrosis
E. Arterial hypotension
13. What disorders of peripheral blood circulation will be observed after cutting of sympathetic (adrenergic)
fibres?
. Ischemia
. Venous hyperemia
. Arterioly spasm
D. Arterial hyperemia
. Stasis
14. At the design of inflammation on mesentery of frog looked after arterial vessels dilation, acceleration of blood
stream under a microscope. What type of hyperemia arised up here?
A. Working
. Postischemic
. Venous
D. Reactive
. Metabolic arterial
15. At the design of inflammation on mesentery of frog looked after arterial hyperemia. What is the main factor of
this disorder of blood circulation pathogenesis?
. Biologically active substances elimination
. Accumulation of potassium at the area of inflammation
. Decreasing of potassium amount at the area of inflammation
D. Increasing of - adrenergic nerves tone
. Vasodilators paralysis
16. A patient, 54 age old, entered to the surgical department with complaints of pain in lower extremities, which
increases at the walking. Objectively: the skin of lower extremities is pale, cold, the pulsation is weak, the
sensitiveness is reduced. Obliterating endarteritis was diagnosed and postganglionic sympathectomy was planned.
What type of arterial hyperemia will arise up as a result of the conducted operation?
A. Neuroparalitic
. Neurotonic
. Metabolic
D. Working
. Reactive
17. A 42-year-old woman, shop assistant by profession, complains of edema of the lower extremities at the end of a
workday. Her legs are cyanotic, their temperature is decreased. Venous hyperemia of lower extremities was
established, which is due to constitutional weakness of the elastic apparatus of veins and the occupation. What is
the major pathogenic factor causing local changes in venous hyperemia?

A. Disorders of metabolism
B. Hypoxia
C. Atrophy
D. Dystrophy
E. Sclerosis
18. A 57-year-old man complains of heart pain that has developed after prolonged negative emotions. An
emergency doctor diagnosed ischemic heart disease manifesting by stenocardia. What kind of ischemia is the most
probable?
A. Compressive
B. Obliterative
C. Angiospastic resulting from deficiency of vasodilators
D. -E. Obturative

19. A patient was admitted to the hospital with diagnosis of acute left-ventricle heart failure. Patients
condition suddenly became worse and edema of lungs developed in him. What kind of disorders of peripheral
blood circulation causes the lungs edema?
A. Arterial hyperemia neurotonic type
B. Arterial hyperemia neuroparalytic type
C. Arterial hyperemia metabolic type
D. Ischemia
E. Venous hyperemia
20. Edema and cyanosis of low extremities appear in a food shop assistant at the end of workday. What is the
main factor of edema development in this patient?
A. Dilatation of resistant vessels
B. Increase of venous pressure
C. Increase of number of functional capillaries
D. Increase of collateral blood flow
E. Increase of tissue drainage
21. A 23-year-old woman had intense psycho-emotional excitement. Hyperemia of face skin, tachycardia and
increase in BP were observed in her. What is the mechanism of redness of her face skin?
A. Congestive venous hyperemia
B. Neuroparalytic arterial hyperemia
C. Post-ischemic arterial hyperemia
D. Neurotonic arterial hyperemia
E. Stasis
22. Destruction of endothelial layer of vessel leads to vasoconstriction. Decreased secretion of endothelialderived factor plays the leading role in this phenomena development. What is this factor?
A. Nitric oxide
B. Adenosine
C. Histamine
D. Bradykinin
E. Adenosine monophosphate
23. Patients arm was put in plaster cast on account of simple fracture of humeral bone. Swelling, cyanosis,
and decrease in temperature of the traumatized arm appear next day. What disorder of peripheral blood flow
do these symptoms testify to?
A. Thrombosis
B. Venous hyperemia
C. Ischemia
D. Embolism
E. Arterial hyperemia
24. The patient which injured the left hand a few days before appealed to the doctor. He complaint of the pain
in the site of damage, limitation of movement of fingers. Objectively: increase of hand volume, hyperemia of
the skin, increase of local skin temperature. What pathological process these signs testify?
A. Thrombosis
B. Tumour
C. Inflammation
D. Embolism

E. Lymphostasis
25. At patient with the ancle joint fracture after the removal of plaster bandage an edema of foot, cyanosis,
decrease of local temperature, increase of organ volume were developed. What type of blood circulation
disorders is observed here?
A. Working hyperemia
B. Arterial hyperemia
C. Venous hyperemia
D. Reactive hyperemia
E. Ischemia
26. Convulsions and loss of consciousness develop in a diver during the emergency raising from the depth.
What is the major pathogenic mechanism in these disorders development?
A. Gas embolism
B. Hypoxia
C. Toxic action of oxygen
D. Toxic action of nitrogen
E. Hypercapnia
27. Acute heart arrest develops in a patient with thrombophlebitis of calf profound veins. What is its reason?
A. Pulmonary thromboembolism
B. Left ventricle hypertrophy
C. Myocardial dystrophy
D. Endocarditis of mitral valve
E. Atherosclerosis
28. Gas embolism developed in a diver who was lifted to the surface very fast. It results from fast change:
A. From increased atmospheric pressure to normal
B. From normal atmospheric pressure to increased
C. From normal atmospheric pressure to decreased
D. From decreased atmospheric pressure to normal
E. -29. Pulmonary embolism developed in a patient with coxal bone fracture. What kind of embolism does this
patient have?
A. Fat
B. Thromboembolism
C. Tissue
D. Gas
E. Air
30. A patient has acute pain in his chest, dyspnea, tachycardia, cyanosis, and decreased BP. Pulmonary
infarction was diagnosed in this patient. Which factor is the most common cause of pulmonary infarction?
A. Embolism by thrombus from veins of lower extremities
B. Congestion in the pulmonary circulation
C. Increase in number of platelets
D. Activation of fibrinolytic system
E. Pneumothorax
31. Instantaneous death of pilots occurs under depressurization of airplane cabin at the altitude of 19 km.
What is the reason for it?
A. Multiple gas embolisms
B. Hemorrhage to the brain
C. Gas embolism of cerebral vessels
D. Bleeding
E. Paralysis of respiratory center
32. Dyspnea, acute pain in the chest, cyanosis, and jugular venous distention suddenly develop in a patient
with thrombophlebitis of lower extremities. What is the most possible disorder of blood circulation developed
in the patient?
A. Thromboembolism of coronary vessels
B. Thromboembolism of mesenteric vessels

C. Thromboembolism of cerebral vessels


D. Thromboembolism of portal vein
E. Thromboembolism of pulmonary artery
33. A pilot experienced aircraft decompression on the altitude 14000 m. What type of embolism could
develop?
A. Gas
B. Tissue
C. Thromboembolia
D. Air
E. Fat
34. The state of a patient, 27 years old, with polytrauma (thorax closed injury, closed fracture of the femoral
bone) in two hours after conducting of skeletal extraction was sharply deteriorated. Death came on the base
of acute pulmonary-cardiac insufficiency. The drops of orange color, which obturate the vessels lumen, were
founded at histological research of lungs and cerebrum blood vessels, coloring by sudan . What
polytrauma complication developed?
A. Thromboembolism
B. Air embolism
C. Bacterial embolism
D. Fatty embolism
E. Gas embolism
1
1
0
1
9
2
8

A
A

2
11

C
D

2
0
2
9

3
1
2
2
1
3
0

A
C
D
A

4
1
3
2
2
3
1

Correct answers
B 5
A 6
D 1
E 1
4
5
A 2
B 2
3
4
A 3
E 3
2
3

A
A
C
A

7
1
6
2
5
3
4

A
A
C

8
1
7
2
6

B
B
A

9
1
8
2
7

D
C
A

*** *** ***

1. Necrotic focus has appeared on burn, swallowing and red skin. What is the main mechanism of necrobyosis
improvement in inflammatory area?
A. Secondary alteration
B. Primary alteration
C. Emigration of leucocytes
D. Dyapedesis of erythrocytes
E. Fibroblasts proliferation
2. Painfulness of tooth and edema of lower part of face at the side of ill tooth are present in a patient with
acute pulpitis. What is leading mechanism of edema development in this case?
A. Increase in production of aldosterone
B. Disturbances of trophic function of nervous system
C. Disorder of neural regulation of water-salt metabolism
D. Disorders of microcirculation at the focus of injury
E. Hypoproteinemia
3. A cook burnt his arm with steam. What substance increased and led to development of redness, edema and
painfulness of affected area of skin?
A. Lysine
B. Histamine
C. Thiamine
D. Galactosamine
E. Glutamine
4. Enlargement and deformation of joints were revealed in a patient with rheumatism. What type of
inflammation underlies these changes?
A. Alterative

B. Proliferative
C. Exudative
D. Fibrinous
E. Hemorrhagic
5. Prevalence of proliferative processes was revealed in a patient with chronic inflammation of skin and
subcutaneous adipose tissue. Which hormone deficiency can lead to this situation?
A. Cortisol
B. Aldosterone
C. Insulin
D. Growth hormone
E. Thyroxin
6. Indicate inflammatory mediators which have to be inhibited for decrease in exudation:
A. Catecholamines
B. Histamine
C. Heparine
D. Thromboxan
E. Interleukine-1
7. Condition of biological active substances (BAS) prevalence over there inhibitors usually occurs in
inflammation. Indicate correct correspondence of BAS to their inhibitor
A. Histamine - carboxypeptidase
B. Catecholamines - cholinesterase
C. Kinins - monoaminooxidase
D. Leukotriens - arylsulfatase
E. Serotonin-protease inhibitor
8. A patient has high body temperature, redness, edema, painfulness on her right forearm. What biological
active substances intensify inflammatory reaction?
A. Vasopressin
B. Prostacyclins
C. Phospholipase D
D. Proteolysis inhibitors
E. Kinins
9. There are edema, redness and soreness in a place of injury. What is the reason for inflammations
symptoms appearance?
A. Sympathetic nervous system activation
B. Monoamine oxidase activation
C. BAS discharged by cells
D. Carboxypeptidase activation
E. Limited proteolysis inhibition
10. What inflammatory mediator is formed due to limited proteolysis of plasma globulins?
A. Histamine
B. Leukotriens
C. Bradykinin
D. Prostaglandins
E. Lymphokines
11. Fever and increase of antibodies and leukocytes have appeared in animal under experimental modeling of
inflammation. What substances conditioned to all these common reactions in inflammation?
A. Leukotriens
B. Interleukins
C. Mineralocorticoids
D. Glucocorticoids
E. Somatomedins

12. Which of following inflammatory mediators are formatted under the influence of lypooxygenases?
A. Leukotriens

B. Prostaglandins E1, E2
C. Prostacyclins
D. Thromboxans
E. Thrombocytes activation factor
13. A male patient, 16, was admitted to the hospital with acute appendicitis. What typical pathological process
is the basis of this disease?
A. Inflammation
B. Hypoxia
C. Fever
D. Tumor
E. Allergy
14. The secondary alteration is caused by:
A. Lysosomal enzymes
B. Kinins
C. Lympokynes
D. Prostaglandins
E. Complement components C3a and C5a
15. Glucocorticoid treatment is usefull in rheumatic arthritis. What is the basal action of glucocoricoids?
A. Histamine decreasing
B. Emigration inhibition
C. Phagocytosis inhibition
D. Prostaglandins synthesis inhibition
E. Cell membranes stabilization
16. It is known that inflammatory mediators are cellular or humoral. What from the following is a humoral
mediator?
A. Kallidine
B. Histamine
C. Serotonin
D. Interleukine-2
E. Hydrogenium peroxide
1
9

A
C

2
10

D
C

3
11

B
B

Correct answers
4
B
5
A
12 A
13 A

6
14

B
A

7
15

D
E

8
16

E
A

*** *** ***

1. A patient sustained trauma of knee joint and posttraumatic hemorrhagic bursitis appeared. After 3 months
passive movements limitation in extend was observed. Limitation was a result of scar formation. What
inflammatory component was a basis of this complication?
A. Secondary alteration
B. Exudation
C. Tissue hyperplasia
D. Proliferation
E. Primary alteration
2. Gout is often associated with increasing and deformation of the joints. What type of inflammation is at the
base of these changes?
A. Proliferative
B. Alternative
C. Exudative
D. Fibrinous
E. Mixed
3. Youth, 17 years old, became ill sharply. The body temperature rose to 38,5 0, the cough, nasal catarrh,
lacrimation, nasal effluent appeared. What kind of inflammation developed at the youth?
A. Purulent
B. Catarrhal
C. Hemorragic

D. Fibrinous
E. Serous
4. The growing of "acute phase" proteins is characteristic for the inflammatory processes. What substances
are the stimulants of their synthesis?
A. Angiotensin II
B. Immunoglobulins
C. Interferons
D. Biogenic amines
E. Interleukin 1
5. A patient with inflammation of a forefinger has acute pain, edema, enlargement of local lymph nodes,
temperature rising to 38,5oC. What factors lead to exudation in the inflammatory focus?
A. Lymph flow augmentation
B. Proliferation
C. Resistive vessels tone increase
D. Increase of volumetric flow rate of blood
E. Increasing in endothelial permeability
6. In a patient with eczema there are 5 typical symptoms of inflammation (Celsius - Halen pentad). Find the
one of them:
A. Cyanosis
B. Jaundice
C. Pigmentation
D. Redness
E. Albinism
7. An 8-year-old child was admitted to the infectious department with fever (up
to 38C) and punctuate bright-red skin rash. The child was diagnosed as having
scarlet fever. Objectively: mucous membrane of pharynx is apparently hyperaemic and edematic, the tonsils
are enlarged and have dull yellowish-grey foci with some black areas. What inflammation is the reason for the
pharynx alterations?
A. Haemorrhagic
B. Serous
C. Purulent necrotic
D. Catarrhal
E. Fibrinous
8. In female patient, 28, abscess opening was performed and fast wound reparation was observed. Reparation
was preceded without score formation. What cells play the main role in proliferation?
A. Eosinophils
B. Neutrophils
C. Fibroblasts
D. Lymphocytes
E. Monocytes
9. In examination of abscess punctate under a microscope different blood cells were revealed. Which of them
appears the first in inflammatory focus?
A. Monocytes
B. Mast cells
C. Eosinophils
D. Neutrophils
E. Lymphocytes
10. It is known, that inflammatory mediators play the main role in inflammation pathogenesis. What is the
histamine action on inflammation?
A. Chemotaxis
B. Blood vessel permeability increasing
C. Thrombocytes aggregation
D. Blood vessels constriction
E. Blood coagulation

11. A patient 6 years old was admitted to the hospital with asphyxia. The membranous coats on mucosa of
fauces were revealed. The coats can be removed easy. What type of inflammation is in patient?
A. Fibrinous
B. Necrotic
C. Pus
D. Cattharal
E. Hemorrhagic
12. There are lots of neutrophils in abdominal pus exudates in a patient with peritonitis. What is the main
function of neutrophils in inflammatory area?
A. Phagocytosis
B. Prostaglandins secretion
C. Degranulation
D. Histamine liberation
E. Local blood flow regulation
13. Keloid cicatrix has formatted in a place of abscess. Name the stage of inflammation, which caused scar
appearing.
A. Proliferation
B. Exudation
C. Primary alteration
D. Secondary alteration
E. Emigration
14. Eyeball inflammation resulted pus in the anterior chamber of the eye (hypopyon). What stage of
inflammation was it?
A. Exudation
B. Proliferation
C. Primary alteration
D. Secondary alteration
E. Emigration
15. In a patient with skin pathological process which has 5 typical symptoms: tumor, rubor, calor, dolor,
functio laesa. The development of such process includes 3 stages: Alteration, .......... , proliferation. What is the
second one?
A. Exudation
B. Regeneration
C. Reparation
D. Fibrosis
E. Petrification
16. The woman, 30 years old, is ill about year, when pain soft-tissues swelling, redness of skin appeared in the
area of joints first time. Previous diagnosis is rheumatoid arthritis. Changing of what component of
connecting tissue protein structure is one of reasons of this disease?
A. Collagen
B. Ovoalbumin
C. Mucin
D. Myosin
E. Troponin
18. At patient, 38 years old, rheumatism is in an active phase. Determination of what blood serum laboratory
index has a diagnostic value at this pathology?
A. Urea
B. Transferrin
C. C-reactive protein
D. Uric acid
E. Creatinine
1
1
0

D
B

2
1
1

A
A

3
1
2

B
A

4
1
3

Correct answers
E 5
E 6
D
A 1
A 1
A
4
5

7
1
6

C
A

8
1
7

C
C

*** *** ***

1. A 25 year old man has spent a long time in the sun under high air humidity. As a result of it his body
temperature rose up to 39oC. What pathological process is it?
A Hyperthermia *
B Infectious fever
C Hypothermia
D Noninfectious fever
E Burn disease
2. After transfusion of 200 ml of blood a patient presented with body temperature rise up to 37,9 oC. Which of
the following substances is the most likely cause of temperature rise?
A Interleukin-1 *
B Interleukin-2
C Tumour necrosis factor
D Interleukin-3
E Interleukin-4
3. On simulation of inflammation of the lower extremity the animal exrerienced raise of the temperature,
increase of amount of antibodies and leucocytes in the blood. What substances caused this general reaction of
the organism on inflammation?
A Interleukin*
B Glucocorticoid
C Mineralcorticoid
D Leucotriens
E Somatomedins
1. At the patients with relapsing fever occurs fever that is characterized by periods of several days of high
temperatures, alternating with periods of normal temperature. This temperature curve is called:
A. Febris recurrens
B. Febris hectica
C. Febris intermittens
D. Febris continua
E. Febris atypica
2. Body temperature at the patient with an infection disease was increased to a day 39,5-40,5 0C and has been
kept at this altitude about an hour and then returned to baseline. What type of temperature curve described
in this case?
A. Remittent
B. Constant
C. Intermittens
D. Hectica
E. Atypical
3. Body temperature at the patient with an infection disease was increased to a day 39,5-40,5 0C and has been
kept at this altitude about an hour and then returned to baseline. At which disease does this type of
temperature curve occur?
A. Malaria
B. Tuberculosis
C. Influenza
D Peritonitis
E. Brutselosis
4. Woman came to hospital with a diagnosis of acute pneumonia. She is ill during last 2 days when the body
temperature rised up to 390C and she fell the weakness, dry cough. Which of the mediators of inflammation
has the properties of endogenous pyrogens?
A. Interleukin 1
B. Thromboxane A2
C. Histamine
D. Serotonin
E. Bradykinin
5. A patient with the pneumonia lungs has an increase of body temperature. What biologically active
substance plays the main role in the origin of this manifestation?
A. Interleukin -

B. Serotonin
C. Bradykinin
D. Histamine
E. leukotriene
6. During the examination of the patient following symptoms were revealed: redness of skin, skin is hot and
dry to touch, heart bit rate is 92 per minute, respiratory rate is 22 per minute, body temperature is 39,2 oC
(102.5 degree Fahrenheit). What is the correlation between heat production and heat loss in described period
of fever?
A. Heat production surpasses heat loss
B. Heat production equals to heat loss
C. Heat production is less than heat loss
D. Decreasing of heat production without changes of heat loss
E. Increasing of heat production without changes of heat loss
7. After transfusion of 200 ml of blood a patient presented with body temperature rise up to 37,9C. Which of
the following substances is the most likely cause of temperature rise?
A. Intcrleukin-4
B. Interleukin-2
C. Tumour necrosis factor
D. Interleukin-3
E. Interleukin-l
8. The body temperature of the patient suddenly rose up to 39 C in the afternoon and after 6 hours returned
to normal. On the second day attack doubled: temperature climbed to 41C and period apyrexia stepped
after 8 hours. Name the type of temperature curve.
A. Intermitting
B. Recurent
C. Hectic
D. Septic
E. Constant
9. The body temperature of a patient rose up to 390C after long finding the sun with high humidity of air.
What pathological process does observe in this case?
A. Hyperthermia.
B. Infectious fever.
C. Hypothermia.
D. Noninfectious fever.
E. Burn disease
10. A patient has felt cold, chills, goose flesh, increase of body temperature. Which else changes
characterize the first period of rapid elevation of body temperature?
A. Tachycardia
B. Equilibration between heat production and heat loss
C. Dilation of skin vessels
D. Decrease of arterial pressure
E. Increase of metabolism on 100-200%
11. Body temperature of patient becomes pyretic. Which substances have to act to neurons of
thermoregulation for fever development?
A. Interferon
B. Kallidinum
C. Prostaglandins
D. Free radicals
E. Leucotriens
12. Fever in a patient develops in following succession of stages:
A. Incrementi; fastigii; decrementi
B. Incrementy; decrementy; fastigii
C. Fastigii; decrementi; incrementi
D. Fastigii; incrementi; decrementi
E. Decrementi; fastigii; incrementi

13. A patient had fever after injection of pyrogenal. His skin has become pale, cold; chill appeared in him,
oxygen consumption increased. How do the processes of thermoregulation change in described period of
fever?
A. Increase of heat production and decrease of heat loss
B. Decrease of heat loss
C. Heat loss is equal heat production
D. Decrease of heat production and increase of heat loss
E. Decrease of heat production
14. A patient suffers from osteomyelitis of maxilla. His body temperature increases to 40 0C and then sharply
decreases to 35.6 0C every day. Which type of fever curve is characterized by these changes?
A. Continua
B. Intermittent
C. Reccurens
D. Atypica
E. Hectica
15. Pallor of the skin, goose flesh and increase of oxygen consumption appeared in the patients skin after
injection of pyrogenal. Which stage of fever is characterized by these changes?
A. Stadium incrementi
B. Stadium fastigii
C. The stage of falling temperature by crisis
D. The stage of falling temperature by lysis
16. Acute increase of body temperature, dyspnea, tachycardia, nausea, convulsions, and loss of conciousness
developed in a worker, working in the thick uniform in summer. What was the most possible reason of
development of those symptoms?
A. Equilibration between heat loss and heat production
B. Decrease of heat production
C. Decrease of heat loss
D. Increase of heat production
E. Increase of heat loss
17. Inclination of the set point of thermoregulation to higher level due to action of IL-1 is in a patient. What is
the name of this typical pathological process?
A. Fever
B. Hyperthermia
C. Hypothermia
D. Inflammation
E. Hypoxia
18. The body temperature of a patient with crupous pneumonia is 39 0C. The difference between the morning
and evening temperature of his body didnt exceed 1 0C during 9 days. Which type of the fever curves was
that?
A. Continua
B. Hectica
C. Intermittent
D. Hyperpyretic
E. Reccurens
19. A patient has fever with following stages: incrementi, fastigii, decrementi. Which disease these features
can characterize?
A. Acute pneumonia
B. Acromegaly
C. Diabetes mellitus
D. Hyperaldosteronism
E. Myocardial hypertrophy
20. The body temperature of a patient with pneumonia was keeping on the level 38.3-38.5 0C all the first week
of disease. Such fever is called:

A. Febrile
B. Hyperpyretic
C. Pyretic
D. Subfebrile
21. Increase of acute phase proteins level in blood such as ceruloplasmin, fibrinogen, C-reactive protein is
typical for development of fever. Indicate the possible mechanism of this phenomenon.
A. Stimulative influence of IL-1 on hepatocytes
B. Destructive action of elevated temperature to the cells of the organism
C. Proliferate action of IL-2 to T-lymphocytes
D. Adaptive reaction of the organism to pyrogen
E. Degranulation of mast cells
22. Body temperature of a patient is 39 0C for several hours (stadium fastigii). Indicate which changes of
physiological functions are the most typical for this stage of fever.
A. Bradycardia
B. Ingibition of phagocytosis
C. Increase of heat loss
D. Increase of heat production
E. Heat production is equal heat loss
23. In a patient with prolonged fever after the course of treatment body temperature begins decreasing. What
is the possible mechanism of temperature decrease?
A. Protective activation of immune system
B. Decrease of heat production due to reducing metabolic rate
C. Decrease of production of pyrogens
D. Increase of resistance of organism to action of the pyrogens
E. Increase of heat loss due to peripheral vasodilatation
24. Animal was injected with pyrogen to reproduce fever. What mechanism starts process of temperature
increasing?
A. Activation of non-shivering thermogenesis
B. Activation of shivering thermogenesis
C. Rise of set point of thermoregulation in hypothalamus
D. Reduction of heat loss
E. Dissociation of oxidation and oxidative phosphrilation in tissues
25. After blood trasfusion patient complaints feeling of heat, rigor, increase of body temperature to +40 0C. Its
known the cause of elevation temperature is secretion of endogenous pyrogens. Which cells produce
endopyrogens?
A. Erythrocytes
B. Platelets
C. Endotheliocytes
D. B-lymphocytes
E. Macrophages
26. Most infectious diseases are characterized by development of fever. It can be explained:
A. Formation of IL-1 during phagocytosis of microorganisms
B. Intoxication of the organism
C. Degranulation of mast cells
D. Activation of T- and B-lymphocytes
E. Processes of exudation
27. A man in light clothes is staying in a room with air temperature +14 0C. Windows and doors are closed.
Which way of heat loss is the most considerable in this case?
A. Evaporation
B. Perspiration
C. Conduction
D. Radiation
E. Convection

28. Sharp increase of the temperature to 38.7 0C was marked in a patient with acute purulent periodontitis.
His body temperature has decreased to normal level after opening the pulp cavity. Which type of fever was in
the patient?
A. Efemeral
B. Septic
C. Recurrent
D. Remittent
E. Continua
29. After being in the room with air temperature 40 0C and humidity 80% a patient has been brought to
hospital in grave condition. He was unconscious; he had tachypnea, tachycardia, and body temperature 41
0
C. Reanimation was failed. The patient has died. What is the most possible direct reason of death in this
case?
A. Paralysis of the breath center
B. Collapse
C. Coagulation of blood and decrease of volume of circulating blood
D. Dehydration
E. Heart failure
30. The temperature of the ambient environment is 38C and relative air humidity is 50%. What ways of heat
emission provide maintaining a constant temperature of the human body?
A. Convection
B. Radiation
C. Evaporation
D. Convection and conduction
E. Heat conduction
1

1
1
2
1

1
2
2
2

1
3
2
3

1
4
2
4

Correct answers
5
A 6
B

1
5
2
5

1
7
2
7

1
8
2
8

1
9
2
9

A
E

1
6
2
6

C
A

1
0
2
0
3
0

E
A
C

*** *** ***


1. There is only one source of water for the body at absolute starvation - a process of organic compounds
oxidation. Which of the following substances in these conditions is the main source of endogenous water?
A. Fats
B. Proteins
C. Carbohydrates
D.Glycoproteins
E. Lipoproteins
2. Negative nitrogen balance, hypoproteinemia, violation of water-solt metabolism combined with normal
function of digestive system were founded at the vegetarians. Name the reason of this state.
A. Monotonous protein diet
B. Monotonous carbohydrate diet
C. Lack of unsaturated fatty acids
D. Deficiency of phospholipids in the food
E. Lack of vitamin E in the food
3. At 10th day of medical starvation the patient suffers from excitation, deep, noisy breathing, blood pressure
dropped to 90/60 mmHg, oligouria, urine with a smell of acetone. Name the reason of this state.
A. Ketosis
B. Non-gas alkalosis
C. Hyperglicemia
D. Hypoglycemia
E. Gas acidosis
4. Esophagus stenosis developed at a patient after a chemical burn. Sharp weight loss arised. In the blood:
erythrocyte.-3,01012/l, Hb-106 g/l, whole [crude] protein - 57 g/l. What type of starvation developed at a
patient?
A. Absolute
B. Albumen
C. Water
D. Incomplete
E. Complete
5. When is duration of starvation multiplied?
. At multiplying the body surface
. At increasing of basic methabolism
. At action of low temperature
D. At hyperfunktion of thyroid gland
. At decreasing of basic metabolism
6. Basic metabolism is increase at starvation. The energy demand of the organism are provided due to the
carbohydrates of reserves. RC=1. Synthesis of albumen is limited. The biosynthesis of amino acids from
ketonic acid and ammonia is reduced. What is the period of starvation?
. Indifference
. Oppression
. Paralysis
D. Terminal
. Excitation
7. Patient, 28 years, middle fatness starves 48 hours. What substances are used by muscular tissues as an
energy source in this case?
A.
Glucose
B.
Amino acid
C.
Keton bodies and fat acids
D.
Laktate
E.
Piruvate
8. At the long starvation hungry edema appear at people. What changes in an organism results it?
A.
Decline of onkotic pressure of blood
B.
Decline of osmotic pressure of blood
C.
Increase of secretion of ADG
D.
Decline of secretion of ADG
E.
Increase of onkotic pressure of blood

9. At the
pressure,
patient?
A.
B.
C.
D.
E.

inspection of woman 50 years, who long time was on a vegetable diet, hypothermia, low blood
muscular weakness, negative nitrous balance were founded. What factor caused such state of
Increased amount of carbohydrates in a ration
An insufficient amount of carbohydrates in a ration
Increased using of liquid
An insufficient amount of fats in a ration
An insufficient amount of albumens in a ration

10. In an experiment in the conditions of absolute starvation water insufficiency is partly compensated by
formation of endogenous water. What from the resulted matters at oxidization give the most account of
water?
A.
Glucose
B.
Fats
C.
Glycerin
D.
Glikogen
E.
Protein
11. A patient presents with twilight vision impairment. Which of the following vitamins should be
administered?
A. Retinol acetate
B. Cyanocobalamin
C. Pyridoxine hydrochloride
D. Nicotinic acid
E. Ascorbic acid
12. After a disease a 16-year-old boy is presenting with decreased function of protein synthesis in the liver as a
result of vitamin K deficiency. This may cause disorder of:
A. Blood coagulation
B. Osmotic blood pressure
C. Erythrocyte sedimentation rate
D. Anticoagulant production
E. Erythropoetin production
13. In patients with the biliary tract obstruction the blood coagulation is inhibited; the patients have frequent
haemorrhages caused by the subnormal assimilation of the following vitamin:
A. A
B. K
C. E
D. D
E. C
14. While determining power inputs of a patient's organism it was established that the respiratory coefficient
equaled 1.0. This means that in the cells of the patient the following substances are mainly oxidized:
A. Proteins and carbohydrates
B. Carbohydrates and fats
C. Proteins
D. Carbohydrates
E. Fats
1
8

A
A

2
9

B
E

3
10

A
A

Correct answers
4
D
5
11
A
12

E
A

6
13

E
B

7
14

A
D

*** *** ***

1. Lung cancer developed at the patient who smoked tobacco for a long time. What of the carcinogens
contained in tobacco smoke and concern to PAH?
A. Benzpiren
B. Dymethylaminoazobenzol
C. - naphthylamin

D. Diethylnitrozamin
E. Ortoaminoazotoluol
2. There is high stage of interaction between lung cancer and tobacco smoking. What chemical carcinogen is
contained in tobacco smog?
A. 3,4-benspyren
B. Orhtoaninotoluol
C. Aphlatoxin
D. Methylcholatren
E. Dyethylnitrosamine
3. Patient, 62 years old, a fireman, complains of general weakness, sudden weight loss, husky voice, shortness
of breathing, dry cough. Laryngeal tumor that germinates vocal cords and epiglottis was discovered at
laryngoscopy. Name the most likely cause of the tumor in this patient:
A. Polycyclic aromatic hydrocarbons
B. Nitrosamines
C. Aromatic amine and amides
D. Retrovirus
E. Ionizing radiation
4. In a patient with metastases of lung carcinoma introduction of cytostatics led to suspension of metastases
growth at first but later metastases resumed spread. What is the most possible mechanism of secondary
growth of metastases?
A. Absence of contact braking
B. Absence of Heifliks limit
C. Rise of genetic heterogeneity of tumor cells
D. Increased glucose consumption by tumour
E. Increased amino acids consumption by tumour
5. A 56-years-old patient, who had contact with diethylnitrozamine at his work place, complains of pain in
right subcostal area, weakness, loss of appetite, and decreased workability. At examination of this patient:
surface of his liver is rough, splenomegaly and ascites are present in him; his body temperature is 37.2 oC; in
his blood analysis ESR is 25 mm/hour, besides neutrophilic leukocytosis, and hypochromic anemia were
found. What disease developed in the patients organism?
A. Cancer of liver
B. Hepatitis
C. Cirrhosis of liver
D. Gallstone disease
E. Dyskinesia of bile ducts
6. Malignant tumor of lung was diagnosed in a patient. What feature of tumour growth testifies its
malignancy?
A. Infiltrative growth
B. Unregulated growth
C. Unlimited growth
D. Expansive growth
E. Appearance from one cell
7. The woman complained to the doctor for changing of voice, appearance of hair on the face, and reduction
of breast. Where would a tumor develop that could lead to these symptoms?
A. Tumor of zona reticulata of adrenal glands
B. Tumor of ovaries
C. Tumor of anterior lobe of pituitary gland
D. Tumor of zona glomerulata of adrenal glands
E. Tumor of zona fasciculata of adrenal glands
8. Following changes can occur in development of tumor:
A. Pheochromocitoma hypotension
B. Insulinoma hypoglycemia
C. Aldosteroma hypohydratation
D. Tumor of zona reticulata of adrenal glands inhibition of sexual growth

E. Tumor of thyroid gland hypothyroidism


9. A male patient, 40, has stenotic (without metastases) esophagus cancer. The following changes were
revealed in that patient: muscular and fat tissue atrophy, brownish color of the skin, thin epidermis, and
cardiac atrophy. Whats the reason of such symptoms?
A. Alimentary cachexy
B. Myasthenia
C. Addisons disease
D. Cancer cachexy
E. Brown induration
10. In 1910 Rhauss managed to cause sarcoma in chickens by cell-free infiltrate inserting. What was the
method of experimental modeling?
A. Induction
B. Explantation
C. Isotransplantation
D. Homotransplantation
E. Heterotransplantation
11. What biological process augmentation is typical for tumor cells?
A. Anaerobic glycolysis
B. Decarboxilation
C. Tissue respiration
D. Lipolysis
E. Gluconeogenesis
12. Erlichs tumor was transplanted to animal. What is the evidence of tumor progression?
A. Unlimited growth
B. Resistance to cytostatics
C. Anaplasia
D. Infiltration
E. Tumor weight increasing
13. Unpainted formation under the jaw was appeared in liquidator of Chernobyls disaster after 12 years of
accident. The size of it has increased till last month. The blood analysis is in norm. What pathological process
is most suspicious in this case?
A. Lymphadenitis
B. Syaloadenitis
C. Abscess
D. Malignant tumor
E. Cyst
14. There is a tumor of tongue in patient. What characteristics of tumor may be considered as malignant?
A. Dysplasia
B. Expansion
C. Infiltration
D. Pasteurs positive effect
E. Mitosis increasing
15. What cell structure is a target for chemical cancerogens?
A. Nuclear DNA
B. Lysosomes
C. Mitochondria
D. Cytoplasmic membrane
E. Ribosomes
16. A patient with lung cancer has been smoking 30 cigarettes per day for 20 years. What the group of
cancerogens is in tobacco smog?
A. Polycyclic carbohydrates
B. Aminoasosubstances
C. Nitrosamines
D. Amines
E. Heterocyclic carbohydrates
17. A patient with urinary bladder cancer was working in coke factory. What substance was the most
probable reason of this pathological condition?
A. Naphtylamine

B. Dichlorethane
C. Vinegar acid
D. Alcohol
E. Pethroleynic aether
18. It is established that tumor tissue receives in 20-25 times less of glucose that intact tissue in equal glucose
amount. What metabolic changings lead to such event?
A. Aerobic glycolysis enhancement
B. Oxydation improvement
C. Normal interaction of these processes
D. Tissue respiration improvement
E. Decreasing of anaerobic glycolysis
19. They got nitrogenous nitrite to experimental animals. A tumor was developed in 80% of animals. What
was the group of cancerogens?
A. Nitrosamines
B. Aminoasosubstances
C. Polycyclic carbohydrates
D. Simple chemical substances
E. Hormones
20. After Chernobyl disaster morbidity of tumors has been increasing. What action of the radiation has been
appearing?
A. Oncogenic
B. Thermal
C. Mutagenic
D. Cytostatics
E. Immunostimulative
1

1
1

1
2

1
3

1
4

***

***

Correct answers
5
A 6
A

1
5

1
7

1
8

1
9

1
6

1
0
2
0

A
A

***

1. Inflamation is characterised by increasing penetration of vessels of microcirculation stream, increasing of


their fluid dynamic blood pressure. Increasing of the osmotic concentration and dispersity of protein
structures present in the intercellular fluid. What kind of edema will appear in this case?
A Mixed
B Hydrodynamic
C Colloid-osmotic
D Lymphogenic
E Membranogenic

2.A patient who suffers from heart failure has enlarged liver, edemata of lower extremities, ascites. What is
the leading mechanism in the development of this edema?
A Hydrodynamic *
B Colloid osmotic
C Lymphogenous
D Membranogenic
E 3. A 56 year old patient suffering from cardiac insufficiency has edema of feet and shins,
edematous skin is pale and cold. What is the leding mechanism of edema pathogenesis?
A Rise of hydrostatic pressure in venules *
B Drop of oncotic pessure in capillaries
C Increase of capillary permeability
D Disorder of lymph outflow
E Positive water balance

4. A 50-year-old patient complains of thirst, drinking of a lot of water, marked polyuria. Blood glucose is 4,8
mmol/L, urine glucose and acetone bodies are absent, urine is colorless, specific gravity is 1,002-1,004. What
is the cause of polyuria?
A Vasopressin insufficiency*
B Hypothyroidism
C Insulin insufficiency
D Aldosteronism
E Thyrotoxicosis
5. A patient with nephrotic syndrome has massive edemata of his face and limbs. What is the leading
pathogenetic mechanism of edemata development?
A Drop of oncotic blood pressure *
B Increase of vascular permeability
C Rise of hydrodynamic blood pressure
D Lymphostasis
E Increase of lymph outflow
6. Hepatitis has led to the development of hepatic failure. Mechanism of edemata formation is activated by
the impairment of the following liver function:
A Protein-synthetic *
B Barrier
C Chologenetic
D Antitoxic
E Glycogen-synthetic
7. A patient was stung by a bee. Examination revealed that his left hand was hot, pink, edematic, there was a
big red blister on the site of sting. What is the leading mechanism of edema development?
A Increased vessel permeability *
B Reduced vessel filling
C Injury of vessels caused by the sting
D Drop of oncotic pressure in tissue
E Drop of osmotic pressure in tissue

8. A patient ill with enteritis accompanied by massive diarrhea has low water rate in the
extracellular space, high water rate inside the cells and low blood osmolarity. What is such disturbance of
water-electrolytic metabolism called?
A Hypo-osmolar hypohydration *
B Hyperosmolar hypohydration
C Osmolar hypohydration
D Hypo-osmolar hyperhydration
E Hyperosmolar hyperhydration

9.A patient was admitted to the infectious department. His symptoms: dry skin, decreased skin turgor, ricewater stool. The patient was diagnosed with cholera. What disorder of water-electrolytic balance is most often
observed in this disease?
A Isoosmotic hypohydration *
B Hyperosmotic hyperhydration
C Hypoosmotic hypohydration
D Hyperosmotic hypohydration
E Hypoosmotic hyperhydration
10. Chronic glomerulonephritis was diagnosed in a 34-year-old patient 3 years ago. Edema has developed in
the last 6 monthes. What caused it?
A Proteinuria *
B Hyperproduction of vasopressin
C Disorder of albuminous kidneys function
D Hyperosmolarity of plasma
E Hyperaldosteronism

1. A 56 year old patient suffering from cardiac insufficiency has edema of feet and shins, edematous skin is
pale and cold. What is the leding mechanism of edema pathogenesis?
A
Rise of hydrostatic pressure in venules
B
Drop of oncotic pessure in capillaries
C
Increase of capillary permeability
D
Disorder of lymph outflow
E
Positive water balance
2. A patient with nephrotic syndrome has massive edemata of his face and limbs. What is the leading
pathogenetic mechanism of edemata development?
A
Drop of oncotic blood pressure
B
Increase of vascular permeability
C
Rise of hydrodynamic blood pressure
D
Lymphostasis
E
Increase of lymph outflow
3. A patient was stung by a bee. Examination revealed that his left hand was hot, pink,
edematic,
there
was a big red blister on the site of sting. What is the leading mechanism of edema development?
A
Increased vessel permeability
B
Reduced vessel filling
C
Injury of vessels caused by the sting
D
Drop of oncotic pressure in tissue
E
Drop of osmotic pressure in tissue
4. A patient ill with enteritis accompanied by massive diarrhea has low water rate in the extracellular space,
high water rate inside the cells and low blood osmolarity. What is such disturbance of water-electrolytic
metabolism called?
A
Hypo-osmolar hypohydration
B
Hyperosmolar hypohydration
C
Osmolar hypohydration
D
Hypo-osmolar hyperhydration
E
Hyperosmolar hyperhydration
5. A patient was admitted to the infectious department. His symptoms: dry skin, decreased skin turgor, ricewater stool. The patient was diagnosed with cholera. What disorder of water-electrolytic balance is most often
observed in this disease?
A
Isoosmotic hypohydration
B
Hyperosmotic hyperhydration
C
Hypoosmotic hypohydration
D
Hyperosmotic hypohydration
E
Hypoosmotic hyperhydration
6. Edema was modeling to the white rat by the injection of adrenalin. What pathogenetic mechanism of
edema development?
A. Oncotic
B. Hydrodynamic
C. Membranogenic
D. Lymphogenic
E. Colloid-osmotic
7. Patient of D., 35 years, complains about permanent thirst, decrease of appetite, headache and convulsion.
He drinks 9 l during a day. Day's diuresis is increased, urine without pathological changes, specific gravity 1005. The reason of development such pathology is the damage of:
A. The epithelium of kidney tubule
B. Adenohypophis
C. Epyphis
D. Hypothalamic nucleus
E. Basal membrane of glomerulus capillaries
8. Inflamation is characterised by increasing penetration of vessels of microcirculation stream, increasing of
their fluid dynamic blood pressure. Increasing of the osmotic concentration and dispersity of protein
structures present in the intercellular fluid. What kind of edema will appear in this case?
A
Mixed

B
C
D
E

Hydrodynamic
Colloid-osmotic
Lymphogenic
Membranogenic

9. Inflammation of a patient's eye was accompanied by accumulation of turbid liquid with high protein at the
bottom of anterior chamber that was called hypopyon. What process underlies the changes under
observation?
A
Disturbance of microcirculation
B
Primary alteration
C
Secondary alteration
D
Proliferation
E
10. A patient who suffers from severe disorder of water-salt metabolism experienced cardiac arrest in
diastole. What is the most probable mechanism of cardiac arrest in diastole?
A
Hyperkaliemia
B
Hypernatremia
C
Organism dehydratation
D
Hypokaliemia
E
Hyponatremia
11. An animal with aortic valve insufficiency got hypertrophy of its left heart ventricle. Some of its parts have
local contractures. What substance accumulated in the myocardiocytes caused these contractures?
A
Calcium
B
Potassium
C
Lactic acid
D
Carbon dioxide
E
Sodium
12. A patient who suffers from heart failure has enlarged liver, edemata of lower extremities,
What is the leading mechanism in the development of this edema?
A
Hydrodynamic
B
Colloid osmotic
C
Lymphogenous
D
Membranogenic
E
-

ascites.

13. Periodic renal colics attackes are observed in the woman with primery hyperparathyroidizm. Ultrasonic
examination revealed small stones in the kidneys. What is the cause of the formation of the stones?
A Hypercalcemia
B Hyperphosphatemia
C Hypercholesterinemia
D Hyperuricemia
E Hyperkalemia
14. Transmural myocardial infarction in the patient was complicated with progressive acute left ventricle
insufficiency. What is the most typical for this state?
A Edema of the lungs
B Edema of the extremities
C Cyanosis
D Ascites
E Arterial hypertension
15. Patient suffered from cirrhosis of liver was intravenous injected 500 ml of 5% glucose solution. What
disturbances of water-salt balance may appear in this patient?
A. Hypoosmolar hyperhydration
B. Hyperosmolar hyperhydration
C. Isoosmolar hyperhydration
D. Hypoosmolar hypohydration
E. There is no dyshydration

16. A person, who has been on vegetable diet for along time, has edemas. What is the main mechanism of
edema development in this case?
A. Hypoaminoacidemia
B. Hypoproteinemia
C. Decrease of quantity of microelements in blood
D. Hypoglycemia
E. Anemia
17. Persons, 32, during 4 years suffers on chronic glomerulonephritis. Edemas are marked on face, lately
edemas appeared in trunk, that glomerulonephritis with a nephrotic syndrome. What pathogenetic factor of
edemas development for this patient?
A. Increase of tissue liquid oncotic pressure
B. Difficulty of lymph outflow
C. Decrease of oncotic blood pressure
D. Increase of hydrodynamic blood pressure in capillaries
E. Increase of capillaries permeability
18. As a result of continuous starvation the glomerular filtration rate has increased by 20%. The most
probable cause of the glomerular filtration alteration under the mentioned conditions is:
A. Increase in the systemic arterial pressure
B. Increase in the permeability of the renal filter
C. Increase of the renal blood flow
D. Decrease in the oncotic pressure of blood plasma
E. Increase of the filtartion quotie
19. After a surgery a 36-year-old woman was given an intravenous injection
concentrated albumin solution. This has induced intensified water movement in the following direction:
A. From the intercellular fluid to the cells
B. From the intercellular fluid to the capillaries
C. No changes of water movement will be observed
D. From the cells to the intercellular fluid
E. From the capillaries to the intercellular fluid

of

20. A patient has osmotic pressure of blood plasma at the rate of 350 mOsmol/l (norm is 300 mOsmol/l). This
will cause hypersecretion of the following hormone:
A Cortisol
B Adrenocorticotyropin
C Vasopressin
D Natriuretic
E Aldosterone
21. Patient, 62 years, during 15 years suffers on insufficiency of mitral valve. Last month the edema of lower
extremities began to arise up. What is the mechanism of edema development?
A. Increase of permeability of capillaries
B. Decrease of oncotic blood pressure
C. Increase of oncotic pressure of tissue liquid
D. Violation of lymphatic vessels
E. Increase of hydrostatical blood pressure
22. A patient with chronic cardiac insufficiency have edema of lower extremities. What is the leading link of
their pathogenesis?
A. Hypothalamo-hypophysar system
B. Renin-angiotensin-aldosteron system
C. Sympato-adrenal system
D. Parasympathetic system
E. Kalikrein-kinin system
23. A person, after the long period of starvation has edema. What is the main mechanism of edema
development in this case?
A. Decrease of oncotic pressure in the blood
B. Increase of oncotic pressure in tissues
C. Increase of hydrostatic pressure of venous blood
D. Decrease of hydrostatic pressure of tissues

E. Decrease of blood circulating volume


24. Pulmonary hypertension and right-ventricle heart insufficiency with ascites and edemas develop in a
patient suffered from pneumosclerosis. What is the main mechanism of edema development in this case?
A. Increase of hydrostatic pressure in veins
B. Increase of oncotic pressure of tissues
C. Decrease of oncotic pressure of blood
D. Reduction of heart stroke volume
E. Increase of vascular permeability
25. Which of followings may cause the isoosmolar hypohydration?
A. Acute blood loss
B. Diarrhea
C. Vomiting
D. Perspiration
E. Hyperventilation
26. A patient has uneasiness in the chest and difficult breathing after physical exertion. Some time later cough
with foamy liquid phlegm appears. Significant cyanosis develops in the patient. What is the leading
mechanism for edema development in this case?
A. Hydrodynamic
B. Colloid
C. Membranogenous
D. Lymphogenic
E. Osmotic
27. Development of toxic edemat is predefined:
A. Hypoproteinemia
B. By the promoted permeability of vessels
C. Hypernatriemia
D. Acidosis
E. By the promoted hydrostatic pressure
28. Patient had trauma, hyperemia of skin is appeared in place of damage, she became hot. Pain, limitation
extremity motions, edema appeared. Name the initiating pathogenetic factor of inflammatory edema?
A. Increase of microvessels permeability
B. Decrease of oncotic blood pressure
C. An increase of blood pressure in capillaries
D. Violation of lymph outflow
E. An increase of osmotic pressure in the region of inflammation
29. A patient with severe nephropathy accompanied by severe oedema syndrome that develops as
complication of bronchiectasis. Laboratory examination of this patient displays abundant proteinuria,
cylinderuria, distinct decrease of protein content in blood serum, hyperlipidemia, hypokalemia, and other
pathological changes. What is the most important link in development of edemas in this patient?
A. Decrease of oncotic pressure of blood
B. Increase of osmotic pressure of interstitial fluid
C. Increase of hydrostatic pressure of blood
D. Blockade of lymphatic drainage
E. Increase of microvessel permeability
30. At complete starvation (with taking water) generalized edemas develop. What is the leading pathogenic
factor in this case?
A. Increase of oncotic pressure of interstitial fluid
B. Decrease of osmotic pressure of blood plasma
C. Decrease of oncotic pressure of blood plasma
D. Increase of osmotic pressure of interstitial fluid
E. Decrease of hydrostatic pressure of interstitial fluid
31. What is the leading factor of edema development in case of nephrotic syndrome?
A. Increase of hydrostatic pressure in capillaries
B. Increase of vascular permeability
C. Hypoalbuminemia
D. Dynamic lymphatic insufficiency
E. Increase of blood volume

32. A patient complain about a muscular weakness, feeling of weight in the epigastral area. At an inspection
found out taxicardia, arterial hypotension, on ECG - lengthening the interval of PQ, decrease of T voltage.
What violation of mineral metabolism causes development of these disorders?
A. Hypernatriemia.
B. Hypokaliemia
C. Hyperkaliemia.
D. Hyponatriemia.
E. Deficit of iron.
33. A patient was admitted to the infectious hospital with complaints about unrestrained vomit. What
violations of water-salt metabolism are there?
A. Isoosmolar dehydration
B. Hyperosmolar dehydration
C. Hypoosmolar hyperhydration
D. Hyperosmolar hyperhydration
E. Hypoosmolar dehydration
34. People found themselves on the island after a catastrophe in the ocean, without fresh water. What form of
water-salt metabolism violations will develop?
A. Hyperosmolar hyperhydration
B. Isoosmolar hypergidration
C. Hypoosmolar hyperhydration
D. Hyperosmolar dehydration
E. Hypoosmolar dehydration
35. Osmotic diuresis develops at diabetes mellitus. What violations of water-electrolyte balance are observed?
A. Isotonic dehydration
B. Hypoosmolar dehydration
C. Hyperosmolar hyperhydration
D. Hyperosmolar dehydration
E. Isoosmolar hyperhydration
36. Patient carried heavy infectious disease, after that appeared the signs of diabetes insipidus. Day's diuresis
was increased to 10l. Afterwards dehydration of organism purchased threatening character. What mechanism
of development?
A. Increase of ultrafiltrate osmolarity
B. Braking of water suction in the intestine
C. Decrease of reabsorbtion of water in kidney
D. Decrease of reabsorbtion of sodium in kidney
E. Decrease of plasma oncotic pressure
37. Patient 43 years, appeared the signs of cardiac activity decompensation with the origin of edema and ascyt
after the sharpening of rheumatic heart disease. The increase of production of what matter may cause those
violation
A. Aldosteron
B. Insulin
C. Cortisol
D. Vasopresin
E. Corticothropin
38. At a examination the edema is discovered in the area of left forearm, which arose up after the bite of bee.
Name the leading pathogenetic mechanism of this edema:
A. Hydrodynamic
B. Oncotic
C. Osmotic
D. Lymphogenic
E. Membranogenous

39. When treating for dehydration by means of salt-poor fluids at the background of sharply reduced
excretory renal function resulted from tubular necrosis the worsening of general condition, confused
consciousness, convulsive readiness, and brain edema with vomiting develop. What kind of water-salt
metabolism disturbances takes place in this case?
A. Hypoosmolar hyperhydration
B. Isoosmolar hyperhydration
C. Hyperoosmolar hyperohydration
D. Hypoosmolar hypohydration
E. Hyperosmolar hypohydration
40. Edemas are developed in a patient with renal disease. High quantity of albumen in the analyses of urine.
What mechanism is it possible to explain the origin of edemas for such patient?
A. By the decrease of lymph oncotic pressure
B. By the decrease of filtration pressure in buds
C. By the decrease of blood plasma oncotic pressure
D. By the decrease of interstitial oncotic pressure
E. By the increase of blood plasma osmotic pressure
41. Person, 64years, complain about the dysphnoe, frequent heart beating, rapid fatigueability. In the evening
edemas appear on legs. What main pathogenetic factor in the development of edemas?
A. Increase of tissue liquid oncotic pressure
B. Violation of lymph outflow
C. Decrease of oncotic blood pressure
D. Increase of hydrodynamic pressure
E. Increase of capillaries permeability
42. Edema of Kvinke (common edema of tissues) developed in the patient with allergy. What pathogenetic
factor is starting in this case?
A. Decrease of hydrostatical pressure in tissues
B. Decrease blood plasma oncotic pressure
C. Increase of permeability of capillaries walls
D. An increase of hydrodynamic blood pressure
E. An increase of osmotic pressure in tissues
43. Considerable part of alimentar starvation cases are accompanied with edema. What is the main
pathogenetic factor of edema in this case?
A. Decrease of tissues hydrostatical pressure
B. Decrease of blood plasma oncotic pressure
C. Increase of blood osmotic pressure
D. Increase of intercellular liquid oncotic pressure
E. Increase of blood hydrodynamic pressure
44. Patient with burn shock next to anaestetic drugs it is vitally necessary to enter antishock solutions in
connection with water-electrolyte metabolism violations. What water-electrolyte metabolism violation most
probably arise up at burn illness?
A. Hypoosmolar of hypohydria
B. Hypoosmolar hyperghydration
C. Isoosmolar dehydration
D. Hyperosmolar dehydration
E. Hyperosmolar hyperhydria
45. Patient, 35 years, carried hepatitis and continued to adopt an alcohol, the signs of liver cirrhosis
developed with ascyt and edemata of lower extremities. What blood composition changes did become deciding
in development of edema?
A. Hypokaliemia
B. Hypoglobulinemia
C. Hypoalbuminemia
D. Hypocholesterinemia
E. Hypoglycemia
46. Patient carried operative interference concerning intestinal impassability, after the signs of considerable
dehydration appeared. What blood ions must be appointed in the first turn for water-salt balance correction?
A. Chlor

B. Cuprum
C. Sodium
D. Calcium
E. Magnesium
47. A woman ate orange, after she has edema of palpebra, lips, neck, and tongue. Before on oranges there
were pouring out on a skin, itch. What pathogenetic mechanism lies in development of edema?
A. An increase of hydrodynamic blood pressure in capillaries
B. Violation of lymph outflow
C. Increase of capillaries permeability
D. Decrease of oncotic blood pressure
E. Increase of tissue fluid oncotic pressure
48. Patient, 35 years, who drinks alcohol often, has strong muscular and cardiac weakness, vomit, diarrhea,
AP-100/60 mm Hg. All those symptoms developed during diuretics treatment. Reason of such state is the
increased selection with urine:
A. Phosphates
B. Na
C.
D. Cl
E. Ca
49. Hypertensive solution of glucose was intravenously entered a patient. It will increased motion of water:
A. From intercellular liquid to the capillaries
B. There were no changes
C. From capillaries to the intercellular liquid
D. From cell to the intercellular liquid
E. From intercellular liquid to the cell
50. Patient has reduced synthesis of vasopresin, that why he has polyuria and dehydration of organism. In
what does consist mechanism of polyuria development of polyuria?
A. Increase of glomerular filtration speed
B. Decrease of water reabsorbtion in tubuls
C. Decrease of Na+ reabsorbtion in tubuls
D. Decrease glucose of reabsorbtion
E. Decrease of protein reabsorbtion in tubuls
1-A
11-A
21-E
31-C
41-D

2-A
12-A
22-B
32-B
42-C

3-A
13-A
23-A
33-E
43-B

4-A
14-A
24-A
34-A
44-C

Correct answers
5-A
6-B
15-A
16-A
25-A
26-A
35-D
36-C
45-C
46-C

7-D
17-C
27-B
37-A
47-C

8-A
18-D
28-E
38-E
48-C

9-A
19-B
29-A
39-A
49-D

10-A
20-C
30-C
40-C
50-B

*** *** ***


1. After taking poor-quality food a patient developed repeated episodes of diarrhea. On the next day he
presented with decreased arterial pressure, tachycardia, extrasystole. Blood pH is 7,18. These abnormalities
were caused by the development of:
A Nongaseous acidosis *
B Gaseous acidosis
C Nongaseous alkalosis
D Gaseous alkalosis
E Metabolic alkalosis
2. A patient with diabetes developed a diabetic coma due to the acid-base imbalance. Specify the kind of this
imbalance:
A Metabolic acidosis *
B Metabolic alkalosis
C Respiratory acidosis
D Gaseous alkalosis
E Non-gaseous alkalosis

3. A newborn child with pylorostenosis has often repeating vomiting accompanied by apathy, weakness,
hypertonicity, sometimes convulsions. What disorder form of acid-base balance is it?
A Nongaseous alkalosis *
B Gaseous alkalosis
C Gaseous acidosis
D Metabolic acidosis
E Excretory acidosis
4. Disorder of the airways passage in small and middle bronchi was revealed in the patient. What disorder of
the acid-base equilibrium can be detected in the blood?
A Respiratory acidosis *
B Metabolic acidosis
C Respiratory alkalosis
D Metabolic alkalosis
E -1. Trauma of the head brain a patient is accompanied by repeated vomiting and shortness of breathing. At an
inspection it is marked: = 7,62; p2 = 40 mm of Hg. What is violation of the acid-basic state at a patient?
A
Ungas alkalosis
B
Gas alkalosis
C
Ungas acidosis
D
Gas acidosis
E
2. Hypoxemia and hypercapnia are educed at the research of blood gas composition at the patient with the
chronic disease of respiratory system on a background of the shortness of breathing, tachicardia and
cyanosys.Because of what violations of the external breathing these changes were arise up?
A
Hypoventilation
B
Hyperventilation
C
Hyperperfusion
D
Hypoperfusion
E
Hyperdiffusion
3. A newborn child with pylorostenosis has often repeating vomiting accompanied by apathy, weakness,
hypertonicity, sometimes convulsions. What disorder form of acid-base balance is it?
A
Nongaseous alkalosis
B
Gaseous alkalosis
C
Gaseous acidosis
D
Metabolic acidosis
E
Excretory acidosis
4. After taking poor-quality food a patient developed repeated episodes of diarrhea. On the next day he
presented with decreased arterial pressure, tachycardia, extrasystole. Blood pH is 7,18. These abnormalities
were caused by the development of:
A
Nongaseous acidosis
B
Gaseous acidosis
C
Nongaseous alkalosis
D
Gaseous alkalosis
E
Metabolic alkalosis
5. Disorder of the airways passage in small and middle bronchi was revealed in the patient. What disorder of
the acid-base equilibrium can be detected in the blood?
A
Respiratory acidosis
B
Metabolic acidosis
C
Respiratory alkalosis
D
Metabolic alkalosis
E
-6. 48 y.o. patient with diabetes mellitus was admitted to the hospital in severe pre-coma state. When
examining of acid-base balance metabolic acidosis was revealed. What is the main possible mechanism of

found changes development?


A. Disorders of O2 using in cells
B. Disorders of buffer systems of blood
C. Decrease of CO2 removing
D. Excretion of alkali elements with urine
E. Formation of products of incomplete oxydation
7. A group of mountain climbers went through the blood analysis at the height of 3000 m. It revealed decrease
of HCO3_ to 15 micromole/l (standard is 22-26 micromole/l). What is the mechanism of HCO3_ decrease?
A
Hyperventilation
B
Intensification of acidogenesis
C
Hypoventilation
D
Decrease of ammoniogenesis
E
Decrease of bicarbonate reabsorption in kidneys
8. For a patient with respiratory insufficiency of blood is 7,35. Determination of 2 showed the
presence of hypercapnia. At research of of urine the increase of her acidity is marked. What form of
violation of the acid-basic state in this case?
A
Compensated gas acidosis
B
Compensated metabolic acidosis
C
Decompensated metabolic acidosis
D
Compensated gas alkalosis
E
Decompensated gas alkalosis
9. Toxicosis with the heavy repeated vomiting for a day long developed at a pregnant woman. After twentyfour hours tetaniform convulsions and dehydration of organism began to show up. What change of pH
caused the described changes?
A
Excretory alkalosis
B
Gas alkalosis
C
Gas acidosis
D
Metabolic acidosis
E
Secretory acidosis
10. In a patient with diabetes mellitus metabolic acidosis developed as a result of accumulation of keton
bodies. Arterial blood pH is:
A. 7,40
B. 7,32
C. 7,48
D. 7,56
E. 7,66
11. Patient of D., 48 y.o., which suffers from obesity, conducted the course of medical starvation in home
terms. The deep, noisy breathing appeared on 10 day, arterial pressure went down to 90/60 mm Hg, a
selection of urine diminished, urine with the smell of acetone. What is conditioned described?
A. Non gas alkalosis
B. Hyperglycemia
C. Ketosis
D. Hypoglycemia
E. Gas acidosis
12. A 65-years-old patient with multiple fractures of ribs was admitted to the hospital. What type of acid-base
balance disorder may develop in him?
A. Gas acidosis
B. Gas alkalosis
C. Non-gas acidosis
D. Non-gas alkalosis
E. There are no disorders of acid-base balance
13. A pregnant woman has toxicosis, which accompanied by prolonged vomiting. Data of her biochemical
analyses show: pH of blood 7.38, pCO2 of arterial blood 46 mmHg, SB 17 mmol/L, BE (+ 6 mmol/L).
What type of acid-base balance disorder takes place in this case?
A. Compensated non-gas alkalosis
B. Decompensated non-gas alkalosis

C. Compensated non-gas acidosis


D. Decompensated non-gas acidosis
E. Compensated gas alkalosis
14. A 58-years-old female patient was admitted to the hospital in severe state. Data of her biochemical
analyses show: pH of blood 7.33, pCO 2 of arterial blood 36 mmHg, SB 17 mmol/L, BE ( - 6 mmol/L).
What type of acid-base balance disorder takes place in this case?
A. Compensated non-gas alkalosis
B. Decompensated non-gas alkalosis
C. Compensated non-gas acidosis
D. Decompensated non-gas acidosis
E. Compensated gas alkalosis
15. What is the reason for gas alkalosis?
A. Pulmonary hyperventilation
B. Loss of gastric juice
C. Loss of intestine juice
D. Pulmonary hypoventilation
E. Hyperaldosteronism
16. What kind of acid-base balance disturbances may be observed in case of diabetes mellitus?
A. Non-gas acidosis
B. Gas acidosis
C. Gas alkalosis
D. Non-gas alkalosis
E. Excretory acidosis

17. At examination of patient following were found: hyperglycemia, ketonuria, polyuria, hyperstenuria, and
glucosuria. What kind of acid-base balance disturbances occurs in this case?
A. Gas alkalosis
B. Non-gas alkalosis
C. Metabolic alkalosis
D. Metabolic acidosis
E. Gas alkalosis
18. At getting up in mountains euphoria, head pain, dizziness, heart beating, dyspnoe, which was alternated
from apnoe, developed for an alpinist. What did violation of the acid-basic state develop for an alpinist?
A. Metabolic alkalosis
B. Gas alkalosis
C. Non gas alkalosis
D. Gas acidosis
E. Non gas acidosis
19. Respiratory alkalosis developed in group of alpinists during ascending to on Everest. Thus pressure of
2 in arterial blood:
A. 40 mm Hg
B. 50 mm Hg
C. 60 mm Hg
D. 70 mm Hg
E. 30 mm Hg
20. A patient suffered from diabetes mellitus was admitted to the hospital because of worsening of his
condition. He has general malaise, polyuria, lethargy, and sleepiness. Kussmaul respiration, heart
arrhythmia, and acetone scent in expired air are noticed in this patient. What kind of shift of acid-base
balance contributes these symptoms?
A. Gas alkalosis
B. Gas acidosis
C. Non-gas metabolic alkalosis
D. Non-gas metabolic acidosis
E. Non-gas excretory alkalosis

21. What violation of the acid-basic state does take place at blood pH 7,48?
A. Alkalosis decompensate
B. Alkalosis compensated
C. Metabolic decompensated acidosis
D. Acidosis compensated
E. Acidosis excretory decompensated
22. What acid basic balance disorder can develop in 2 hours after repeated vomiting?
A. Excretory acidosis
B. Excretory alkalosis
C. Eczogenic alkalosis
D. Metabolic acidosis
E. Eczogenic acidosis
23. Patient, who suffers from severe diarrhea, was admitted to the hospital with consequences disorder,
Kussmaul respiration. Blood pH 7,30, deficiency of base. Acidic reaction of urine, contain many phosphates
and ammonium salts. What is acid-basic balance disorder develop in this case?
A. Non-gas alkalosis
B. Non-gas acidosis
C. Excretory alkalosis
D. Gas alkalosis
E. Gas acidosis
24. Repeated vomiting occurs in patient suffered from pylorostenosis which is bring to severe state. Appeared
appation, weakness, increased muscular tonus, cramps. What is acid-basic balance disorder develop in this
case?
A. Non-gas alkalosis
B. Non-gas acidosis
C. Excretory alkalosis
D. Gas alkalosis
E. Gas
25. In the case of development of mountain disease compensatory lung hyperventilation is developed. What is
acid-basic balance disorder develop in this case?
A. Non-gas alkalosis
B. Non-gas acidosis
C. Excretory alkalosis
D. Gas alkalosis
E.Gas acidosis
26. During bronchial asthma attack patient developed gas acidosis (hypercapnia). What type of acid-base
balance disorder takes place in this case?
A. Gas alkalosis
B. Gas acidosis
C. Non-gas metabolic alkalosis
D. Non-gas metabolic acidosis
E. Non-gas excretory alkalosis

27. Patient was appointed solution of glucose intravenously with potassium bicarbonat. Such indexes are
determined: pH- 7,43; 2 - 61 mm Hg; SB - 31,5 mecv/l; BB - 59 mecv/l; BE +8,5 mecv/l . What ABB
violation does take place in this case?
A. Non gas alkalosis
B. Non gas acidosis
C. Excretory alkalosis
D. Gas acidosis
E. Gas alkalosis
28. Patient had such laboratory parameters pH-7,32, pCO2 35 mm Hg, SB- 16,5mecv/l, BB 35,0 mecv/l,
BE 9,0 mecv/l, TK day urine - 8,0 mecv/day. What is acid-basic balance disorder develop in this case?

A. Non-gas alkalosis
B. Non-gas acidosis
C. Excretory alkalosis
D. Gas alkalosis
E. Gas acidosis
29. Patient get head trauma, which accompanied with repeated vomiting and dyspnoe. Laboratory tests: pH
-7,48, pCO2 30 mm Hg, SB 27 mecv/l, BB- 50,0 mecv/l., BE -+ 3,0 mecv/l. What is acid-basic balance
disorder develop in this case?
A. Non-gas alkalosis
B. Non-gas acidosis
C. Excretory alkalosis
D. Gas alkalosis
E. Gas acidosis
30. For a patient with nefrotic syndrome polyuria observed, hyponatriemia (patient gets diacarb). Patient had
such laboratory parameters: pH - 7,30; 2 - 36 mm Hg;SB- 17,0 mecv/l; BB - 42 mecv/l; BE - 8,0 mecv/l.
What ABB violation does take place in this case?
A. Non gas alkalosis
B. Non gas acidosis
C. Excretory alkalosis
D. Gas acidosis
E Gas alkalosis
31. Patient in comatose state. Laboratory tests: keton bodies 58 mg%, TK day urine- 40 mecv/day, pH
7,30, p CO2 50 mmHg, SB 15,5 mecv/l, BB 38,0 mecv/l, BE 13,0 mecv/l. What is acid-basic balance
disorder developing in this case?
A. Non-gas alkalosis
B. Non-gas acidosis
C. Excretory alkalosis
D. Gas alkalosis
E. Gas acidosis
32. Patient suffers from severe toxic damage of kidney with anuria. Laboratory tests: lactic acid 20 mg%,
pH -7,25, p CO2 - 47 mecv/l, SB -18,5 mecv/l, BB 40,5 mecv/l, BE -70 mecv/l. What is acid-basic balance
disorder develop in this case?
A. Non-gas alkalosis
B. Non-gas acidosis
C. Excretory alkalosis
D. Gas alkalosis
E. Gas acidosis
33. An operation is conducted with artificial ventilation of lungs. Patient had such laboratory parameters: pH
-7,47; 2-75 mm Hg; SB-27,0 mecv/l; BB-49,0 mecv/l; +3,5 mecv/l. What ABB violation does take place
in this case?
A. Non-gas alkalosis
B. Non-gas acidosis
C. Excretory alkalosis
D. Gas alkalosis
E. Gas acidosis
34. An operation is conducted with artificial ventilation of lungs. Patient had such laboratory parameters: pH
-7,32; 2-25 mm Hg; SB-20 mecv/l; BB-40 mecv/l; +3,0 mecv/l. What ABB violation does take place in
this case?
A. Non-gas alkalosis
B. Non-gas acidosis
C. Excretory alkalosis
D. Gas alkalosis
E. Gas acidosis
35. Patient suffers from sharp insuffiency of left ventricle of heart. Edema of lung developed. Patient had
such laboratory parameters: pH - 7,32; 2 - 51 mm Hg; SB - 18,0 mecv/l; BB - 45 mecv/l; BE +8.0 mecv/l.
What ABB violation does take place in this case?
A. Non-gas alkalosis

B. Non-gas acidosis
C. Excretory alkalosis
D. Gas alkalosis
E. Gas acidosis
36. For a patient with diabetes mellitus hyperglicemia, ketonuria, glycosuria is marked, hyperstenuria and
polyuria. What form of ABB violation does take place in this situation?
A. Gas acidocic
B. Metabolic acidosis
C. Gas alkalosis
D. Non gas alkalosis
E. Excretory alkalosis
1-A
10-B
19-E
28-B

2-A
11-C
20-D
29-C

3-A
12-A
21-A
30-B

4-A
13-A
22-B
31-B

Correct answers
5-A
6-E
14-D
15-A
23-B
24-C
32-B
33-E

7-A
16-A
25-D
34-D

8-A
17-D
26-B
35-B

9-A
18-B
27-A
36-B

*** *** ***


1. A patient with the symptoms of acute alcoholic poisoning was brought to the hospital. What carbohydrates metabolism
changes are typical for this condition?
A The gluconeogenesis velocity in liver is decreased
B The gluconeogenesis is increased in liver
C The breakage of glycogen is increased in liver
D The anaerobic glucose metabolism predominates in muscles
E The anaerobic breakage of glucose is increased in muscles

1. A patient, 24, suffers from dryness in a mouth, decline of waight on the base of the increased appetite,
increase of urine amount. What test for the diagnosis is it necessary to make first of all?
A
Determination of glucose level in the day's amount of urine
B
Analysis of urine according to Zimnitskyy
C
General analysis of urine
D
Research of albuminous factions of the blood serum
E
Coagulograme
2. Apatient with a diabetes mellitus delivered in a hospital in the state of unconsciousness. Kussmauls
breathing, blood pressure 80/50 mmHg, the smell of acetone from a mouth. By accumulation of what
substances in the organism it is possible to explain the origin of these disorders?
A
Keton bodies
B
Modified lipoproteids
C
Lactic acid
D
Coal acid
E
Complete carbohydrates
3. Glucose was founded in the purine of patient who suffers from polyuria and polydipsia. Content of dlucose
in the blood plasma is normal. What is the mechanism of glucosuria development for a patient?
A Violation of glucose reabsorption in the tubulis.
B Violation of glucose filtration in the glomerulis.
C Insufficient production of insulin by a pancreas.
D Resistance of cell receptors to insulin.
E Hyperproduction of glucocorticoids by the adrenal gland.
4. A woman 26 years suffers from dryness in to the mouth, thirst. At an inspection: a level of glucose in the
blood is 6,5 mmol/l, glucosuria. For what from the marked below states these symptoms are most
characteristic for?
A Kidney diabetes
B Steroid diabetes
C Diabetes insipidus
D Alimentary glucosuria
E Diabetes mellitus
5. Insulin was entered to a patient with insulindepended diabetes mellitus. After some time general weakness,
irritability, increase of sweating arose up. What basic mechanism of of hypoglycemic comma development?

A
B
C
D
E

Carbohydrate starvation of brain.


Intensification of glycogenolysis.
Intensification of ketogenesis.
Intensification of lipogenesis.
Diminishing of glyconeogenesis.

6. A 12-year-old teenager has significantly put off weight within 3 months; glucose concentration rose up to 50
millimole\l. He fell into a coma. What is the main mechanism of its development?
A Hyperosmolar
B Hypoglycemic
C Ketonemic
D Lactacidemic
E Hypoxic
7. A child 12 years after the carried rubella had loss of weight, permanent feeling of thirst, increase of
appetite, polyuria. Objectively: diuresis 6 l, glucose of blood - 17,8 mmol/L, in urine found out glucose and
acetone. What pathology can be assumed for a patient?
A. Diabetes mellitus type
B. Diabetic nephropathy
C. Diabetes mellitus type
D. Violation of tolerance to glucose
E. Diabetes insipidus
8. Patient with diabetes didn't get insulin injection in time that caused hyperglycemic coma
in the blood 50mmol/L). What mechanism is prevalent in the development of the coma?
A
Hyperosmia
B
Hypokaliemia
C
Hypoxia
D
Hyponatremia
E
Acidosis

(glucose

9. A woman, aged 58, was admitted to the hospital in severe condition. She has confused consciousness; dry
skin, hollow eyes, cyanosis, and scent of acetone from her mouth. At laboratory examination of her: glucose in
blood 15.2 mmol/L, ceton bodies 100 mcmol/l. What is the most possible reason for this condition?
A. Hypoglycemic coma
B. Ketoacidotic coma
C. Hypovolemic coma
D. Uremic coma
E. Anaphylactic shock
10. A patient was admitted to the hospital. Objectively: the state is heavy, consciousness is absent, adynamia.
Skin is dry, hollow eyes, cyanosis, taxicardia, smell of acetone from a mouth. Results of analyses: glucose of
blood -20,1 mmol/L (normal -3,3-5,5 mmol/L), in urine -3,5% (normal - 0). What the most reliable diagnosis?
A. Sharp alcoholic poisoning
B. Hypoglycemic coma
C. naphylactic shock
D.Hyperglycemic coma
E. Cardiac insufficiency
11. A patient suffering from pheochromocytoma complains of thirst, dry mouth, hunger. Blood test for sugar
revealed hyperglycemia. What type of hyperglycemia is it?
A
Adrenal
B
Hypercorticoid
C
Alimentary
D
Somatotropic
E
Hypoinsulinemic
12. Patient has diabetes mellitus. Hyperglycemia over 7,2 mmol/L. The level of what protein of plasma does
allow retrospectively (previous 4-8 weeks to an inspection) to estimate the level of glycemia?
. Ceruloplasmin
. Glycated hemoglobin
. Fibrinogen
D. -reactive protein

E. Albumen
13. In a patient with diabetes mellitus loss of consciousness and cramps were observed after insulin injection.
What is a possible result of blood test for glucose in this case?
A. 5.5 mmol/L
B. 3.3 mmol/L
C. 8.0 mmol/L
D. 10 mmol/L
E. 1.5 mmol/L
14. At diabetes mellitus can develop ketosis as a result of activation of lipid acids oxygenation. What violations
of ABB can bring accumulation of keton bodies in blood?
. Respiratory acidosis
. Respiratory alkalosis
. Exogenous acidosis
D. Metabolic acidosis
. Excretory acidosis
15. Content of glucose in patients blood is: on an empty stomach - 5,6 mmol/l, in 1 hour after taking sugar 13,8 mmol/l, and in 3 hours -9,2 mmol/l. What pathology such indexes characterise?
. Healthy person
. cromegaly
. Ichenko- Kushinga disease
D. Diabetes mellitus
. Thyreotoxycosis
16. Content of glucose in patients blood is: on an empty stomach 5.65 mmol/l, in one hour after taking
sugar 8.55 mmol/l, and in two hours after taking sugar 4.95 mmol/l. These signs are characteristic for:
A. Healthy person
B. Person suffered from hidden diabetes mellitus
C. Person suffered from non-insulin-dependent diabetes mellitus
D. Person suffered from insulin-dependent diabetes mellitus
E. Person suffered from thyrotoxicosis
17. Newborn has been refusing food, having vomiting and diarrhea, and some time later its crystalline lens
become opaque. At examination of newborn: glucose in blood 8.5 mmol/L and in urine 1%. What is the
most possible diagnosis?
A. Galactosemia
B. Phenylketonuria
C. Tyrosinosis
D. Cystinuria
E. Alkaptonuria
18. Introduction of adrenalin in the experiment to the animal causes the increase of glucose in blood. A result
of what hormone action is this hyperglycemia?
A. Increase of absorption of glucose in intestine
B. Decrease of glicogenesis
C. Activation of gluconeogenesis
D. Activation of glycogenolysis in the liver
E. Decrease of hexokinase activation
19. At the inspection of patient glycemia -16 mmol/L, acetone bodies - 0,52 mmol/L, at urine - 2% glucose,
acetone bodies + + +, diures 10 l in day. About what disease it is possible to think?
A. Illness of Itsenko-Kuching
B. Steroid diabetes
C. Kidney diabetes
D. Diabetes mellitus
E. Diabetes insipidus
20. During examination the increase of glucose level is marked in students blood. What mechanism linked in
this case?
A. Increase of products of glucagons
B. Increase of products of STH
C. Increase of insulin production by pancreas

D. Activating of the sympato-adrenal system


E. Increase of tonus of the parasympathic nervous system
21. To the patient with diabetes mellitus with the purpose of hyperglycemic effect appointed insulin. What
mechanism of insulin action?
A.
Warning of suction of glucose in a thin intestine
B.
Activation of gluconeogenesis
C.
Increasing of selection of endogenous insulin by pancreas
D.
By the assistance of penetration of glucose to the cell and its utilization
E.
Activation of anaerobic glicolis
22. A patient was admitted to the hospital in comatose state with the smell of acetone from mouth. Level of
glucose in plasma of blood - 18 mmol/L. What type of coma is the most possible in this patient?
A.
Ketoacidotic
B.
Toxic
C.
Hyperosmolar coma
D.
Lactic acidosis
E.
Hypoglycemic coma
23. Patient has hyperglycemia, polyuria, hyperstenuria, ketonuria, glucosuria. What nosology can connect all
these indexes?
A.
Diabetes mellitus
B.
Diencephalic obesity
C.
Diabetes insipidus
D.
Glikogenosis
E.
Kidney diabetes

24. A patient suffers from frequent head aiche, somnolence, crabbiness. At laboratory tests the level of glucose
of blood is on an empty stomach - 2,8 mmol/L, after taking sugar the maximal concentration of glucose 4,7
mmol/L. The contents of insulin in blood in 10 times exceeds normal indexes, that characteristically for
insulinoma. Activity of which processes is most promoted at this pathology?
A.
Lipolis
B.
Suction of glucose in an intestine
C.
Absorption of glucose by cell
D.
Synthesis of glicogenes
E.
Litogenesis
25. A man, aged 38, is under the course of treatment for schizophrenia at in-patient department. Contents of
glucose, ketone bodies, and urea in his blood are normal. Shock therapy with regular injections of insulin has
led to development of insular coma, and after that state of patient becomes better. What is the most possible
reason for insulin coma?
A. Glucosuria
B. Dehydration of tissues
C. Hypoglycemia
D. Metabolic acidosis
E. Ketonemia
26. Man, 28 years, suffers diabetes mellitus from 6 years age, he hospitalized in the comatose state which
arose up after the physical loading. Skin moisture, tone of muscles and reflexes of tendons promoted,
breathing is superficial, taxicardia, low blood pressure, level of glucose in blood - 1,88 mmol/L. This
symptoms most characteristic for?
A.
Diabetic ketoacidosis
B.
Lactic acidosis
C.
Hypoglycemic coma
D.
Collapse
E.
Hyperosmolar coma
27. Man, 43years, delivered to the hospital in the comatose state. Objectively: a skin is dry, breathing is
superficial frequent, the smell of acetone is absent, taxicardia, hypotension. Level of glucose in blood - 48
mmol/L, the reaction of urine on the acetone is negative. These symptoms most characteristic for?

A.
B.
C.
D.
E.

Hypertensive cris
Ketoacidotic coma
Lactic coma
Collapse
Hyperosmolar coma

28. Person, 42, was admitted to the hospital in the comatose state. Objectively: a skin is dry, pale, smell of
acetone from the mouth, Kussmaul respiration, taxicardia, low blood pressure, level of glucose in blood - 27,7
mmol/L. What kind of coma has developed in this patient?
A.
Starvation
B.
Ketoacidotic coma
C.
Hyperosmolar coma
D.
Lactic coma
E.
Alcoholic intoxication
29. Woman, 45 years, the symptoms of diabetes are absent, but the level of glucose on an empty stomach
determined in blood (7,5 mmol/L). What index must be conducted for clarification of presence of diabetes?
A.
Remaining nitrogen of blood
B.
Glucose of blood on an empty stomach
C.
Acetone bodies in urine, blood
D.
Tolerance to glucose
E.
Glycated hemoglobin
30. Patient with the Itsenko-Cushing syndrome has hyperglycemia because of hyperprodaction of
glycocorticoids. What leading mechanism of hyperglycemia in this case?
A.
Decrease of activity of hexokinas
B.
Activating of glycogenolisis in liver
C.
Activating of glyconeogenesis in liver
D.
Strengthening of absirbtion of glucose in an intestine
E. Activating of insulinase in liver
31. A patient with diabetes mellitus did not take the dose of insulin in time. Thus hyperglycemic coma
developed (content of glucose in patients blood is 50 mmol/L). What is the leading factor for such coma
development?
A. Hyperosmolarity of blood plasma
B. Hypokalemia
C. Hypoxia
D. Hyponatremia
E. Acidosis
32. On the second week after gastrectomia for a patient in a hour after taking sweat food on the empty
stomach develop sharp hypoglycemic state. What phenomenon does lie in basis of this state?
A.
Alimentar hypoglycemia
B.
Glikogenos
C.
Phermentopatia
D.
Adrenal insufficiency
E.
Dumping-syndrome
33. During the experiment rat was injected with 5% alloxan solution in dose 200 mg per kg of rat weight.
What kind of pathology arises in this case?
A. Arterial hypertension
B. Acute renal failure
C. Diabetes insipidus
D. Diabetes mellitus
E. Hepatic failure
34. After the break of diet (taking the easy for assimilation carbohydrates) in a woman, who has been
suffering from diabetes mellitus for a long time, general malaise and increase of blood pressure gradually
develop and hallucinations and cramps appear. Woman has dry skin and distinct signs of dehydration. What
is the reason for worsening of patients condition?
A. Hypoglycemic coma

B. Hyperosmolar hyperglycemic coma


C. Diabetic ketoacidosis
D. Heart failure
E. Respiratory failure
35. What is the reason for glucosuria appearance under diabetes mellitus?
A. Hyperglycemia
B. Increase of renal threshold
C. Rise of hexokinase activity
D. Increase of glomerular membrane permeability
E. Polyuria
36. A patient, aged 50, complains of increased appetite, thirst, and loss of body weight, weakness. At
laboratory examination rise of amount of glucose in his blood revealed. What type of cells is injured in case of
this disease development?
A. Lipotropocytes
B. Thyrocytes
C. B-cells of Langerhans islets
D. A-cells of Langerhans islets
E. Pancreatocytes
37. A patient address to a doctor with complaints of constant thirst. Hyperglycemia, polyuria and increased
content of 17-ketosteroids in urine were revealed. What is the most probable disease in this case?
A. Addisons disease
B. Mychedema
C. Glycogenosis of I type
D. Insulin dependent diabetes mellitus
E. Steroid diabetes
38. A patient was admitted to the hospital in comatose state. Accompanying people said that hi lost
consciousness at training while he was finishing the Marathon distance. What sort of coma is the most
possible in this patient?
A. Hyperglycemic
B. Hypoglycemic
C. Hypothyroid
D. Hepatic
E. Diabetic ketoacidosis
39. In 18-years-old patient, while laboratory examining presence of glucose in urine and normal
concentration of glucose in blood plasma were revealed. What disorder is the most possible cause of these
changes?
A. Disorders of glomerular filtration
B. Disorders of tubular secretion
C. Disorders of glucocorticoids secretion
D. Disorders of insulin secretion
E. Disorders of tubular reabsorption
40. A 19-years-old patient has suffered from diabetes mellitus since he was 8. He took cure irregularly. He was
admitted to the hospital in connection to diabetes ketoacidosis development. What kind of respiration is the
most possible in this condition?
A. Kussmaul respiration
B. Biots respiration
C. Chane-Stocks respiration
D. Inspiratory breathlessness
E. Expiratory breathlessness
41. In a patient, who has poisoning with flodzine, glucosuria was revealed; concentration of glucose in
patients blood is 5.6 mmol/L. What is the mechanism of glucosuria in this case?
A. Disturbances of tubular reabsorption
B. Reduction of glomerular filtration
C. Increase of glomerular filtration
D. Disorders of tubular secretion
E. Rise of oncotic pressure of blood

42. Woman, 45 years disease of Kuchinga - steroid diabetes. At a biochemical inspection: hy perglycemia,
hypochloremia. What processes is activated first of all?
A. Glicogenolis
B. ransportation of glucose in a mew
C. Glicolis
D. Gluconeogenesis
E. Reabsorption of glucose
43. A patient suffers from neurodermitis for a long time, use prednisolon. During examination increase of
glucose in blood was found. In what link of carbohydrate metabolism prednisolon influence ?
A. Increase of suction of glucose in an intestine
B. Activation of gluconeogenesis
C. Decrease of synthesis of glicogen
D. Activating of breaking up of insulin
E. Activating of glycogenogenesis
44. Patient with diabetes mellitus. In blood increase of level of free lipid acids. Reason of what it can be:
A. Increase of thriglyceridlipase of adipocytes activity
B. Accumulation in cytosolum palmitin-KoA
C. Activating of apolipoprotein -1, -2, -4synthesis
D. Decrease of activity of phosphatidilcholin cholesterin - acetyltransferase of plasma
E. Activating of utilization of keton bodies

*** *** ***


1. A 58-year-old patient suffers from the cerebral atherosclerosis. Examination revealed hyperlipoidemia.
What class of lipoproteins will most probably show increase in concentration in this patients blood serum?
A.Low-density lipoproteins
B.High-density lipoproteins
C.Fatty acid complexes with albumins
D.Chylomicrons
E.Cholesterol
2. A patient who had been working hard under conditions of elevated temperature of the environment, has
now a changed quantity of blood plasma proteins. What penomenon is the case?
A.Relative hyperproteinemia
B.Absolute hyperproteinemia
C.Absolute hypoproteinemia
D.Disproteinemia
E.Paraproteinemia
3. A patient being treated for viral hepatitis type B got symptoms of hepatic insufficiency. What blood changes
indicative of protein metabolism disorder will be observed in this case?
A.Absolute hypoalbuminemia
B.Absolute hyperalbuminemia
C.Absolute hyperfibrinogenemia
D.Proteinic blood composition is unchanged
E.Absolute hyperglobulinemia
4. A 1,5-year-old child presents with both mental and physical lag, decolorizing of skin and hair, decrease in
catecholamine, concentration in blood. When a few drops of 5% solution of trichloroacetic iron had been
added to the child's urine it turned olive green. Such alteration are typical for the following pathology of the
amino acid metabolism:
A. Tyrosinosis
B. Alkaptonuria
C. Phenylketonuria
D. Albinism
E. Xanthinuria

5. Nappies of a newborn have dark spots being the evidence of homogentisic acid formation. This is caused by
the metabolic disorder of the following substance:
A. Cholesterol
B. Methionine
C.Galactose
D. Tyrosine
E. Tryptophan
6. Patient suffers from pellagra. Disorder of metabolism of what amino acid lie in basis of this disease?
. Thrypthofan
. hyrosin
. Phenylalanine
D. Cystein
E. Valin
7. A child with phenylcetonuria suffers from mental insufficiency. What mechanism will be main in
development of damage of the central nervous system?
. Increase ecscreation with urine of phenylceton bodies
. Increase of thyrosin synthesis
. Decrease of the synthesis of thyreiod hormones
D. Decline of synthesis of melanin
. Accumulation of phenylalanine in the blood
8. Child, 8 months, found disorders in mental and physical development. What amino acid must be excluded
from the meal of child?
. Hystidin
. Leicin
. hyrosin
D. Thrypthofan
. Phenilalanin
9. Colouring of sclera, mucous membranes were founded during examination of infant. Darkening of urine in
air was determined. Homogenthisin acid was founded in the blood and urine. For what state it is
characteristically?
. Cystinuria
. Hystidinemia
. lcaptonuria
D. Albinism
. Galakozemia

10. Albinos do not carry sun influencing- sunburn does not develop, and burns appear. Violation of what
aminoacid lies in a basis of this phenomenon?
. Thrypthofan
. Hystidin
. thionin
D. Glyutamin acid
. Phenylalanine
11. To the woman with pathological obesity for the decline of body mass conducted an operation, as a result
she lost 30 kg That it was remote during an operation.
. Part of stomach
. Gall blader
. Part of pancreas
D. Part of colon
. Part of thin bowel
12. Man has cerebral obesity developed after the carried encephalitis. What is the leading mechanism of this
obesity?
. Decrease of basic metabolism
. Increase converting of glucose into lipids
. Decrease of lipolis

D. Hyperinsulinemia
E. Bulemia, polyphagia
13. Why develops obesity at hyperinsulinism?
. Decrease of pentozophosphatic way
. Proteins convert into lipids
. Activaiting of glycogenesis
D. Glyconeogenes increases
. Carbohydrates converts into lipids
14. A sportsman after stopping of trainings had obesity. What is pathogenetic mechanism?
. Increase of the tonus of the parasympathetic nervous system
. Decrease of the tonus of the sympathetic nervous system
. Insufficiency of mobilizations lipids from a depo
D. Oppressing the center of saturation
. Uncorrespondence of using energy to the amount of food which is
used
15. Child, 10 months (parents has dark hair), has very light skin and blue eyes. During the last 3th months
there were violations of cerebral blood circulation. Reason for such state can be:
. Sharp porphyria
. Hystidinemia
. Glicogenom
D. Phenilcetonuria
. Galakozemia
16. A patient, 60 years, suffered from pain in the small joints of feet and hands. Joints are increased, have the
appearance of the thickened nodes. Maintenance of urats is increased in blood. What metabolism disorders
can be the reason of this state?
A. Pyrimidin
B. Amino acid
C. Purins
D. Lipid
E. Carbohydrates
17. Multiplying maintenance of phenylalanine is discovered in the pregnant women blood. What disorder can
arise up in a child?
A. Development of oligofrenia
B. Development of galaktozemia
C. Birth of child is with the Turner syndrome
D. Development of Vilson disease
E. Birth of child is with the Dawns syndrome
18. Man, 65 years, complaints about acute pain in large fingers of legs. He drinks beer often. There was
suspicion on a gout. Level of what acid is it necessary to define in blood for confirmation of diagnosis?
A. Urinary acid
B. Bilirubin
C. Keton bodies
D. Urea
E. Lactate
19. It is known that phenylcetonuria - is result of gene mutation, responsible for transformation of
phenylalanine and disintegration of it to the finished products of metabolism - 2 and 2. What way of
phenylalanine metabolism will result development of phenylcetonuria?
A. Phenylalanine - thyrosin - thyroxin
B. Phenylalanine - thyroxin - noradrenalin
C. Phenylalanine - thyroxin
D. Phenylalanine - phenylpiruvat - keton acid
E. Phenylalanine - thyrosin - melanin
20. Sick K., 48 years, suffers from obesity, conducted the course of medical starvation at home. The deep
noisy breathing appeared on 10th day, arterial pressure 90/60 mm Hg, a selection of urine diminished, urine
with the smell of acetone. What does this phenomenon testify about?
A. Nongas alcalos

B. Hyperglycemia
C. Ketosis
D. Hypoglycemia
E. Gas acidosis
21. Slow physical and psychical development, pale skin, hairs, iris of eye, positive Felings test were marked at
child 6 months. What inherited disease found out at a child?
A. Galaktozemia
B. Alcaptonuria
C. Illness of Dauna
D. Albinism
E. Phenylcetonuria
22. Patient suffers from alcaptonuria. Deficiency of what enzyme was present in this case?
A. Thyroxinhydroxylase
B. Oxydase of homogenthisin acid
C. Phenylalaninhydroxylase
D. Thyrosinase
E. Monoaminooxydase
23. A month ago patient was delete of considerable part of thin bowel. In his blood explored maintenance of
albumen in plasma, which made 3,5 g/l. How to classify the changes of albuminous composition of blood?
A. Desproteinemia
B. Hyperproteinemia
C. Paraproteinemia
D. Hypoproteinemia
E. Hypernitrogenemia
24. The promoted maintenance of low density lipoproteids was founded in the blood at an inspection of
patient. What disease can be expected in this patient?
A. Pneumonia
B. Acute pancreatitis
C. Gastritis
D. Atherosclerosis
E. Damage of kidneys
25. A patient with a fever lost weight during three weeks. Rest nitrogen of blood is 70 mmol/L. What is nitrous
balance in a patient?
A. Positive
B. Negative
C. Normal
D. Relative
26. The concentration of albumen in patient with infectious disease blood is 120 g/l. What quantitative change
of blood albuminous composition is present?
A. Hyperproteinemia
B. Hypoproteinemia
C. Paraproteinemia
D. Dysproteinemia
E. Hypernitrogenemia
27. Maintenance of common protein in the patient with hepatic coma blood is 40 g/l, maintenance of albumen
and fibrinogen is reduced, -globulin is promoted. Name the changes of blood albuminous composition?
A. Hypoproteinemia
B. Hyperproteinemia
C. Dysproteinemia
D. Hypernitrogenemia
E. Paraproteinemia
28. The maintenance of lipids in the blood of patient at starvation (in the second period) can increase. What is
the form of hyperlipidemia in this case?
A. Cerebral
B. Retentional
C. Transport
D. Alimentary
E. Diencephalic

29. Ventromedial nucleos of hypothalamus were ruined at animal in an experiment. As a result it has an
obesity. What is the type of obesity in this case?
A. Alimentary
B. Cerebral
C. Hormonal
D. Hyperplastic
E. Hypertrophic
30. Maintenance of urinary acid in a patient blood is promoted, that causes the pain syndrome development
as a result of urats deposit in the joints. What is the reason of this acid appearance?
A. Catabolism of HEM
B. Pirimidin nucleotids disintegration
C. Purin bases reutilisation
D. Purin nukleotids disintegration
E. Protein disintegration
31. Hyperlipidemia was founded at patient 67 years, suffers from atherosclerosis of cerebrum vessels. What
class of blood lipoproteids of probably will be promoted at biochemical research?
A. Low density lipoproteids
B. High density lipoproteids
C. -lipoproteids
D. Chilomicrons
E. Lipid acids and albumin complexes
32. Negative nitrous balance was founded at the inspection of man 45 years, which was long time on a
vegetable diet. What is the reason of this phenomenon?
A. Insufficient quantity of proteins
B. Surplus quantity of water
C. Surplus quantity of carbohydrates
D. Insufficient quantity of lipids
E. Insufficient quantity of lipids and proteins
33. Child, 2 years, was delivered to the hospital with slow mental and physical development, frequent
vomiting after taking a meal. Phenylpirovinograde acid was founded in urine. Violation of what metabolism is
in this pathology?
A. Carbohydrate
B. Amino acid
C. Water-salt
D. Phosphoric-calcium
E. Lipid metabolism
34. A prisoner declared starvation. Hypoglycemia, hypoproteinemia, hyperlipemia and ketonemia were
founded in his blood after the week of starvation. What form of hyperlipemia developed?
A. Alimentary
B. Food
C. Transport
D. Esential
E. Retentional
35. Hypotension, muscular weakness, negative nitrous balance were founded at the inspection of woman 50
years, who long time was on a vegetable diet. What factor resulted such state?
A. Surplus carbohydrates amount in a food
B. Insufficient carbohydrates amount in a food
C. Surplus consumption of water
D. Insufficient lipids amount in a food
E. Insufficient protein amount in a food
36. A patient with normal maintenance of general protein in blood has changes of separate albuminous
fractions correlation. Name this phenomenon.
A. Hyperproteinemia

B. Dysproteinemia
C. Paraproteinemia
D. Hypoproteinemia
E. Monoproteinemia
37. Polyunsaturated - 3-lipid acids which are contained in marine fish, and preparations, created on their
basis, are recommended for the atherosclerosis prophylaxis. What is the mechanism of the lipid acids action?
A. In the human organism these acids transform on eycosanoids
B. There are vasoconstrictors and activators of trombocytes agregation
C. There are vasodilyatators and inhibitors of trombotocytes agregation
D. Activate lecitin -cholestirin-acetyltransferase, that is contained in LDLP
. Inhibitor of Co-A- reductase
38. Apoprotein - is:
. Protein cellular receptors to lipoprotein of blood plasma
. Variant of modificated lipoproteids
. Lipoproteins of blood plasma without albuminous part
D. Albuminous component of blood plasma lipoproteins
. Anomal proteins with characteristics of lipoproteins
39. Woman, 62years, suffers from frequent pain in the area of thorax and vertebra, breaks of ribs. A doctor
assumed mielom disease (plasmocytoma). What laboratory indexes will have a main diagnostic value?
A. Hypoproteinemia
B. Paraproteinemia
C. Proteinuria
D. Hyperalbumenemia
E. Hypoglobulinemia
40. The quantity of plasma albumens changed in a man which executed a physical work in the conditions of
high temperature. How did quantity of plasma albumins change?
A. Relative hyperproteinemia
B. Paraproteinemia
C. Dysproteinemia
D. Absolute hyperproteinemia
E. Absolute hypoproteinemia
41. Increase of free lipid acids in patients with diabetes mellitus blood was observed. What is the reason of
this state?
A. Increase of adipocytes trigliceridlipase activity
B. Accumulation of palmytin Co-A in cytosolum
C. Activating of apolipoproteins of -1, -2, -4 synthesis
D. Decline of phosphatidilcholin cholesterin acetyltransferase activity in the blood
E. Activating of keton bodies utilization
42. Man, 70 years, suffers from atherosclerosis of lower extremities vessels and ischemic heart disease.
Violation of lipid composition of blood was founded during examination. What lipoprotein is the main link in
atherosclerosis pathogenesis?
A. Low density
B. Cholesterol
C. High density
D. Intermediate density
E. Chilomicrons

*** *** ***

1. Female, 55 years old, complaints of prolonged cyclical uterine bleeding


during the year, weakness, dizziness. Objective: pale skin. In the blood: Hb 70 g/l, er.-3,2x1012/l, CI - 0,6, leuk.-6,0x109 /l, reticulocytes - 1%; hypochromia
of erythrocytes. What is anemia in patients?
A. Chronic posthemorrhagic
B. Hemolytic
C. Iron-deficiency
D. Aplastic
E. B12-deficiency
2. Patient, 32 years old, brought to the clinic with massive blood loss due to
road trauma. Pulse - 110 / min., RR - 22 / min., AP - 100/60 mm Hg/ What will
be the typical in 1 hour after hemorrhage?
A. Erythropenia
B. Hypoproteinemia
C. Hypovolemia
D. Leukopenia
E. Hypochromia of erythrocytes
3. In the blood of patient, 36 years, who treated the respiratory viral infection
by the sulfanilamides, there is hyporegenerative normochromic anemia,
leukopenia, thrombocytopenia. In the bone marrow - reducing of
myelokariocytes. What is the kind of anemia?
A Hypoplastic
B Hemolytic
C Posthemorrhagic
D-B12 - deficiency
E Iron - deficiency
4. The patient entered into the hospital with complains on general weakness,
dizziness, shortness of breathing. Shortly before the going to the clinic he took
Levomicetin to prevent intestinal infections. Blood: red cells - 1,9 T/l, Hb - 58
g/l, CI - 0,9, leukocytes - 2,2 G/ l. What is the kind of anemia it?
A Hypoplastic
B Metaplastic
C Aplastic
D Hemolytic
E Iron
5. The patient has chronic posthemorrhagic anemia, accompanied by lower
concentrations serum iron, hypochromia of erythrocytes, poikilocytosis and
anisocytosis. What is the value of color index at this anemia?
A 0,7
B 0,8
C 0,9

D 1,0
E 1,1
6. Anaemia was revealed in a patient with diffuse glomerulonephritis. What
underlies pathogenesis of this anaemia?
A. Decreasing production of erythropoietin
B. Suppression of red bone marrow function
C. Deficit of internal Kastl factor
D. Increased hemolysis
E. Presence of antibodies to the cell of peripheral blood.
7. A B-12 -deficiency anaemia developed in a patient following the stomach
resection. What colour index of blood is typical for this pathology?
A. 1.4
B. 1.0
C. 0.8
D. 0.5
E. 0.2

1. A 38-year-old patient with an uterine haemorrhage lasting for 2 days was


delivered to the admission ward. Which of the following will be revealed in the
patient's blood?
A. Leukocytosis
B. Increase in the colour index
C. Decrease in the haematocrite index
D. Deceleration in ESR
E. Eosinophilia
2. It was established that agglutination of the recipient's blood erythrocytes
had been caused by the standard sera from the I and II groups. Serum from
the III group as well as anti-Rh serum hadnt provoke any agglutination.
Which blood group and rhesus is allowed to be transfused this recipient?
A B, (III) RhB 0, , (I) Rh+
C A, (II) Rh
D AB (IV), Rh+
E AB (IV), Rh1. Anisochromia is:
. Changes of red corpuscles forms.
. Changes of red corpuscles sizes.
. Increase of red corpuscles amounts

D. *Different degree of red corpuscles saturation by haemoglobin


. Decrease of red corpuscles amounts
2. The presence of what cells in blood testifies of physiological regeneration
development:
A. Erythroblasts.
B. Megalocytes.
C. Megaloblasts.
D. *Reticulocytes
. Erythrocytes
3. Hb 48 g/l, amount of red corpuscles - 3,0 10 12/l, GV - 0,7, leucocytes 7,5x109/l, thrombocytes 280x109/l were discovered for a young man during a
prophylactic inspection. Give description of the patient state.
A. Thrombocytopenia
B. Leukopenia
C. Leukocytosis
D. Erythrocytosis
E. *Anaemia
4. Iron-deficiency anaemia developed at a patient with pathology of liver.
What globular value is characteristic for this disease?
A. 1,2
B. 0,8
C. 1,0
D. *0,6
E. 1,4
5. The amount of reticulocytes grew in peripheral blood at a man, 50 years
old, on 5th day after the acute gastric bleeding. What this phenomenon testify
about?
. About presence of oxygen deficiency in an organism.
. About presence of inflammatory reaction in a stomach.
. About multiplying the albumens synthesis in a liver.
D. *About increase of hemopoietic activity of bone marrow.
. About presence of allergic reaction.
6. Hypochromic anaemia is diagnosed at a woman after the complicated labor
which was accompanied the considerable bleeding. What pathological forms
of red corpuscles are characteristic for such anaemia?
. Macrocytes

. Targed red corpuscles.


. Sickle red corpuscles.
D. Spherocytes.
E. *Anulocytes.
7. Victim lost much blood as a result of accident. What violation of general
blood volume takes place?
. Polycythemic normovolemia
. Polycythemic hypovolemia
. Oligocythemic normovolemia
D. *Simple hypovolemia
. Oligocythemic hypovolemia
8. A child got thermal burns of 40% surface of body. What form of general
blood volume violation is observed here?
. Polycythemic hypovolemia.
. Polycythemic hypervolemia.
. Oligocythemic hypovolemia.
D. Normocythemic hypovolemia.
. Oligocythemic hypervolemia.
9. What violation of general volume of blood does arise up in case of absolute
erythrocytosis?
. Simple hypervolemia.
. Oligocythemic hypervolemia.
. Oligocythemic normovolemiya.
D. Policythemic hypervolemia.
. Simple hypovolemia.
10. A patient lost a 25% volume of blood as a result of wound. Name the
urgent mechanism of blood loss compensation.
. *Entry of intercellular liquid into the vessels
. Restoration albuminous composition of blood
. Increase of reticulocytes numbers
D. Restoration of red corpuscles number
. Activation of erythropoiesis
11. Signs of increase of bone marrow Regenerativey possibilities was found on
a 6th day at a patient which is on stationary treatment concerning acute
posthemorrhagic anaemia. Name the organ which is chargeable for
erythropoiesis regulation (erythropoietins synthesis) in this case.

A. Adrenal gland
B. *Kidneys
C. Spleen
D. Thyroid gland
E. Hypophysis
12. Blood test is done on 30th days after bleeding at a patient with the wound
of subclavian artery. What is testify about activation of erythropoiesis in the
blood smear?
A. Poikilocytosis
B. Anisocytosis
C. *Reticulocytosis
D. Anisochromia
E. Hypochromia
13. ESR is promoted at young pregnant woman. What results the increase of
ISR?
A. Considerable diminishing of amount of red corpuscles
B. *Increase of concentration of fibrinogen
C. Multiplying of lipoproteids concentration
D. Increase of imunoglobulins concentration
E. Increase of albumins concentration
14. Such indexes found out at a patient at the inspection of blood: erythrocytes
- 2,01012/l, Hb-60 g/l, GV - 1,0, reticulocytes are absent, platelites. - 5010 9/l,
leukocytes - 2,0109/l. What is the type of anaemia?
A. Posthemorrhegic
B. Pernicious
C. Iron-deficiency
D. *Aplastic
E. Hemolytic
15. Single oxyphyle normocytes appeared in the patient blood after one day
after a loss of 15% of blood. 25% of reticulocytes was discovered at the blood
smear. What type of anaemia at this patient by the ability of bone marrow to
regeneration?
A. Hyporegenerative
B. Aregenerative
C. *Hyperregenerative
D. Regenerative
E. 16. Signs of anaemia found out at a patient with chronic glomerulonephritis.
What stimulated their appearance?
A. *Decline of erythropoetins synthesis
B. Loss of red corpuscles with urine

C. Destruction of normal red corpuscles


D. Hemolysis of red corpuscles
E. Insufficiency of iron for the haemoglobin synthesis
17. A man was living in the mountains for a long time. What changes in the
blood will he have?
A. *Multiplying the amount of haemoglobin
B. Multiplying of leukocytes amount
C. Decline of leukocytes amount
D. Diminishing of blood vessels diameter
E. Multiplying of vessels diameter
18. A 58-year-old woman complains of fatigability, sleepiness, dyspnea when
walking fast. The analysis of the blood is as follows: erythrocytes 4x1012/l,
hemoglobin 92.0 g/1, Colour Index - 0.6, plenty of anulocytes and microcytes.
What anemia is it?
A. Thalassemia.
B. *Iron deficiency.
C. Hemolytic.
D. Pernicious.
E. Sicklemia.
19. The erythrocytes of a patient with hypochromic anemia contain 45 % Hb
S and 55 % Hb A,. What form of anemia is it?
A. Microspherocytic.
B. -Thalassemia.
C. Addison-Birmer illness.
D. Glucose-6-phosphatdehydrogenase deficiency.
E. *Sicklemia.
20. Degenerative and regenerative forms of erythrocytes were found in the
peripheric blood of a patient with toxic hemolytic anemia. What cells refer to
regenerative?
A. Spherocytes.
B. Microcytes.
C. *Reticulocytes.
D.Poikilocytes.
E. Hyperchromic erythrocytes.
21. A patient with chronic diffuse glomerulonephritis suffers from anemia.
What is the pathogenesis of this anemia?
A. Presence of antibodies to the cells of the peripheric blood.
B. Iron deficiency.
C. Increased deficiency of the internal Castle factor.

D. Hemolysis of erythrocytes.
E. *Diminished production of erythropoietin.
22. A 40-year-old man was diagnosed with sicklemia. What is the mechanism of
decreasing the erythrocytes number in the patient's blood?
. *Intracellular hemolysis.
B. Intravascular hemolysis.
C. Bleeding.
D. Inhibition of erythropoiesis.
E. Disturbance of DNA synthesis.
23. A 34-year-old woman is diagnosed with hereditary hemolytic
microspherocytic anemia (Minovski Shoffar disease). What is the cause of
hemolysis of erythrocytes in this case?
A. Fermentopathia.
B. Endogenic intoxication.
C. Hemoglobinopathia.
D. Autoimmune lesion.
E. *Membranopathia.
24. In the 6-th month of pregnancy a woman had the following blood
findings: the amount of erythrocytes and hemoglobin is reduced, Colour
Index is 1.4, megalocytes, oxyfile megaloblasts are present. What type of
anemia is it?
A. Iron deficiency.
B. *B12 and folic deficiency.
C. Myelotoxic.
D. Aplastic.
E. Metaplastic.
25. A woman complains of headache, giddiness, dyspnea during physical activity.
For the last 3 years she has been having extensive menstrual bleedings. The
patient has normal body type, the skin is pale and dry. The analysis of the
blood is as follows: hemoglobin 90.0 g/1, erythrocytes-3.7-1012/l, Colour
Index - 0.7, ESR - 20 mm/h, significant hypochromia of erythrocytes,
anisocytosis, poikilocytosis. What type of anemia is it?
A. Hypoplastic.
B. Hemolytic.
C. Acute posthemorrhagic.
D.B12 and folic deficiency.
E. *Chronic posthemorrhagic.
26. After the resection of the stomach a patient developed B)2 deficiency anemia.
What color index is characteristic of this anemia?
A. 0.4.
B. 0.5.

C. 0.8.
D.1.0.
E. *1.4.
27. Microcythemia, poikilocytosis, and anulocytosis are found in the patient's
blood smear. What anemia are these changes characteristic of?
A. Microspherocytic.
B. B12 deficiency.
C. Hypoplastic.
D. Sicklemia
E. *Iron deficiency.
*** *** ***

1. Substitution of the glutamic acid on valine was revealed while examining


initial molecular structure. For what inherited pathology is this typical?
A Sickle-cell anemia
B Thalassemia
C Minkowsky-Shauffard disease
D Favism
E Hemoglobinosis
2. A 34 year old woman was diagnosed with hereditary microspherocytic
hemolytic anemia (Minkowsky-Shauffard disease). What mechanism caused
haemolysis of erythrocytes?
A Membranopathy
B Enzymopathy
C Hemoglobinopathy
D Autoimmune disorder
E Bone marrow hypoploasia
3. 2 years ago a patient underwent resection of pyloric part of stomach. He
complains of weakness, periodical dark shadows beneath his eyes, dyspnea. In
blood: Hb - 70 g/l, erythrocytes - 3,0*1012l, colour index - 0,7. What changes of
erythrocytes in blood smears are the most typical for this condition?
A Microcytes
B Megalocytes
C Schizocytes
D Ovalocytes
E Macrocytes
4. A 55 y.o. woman consulted a doctor about having continuous cyclic uterine
hemorrhages for a year, weakness, dizziness. Examination revealed skin
pallor. Hemogram: Hb- 70 g/l, erythrocytes - 3,2*1012/l, color index - 0,6,

leukocytes - 6,0*109/l, reticulocytes - 1%; erythrocyte hypochromia. What


anemia is it?
A Chronic posthemorrhagic anemia
B Hemolytic anemia
C Aplastic anemia
D B12-folate-deficiency anemia
E Iron-deficiency anemia
5. A 56 year old patient came to a hospital with complaints about general
weakness, tongue pain and burning, sensation of limb numbness. In the past
he underwent resection of forestomach. In blood: Hb- 80 g/l; erythrocytes 2,0*1012/l; colour index - 1,2, leukocytes - 3,5*109/l. What anemia type is it?
A B12-folate deficient
B Hemolytic
C Posthemorrhagic
D Aplastic
E Iron-deficient
6. Blood test of a patient suffering from atrophic gastritis gave the following
results: RBCs - 2,0*1012/l, Hb- 87 g/l, colour index - 1,3, WBCs - 4,0*10 9/l,
thrombocytes - 180*109/l. Anaemia migh have been caused by the following
substance deficiency:
A Vitamin B12
B Vitamin A
C Vitamin K
D Iron
E Zinc
7.
A 26 year old pregnant woman is under treatment at an in-patient
hospital. After a continuous attack of vomiting she was found to have reduced
volume of circulating blood. What kind of change in general blood volume is
the case?
A Polycythemic hypovolemia
B Simple hypovolemia
C Oligocythemic hypovolemia
D Polycythemic hypervolemia
E Oligocythemic hypervolemia
8. A 15 year old girl has pale skin, glossitis, gingivitis. Blood count:
erythrocytes - 3,3*1012/l, hemoglobin - 70 g/l, colour index - 0,5. Examination
of blood smear revealed hypochromia, microcytosis, poikilocytosis. What type
of anemia is it?
A Iron-deficient
B B12-folic acid-deficient
C Sickle-cell

D Hemolytic
E Thalassemia

9. On the fifth day after the acute blood loss a patient has been diagnosed with
hypochromic anemia. What is the main mechanism of hypochromia
development?
A Release of immature red blood cells from the bone marrow
B Impaired iron absorption in the intestines
C Increased destruction of red blood cells in the spleen
D Impaired globin synthesis
E Increased excretion of body iron
10. A 25 year old Palestinian woman complains of weakness, dizziness,
dyspnea. In anamnesis: periodically exacerbating anemia. In blood: Hb - 60
g/l, erythrocytes - 2,5*1012/l, reticulocytes - 35o/oo, anisocytosis and
poikilocytosis of erythrocytes, a lot of target cells and polychromatophils.
What type of anemia is it?
A Thalassemia
B Sickle-cell anemia
C Minkowsky-Shauffard disease
D Addison-Biermer disease
E Glucose 6-phosphate dehydrogenase-deficient anemia

11. A 32-year-old patient was admitted to the hospital with gross bloodloss
due to auto accident trauma. Ps 110 Bpm, RR- 22 pm, BP- 100/60mm Hg.
What changes in the blood will occur in an hour after the bloodloss?
A Hypovolemia
B Erythropenia
C Hypochromia of erythrocytes
D Leukopenia
E Hypoproteinemia
12. Patient with hypochromic anemia has splitting hair and loss of hair,
increased nail brittling and taste alteration. What is the mechanism of the
development of these symptoms?
A Deficiency of iron-containing enzymes
B Deficiency of vitamin 12
C Decreased production of parathyrin
D Deficiency of vitamin
E Decreased production of thyroid hormones
1. An immune haemolytic anaemia developed in a patient. What index of the
blood serum will increase the most significantly&
A. Indirect bilirubin

B. Direct bilirubin
C. Stercobilinogen
D. Mesobilinogen
E. Protoporphin
2. Hereditary spherocytosis (Minkowsky-Shauffard disease) was diagnosed in
a 34-year-old woman. What mechanism caused haemolysis in this patient?
A. Membranopathy
B. Enzymopathy
C. Hemoglobinopathy
D. Autoimmune damage
E. Bone marrow hypoplasia
3. A patient complains that his hair split and fall out, his nails are brittle, his
taste is broken. What is the mechanism of the mentioned symptoms?
A. Iron-containing enzymes deficit
B. Vit B-12 deficit
C. Decreased paratyrin production
D. Vit A deficit
E. Decreased thyroid hormones production
4. A B-12 folio-deficit anaemia developed in a patient following the stomach
resection 5 years ago. What mechanism is typical for this pathology
development?
A. Absence of the internal Casls factor
B. Absence of the external Casls factor
C. Malabsorption of the vit B-12 in small intestine
D. Deficit of folic acid
E. Deficit of transcobalamin
5. Hypochromic anaemia was diagnosed in a 54-year-old patient, who had
permanent contact with lead due to professional activities. Treatment with
iron preparations during a month had no effect. In addition, increased level of
iron was detected in blood serum. What is the cause of the anaemia?
A. Disorders of porthyrin synthesis
B. Vit B-12 deficit
C. Deficit of folic acid
D. Bone marrow hypoplasia
E. Deficit of proteins
6. A 20-year-old man complains of periodic skin and sclera icteritiousness,
fatigue. Minkowsky-Shauffard disease was diagnosed. What is typical for
blood film in this case?
A. Microspherocytosis
B. Anulocytosis
C. Agranulocytosis

D. Macrocytosis
E. Thrombocytosis
7. A woman was hospitalised for examination. Decreasing of haemoglobin
levels down to 90-95 g/l has been taking place from childhood. Treatment with
iron medications had no effect. Blood test results at the moment of
hospitalisation: erythrocytes 3.2 x 1012/l, Hb 85 g/l, colour index of blood
0.78, anisocytosis, poikilocytosis, target cell erythrocytes, reticulocytes 16%.
Thalassemia was diagnosed. What type of haemolytic anaemia takes place?
A. Hereditary hemoglobinopathy
B. Hereditary membranopathy
C. Acquired membranopathy
D. Hereditary enzymopathy
E. Acquired enzymopathy
8. A woman developed severe iron-deficiency anaemia on the 6 th month of
pregnancy. It was diagnosed due to presence in her blood
A. Anulocytes
B. Macrocytes
C. Poikilocytes
D. Reticulocytes
E. Normocytes
9. A B-12 folio-deficit anaemia with disorder of haemopoiesis and appearance
of changed forms of erythrocytes in blood developed in a patient following the
stomach resection. As an illustration of this pathology might serve presence in
blood
A. Megalocytes
B. Microcytes
C. Ovalocytes
D. Normocytes
E. Anulocytes
10. A patient develops decreasing of haemoglobin level and erythrocyte count
from time to time. It was figured out from anamnesis that these episodes
appear after eating of field beans. What type of anaemia takes place?
A. Hereditary enzymopathy
B. Hereditary membranopathy
C. Iron-deficiency anaemia
D. Hereditary hemoglobinopathy
E. Acquired hemolytic anemia
11. During the study of globin molecule structure it was revealed that glutamic
acid was replaced by valine. What hereditary pathology takes place?
A. Sickle-cell anemia
B. Thalassemia
C. Minkowsky-Shauffard disease
D. Favism

E. Haemoglobinosis
12. A hypochromic anemia takes place in a patient with hypoacid gastritis.
Anulocytes, microanisocytosis, and poikilocytosis were revealed in a blood
smear. What type of anaemia takes place?
A. Iron-deficiency
B. Acute posthaemorrhagic
C. Thalassemia
D. Sickle-cell
E. Pernicious
1. A 36-year-old patient with respiratory viral infection was treated with
sulphanilamide drugs. Blood test findings revealed hyporegenerative
normochrome anemia, leukopenia, and thrombocytopenia. In the bone
marrow decreased amount of myelokariocytes was found. What anemia is it?
A. Posthemorrhagic.
B. Hemolytic.
C. *Hypoplastic.
D. B12 and folic deficiency.
E. Iron deficiency.
2. A patient, who had arrived from Tunis, revealed -thalassemia with hemolysis
of erythrocytes and icterus. The presence of what cells in the blood is
typical of this illness?
A. *Target-like erythrocytes.
B. Granular erythrocytes.
C. Polychromatophilous erythrocytes.
D. Normocytes.
E. Reticulocytes.
3. A 3-year-old child was hospitalized with hemoglobin opathy (sicklemia).
What amino acid replaced glutamin acid in the -chain of hemoglobin S?
A. Arginine.
B. Serine.
C. Thyrosin.
D. Phenylalanine.
E. *Valine.
4. Beta-thalassemia was revealed in a patient, who came from Tunis. The
disease was accompanied by hemolysis and jaundice. The disease was
diagnosed on the base of presence in blood:
A. *Target-like erythrocytes
B. Grained erythrocytes
C. Polychromatophil erythrocytes
D. Normocytes
E. Reticulocytes

5. During the examination of adolescents that reside in mounting region


increase in level of erythrocytes and hemoglobin in peripheral blood was
found out. What is the reason for indicated erythrocytosis?
A. *Exogenous hypoxia
B. Diseases of lungs
C. Congenital heart disease
D. Condensation of blood due to large loss of water
E. Vakess disease
6. Content of hemoglobin and number of erythrocytes significantly decreases
in a patients blood from time to time. It was found out that such attacks
appear after taking some horse beans. What kind of anemia takes place in this
case?
A. *Enzymopathy
B. Membranopathy
C. Iron deficiency anemia
D. Hemoglobinopathy
E. Acquired hemolytic anemia
7. In blood analysis of 37-years-old woman following data were revealed:
content of hemoglobin is 60 g/L, number of erythrocytes is 3.0x10 12/L, and
color index of 0.6; differential count of leukocytes without any changes;
number of platelets is 200x109/L; reticulocytes count of 0.3%; ESR of 18
mm/hour; microcytosis and poikilocytosis of erythrocytes. Indicate the most
probable type of anemia according to mechanisms of its development.
A. Hypoplastic anemia
B. Hemolytic anemia
C. Acute post-hemorrhagic anemia
D. *Iron deficiency anemia
E. B12-folate deficiency anemia
8. A patient aged 20, has periodically yellowness of sclera and skin that is
accompanied by weakness. Minkovski-Shoffar disease is diagnosed. What is
the most typical for blood picture in this disease?
A. Reticulositosis
B. Agranulocytosis
C. Macrocytosis
D. *Microspherocytosis
E. Thrombocytosis

9. Posthemorrhagic anemia has developed in a patient with periodical


bleeding due to uterus fribromyoma. What type of chronic post hemorrhagic
anemia takes place in this case?
A. Megaloblastic, hyperchromic, hyperegenerative.
B. Megaloblastic, hypochromic, hyporegenerative.
C. Erythroblastic, hyperchromic, hyporegenerative.
D. Erythroblastic, hypochromic, hyperregenerative.
E. *Erythroblastic, hypochromic, hyporegenerative.
10. Hypochromic anemia was found out in a patient aged 54, who had a
prolonged contact with lead at his work. Treatment with iron preparations for
a month didnt give any effect. The increased amount of iron was determined
in blood serum. What is this anemia due to?
A. Vitamin B12 deficiency
B. Porphirin synthesis impairment
C. Folic acid deficiency
D. *Hypoplasia of bone marrow
E. Erythrocyte hemolysis
11. Excessive flow of estrogens into the blood due to follicle persistence (a state
when follicle does not reach complete maturation and ovulation does not take
place) often cause uterine bleeding. What anemia may develop in this case?
A. *Iron deficiency
B. Sideroachrestic
C. Sickle cell
D. Hypoplastic
E. Metaplastic.
12. Which of the below named anemias relevant to hemoglobinopathies?
A. Minkovsky-Shoffar disease
B. Iron deficiency anemia
C. B12 deficiency
D. *Thalassemia
E. Iron refractory anemia
13. Evaluate the blood analysis: erythrocytes 3.10; Hb 90g\L reticulocytes
0.5%. In the smear there are poikilocytes hypochromatic erythrocytes. Blood
serum iron is 80 micromol/l. What pathology is this typical for?
A. Minkovsky-Shoffer disease
B. Iron deficiency anemia
C. B12 deficiency
D. Sickle cell anemia
E. *Iron refractory anemia

14. The experiment was carried out on a rabbit. The increase of the number of
erythrocytes and hemoglobin in the blood due to the stimulation of
erythropoiesis by erythropoietin was determined 2 weeks later after the
narrowing of renal artery. What increases the formation of erythropoietin?
A. Hypoosmia
B. Hypercapnia
C. *Hypoxemia
D. Hyperosmia
E. Hypovolemia
15. Hemolytic anemia with decrease of osmotic erythrocyte resistance that
averaged 0.6-0.5 was revealed in a smear of venous blood during microscopic
examination. What substance accumulation in the blood plasma may also
indicate the development oh hemolytic anemia?
A. Creatinine
B. Urea
C. *Indirect bilirubin
D. Lactic acid
E. Inorganic phosphate
16. Hypochromia of erythrocytes, micro-, anisocytosis, poikilocytosis, are
determined in the blood in case of development of iron deficiency and iron
refractory anemia. What index must be determined to carry out differential
diagnosis of these anemias?
A. Serum chlorine
B. Serum phosphorous
C. Serum magnesium
D. Serum calcium
E. *Serum iron
17. A patient aged 32 with massive hemorrhage due to car accident trauma
was admitted to the hospital. Pluse-100 beats per min, respiratory rate-22 per
2 min, BP-100/60 mmHg. What blood change will be the most characteristic in
an hour after hemorrhage?
A. Erythropoenia
B. Hypoproteinemia
C. *Hypovolemia
D. Leucopoenia
E. Erythrocyte hypochromia
18. Singular oxyphilic normoblasts appeared in the blood of a patient after
acute post-traumatic hemorrhage composing 15% of blood volume. On
supravital staining 25% of reticulocytes were found. What is the patient
anemia according to its ability of regeneration?
A. *Hyperregenerative

B. Regenerative
C. Hyporegenerative
D. Aregenerative
E. Hypo- and aregenerative
19. What index of blood analysis is the most typical for beta-thalassemia?
A. Considerable decrease of erythrocytes and hemoglobin
B. Erythrocytes with basophilic stippling
C. *Increase of fetal hemoglobin
D. Target-like erythrocytes
E. Increase of met-hemoglobin
20. A patient had anemia due to profuse blood loss. What blood changes are
typical at the beginning of development of acute post-hemorrhagic anemia?
A. Presence of megalocytes in the blood
B. Absence of reticulocytes
C. Poikilocytosis, anisocytosis
D. Hyperchromia
E. *Normochromia
21. Erythropenia, hyperchromia, normocytes, macrocytes, megalocytes,
poikilocytosis were found out in a patients blood at examination. What is the
cause of this pathology?
A. Ascariasis
B. *Deficiency of gastromucoprotein
C. Iron deficiency in food
D. Trichocephaliasis
E. Fequent loss of blood
22. In an infant, who is under an artificial nutrition with cow milk, severe
anemia has developed. At the blood count of the infant: number of
erythrocytes is 4x1012/L, content of hemoglobin is 68 g/L, reticulocytes of 0%.
What kind of anemia developed in the infant?
A. Sickle-cell anemia
B. Inborn hemolytic anemia
C. B12-deficiency anemia
D. Hypopastic anemia
E. *Iron-deficiency anemia
23. Predomination of erythroblasts, normoblasts and megaloblasts was
revealed in blood analysis of a patient with anemia. The same cells were found
in bone marrow. What type of anemia do these changes characteristic for?
A. Post-hemorrhagic
B. Hemolytic
C. Aplastic

D. *B12-folate deficiency anemia


E. Iron-deficiency anemia
24. Funicular myelosis and hyperchromic anemia developed in a man 7 years
later after the stomach resection due to ulcer. What pathogenic mechanism of
changes in spinal cord is the most possible one?
A. Hypoxia impairment in anemia
B. *Accumulation of methylmalonic acid in cyanocobalamin deficiency
C. Impairment of DNA in cyanocobalamin deficiency
D. Deficiency of folic acid
E. Deficiency of iron containing enzyme
25. Examining the oral cavity of a patient, a dentist paid attention to the
presence of inflammatory-dystrophy process in the mucous membrane
(Gunters glossitis, atrophic stomatitis). Blood analysis revealed hyperchromic
anemia. What factor is a cause of this disease?
A. Hypovitaminosis B6
B. Hypovitaminosis A
C. Increase of stomach juice acidity
D. Hypovitaminosis B1
E. *Hypovitaminosis B12
26. A female patient complains of malaise, weakness, breathlessness, rapid
fatigability, and dizziness. Her blood test data: erythrocytes-1.8x1012/L, Hb-80
g/L, leukocytes-3.2x109/L, color index-1.5. Anisocytosis, poikilocytosis,
megaloblasts, megalocytes were found in smear. What is the possible
diagnosis?
A. *B12-deficiency anemia
B. Posthemorragic anemia
C. Acute leukemia
D. Iron deficiency anemia
E. Immunohemolytic anemia
27. Anemia, leuko- and thrombocytopenia, color index-1.3, presence of
megaloblasts and megalocytes were determined in the laboratory analysis of
blood of a patient a year later after he was operated on for subtotal resection
of the stomach for the ulcer of lesser curvature of the stomach. What factor
deficiency results in these changes?
A. *Gastromucoprotein
B. Gastrin
C. Pepsin
D. Chlorine hydrate
E. Mucin
28. Amino acids replacement in alpha and beta chains of hemoglobin takes
place in a number of hemoglobimopathies. Which of them is typical for HbS
(sickle-cell anemia)?

A. Glycine to serine
B. Aspartate to lysine
C. Methionine to histidine
D. *Glutamate to valine
E. Alanine to serine
29. Hereditary microspherocytic hemolytic anemia (Mincovsky-Shoffar
disease) was diagnosed in a woman aged 34. What mechanism caused
hemolysis of erythrocytes in the patient?
A. Enzymopathy
B. Hemoglobinopathy
C. Autoimmune impairment
D. *Membranopathy
E. Hypoplasia of bone marrow
30. Megaloblastic anemia was diagnosed in a patient. What substance
deficiency may cause the development of this disease?
A. *Cyanocobalamin
B. Cholecalciferol
C. Magnesium
D. Glycine
E. Copper
31. Three years ago a man aged 45 was operated on for stomach resection.
After the operation the content of erythrocytes in the blood is 2.0x10 12, Hb 85
g/l, color index-1.27. What vitamin absorption is impaired that causes the
change of erythropoiesis?
A. C
B. P
C. A
D. B6
E. *B12

A
B
C
D
E

32. Substitution of the glutamic acid on valine was revealed while examining
initial molecular structure. For what iherited pathology is this typical?
Minkowsky-Shauffard disease
*Sickle-cell anemia
Favism
Talassemia
Hemoglobinosis
33. A 58-year-old woman complaints of increased tiredness, decreased capasity
for work, somnolence and dyspnea during fast walking. Blood test revealed:
erythrocytes -4,6x1012/l, hemoglobin - 92 g/l, colour index 0,6. Blood smear
demonstrated high contents of microcytes and anulocytes. What anemia is it
typical for?
A Hemolytic

B
C
D
E

*Iron deficiency
Acute posthemorragic
Penicious
Sickle cell

34. A 43-year-old man suffers from chronic atrophic gastritis and Megaloblastsc
hyperchromic anemia. He also has methylmalonic aciduria. Insufficiency of what
vitamin led to the development of such complex of symptoms?
A Vitamin B5
B Vitamin B2
C *Vitamin B12
D. Vitamin B3
E. Vitamin B1

A
B
C
D
E
A
B
C
D
E

35. A woman with III (B), Rh- blood group born a child with II (A) blood
group. The child is diagnosed with hemolytic disease of newborn as a result of
rhesus incompatibility. What blood group is the childs father likely to have?
I (0), Rh+
I (0), RhII (A), Rh*II (A), Rh+
III (B), Rh+
36. Examination of a 43 y.o. anephric patient revealed anemia symptoms.
What is the cause of this symptoms?
Iron deficiency
Vitamin B12 deficiency
Enchanced destruction of erythrocytes
Folic acid deficiency
*Reduced synthesis of erythropoetins
37. Surgical removal of a part of stomach resulted in disturbed absorption of
vitamin B12, it is excreted with feces. The patient was diagnosed with anemia.
What factor is necessary for absorption of this vitamin/
A
Gastrin
B
*Gastromucoprotein
C
Pepsin
D
Folic acid
E
Hydrochloric acid
38. A 56 year old patient came to a hospital with complaints about general
weakness, tongue pain and burning, sensation of limb numbness. In the past
he underwent resection of forestomach. In blood: Hb-80 g/l; erythrocytes2,0x1012/l; colour index-1,2, leucocytes-3,5x109/l. What anemia type is it?
A
*B12-folate deficient
B
Iron-deficient

C
D
E

Posthemorragic
Aplastic
Hemolytic

39. Packed cell volume of a man was 40% before the trauma. What packed
cell volume will be observed 24 hours after blood loss of 750 ml?
A *30%
B 40%
C 55%
D 50%
E 45%

A
B
C
D
E

40. A patient was diagnosed with autoimmune hemolitic cytotoxic anemia.


What substances are antigens in II type allergic reactions?
Hormones
*Modified receptors of cell membranes
Antibiotics
Serum proteins
Inflammation modulators
41. A 55 y.o. woman consulted a doctor about having continuous cyclic uterine
hemorrhages for a year, weakness, dizziness. Examination revealed skin
pallor. Hemogram: Hb-70 g/l, erythrocytes-3,2x1012/l, color index-0,6,
leucocytes-6,0x109/l,reticulocytes-1%; erythrocyte hypochromia. What anemia
is it?
A
Iron-deficiency anemia
B
Aplastic anemia
C
Hemolytic anemia
D
*Chronic posthemorhagic anemia
E
B12-folate-deficiency anemia
42. Patient, 52 years old, had the operation of stomach resection three years.
In the blood: erythrocytes - 2,01012/l, Hb-85 g/l, GV - 1,27. Violation of what
vitamin uptake caused such changes?
A. P
B. *B12
C. B6
D. A
E. C
43. Inkcretory function of kidneys is violated at a patient with chronic
glomerulonephritis. To the deficiency of what uniform elements of blood will it
lead?
A. *Red corpuscles

B. Red corpuscles and leukocytes


C. Thrombocytes
D. Leukocytes
E. Leukocytes and thrombotcytes
44. Long-term starvation cure of a patirnt resulted in deminieshed ratio of
albumines and globulines in plasma. What of the following will be result of
these changes?
A
Decrease of ESR
B
Hypercoagulation
C
*Increase of ESR
D
Increase of hematocrit
E
Decrease of hematocrit

A
B
C
D
E

A
B
C
D
E

45. 2 years ago a patient underwent resection of pyloric part of stomach. He


complains of weakness,periodical dark shadows beneath his eyes, dyspnea. In
blood: Hb:-70 g/l, erythrocytes-3,0x1012/l, colour index-0,7. What changes of
erythrocytes in blood smears are the most typical for this condition?
*Microcytes
Shizocytes
Megalocytes
Ovalocytes
Macrocytes
46. A three-year child with the fever after taking an aspirin has increased
hemolysis of red corpuscles. Insufficiency of what enzyme could cause
hemolytic anaemia at a child?
A. *Glucose 6-phosphate dehydrogenase
B. - glutamintransferase
C. Glycerophosphatedehydrogenase
D. Glucose 6-phosphatase
E. 47. A 20 year old patient complains of general weakness, dizziness, quick
fatigability. Blood analysis results: Hb-80 g/l. Microscopical examination
results: erythrocytes are of modified form. This condition might be caused by:
Acute intermittent porphyria
Hepatocellular jaundice
Obturative jaundice
Addisons disease
*Sickle-cell anemia
*** *** ***

1. A 23 y.o. patient complains of weakness, temperature rise up to 38-40 0C.


Objectively: liver and spleen are enlarged. Hemogram: Hb- 100 g/l,
erythrocytes - 2,9*1012/l, leukocytes - 4,4*109/l, thrombocytes 48*109/l,
segmentonuclear neutrophils - 17%, lymphocytes - 15%, blast cells - 68%. All

cytochemical reactions are negative. Make a hematological conclusion:


A Undifferentiated leukosis
B Chronic myeloleukosis
C Acute myeloblastic leukosis
D Acute lymphoblastic leukosis
E Acute erythromyelosis
1. A 21-year-old man complains of fatigue and fever up to 38-40C. Liver and
spleen are enlarged, Hb 100 g/l, erythrocytes 2.9 x 10 12/l, leukocytes 4.4 x
109/l, platelets 48 x 109/l, polymorphonuclear neutrophils 17%,
lymphocytes 15%, blast cells 68%. All cytochemical tests are negative.
Provide hematological conclusion.
A. Undifferentiated leucosis
B. Chronic myeloleukemia
C. Acute myeloblastic leucosis
D. Acute lymphoblastic leucosis
E. Acute erythromyelosis
2. A blood test was taken in a patient with leukaemia. What is the most typical
for acute myeloblastic leukaemia?
A. Leukemic gap
B. Leucocytosis
C. Appearance of myeloblasts in the blood
D. Anaemia
E. Leukocytes degeneration
1. What blood pathology is the presence of Philadelphia chromosomes in the
blood cells and bone marrow cells typical for?
A. Acute myelogenous leukemia
B. *Chronic myelogenous leukemia
C. Hodgkins disease
D. Burkitts lymphoma
E. Chronic lymphocyte leukemia
2. In the patients blood analysis the number of leukocytes is 250*10 9/L. What
syndrome does this patient have?
A. *Leukemia
B. Leucocytosis
C. Leucopoenia
D. Leucomoid reaction
E. Hyperleucocytosis.
3. Patient M, aged 20 was admitted to the hospital complaining of high
temperature, pain in the bones, and hemorrhage from his gums. Blood
analysis of this patient shows: erythrocytes-2.5x1012/L; Hb-80g/L; leucocytes2.0x109/L; thrombocytes-6.0x109/L; differential count: eosinophils-1%; stab

neutrophils-1%; segmented neutrophils-10%; lymphocytes-10%; monocytes3%; blast cells-75%. What pathology is this blood analysis typical for?
A. Hodgkins disease
B. Burkitts lymphoma
C. *Acute leukemia
D. Infections mononucleosis
E. Chronic leukemia
4. General amount of leucocytes is 90x10 9/l. In differential count: eosinophils1%; basophils-0%; juvenile neutrophils-0%; stab neutrophils-2%; segmented
neutrophils-20%; prolymphocytes-2%; lymphocytes-70%; Botkin-Gumprecht
cells. Cervical, submandibular lymph nodes are enlarged. What pathology is
such blood picture typical for?
A. Acute lymphoblastic leukemia
B. Hodgkins disease
C. Infectious mononucleosis
D. *Chronic lymphocytic leukemia
E. Chronic myelogenous leukemia
5. Hemiparesis appeared in a patient with acute promyelocytic leukemia.
What is the main mechanism of the impairment of CNS in this case?
A. Intoxication by leukemic cells decay products;
B. *Formation of leukemic infiltrates;
C.Impairment of desintoxicative function of the liver;
D. Cachexia;
E. Increase of thrombogenesis.
6. A patient with leukemia has general number of leukocytes of 120.0x10 9/L.
What kind of leukemia does this patient have?
A. *Leukemic
B. Leucopenic
C. Subleukemic
D. Aleukemic
E. Erythremia
7. Patient with chronic leukemia has acutely increased temperature,
breathlessness, marked muscular weakness at insignificant physical exertion,
increased sweating, cough. What mechanism of leukemia influence upon
organism underlies complications in this patient?
A. *Immunodeficiency due to functional inability of leukocytes
B. Internal bleeding because metastases into vessel wall
C. Anemia
D. Tumor progression
E. Airway obstruction because of development of metastases
8. Blood examination revealed leukocytosis, lymphocytosis, Botkin Gumpreht
shades, and anemia. What disease is recognized by these findings?

A. Acute myeloleukemia.
B. *Chronic lymphoid leukosis.
C. Myelosis.
D. Infectious mononucleosis.
E. Acute leukosis.
10. The test findings of the peripheric blood of a 42-year-old patient are:
hemoglobin 80.0 g/1, erythrocytes - 3.2 -10 1 2 /l, leukocytes - 25 -109/l, the
leukocytic formula: basophils 5 %, eosinophils 9 %, myeloblasts 3 %,
promyelocytes 8 %; neutrophils: myelocytes 11 %, metamyelocytes
22 %, stab neutrophile 17 %, segmentonuclear 19 %, lymphocytes 3 %,
monocytes 3 %. What kind of blood pathology does the patient have?
A. Eritromielosis.
B. Myeloblastsc leukosis.
C. *Chronic myeliod leukemia.
D. Promielocytic leukosis.
E. Panmyelophthisis.

A
B
C
D
E

11. A patient with acute myeloblast leukosis has developed liver and spleen
enlargement, anemia, myeloblasts in peripheral blood. What principal sign
allows to differ myeloblast leukosis from chronic one?
Anemia
Pancytopenia
*Leukemic collapse
Leukemic cells in peripheral blood
Thrombocytopenia
12. Extraction of a tooth, in a patient with chronic lymphocytic leukemia, was
complicated by prolonged bleeding. What may cause the hemorrhagic
syndrome in this patient?
A. Anemia
B. Lymphocytosis
C. Eosinopoenia
D. Neutropoenia
E. *Thrombocytopoenia
13. A patient with acute myeloblast leukosis has developed liver and spleen
enlargement, anemia, myeloblasts in peripheral blood. What principal sign
allows to differ myeloblast leukosis from chronic one?
A Anemia
B Pancytopenia
C *Leukemic collapse
D Leukemic cells in peripheral blood
E Thrombocytopenia
14. Following changes were discovered in peripheral blood of patient:
erhytrocytes - 3,0x1012/l Hb-80g/l, leukocytes - 1,0x109/l. Leukocyte formula:

basophyles - 0%, eosinophyles - 0%, myeloblasts - 64%, promyelocytes - 0%;


myelocytes - 0%, metamyelocytes - 0%, stab - 1%, segmentonuclear - 8%,
lymphocytes - 24%, monocytes- 3%. Make the diagnosis due to blood picture.
A.
Chronic myeloid leukemia.
B.
Erythremia.
C.
Neutrophyle leukocytosis.
D.
*Acute myeloid leukemia
E.
Lymphocytosis
15. Following changes were discovered in peripheral blood of patient:
erhytrocytes- 3,2x1012/l, Hb 80g/l, leukocytes. - 25x109/l. Leukocyte formula:
basophyles - 5%, eosinophyles - 9%, myeloblasts - 3%, promyelocytes - 8%;
myelocytes - 11%, metamyelocytes - 22%, stab - 17%, segmentonuclear 19%, lymphocytes - 3%, monocytes - 3%. Make the diagnosis due to blood
picture..
A.
Acute myeloid leukemia
B.
Erythromyelosis
C.
Neutrophyle leukocytosis
D.
Eosinophyle leukocytosis
E.
*Chronic myeloid leukemia
16. Patient marks the promoted fatigueability, general weakness during the
last year. Following changes were discovered in peripheral blood of patient:
erhytrocytes -4.11012/l, Hb-119g/l, GV-0.87, leukocytes - 57109/l, Leukocyte
formula: metamyelocytes -0%, stab -0%, segmentonuclear -9%, eosinophyles
-0%, basophyles -0%, lymphoblastes -2%, prolymphocytes -5%, lymphocytes
-81%, -3%, platelits-160109/l, Botkin-Gumpreht bodies in the blood smear.
Make the diagnosis due to blood picture.
A.
*Chronic lymphoid leukemia.
B.
Chronic myeloid leukemia
C.
Acute lymphoblast leukemia
D.
Acute myeloid leukemia
E.
Chronic monoleukemia
17. What changes from the side of leukocyte formula can confirm a
diagnosisin the blood at the patient with chronic myeloid leukemia?
A.
Leukemic gap
B.
Botkin-Gumpreht bodies
C.
*Hyperregenerative shift of leukocyte formula to the left side
D.
Degenerative change of formula to the left
E.
Presence of metamyelocytes
18. Following changes were discovered in peripheral blood of patient:
erhytrocytes - 2,81012/l, Hb 80g/l, GV - 0,85, thrombocytes - 160109/l,

leukocytes - 60109/l; basophyles - 2%, eosinophyles - 8%, promyelocytes 5%, myelocytes - 5%, metamyelocytes - 16%, stab - 20%, segmentonuclear 34%, lymphocytes - 7%, monocytes - 3%. Make the diagnosis due to blood
picture.
A.
Acute myeloid leukemia.
B.
*Chronic myeloid leukemia
C.
Hypopoplastic anaemia.
D.
Undifferentiated leukemia.
E.
Hemolitic anaemia.
19. At patient with myeloid leukemia neutrophyle leukocytosis is determined
in blood. What variant of nuclear shift of leukocyte formula to the left is most
typical for chronic myeloid leukemia?
A.
*Hyperregenepative
B.
Regenerative
C.
Degenerative
D.
Regenerative-degenerative
E.
20. Amount of leucocytes-90109/l. In leukocyte formula: eosinophyles -1%,
basophyles -0%, metamyelocytes -0%, stab -2%, segmentonuclear -20%,
prolymphocytes -2%, lymphocytes -70%, monocytes -5%, Botkin-Gumpreht
bodies. Lymphatic nodes are ancreased. Make the diagnosis due to blood
picture.
A.
Acute lymphoid leukemia.
B.
Lymphogranulematosis.
C.
Infectious mononucleosis.
D.
*Chronic lymphoid leukemia
E.
Chronic myeloid leukemia.
*** *** ***

1. A patient underwent a surgery for excision of a cyst on pancreas. After this


he developed haemorrhagic syndrome with apparent disorder of blood
coagulation. Development of this complication can be explained by:
A Activation of fibrinolytic system
B Insufficient fibrin production
C Reduced number of thrombocytes
D Activation of anticoagulation system
E Activation of Christmas factor
2. A 43-year-old patient has thrombopenia, reduction of fibrinogen, products
of degradation of fibrin presented in the blood, petechial haemorrhage along
with septic shock. What is the most likely cause of the changes?

A DIC-syndrom
B Autoimmune thrombocytopenia
C Haemorrhagic diathesis
D Disorder of thrombocytes production
E Exogenous intoxication
3. A 70-year-old patient suffers from atherosclerosis complicated by the lower
limb thrombosis that has caused gangrene on his left toes. What is the most
likely cause of the thrombosis origin?
A Thrombocyte adhesion
B Prothrombinase activation
C Transformation of prothrombin into thrombin
D Transformation of fibrinogen into fibrin
E Impaired heparin synthesis
4. A patient with tissue trauma was taken a blood sample for the
determination of blood clotting parameters. Specify the right sequence of
extrinsic pathway activation.
A III VIIa Xa
B III IV Xa
C IV VIII: TF Xa
D IV VIIa Xa
E III VIII: TF Xa
5. A disaster fighter at a nuclear power plant developed hemorrhagic
syndrome on the background of acute radiation disease. What is the most
important factor of syndrome pathogenesis?
A Thrombocytopenia
B Vascular wall damage
C Increased activity of fibrinolysis factors
D Increased activity of anticoagulative system factors
E Decreased activity of coagulative factors
6. A patient was ill with burn disease that was complicated by DIC syndrome.
What stage of DIC syndrome can be suspected if it is known that the patient's
blood coagulates in less than 3 minutes?
A Hypercoagulation
B Transition phase
C Hypocoagulation
D Fibrinolysis
E Terminal
7 .A patient suffers from the haemorrhagic syndrome that shows itself in
frequent nasal bleedings, posttraumatic and spontaneous intracutaneous and
intra-articular haemorrhages. After a laboratory study a patient was

diagnosed with the type B haemophilia. This disease is provoked by the deficit
of the following factor of blood coagulation:
A IX
B VIII
C XI
DV
E VII
8. A 3-year-old boy with pronounced hemorrhagic syndrome doesn't have
antihemophilic globulin A (factor VIII) in the blood plasma. Hemostasis has
been impaired at the following stage:
A Internal mechanism of prothrombinase activation
B External mechanism of prothrombinase activation
C Conversion of prothrombin to thrombin
D Conversion of fibrinogen to fibrin
E Blood clot retraction
1. A patient with burn disease developed disseminated intravascular
coagulation syndrome (DIC). Time of blood coagulation is less than 3 min.
What stage of DIC might be diagnosed?
A. Hypercoagulation
B. Transitional
C. Hypocoagulation
D. Fibrinolytic
E. Terminal
2. A Chornobyl liquidator developed hemorrhagic syndrome as a sign of acute
radiation sickness. What is the most significant in pathogenesis of the
syndrome?
A. Thrombocytopenia
B. Impairment of the structure of vessels wall
C. Increasing activity of fibrinolytic factors
D. Increasing activity of blood anticoagulation factors
E. Decreasing activity of blood coagulation factors
3. Haemophilia was diagnosed in a child with hemorrhagic syndrome. It is
caused by deficit of what factor?
A. IX (Kristmass)
B. II (prothrombin)
C. VIII (antihaemophilic globulin)
D. XI (prothromboplastin)
E. XII (Hageman's)
4. A hereditary type of coagulopathy with defect of VIII factor of blood
coagulability was diagnosed in a patient. Name the phase of blood
coagulability in which the primary disorders of coagulation take place.

A. Thromboplastin formation
B. Thrombin formation
C. Fibrin formation
D. Clot retraction
E. Clot sedimentation
1. A patient was admitted to the hospital with abundant hemorrhoid bleeding.
This patient has been suffering from hepatic cirrhosis for a long time. What is
the reason for hemorrhage development under hepatic cirrhosis?
A. Activation of fibrinolysis
B. Plasmin deficiency
C. *Prothrombin deficiency
D. Low concentration of thrombostenin in blood
E. Excess of heparin
2. A patient was ill with burn disease that was complicated by DIC syndrome.
What stage of DIC syndrome can be suspected if it is known that the patients
blood coagulates in less that 3 minutes?
A *Hypercoagulation
B Hypocoagulation
C Terminal
D Fibrinolysis
E Transition phase
3. Hemorrhagic syndrome connected to disorders of the third phase of
coagulation developed in a patient after hi was operated on pancreas. What is
the possible mechanism of development of hemostasis disorder?
A. Elevation in content of heparin in patients blood
B. Reduction in fibrinogen synthesis
C. *Activation of fibrinolysis
D. Deficit of fibrin stabilizing factor
E. Reduction in prothrombin synthesis
4. Shonlein-Henoch disease was diagnosed in a patient. What changes in blood
cells number are characteristic for this disease?
A. Eosinophilia
B. *Thrombocytopenia
C. Erythropoenia
D. Polycytemia
E. Eosinopoenia
5. Patient has hemorrhage from gums, subcutaneous hemorrhages, and
frequent nasal bleedings. Thrombocytopenia was revealed in blood test of this
patient. What is the reason for bleeding development in case of
thrombocytopenia?
A. Thrombin deficiency

B. *Reduction of thromboplastin formation


C. Excessive heparin formation
D. Activation of fibrinolysis
E. Excess of prostacyclins
6. A boy has congenital disorder of hemostasis: he has prolonged hemorrhage
even in case of insignificant injuries, subcutaneous bruises, and bleedings in
joint cavities, which restrict the movement activity. Patients blood does not
coagulate for a long time if it is taken out from the organism and it does not
contain coagulation factor VIII. Blood cells count including number of
platelets is within norm. What underlies the congenital disease of this boy?
A. *Hereditary gene defect linked with X chromosome
B. Toxicosis of pregnancy in boys mother
C. Intrauterine infection
D. Intrauterine intoxication
E. Intrauterine immune conflict
7. Hemophilia B was diagnosed in a child who has hemorrhagic syndrome.
This type of hemophilia results from absence of:
A. *Coagulation factor IX (Christmass factor)
B. Coagulation factor II (prothrombin)
C. Coagulation factor VIII (antihemophilic globulin)
D. Coagulation factor XI (thromboplastin)
E. Coagulation factor XII (Hagemans factor)
8. Hemorrhagic syndrome connected to disorders of the third phase of
coagulation developed in a patient after hi was operated on pancreas. What is
the possible mechanism of development of hemostasis disorder?
A. Elevation in content of heparin in patients blood
B. Reduction in fibrinogen synthesis
C. *Activation of fibrinolysis
D. Deficit of fibrin stabilizing factor
E. Reduction in prothrombin synthesis
9. A female patient, aged 25, was admitted to the hematological department
with complains of the appearance of hemorrhages of different sizes on the
body; during menstruation there are uterine bleedings. She has been ill for ten
years. Paleness of skin and mucous membranes were determined on
examination; there are hemorrhages of different size and color on the upper
and lower extremities. Pulse = 100 beats/minute, AP 110/70 mmHg. Blood
analysis shows: erythrocytes 3.3*1012 /L, Hb 80g/L, thrombocytes 33*109/L;
time of blood coagulation: beginning is at 2nd minute, end is at 6th minute;
time of bleeding (according to Duke) - 15 minutes. What is the possible
diagnosis?
A. Marchiafava-Michelli disease.

B. *Thrombocytopoenic purpura.
C. Glanzmans thrombasthenia.
D. Willeberandt-Yurgens thrombocytopathia
E. Chronic myelogenous leukemia.
10. Such appearances as petechiae and ecchymoses develop in a boy aged 7,
who often fall ill with acute respiratory diseases. Pathology of internal organs
is absent in this patient. What pathology is present in this case?
A. Hypoplastic anemia
B. *Thrombocytopenia
C. Acute leukemia
D. Hemophilia
E. Chronic leukemia
11. A 30-years-old female patient, who is suffering from megrim, often takes
analgin. Hemorrhages on skin and frequent nasal bleedings appear in her at
the recent time. At her blood analysis: number of platelets is 30x10 9/L;
bleeding time is increased. What do these changes result from?
A. *Autoimmune thrombocytopenia
B. Hemorrhagic vasculitis
C. Hemolytic anemia
D. Angiohemophilia
E. Thrombocytopathy
12. Antihemophilic globulin A (factor VIII) is absent in the blood plasma of a
boy with significant hemorrhagic syndrome. What phase of hemostasis is
infringed primarily in this boy?
A. Retraction of blood clot
B. Conversion of fibrinogen to fibrin
C. Conversion of prothrombin to thrombin
D. Extrinsic pathway of prothrombinase (thrombokinase) activation
E. *Intrinsic pathway of prothrombinase (thrombokinase) activation
13. A 12-years-old patient was admitted to the hospital with hemarthrosis of
knee joint (hemorrhage into joint cavity). This patient has been suffering from
hemorrhages since early childhood. What disease does this boy suffer from?
A. *Hemophilia
B. Hemorrhagic vasculitis
C. Iron deficiency anemia
D. Vitamin B12 deficiency anemia
E. Thrombocytic purpura
14. A worker at pharmaceutical plant addresses the doctor with complaints of
general malaise, significant hemorrhages from gums, nasal bleedings, and
numerous subcutaneous hemorrhages. At blood analysis of this patient

following was revealed: number of erythrocytes is 2.2x1012, content of


hemoglobin is 48 g/L, presence of neutropenia with relative lymphocytosis,
and number of platelets is 35x109/L. What is the possible pathogenesis of
thrombocytopenia in this patient?
A. *Decrease of platelet production
B. Enhanced platelet destruction
C. Increase platelet utilization
D. Redistribution of platelets
E. Increased loss of platelets
15. Extraction of a tooth, in a patient with chronic lymphocytic leukemia, was
complicated by prolonged bleeding. What may cause the hemorrhagic
syndrome in this patient?
A. Anemia
B. Lymphocytosis
C. Eosinopoenia
D. Neutropoenia
E. *Thrombocytopoenia
16. Changes of some indices of blood were revealed at examination of a patient
suffered from hemophilia. What of enumerated indices corresponds to this
condition?
A. Thrombocytopenia
B. Bleeding time by Duke takes longer
C. Eosinophilia
D. Time of coagulation takes longer
E. Afibrinogenemia
17. A patient with liver disease revealed the decreasing of prothrombin level in
the blood. It can, first of all, result in the impairment of:
A The first phase of the coagulatory hemostasis
B Fibrinolysis
C Vascular-thrombocytic hemostasis
D The second phase of the coagulatory hemostasis
E Anticoagulative properties of the blood

18. A patient suffers from the haemorrhagic syndrome that shows itself in
frequent nasal bleedings, posttraumatic and spontaneous intracutaneous and
intra-articular haemorrhages. After a laboratory study a patient was

diagnosed with the type B haemophilia. This disease is provoked by the deficit
of the following factor of blood coagulation:
A.IX
B.VII
C.VIII
D.XI
E.V
19. A patient underwent a surgery for excision of a cyst on pancreas. After this
he developed haemorrhagic syndrome with apparent disorder of blood
coagulation.
Development of this complication can be explained by:
A. Reduced number of thrombocytes
B. Activation of Christmas factor
C. Insufficient fibrin production
D .Activation of fibrinolytic system
E. Activation of anticoagulation system
*** *** ***

***

***

1. ECG of a 44-year-old patient shows signs of hypertrophy of both ventricles


and the right atrium. The patient was diagnosed with the tricuspid valve
insufficiency. What pathogenetic variant of cardiac dysfunction is usually
observed in case of such insufficiency?
AHeart overload by volume
B Heart overload by resistance
CPrimary myocardial insufficiency
D Coronary insufficiency
E Cardiac tamponade
2. Dystrophic changes of the heart muscle are accompanied with cardiac
cavity enlargement, decrease of the strength of heart contraction, increased
amount of blood, which remains in the heart during systolic phase, overfilled
veins. For what state of heart is it characteristic?
AMyogenic dilatation
B Tonogenic dilatation
CEmergency stage of hyperfunction and hypertrophy
D Cardiosclerosis
E Tamponage of the heart
3. After a serious psycho-emotional stress a 45-year-old patient suddenly felt
constricting heart pain irradiating to the left arm, neck and left scapula. His
face turned pale, the cold sweat stood out on it. The pain attack was stopped
with nitroglycerine. What process has developed in this patient?

AStenocardia
B Myocardial infarction
CStroke
D Psychogenic shock
E Stomach ulcer perforation
4. A 50 year old patient suffers from essential hypertension. After a physical
stress he experienced muscle weakness, breathlessness, cyanosis of lips, skin
and face. Respiration was accompanied by distinctly heard bubbling rales.
What mechanism underlies the development of this syndrome?
AAcute left-ventricular failure
B Chronic right-ventricular failure
CChronic left-ventricular failure
D Collapse
E Cardiac tamponade
5. After a serious psychoemotional stress a 48 year old patient suddenly
developed acute heart ache irradiating to the left arm. Nitroglycerine relieved
pain after 10 minutes. What is the leading pathogenetic mechanism of this
process development?
ASpasm of coronary arteries
B Dilatation of peripheral vessels
CObstruction of coronary vessels
D Compression of coronary vessels
E Increase in myocardial oxygen consumption
6. A 59 year old patient is a plant manager. After the tax inspection of his
plant he felt intense pain behind his breastbone irradiating to his left arm. 15
minutes later his condition came to normal. Which of the possible mechanisms
of stenocardia development is the leading in this case?
AHigh catecholamine concentration in blood
B Coronary atherosclerosis
CIntravascular aggregation of blood corpuscles
D Coronary thrombosis
E Functional heart overload
7. The patient with acute miocardial infarction was given intravenously
different solutions during 8 hours with medical dropper 1500 ml and oxygen
intranasally. He died because of pulmonary edema. What caused the
pulmonary edema?
AVolume overload of the left ventricular
B Decreased oncotic pressure due to hemodilution
CAllergic reaction
D Neurogenic reaction

E Inhalation of the oxygen


8. A patient who suffers from acute myocarditis has clinical signs of
cardiogenic shock. What of the under-mentioned pathogenetic mechanisms
plays the main part in shock development?
ADisturbance of pumping ability of heart
B Depositing of blood in organs
CReduction of diastolic flow to the heart
D Decrease of vascular tone
E Increase of peripheral vascular resistance
9. A patient suffering from stenocardia was taking nitroglycerine which
caused restoration of blood supply of myocardium and relieved pain in the
cardiac area. What intracellular mechanism provides restoration of energy
supply of insulted cells?
AIntensification of ATP resynthesis
B Reduction of ATP resynthesis
CIncreased permeability of membranes
D Intensification of oxygen transporting into the cell
E Intensification of RNA generation
10. In course of a preventive examination of a miner a doctor revealed
changes of cardiovascular fitness which was indicative of cardiac insufficiency
at the compensation stage. What is the main proof of cardiac compensation?
AMyocardium hypertrophy
B Tachycardia
CRise of arterial pressure
D Dyspnea
E Cyanosis
11. A patient ill with essential arterial hypertension had a hypertensic crisis
that resulted in an attack of cardiac asthma. What is the leading mechanism
of cardiac insufficiency in this case?
AHeart overload caused by high pressure
B Heart overload caused by increased blood volume
CAbsolute coronary insufficiency
D Myocardium damage
E Blood supply disturbance
12. Transmural myocardial infarction in the patient was complicated with
progressive acute left ventricle insufficiency. What is the most typical for this
state?
A Edema of the lungs
B Edema of the extremities
C Cyanosis
D Ascites

E Arterial hypertension
13. Arterial hypertention is caused by the stenosis of the renal arteries in the
patient. Activation of what system is the main link in the pathogenesys of this
form of hypertension?
A Renin-angiotensin
B Sympathoadrenal
C Parasympathetic
D Kallikrein-kinin
E Hypothalamic-pituitary

1. ECG of a 44-year-old patient shows signs of hypertrophy of both ventricles


and the right atrium. The patient was diagnosed with the tricuspid valve
insufficiency. What pathogenetic variant of cardiac dysfunction is usually
observed in case of such insufficiency?
A Heart overload by volume
B Heart overload by resistance
C Primary myocardial insufficiency
D Coronary insufficiency
E Cardiac tamponade
2. Dystrophic changes of the heart muscle are accompanied with cardiac
cavity enlargement, decrease of the strength of heart contraction, increased
amount of blood, which remains in the heart during systolic phase, overfilled
veins. For what state of heart is it characteristic?
A Myogenic dilatation
B Tonogenic dilatation
C Emergency stage of hyperfunction and hypertrophy
D Cardiosclerosis
E Tamponage of the heart
3. The patient with acute miocardial infarction was given intravenously
different solutions during 8 hours with medical dropper 1500ml and oxygen
intranasally. He died because of pulmonary edema. What caused the
pulmonary edema?
A Volume overload of the left ventricular
B Decreased oncotic pressure due to hemodilution
C Allergic reaction
D Neurogenic reaction
E Inhalation of the oxygen

1 A patient suffering from stenocardia was taking nitroglycerine which


caused restoration of blood supply of myocardium and relieved pain in
the cardiac area. What intracellular
mechanism
provides
restoration of energy supply of insulted cells?
A Intensification of ATP resynthesis
B Reduction of ATP resynthesis
C Increased permeability of membranes
D Intensification of oxygen transporting into the cell
E Intensification of RNA generation
2 In course of a preventive examination of a miner a doctor revealed
changes of cardiovascular fitness which was indicative of cardiac
insufficiency at the compensation stage. What is the main proof of
cardiac compensation?
A Myocardium hypertrophy
B Tachycardia
C Rise of arterial pressure
D Dyspnea
E Cyanosis
3 A patient ill with essential arterial hypertension had a hypertensic crisis
that resulted in an attack of cardiac asthma. What is the leading
mechanism of cardiac insufficiency in this case?
A Heart overload caused by high pressure
B Heart overload caused by increased blood volume
C Absolute coronary insufficiency
D Myocardium damage
E Blood supply disturbance
4 Transmural myocardial infarction in the patient was complicated with
progressive acute left ventricle insufficiency. What is the most typical
for this state?
A Edema of the lungs
B Edema of the extremities
C Cyanosis
D Ascites
E Arterial hypertension
5 A patient who suffers from acute myocarditis has clinical signs of
cardiogenic shock. What of the under-mentioned pathogenetic
mechanisms plays the main part in shock development?
A Disturbance of pumping ability of heart
B Depositing of blood in organs

C Reduction of diastolic flow to the heart


D Decrease of vascular tone
E Increase of peripheral vascular resistance
**8 *** ***

. A patient has been diagnosed with influenza. His condition became


drastically worse after taking antipyretic drugs. His consciousness is
confused, AP is 80/50 mm Hg, Ps is 140/m, body temperature droped down to
35,8oC. What complication developed in this patient?
ACollapse
B Hyperthermia
CHypovolemia
D Acidosis
E Alkalosis
2. A 70 year old man is ill with vascular atherosclerosis of lower extremities
and coronary heart disease. Examination revealed disturbance of lipidic blood
composition. The main factor of atherosclerosis pathogenesis is the excess of
the following lipoproteins:
ALow-density lipoproteins
B Cholesterol
CHigh-density lipoproteins
D Intermediate density lipoproteins
E Chylomicrons
3. A 58-year-old patient suffers from the cerebral atherosclerosis.
Examination revealed hyperlipoidemia. What class of lipoproteins will most
probably show increase in concentration in this patients blood serum?
ALow-density lipoproteins
B High-density lipoproteins
CFatty acid complexes with albumins
D Chylomicrons
E Cholesterol
1. Shock and signs of acute renal failure (ARF) developed in the patient due
to permanent injury. What is the leading cause of development of ARF in the
case?
A Decreased arterial pressure
B Urine excretion violation
C Increased pressure in the nephron capsule
D Increased pressure in the renal arteries
E Decreased oncotic BP

2. Arterial hypertention is caused by the stenosis of the renal arteries in the


patient. Activation of what system is the main link in the pathogenesys of this
form of hypertension?
A Renin-angiotensin
B Sympathoadrenal
C Parasympathetic
D Kallikrein-kinin
E Hypothalamic-pituitary
3. After a serious psycho-emotional stress a 45-year-old patient suddenly felt
constricting heart pain irradiating to the left arm, neck and left scapula. His
face turned pale, the cold sweat stood out on it. The pain attack was stopped
with nitroglycerine. What process has developed in this patient?
A Stenocardia
B Myocardial infarction
C Stroke
D Psychogenic shock
E Stomach ulcer perforation
4. A 70 year old man is ill with vascular atherosclerosis of lower extremities
and coronary heart disease. Examination revealed disturbance of lipidic blood
composition. The main factor of atherosclerosis pathogenesis is the excess of
the following lipoproteins:
A Low-density lipoproteins
B Cholesterol
C High-density lipoproteins
D Intermediate density lipoproteins
E Chylomicrons
*** *** ***

1. Person has stable HR, not more than 40 bpm. What is the pacemaker of the
heart rhythm in this person?
AAtrioventricular node
B Sinoatrial node
CHis' bundle
D Branches of His' bundle
E Purkinye' fibers
2. In a 45-year-old patient on ECG it was revealed: sinus rhythm, the number
of auricular complexesexceeds number of ventricular complexes; progressing
extension of the P-Q interval from complex to complex; fallout of some
ventricular complexes; waves and QRST complexes are without changes.
Name the type of heart rhythm disfunction.

AAtrioventricular block of the II degree


B Synoauricular block
CAtrioventricular blockade of the I degree
D Intraatrial block
E Complete atrioventricular block
3. Processes of repolarisation are disturbed in ventricular myocardium in
examined person. It will cause amplitude abnormalities of configuration and
duration of the wave:
A
BQ
CR
DS
EP
4. A 49 y.o. woman consulted a doctor about heightened fatigue and dyspnea
during physical activity. ECG: heart rate is 50/min, PQ is extended, QRS is
unchanged, P wave quanity exceeds quantity of QRS complexes. What type of
arrhythmia does the patient have?
AAtrioventricular block
B Extrasystole
CSinus bradycardia
D Ciliary arhythmia
E Sinoatrial block
3. A patient has extrasystole. ECG shows no P wave, QRS complex is
deformed, there is a full compensatory pause. What extrasystoles are
these?
AVentricular
B Atrial
CAtrioventricular
D Sinus
E4. A 45 year old patient was admitted to the cardiological department.
ECG data: negative P wave overlaps QRS complex, diastolic interval is
prolonged after extrasystole. What type of extrasystole is it?
AAtrioventricular
B Sinus
CAtrial
D Ventricular
E Bundle-branch
5. A 67 year old patient complains of periodic heart ache, dyspnea during
light physical activities. ECG reveals extraordinary contractions of

heart ventricles. Such arrhythmia is called:


AExtrasystole
B Bradycardia
CTachycardia
D Flutter
E Fibrillation

1. Person has stable HR, not more than 40 bpm. What is the pacemaker of the
heart rhythm in this person?
A Atrioventricular node
B Sinoatrial node
C His' bundle
D Branches of His' bundle
E Purkinye' fibers
2. In a 45-year-old patient on ECG it was revealed: sinus rhythm, the number
of auricular complexesexceeds number of ventricular complexes; progressing
extension of the P-Q interval from complex to complex; fallout of some
ventricular complexes; waves and QRST complexes are without changes.
Name the type of heart rhythm disfunction.
A Atrioventricular block of the II degree
B Synoauricular block
C Atrioventricular blockade of the I degree
D Intraatrial block
E Complete atrioventricular block
3. Processes of repolarisation are disturbed in ventricular myocardium in
examined person. It will cause amplitude abnormalities of configuration and
duration of the wave:
A
BQ
CR
DS
EP
4. ECG of a patient shows such alterations: P-wave is normal, P-Q-interval is
short, ventricular QRST complex is wide, R-wave is double-peak or twophase. What form of arrhythmia is it?
A WPW syndrome (Wolff-Parkinson-White)
B Frederick's syndrome (atrial flutter)

C Atrioventricular block
D Ventricular fibrillation
E Ciliary arrhythmia
6. A 45 year old patient was admitted to the cardiological department.
ECG data: negative P wave overlaps QRS complex, diastolic interval is
prolonged after extrasystole. What type of extrasystole is it?
A Atrioventricular
B Sinus
C Atrial
D Ventricular
E Bundle-branch
6. A 49 y.o. woman consulted a doctor about heightened fatigue and dyspnea
during physical activity. ECG: heart rate is 50/min, PQ is extended, QRS is
unchanged, P wave quanity exceeds quantity of QRS complexes. What type of
arrhythmia does the patient have?
A Atrioventricular block
B Extrasystole
C Sinus bradycardia
D Ciliary arhythmia
E Sinoatrial block

7. An animal with aortic valve insufficiency got hypertrophy of its left heart
ventricle. Some of its parts have local contractures. What substance
accumulated in the myocardiocytes caused these contractures?
A Calcium
B Potassium
C Lactic acid
D Carbon dioxide
E Sodium
8. A patient has extrasystole. ECG shows no P wave, QRS complex is
deformed, there is a full compensatory pause. What extrasystoles are these?
A Ventricular
B Atrial
C Atrioventricular
D Sinus
E*** ******

1. A 62-year-old patient was admitted to the neurological department due to


cerebral haemorrage. Condition is grave. There is observed progression of
deepness and frequency of breath that turnes into reduction to apnoea,and the
cycle repeates. What respiration type has developed in the patient?

ACheyne-Stockes respiration
B Kussmaul respiration
CBiot's respiration
D Gasping respiration
E Apneustic respiration
2. A patient staying in the pulmonological department was diagnosed with
pulmonary emphysema accompanied by reduced elasticity of pulmonary
tissue. What type of respiration is observed?
AExpiratory dyspnea
B Inspiratory dyspnea
CSuperficial respiration
D Infrequent respiration
E Periodic respiration

3. An unconscious young man with signs of morphine poisoning entered


admission office. His respiration is shallow and infrequent which is caused by
inhibition of respiratory centre. What type of respiratory failure is it?
AVentilative dysregulatory
B Ventilative obstructive
CVentilative restrictive
D Perfusive
E Diffusive

4. A patient with bronchial asthma has developed acute respiratory failure.


What kind of respiratory failure occurs in this case?
AObstructive disturbance of alveolar ventilation
B Restrictive ventilatory defect
CPerfusion
D Diffusion
E Dysregulation of alveolar ventilation
5. A 23-year-old patient has been admitted to a hospital with a craniocerebral
injury. The patient is in a grave condition. Respiration is characterized by
prolonged convulsive inspiration followed by a short expiration. What kind of
respiration is it typical for?
A Apneustic
B Gasping breath
C Kussmaul's
D Cheyne-Stokes
E Biot's

1. A 62-year-old patient was admitted to the neurological department due to


cerebral
haemorrage. Condition is grave. There is observed progression of deepness
and frequency of breath that turnes into reduction to apnoea,and the cycle
repeates. What respiration type has developed in the patient?
A Cheyne-Stockes respiration
B Kussmaul respiration
C Biot's respiration
D Gasping respiration
E Apneustic respiration
2. A 12 y.o. boy who suffers from bronchial asthma has an acute attack of
asthma: evident expiratory dyspnea, skin pallor. What type of alveolar
ventilation disturbance is it?
A Obstructive
B Restrictive
C Throracodiaphragmatic
D Central
E Neuromuscular
1
A group of mountain climbers went through the blood analysis at the
height of 3000 m. It revealed decrease of HCO 3 to 15 micromole/l (standard is
22-26 micromole/l). What is the mechanism of HCO3 decrease?
A Hyperventilation
B Intensification of acidogenesis
C Hypoventilation
D Decrease of ammoniogenesis
E Decrease of bicarbonate reabsorption in kidneys

2
Examination of a miner revealed pulmonary fibrosis accompanied by
disturbance of alveolar ventilation. What is the main mechanism of this
disturbance?
A Limitation of respiratory surface of lungs
B Constriction of superior respiratory tracts
C Disturbance of neural respiration control
D Limitation of breast mobility
E Bronchi spasm
3
A patient staying in the pulmonological department was diagnosed with
pulmonary
emphysema accompanied by reduced elasticity of pulmonary tissue. What
type of respiration is observed?
A Expiratory dyspnea

B Inspiratory dyspnea
C Superficial respiration
D Infrequent respiration
E Periodic respiration
4
An unconscious young man with signs of morphine poisoning entered
admission office. His respiration is shallow and infrequent which is caused by
inhibition of respiratory centre. What type of respiratory failure is it?
A Ventilative dysregulatory
B Ventilative obstructive
C Ventilative restrictive
D Perfusive
E Diffusive
5
While having the dinner the child choked and aspirated the food. Meavy
cough has started, skin and mucose are cyanotic, rapid pulse, rear breathing,
expiration is prolonged. What disorder of the external breathing developed in
the child?
A Stage of expiratory dyspnea on asphyxia
B Stage of inspiratory dyspnea on asphyxia
C Stenotic breathing
D Alternating breathing
E Biot's breathing
*** *** ***

1. Measurements of the arterial pCO2 and pO2 during an attack of bronchial


asthma revealed hypercapnia and hypoxemia respectively. What kind of
hypoxia occurred in this case?
ARespiratory
B Hemic
CCirculatory
D Tissue
E Histotoxic
1. A 29-year-old patient with carbon monoxide poisoning was admitted to the
hospital. He had signs of severe hypoxia: pronounced dyspnea, cyanosis and
tachycardia. Which changes of hemoglobin take place during carbon
monoxide poisoning?
A. Methemoglobin formation
B. Carboxyhemoglobin formation
C. Carbhemoglobin formation
D. Sulfhemoglobin formation
E. Oxyhemoglobin inactivation

2. There are the decrease of blood erythrocytes, hemoglobin, color index, the
concentration of serum iron, mikroanizocytosis, poykilocytosis at the patient
blood. These changes were accompanied by development of hypoxia. What
type of hypoxia observed in this case?
A. Hemic
B. Hypoxic
C. Circulatory
D. Tissues
E. Respiratory
3. The doctor-researcher within the mountaineering expedition climbed to the
base camp situated at an altitude of 5000 meters. On the 3rd day he complains
of breathlessness, palpitation, headache, dizziness, and ringing in his. What
type of hypoxia occurs in this case?
A. Hypoxic
B. Hemic
C. Circulatory
D. Tissues
E. Mixed
4. Woman, 56 years, suffered from thyrotoxicosis for a long time. What type of
hypoxia could occur in this instance?
A. Tissues
B. Hemic
C. Circulatory
D. Respiratory
E. Mixed
1. A man constantly lives in mountains. What changes of blood test indices can
found in him?
A. Increased red blood cells number
B. Decreased reticulocytes count
C. Decreased color index
B. Appearance of erythroblasts in blood
E. Decreased hemoglobin content
2. After autopsy a forensic medical expert established that death of 20-yearold woman resulted from cyanide poisoning. What process disorder caused
the death?
A. Oxygen transport by hemoglobin
B. Tissue respiration
C. Hemoglobin synthesis
D. Urea synthesis
E. Oxidative phosphorilation

3. Increased number of erythrocytes in blood test was revealed in people


residing in village, which is situated in mountains at the altitude of 3000 m.
What is the reason for changes in their blood?
A. Increased vitamin B12 synthesis
B. Blood clotting
C. Change of spleen function
D. Increased erythropoietin production
E. Increased circulating blood volume
4. The group of patients from sanatorium went to the mountains for
excursion. Tachycardia and breathlessness developed in part of them in two
hours after beginning of excursion. Which type of hypoxia do those
disturbances result from?
A. Hemic
B. Hypoxic
C. Respiratory
D. Tissue
E. Circulatory
5. A driver slept in garage in a car with working engine. When he woke he had
headache and later vomiting began in him. What compound formation in
blood caused this state?
A. Carbhemoglobin
B. Methemoglobin
C. Carboxyhemoglobin
D. Desoxihemoglobin
E. Oxihemoglobin
6. A 40-years-old man took cyanic potassium by mistake. He died instantly.
What mitochondrial enzymes do cyanides block?
A. NAD-dependent Dehydrogenases
B. Cytochrome Oxydase
C. Cytochrome B
D. Cytochrome C
E. FAD-dependent enzymes
7. The number of erythrocytes in alpinists blood before going to mountains is
4,5x1012/L. What changes of erythrocytes number can develop at the altitude
of 2500 m above sea level?
A. Absolute erythropenia
B. Absolute erythrocytosis
C. Relative erythropenia
D. Relative erythrocytosis
E. There wont be any changes
8. Dyspnea, increased heart beat rate, weaken of attention, foolishness
occurred in sportsmen without acclimatization during the ascent to the

altitude of 3000 m. Then weakness and disturbance of muscular coordination


occurred. What was the reason for that condition?
A. Circulatory hypoxia
B. Hypoxic hypoxia
C. Respiratory hypoxia
D. Hemic hypoxia
E. Hypercapnia
9. Environmental pollution by nitric compounds occurs after accident at a
chemical factory. People, who live at this region, have sharp weakness,
headache, breathlessness, and giddiness. What does hypoxia result from?
A. Cytochrome Oxidase inactivation
B. Dehydrogenases suppression
D. Carboxyhemoglobin formation
E. Decreased function of FAD-dependent enzymes
C. Methemoglobin formation
10. Total respiratory insufficiency was noticed on examination of blood gases
partial pressure in a patient with bronchial asthma. What is the reason of
respiratory hypoxia in this case?
A. Increased perfusion
B. Decreased diffusion
C. Increased ventilation
D. Decreased ventilation
E. Increased diffusion
11. Total respiratory insufficiency developed in a patient with lung disease. It
manifested by decreased pO2 and increased pCO2 in patients blood. What
caused development of respiratory hypoxia and pronounced respiratory
insufficiency?
A. Excessive hyperventilation
B. Frequent shallow respiration
C. Oxygen deficit in inspired air
D. Uneven ventilation of the lungs
E. Functional blood shunting in the lungs
12. Disorders of oxyhemoglobin formation have led to development of hemic
hypoxia. What are the reasons for hemic hypoxia development?
A. Methemoglobin formation
B. Inhibition of dehydrogenases
C. Pronounced polycytemia
D. Activation of Glutation Peroxidase
E. Deficiency of riboflavinum

13. A man has been living high in mountains for a long time. What changes in
his blood would develop?
A. Increase in number of leukocytes
B. Increase in diameter of blood vessels
C. Decrease in number of leukocytes
D. Increase in quantity of hemoglobin
E. Rare pulse
14. Tachypnea and hypopnoe developed in tourists, which had climbed to the
altitude of 3000 m. These changes are consequence of stimulation of
A. Chemoreceptors of carotid sinus
B. Mechanoreceptors of pulmonary alveoli
C. Baroreceptors of arch of aorta
D. Neurons of the cerebral cortex
E. Motoneurons of spinal cord
15. A dog has poisoning by unknown substance that caused its immediate
death because of oxidation of cytochromes. What is that substance?
A. Potassium chloride
B. Potassium cyanide
C. Potassium sulfate
D. Potassium orotate
E. Potassium permanganate

16. Gas alkalosis developed in a group of alpinists due to rise to the Everests
top. So, carbon dioxide partial pressure in their arterial blood makes up
A. 40 mmHg
B. 50 mmHg
C. 30 mmHg
D. 60 mmHg
E. 70 mmHg
17. Sensitivity of different tissues to oxygen deficit depends on their metabolic
rate, power of glycolytic system, reserve of ATP and ADP, potential possibility
of genetic apparatus to provide the plastic consolidation of hyperdeduction.
Which system is in the worst conditions from these points of view?
A. Nervous system
B. Cardiovascular system
C. Respiratory system
D. Gastrointestinal system
E. Urinary system

18. A patient addressed to the doctor with complaints of the attack of


breathlessness and dizziness. It was revealed that he worked at the chemical
factory, producing senile acid. Those signs can be explained by disturbance of
enzyme
A. Succinate dehydrogenase
B. Cytochrome oxydase
C. Catalase
D. Lactate dehydrogenase
E. Pyruvate dehydrogenase
19. A child has been brought to the admissions office. He had nitrates
poisoning symptoms such as cyanosis, dyspnea, and cramps. What is the
mechanism of those symptoms development?
A. Oxyhemoglobin formation
B. Reduce hemoglobin formation
C. Carbhemoglobin formation
D. Methemoglobin formation
E. Carboxyhemoglobin formation
20. Sense of muscular weakness, dyspnea, and tachycardia appeared in a
tourist at the altitude of 5200 m. What was the reason for appearance of these
symptoms?
A. Decrease in oxygen partial pressure in expired air
B. Decrease in atmospheric pressure
C. Increase in carbon dioxide partial pressure in patients blood
D. Gas embolism
E. Air embolism
21. A fireman with signs of carbon monoxide poisoning has been brought from
the place of accident. Which type of hypoxia is more possible in that situation?
A. Hemic hypoxia
B. Ischemic hypoxia
C. Stagnant type of circulatory hypoxia
D. Hypoxic hypoxia
E. Respiratory hypoxia
22. A 60-year-old man was admitted to the hospital with carbon monoxide
poisoning. What is mechanism of disturbances that man has?
A. Inactivation of hemoglobin respiratory function
B. Insufficiency of lungs ventilation
C. Disturbance of blood circulation in the organs and tissues
D. Decrease in partial pressure of oxygen in alveoli
E. Increase in adrenaline secretion
23. Which reason can lead to hypoxia of exogenous type?
A. Acute bleeding

B. Air embolism
C. Gas embolism
D. Insufficiency of ventilation
E. Decreased partial pressure in inspired air
24. A patient has been suffering from disease of cardiovascular system for a
long time. Chronic blood circulation insufficiency developed in him
eventually. Which mechanism of long-term adaptation to hypoxia takes place
in this patient?
A. Activation of hemopoiesis
B. Tachypnea
C. Tachycardia
D. Mobilization of blood from depots
E. Interorganic redistribution of blood circulation
25. A patient has been taken to resuscitation department. Considerable
content of sulfhemoglobin was revealed in his blood. Which type of hypoxia
does patient have?
A. Hemic
B. Respiratory
C. Circulatory
D. Tissue
E. Exogenous
26. Depressurizing of an airplane cabin happened due to emergency
conditions at the altitude of 5000 m. What is the main mechanism of damages
inflicted on the people that were in the airplane?
A. Hypoxic hypoxia
B. Gas embolism
C. Tissue embolism
D. Respiratory hypoxia
E. Circulatory hypoxia
27. Symptoms of hypoxia appeared in a patient after the course of treatment
with amidopirin and sulfanilamides. What is the reason for development of
this type of hypoxia?
A. Formation of methemoglobin
B. Formation of carboxyhemoglobin
C. Formation of nitroxyhemoglobin
D. Formation of carbhemoglobin
E. Formation of oxyhemoglobin

28. A man has been brought in hospital by ambulance. He was unconscious


because of carbon monoxide poisoning. What substance accumulation in the
blood underlies this condition?
A. Carboxyhemoglobin
B. Oxyhemoglobin
C. Desoxyhemoglobin
D. Methemoglobin
E. Carbhemoglobin
29. A 65-year-old patient suffers from aortic valve stenosis. She has symptoms
of heart failure such as dyspnea, cyanosis, and edema that appeared after
viral infection. She was admitted to the therapeutic department. Which type
of hypoxia is in the patient?
A. Circulatory
B. Hemic
C. Hypoxic
D. Respiratory
E. Tissue
30. A 70-year-old patient underwent medical treatment for ischemic heart
disease, heart failure in cardiological department. Which type of hypoxia was
in the patient?
A. Circulatory
B. Hemic
C. Respiratory
D. Tissue
E. Mixed
31. A 13-year-old girl undergoes treatment for iron-deficiency anemia in
hematological department. Which type of hypoxia does this patient have?
A. Hemic
B. Circulatory
C. Tissue
D. Respiratory
E. Mixed
32. Group of tourists ascended to the altitude of 4200 m. Three alpinists
complained of headache, pain in their ears and frontal sinuses, somnolence,
considerable muscular weakness, irritability during the ascent. The possible
reason for appearance of these symptoms was:
A. Altitude sickness
B. Hyperbaric oxygenetion
C. Caisson disease
D. Gas saturation in blood
E. Altitude decompression

33. Patient, 56 years old, suffering from thyrotoxicosis for a long time. What
type of hypoxia can develop at this disease?
A. Tissue
B. Respiratory
C. Hemic
D. Mixed
E. Circulatory
34. Senile acid and cyanides cover to the strongest poisons. Depending on a
dose death comes after a few seconds or minutes. Is oppressing activity of
what enzyme is reason of death?
A. Cytochrome-c-oxidase
B. Catalase
C. Acetylcholinesterase
D. ATP - synthetase
E. Methemoglobinreductase
35. A woman lost about 800 ml of blood during delivery. Tachycardia,
tachipnoe 28 per minute, BP-100/70 mm Hg are marked. What type of
Hypoxia initially developed at such clinical situation?
A. Mixed
B. Respiratory
C. Hemic
D. Cardiovascular
E. Tissue
36. The blood test was done at the alpinists group on height of 3000 meters.
Decrease of 3 to 15 mmol/l (norm - 22-26 mmol/l). What mechanism of
3 decreasing in the blood?
A. Decreasing of ammoniogenesis
B. Increasing of acidogenesis
C. Hypoventilation
D. Hyperventilation
E. Decreasing of bicarbonate buffer reabsorbtion
***

***

***

1. A 57-year-old patient was admitted to the gastroenterological department


with suspicion on Zollinger-Ellison syndrom because of rapid increase of
gastrin level in the blood serum. What disorder of the secretory function of
the stomach is the most likely?
A Hyperacid hypersecretion
B Hyperacid hyposecretion
C Achylia
D Hypoacid hyposecretion

E Hypoacid hypersecretion
2. To simulate gastric ulcer animal was introduced into gastric artery
atofanom that causes arteriosclerosis. What is the mechanism of damage to
the mucous membrane of the stomach isleading in this experiment?
A * hypoxic
B neurodystrophic
C Mechanical
D Dysrehulyatornyy
E neurohumoral
3. Patient L., complains of belching, heartburn, frequent constipation. In the
titration of gastric juice obtained the following data: total acidity - 88 mlmol /
l., total HCl 83 mlmol / l free HCl - 50 mlmol / l related HCl - 33 mlmol / l,
acid phosphate and organic acid - 5 mlmol / l. Assess the condition of gastric
acidity:
A * hyperacid state
B hypoacid state
C Achilios
D Normacidic state
E hypoCl
4. The patient was diagnosed - irritable bowel syndrome. Level of what
substance will be increase?
A * Motylin
B intestinal peptide
C glucagon
D Urogastron
E secretin
5. A patient with the syndrome Zollinger-Ellison [tumor of the pancreas]
observed increase secretion, motility of the gastrointestinal tract, as well as
diarrhea and peptic ulcer. Which of the following substances secreted by the
tumor indicated this is a set of symptoms?
A * Gastrin
B vasoactive intestinal peptide
C Pepsin.
Trypsin D
E Secretin

6 .A 57-year-old patient was admitted to the gastroenterological department


with suspicion on Zollinger-Ellison syndrom because of rapid increase of
gastrin level in the blood serum. What disorder of the secretory function of
the stomach is the most likely?

A Hyperacid hypersecretion
B Hyperacid hyposecretion
C Achylia
D Hypoacid hyposecretion
E Hypoacid hypersecretion
7. Male 32 years old with dyspeptic disorders found during the examination
ulcer of the gastric mucosa and diagnosed Zolinger-Ellison syndrome. What is
the main pathogenetic mechanism of occurrence of ulcers in this case?
A * increased production of gastrin
B increased production of hydrochloric acid
C reducing the protective properties of gastric mucosa
D increased production of insulin
E reflux of duodenal contents into the stomach
8. The patient complained of pain in the epigastric region zoster character
when laboratory examination revealed a high content of diastase urine and
feces content to a large amount of undigested fat. For what form of
gastrointestinal pathology most characteristic phenomena described?
A * Acute pancreatitis
B Peptic ulcer
C Acute appendicitis
D Inflammation of the large intestine
Infectious hepatitis E
1. A patient ill with enteritis accompanied by massive diarrhea has low water
rate in the extracellular space, high water rate inside the cells and low blood
osmolarity. What is such disturbance of water-electrolytic metabolism called?
A Hypo-osmolar hypohydration
B Hyperosmolar hypohydration
C Osmolar hypohydration
D Hypo-osmolar hyperhydration
E Hyperosmolar hyperhydration
2. A 57-year-old patient was admitted to the gastroenterological department
with suspicion on Zollinger-Ellison syndrom because of rapid increase of
gastrin level in the blood serum. What disorder of the secretory function of
the stomach is the most likely?
A Hyperacid hypersecretion
B Hyperacid hyposecretion
C Achylia
D Hypoacid hyposecretion
E Hypoacid hypersecretion

1. A patient, 58 years, carried a insult. Later there was the gastric


hemorrhage. Patient never suffers on ulcer disease. What from is the leading
mechanism of gastric ulcer development?
A. Increase of glycocorticoid production.
B. Decrease of glycocorticoid production
C. Decrease of local prostaglandin synthesis
D. Damage of mucous barrier.
E. Hypersecretion of gastric juice.
2. A neurological patient has hypersalivation. What can result it?
A. Increase of n. vagus tonus
B. Increase of sympathetic nerves tonus
C. Pathology of cerebellum
D. Oppressing the sensitiveness of tastes receptors
E. Blockade of n. vagus
3. Dumping-syndrome is :
A. Complex of symptoms, which appears after the complete resection of
stomach.
B. Complex of symptoms, which appears after the partial resection of stomach
C. Complex of symptoms, which appears after the resection of bottom of
stomach
D. Complex of symptoms, which appears for some patients with ulcer disease
E. Complex of symptoms, which appears after the considerable resection of
colon
4. Sick L., complains about a belch, heartburn, frequent constipations. At
titruvanni of gastric juice got such information: general acidity - 88 mmol/L,
general HCl - 83 mmol/L, free HCl - 50 mmol/L, linked HCl - 33 mmol/L, acid
phosphates and organic acids - 5 mmol/L. Estimate the state of acidity of
stomach:
A. Hyperacidic state
B. Gipoacidic state
C. Axilia
D. Norma
E. Hypochlorgydria
5. Sick, 66 years, suffers on reumatoid arthritis over 10 years. Acetylsalitcyl
acid and prednisolon adopted in connection with sharpening. Complains
about a stomach-ache, belch, nausea, feeling of overfill in epigastrii,
flatulence. At gastroscopia found out erosion (0.5 sm x 0.5 sm) of mucus
membranes of stomach. What reason of forming of defect of mucous
membrane of stomach:
A. Protracted reception of aspirin and hormones
B. Immune defeat of stomach mucous

C. Development of dysbacteriosis
D. Age-old changes of mucous
E. Protracted hypertonus of stomach musculature
6. A new-born child has diarrhea, signs of intoxication, dehydration of
organism appeared on 5-6 days. Indicated phenomena are disappeared after
passing to the artificial soul-milk mixtures. A conclusion is done about
hereditary insufficiency of galaktose. What process did develop in a child?
A. Disorder of membranous digestion
B. Disorder of cavity digestion
C. Disorder of intestine excretory function
D. Disorder of pancreas secretory function
E. Disorder of stomach secretory function
7. For a patient with stomach ulcer disease at an inspection a presence in
stomach mucous of stomach of Helicobacter of pylori is discovered. What role
of Helicobacter of pylori in development of ulcer disease?
A. Damage of mucous barrier
B. Causes a hypersecretion
C. Damages of stomach wall blood supply.
D. Inhibition of prostaglandin synthesis
E. Brakes development of ulcer.
8. Ulcers of stomach were found out in a patient, who was treated with
glucocorticoids for a long time. What is the main mechanism of ulcer
development in this case?
A. Increase in secretion and acidity of gastric juice
B. Decrease in histamine concentration in stomach wall
C. Increased tonus of sympathetic nervous system
D. Increased production of prostaglandins E
E. Decreased tonus of parasympathetic nervous system
9. Malabsorption syndrome is:
A. Syndrome of membraous maldigestion
B. Syndrome of gastric malabsorption
C. Syndrome of malabsorption in small intestine
D. Syndrome of intestinal malabsorption
E. Malabsorption of proteins
10. A woman, 50 years, complain about pain in epigastrial area, which calm
down after adopting a meal. At an inspection ulcer disease of stomach is
diagnosed. What is main factor in development of this pathology?
A. HCl acid and pepsin
B. Bilious acids
C. Heliobater pylori
D. Decrease of mucous secretion

E. Increase of 2, 2b prostaglandin synthesis


11. At an inspection in a diagnostic center patient with ulcer disease of bulbus
duodenum found out the presence of Heliobater pylori. Indexes: free acidity
- 50 mmol/L, common acidity - 70 mmol/L. What mechanism is main in the
origin of this disease?
A. Stimulation of gastrin and histamin secretions
B. Insufficient secretion of mucus
C. Decrease of 2, 2b prostaglandin synthesis
D. Rapid emptying of stomach
E. Decrease of bicarbonate secretion
12. A woman, 45 years, complain about unbearableness of some products
(eggs, fish, crabs), dyspeptic phenomenon. At an inspection found out the
deficit of iron, dysbacteriosis. In what state acid making function of stomach
in patients?
A. Hypersecretion
B. Hyperacidity
C. hilia
D. Normal acidity
E. Normal secretion
13. Patient B. for a long time adopted salicylats, indometacin for treatment of
rheumatism. Lately pains appeared in epigastrial area and dyspeptic
disorders. During fibrogastroscopif the ulcer of small curvature of stomach is
found. What mechanism of ulcer development in this case?
A. Braking of prostaglandin synthesis
B. Decrease of mucin production
C. Increase of stomach secretory activity
D. Increase of HCl secretions
E. Strengthening of stomach motoric function
14. Patient suffer Zolingera-Elisona syndrome, hypergastrinemia is found out.
What state of stomach secretory function is most credible in this case?
A. Organic axilia
B. Functional axilia
C. Hyposecretion
D. Hypoacidity
E. Hypersecretion
15. At an inspection in gastroenterology for a patient hyperchloremia is
discovered. Such violation of stomach secretory function is characteristic?
A. Cancer of stomach
B. Ulcer disease of duodenum

C. Atrophy gastritis
D. -12-deficiency anemia
E. Iron deficiency anemia
16. Patient complain about the decrease of appetite, belch rotten, frequent
diarrhea, flatulence. General acidity of gastric juice is 10 units. What disease
is most credible in patients?
A. Hyperacidic gastritis
B. Ulcer disease of stomach
C. Ulcer disease of duodenum
D. Hypoacidic gastritis
E. Anacidic gastritis
17. Patient suffers from duodenum ulcer disease. Violations of what stomach
function have a pathogenetic value in the origin of this disease?
A. Motoric
B. Secretory
C. Reservoir
D. Evacuatory
E. Escritoire
18. To the patient suffers from duodenum ulcer disease the operation of
vagotomia was done. What pathogenetic mechanism of this method?
A. Increased of gastrin secretion
B. Reduced synthesis of prostaglandins
C. Reduced mucus secretion
D. Reduced bicarbonates secretion
E. Broken mucus membranes microcirculation
19. A man, 28 years, complain about pain in epigastrial area, dyspeptic
disorders. At gastroscopia found out the numerous ulcer defects of stomach
mucus membranes. Zolingera-Elisona syndrome is diagnosed. What
pathogenetic mechanism takes leading part in the origin of this pathology?
A. Hypersecretion of pepsinogen
B. Decrease of protective factors action in mucus membranes of stomach
C. Hypersecretion of gastrin
D. Reflux in stomach duodenal maintenance
E. Increase of stomach motoric
20. A woman, 47 years, long time suffers on functional disorders of nervous
system. Development of what disease can be pathogenetic related to the
neurosis?
A. Disease of Itsenko-Cushing
B. Chronic glomerulonefritis

C. Ulcer disease of stomach


D. Pancreatitis
E. Hepatitis
21. A patient complain about hungry pains in epigastrial area, feeling of
discomfort after adopting rough, rich, spicy food. At an inspection the defect
of stomach mucus membranes is set, the protracted uncontrolled application
of glycocortikoids is marked in anamnesis. The leading link of pathogenesis
was in this case:
A. Surplus of histamin
B. Lack of prostaglandin
C. Increase n.vagus tonus
D. Surplus of prostaglandin
E. Lack of histamin
22. As a result of gastric juice analyses is set: general acidity - 28 mmol/L, free
HCl acid - 1,5 mmol/L, maintenance of gastromucoprotein is reduced. Lack of
what vitamin is observed in organism?
A. Cyancobalamine
B. Folic acid
C. Pantotenic acid
D. Nicothinamid
E. Bioflavonoid
23. Duodenum was removed for a patient. It is caused the decrease of
gastrointestinal hormones secretion:
A. Histamin
B. Cholecystokin and secretin
C. Gastrin and histamin
D. Neurotenzin
E. Gastrin
24. 150 ml of meat broth were introduced into the stomach cavity of
experimental dog to a probe the content of what substance will increase
quickly in the animals blood?
A. Insulin
B. Vasointestinal peptide
C. Neurotensin
D. Somatostatin
E. Gastrin
25. Patient month ago carried pneumonia with the expressed respiratory
insufficiency. Therapy with cefazolin was conducted. omplaine about the
speed-up (to 6 times for days) selection of the liquid emptying with a specific
smell, stomach-ache, feeling of gurchannya. Specify possible reason of
diarrhea development:
A. Development of dysbacterios

B. Toxic influence of antibiotics on intestine mucous


C. Formation of antigen-antibody complexes
D. Violation of the peristaltic reflex regulation
E. Hypoxic damage of mucus membranes
26. On laboratory examination increased amount of diastase in the urine and
also a large amount of undigested fat in stool were revealed in a patient female
with complain of circular character pain in epigastric area. What form of
gastrointestinal tract pathology are described signs typical for?
A. Inflammation of large intestine.
B. Acute appendicitis.
C. Infectious gastritis.
D. Ulcer disease of the stomach
E. Acute pancreatitis
27. A 8-month child suffers from diarrea, vzduttya stomach, hypotropia,
abundant stinking emptying. Symptoms began to show up and grow after
introduction to the ration of flour wares feed. Emptying is abundant, foamy,
whitishness with a putrid smell, pH - 6. What pathology can be assumed?
A. Pankreatit
B. Celiacia
C. Malabsorbtion syndrome
D. Hypoacidic gastritis
E. Enterocolitis
28. A new-born child after feeding with milk had dyspepsia, vomit. At feeding
with solution of glucose these phenomena disappeared. Insufficient activity of
what enzyme which takes part in overcooking of carbohydrates does result in
the noted disorders?
A. Lactase
B. Amilaze
C. Maltaze
D. Saccaraze
E. Isomaltaze
29. Man, 58 years, has clinical picture of sharp pancreatitis. Appearance of
what matter in urine can confirm diagnosis?
A. Diastase
B. Albumen
C. Remaining nitrogen
D. Urea
E. Urinary acid
30. Man 20 years, complaints about sudden strong diarrhea, tenesms,
flatulence. All these symptoms arose up after he drink milk. The deficit of
what ferment could cause these dyspeptic disorders?
A. Lactase.

B. Lipase.
C. HCl
D. Tripsin.
E. Bile
31. A patient with the Zollinger-Ellison syndrome [tumour of pancreas] have
multiplying secretion of gastric juice, increase of digestive system peristaltic,
diarrhea and peptic ulcers. What matters, secretated by the tumour that, does
cause the complex of symptoms?
A. Gastrin
B. Vasoactive intestinal peptid
C. Pepsinogen [pepsins].
D. Tripsinogen [tripsini]
E. Secretin
32. Intrastomach pH metria is done for patient with chronic gastritis. The
decrease of gastric juice acidity is set by. The function of what cells is
reduced?
A. Endocrinocytes
B. Main eksocrinocytes
C. Parietal eksocrinocytes
D. Neck cell
E. Additional cell
33. At FGDS for a patient found out in antral part a chronic peptic ulcer
associated with Helicobacter pylori. The frequent receives is related with
secretion disorder of:
A. Mucous
B. Pepsin
C. l
D. Lipaze
E. Gastrin
*** *** ***

1. A 48 y.o. patient was admitted to the hospital with complaints about


weakness, irritability, sleep disturbance. Objectively: skin and scleras are
yellow. In blood: conjugated bilirubin, cholalemia. Feces are acholic. Urine is
of dark colour (bilirubin). What jaundice is it?
A
Mechanic
B
Hemolytic
C
Parenchymatous
D
Gilbert's syndrome
E
Crigler-Najjar syndrome
2. An infectious disease unit admitted a patient with signs of jaundice caused
by hepatitis virus. Select an indicator that is specific only for parenchymatous
jaundice:

A
B
C
D
E

Increase in ALT and AST rate


Hyperbilirubinemia
Bilirubinuria
Cholaemia
Urobilinuria

3. A patient being treated for viral hepatitis type B got symptoms of hepatic
insufficiency. What blood changes indicative of protein metabolism disorder
will be observed in this case?
A
Absolute hypoalbuminemia
B
Absolute hyperalbuminemia
C
Absolute hyperfibrinogenemia
D
Proteinic blood composition is unchanged
E
Absolute hyperglobulinemia
4. A patient being treated for viral hepatitis type B got symptoms of hepatic
insufficiency. What blood changes indicative of protein metabolism disorder
will be observed in this case?
A
Absolute hypoalbuminemia
B
Absolute hyperalbuminemia
C
Absolute hyperfibrinogenemia
D
Proteinic blood composition is unchanged
E
Absolute hyperglobulinemia
5. Hepatitis has led to the development of hepatic failure. Mechanism of
edemata formation is activated by the impairment of the following liver
function:
A
Protein-synthetic
B
Barrier
C
Chologenetic
D
Antitoxic
E
Glycogen-synthetic
6. A patient presents with icteritiousness of skin, scleras and mucous
membranes. Blood plasma the total bilirubin is increased, stercobilin is
increased in feces, urobilin is increased in urine. What type of jaundice is it?
A
Haemolytic
B
Gilbert's disease
C
Parenchymatous
D
Obturational
E
Cholestatic
7. Blood analysis of a patient with jaundice reveals conjugated bilirubinemia,
increased concentration of bile acids. There is no stercobilinogen in urine.
What type of jaundice is it?
A
Obstructive jaundice

B
C
D
E

Hepatocellular jaundice
Parenchymatous jaundice
Hemolytic jaundice
Cythemolytic jaundice

8. Patient after mushroom poisoning has the yellow color of the skin and
sclera, dark color of urine. What pigment causes the urine in patients with
hemolytic jaundice?
A * stercobilin
B Monoglyucuronid bilirubin
Unconjugated bilirubin C
D Verdohlobin
E Biliverdyn
9. Patients with chronic hepatitis complains of increased sensitivity to
barbiturates, which before he used without symptoms of intoxication. Name
violation of liver function?
A * Metabolic
B Formation of bile
C hemodynamically
D hematopoietic
E phagocytic
10. A woman after a long painful attack in the right subcostal area appeared
jaundice, and then she appealed to the doctor. There was a suspicion on acute
calculous cholecystitis. Research which blood parameters indicates
obstruction of the bile ducts?
A * Free and bilirubin
B protein fractions
C total lipids
D Uric acid
E residual nitrogen
11. The boy is 15 years old diagnosed with acute viral hepatitis. Research of
which blood counts must be conducted for acute liver cells damage
confirmation?
A * Activity aminotransferase [AST and ALT]
B content of free and bound bilirubin
C Erythrocyte sedimentation rate [ESR].
D Cholesterol level
E content of protein fractions
12. The patient was 25 years old diagnosed with chronic hepatitis. The patient
complains of loss of body weight of 10 kg for 2 months. Objectively: the skin is
dry, peeling, pale yellowish color, punctulata haemorrhages on the skin,

bleeding gums. Violation of what liver function reflect punctulata hemorrhage


and bleeding gums:
A * protein-syntetic
B formation of pigments
C Glicogynsyntetic
D detoxifies
E deposited
13. A patient 48 years, is in a reanimation after poisoning a tetrachloride
carbon. The state is heavy, patient without consciousness, a pulse is threadlike,
weak filling, heart rate - 40 /min., AP - 75/40 mm Hg. Breathing is periodic
(Biota). In blood: decrease of albumen (16 g/l), direct bilirubin - 155 mcmol/L.
In urine high quantity of ammonium and bilirubin. What caused the
development of the described state for a patient?
A.
Heprtic insufficiency
B.
Respiratory insufficiency
C.
Cardiogenic shock
D.
Cardiac insufficiency
E.
Kidney insufficiency
14. Patient 38, who had hepatitis and continued to consume alcohol, developed
signs of liver cirrhosis with ascites and edema of the lower extremities. What
changes in blood composition were decisive in the development of edema?
A * Hypoalbuminaemia
B Hipohlobulinemiya
C Hipoholesterynemiya
Hypokalemia D
E Hypoglycemia
15. Clinical examination of the patient revealed a previous diagnosis of liver
cancer.The presence of which protein in serum allows to confirm the
diagnosis?
A * alpha-fetoprotein
B Properdyn
C paraprotein
D C-reactive protein
E gamma globulin
16. A patient suffering from gall - stone disease due to obstruction biliary
found discolored fat feces. The absence of what component of bile causes
steatorrei phenomenon?
A * Bile acids
B Cholesterol
C cholochrome
D Fatty Acids

E Alkaline phosphatase
17. It was found that the development of hepatoma in it cells often stops
synthesis of bile acids. What type of anaplasia does this mean?
A * Functional
B Energy
Morphological C
D Biochemical
E Physico-Chemical
18. In a patient with hemolytic jaundice in the blood smear of blood present in
the form of red blood cells microspherocytes 1 - 6 in sight. What is the possible
cause of hemolysis of erythrocytes, causes such form of jaundice?
A * Hereditary defect in development of their membranes
B Hereditary defect structure of hemoglobin
C Violation erythrocyte enzyme systems
D Effects on erythrocyte membrane of bile acids
E Effects on erythrocyte membrane bilirubin
19.
In
overweight
women
established
cirrhosis.
Laboratory:
hypoalbuminemia, hyperglobulinemia. Visually: edema of the hands, eyelids
and legs. The most likely cause of edema is modified
A * oncotic pressure of the blood
Buffer B emnosti blood
C Acid-base balance
D detoxification function of the liver
E glycogensytetic liver function
20. Patient '35 developed immune hemolytic anemia. What index of serum is
rises mostly?
A * Indirect bilirubin
B Direct bilirubin
C Sterkobilinogen
D Mesobilinogen
E protoporphyrin
21. Female 55 years complains of fatigue, irritability, insomnia and state of
drowsiness in the day, itching skin. Pulse 58 beats / min., AT-110/65 mmHg.
Feces is colorless, contains a lot of fat. Cholelithiasis is suspected of blockage
of the bile duct stone. What is most likely determines the condition of the
patient?
A *Cholemia
B hyperbilirubinemia
C hypercholesterolemia
D malabsorption of fat-soluble vitamins

E malabsorption of fat
22. The laboratory study of patient with jaundice revealed following data
describing the violation of pigment metabolism: total bilirubin - 80 mmol/L,
direct bilirubin -63mmol/l is present in the urine bilirubin, urobilinogen in
urine and stercobilin are abscente. What is the reason for the absence of
urobilinogen in the urine in this type of jaundice?
A * Lack of receipt of bilirubin into the intestine
B Malabsorption urobilin in the intestine
C Violation kidney allocation urobilin
D Violation binding protein transport urobilin
E deficiency converting enzyme glyukoronidbilirubin in urobilinogen
23. In a patient with tumor obturation of biliary feces contained large amount
of fat (steatorreya). Lack of what component of bile causes this phenomenon?
A * Bile acids
B Fatty Acids
C Cholesterol
D cholochrome
E Alkaline phosphatase
24. The patient observed yellowness of the skin, increase content
blood indirect bilirubin, increased excretion sterkobilina with feces and urine,
decrease in osmotic resistance of erythrocytes. What type of jaundice noted in
this case?
A * Hemolytic
B Mechanical
C Perenhimal
DE1. Blood analysis of a patient with jaundice reveals conjugated bilirubinemia,

icreased concentration of bile acids. There is no stercobilinogen in urine. What


type of jaundice is it?
A Obstructive jaundice
B Hepatocellular jaundice
C Parenchymatous jaundice
D Hemolytic jaundice
E Cythemolytic jaundice
2.A patient presents with icteritiousness of skin, scleras and mucous membranes.
Blood plasma the total bilirubin is increased, stercobilin is increased in feces,
urobilin is increased in urine. What type of jaundice is it?
A Haemolytic

B Gilbert's disease
C Parenchymatous
D Obturational
E Cholestatic
3.Hepatitis has led to the development of hepatic failure. Mechanism of edemata
formation is activated by the impairment of the following liver function:
A Protein-synthetic
B Barrier
C Chologenetic
D Antitoxic
EGlycogen-synthetic
4.A patient being treated for viral hepatitis type B got symptoms of hepatic
insufficiency. What blood changes indicative of protein metabolism disorder will
be observed in this case?
A Absolute hypoalbuminemia
B Absolute hyperalbuminemia
C Absolute hyperfibrinogenemia
D Proteinic blood composition is unchanged
E Absolute hyperglobulinemia
5.An infectious disease unit admitted a patient with signs of jaundice caused by
hepatitis virus. Select an indicator that is specific only for parenchymatous
jaundice:
A Increase in ALT and AST rate
B Hyperbilirubinemia
C Bilirubinuria
D Cholaemia
E Urobilinuria
1. A patient being treated for viral hepatitis type B got symptoms of hepatic
insufficiency. What blood changes indicative of protein metabolism disorder
will be observed in this case?
A Absolute hypoalbuminemia
B Absolute hyperalbuminemia
C Absolute hyperfibrinogenemia
D Proteinic blood composition is unchanged
E Absolute hyperglobulinemia
2. Hepatitis has led to the development of hepatic failure. Mechanism of
edemata formation is activated by the impairment of the following liver
function:
A Protein-synthetic
B Barrier

C Chologenetic
D Antitoxic
E Glycogen-synthetic
3. A patient presents with icteritiousness of skin, scleras and mucous
membranes. Blood plasma the total bilirubin is increased, stercobilin is
increased in feces, urobilin is increased in urine. What type of jaundice is it?
A Haemolytic
B Gilbert's disease
C Parenchymatous
D Obturational
E Cholestatic
4. A 48 y.o. patient was admitted to the hospital with complaints about
weakness, irritability, sleep disturbance. Objectively: skin and scleras are
yellow. In blood: conjugated bilirubin, cholalemia. Feces are acholic. Urine is
of dark colour (bilirubin). What jaundice is it?
A Mechanic
B Hemolytic
C Parenchymatous
D Gilbert's syndrome
E Crigler-Najjar syndrome
1. .In patients with the biliary tract obstruction the blood coagulation is
inhibited; the patients have frequent haemorrhages caused by the subnormal
assimilation of the following vitamin:
A. A
B. K
C. E
D. D
E. C
1. Patient suffers on chronic calculuss choletsistitis complain about sharp
pain in right hypochondrium, an itch and icteric of skin covers, plural petexial
hemorrhages, excrement is fat and discolored (steatorrhea). What type of
icterus is observed for a patient?
A. Hemolytic
B. Hepatocellular
C. Gemic
D. Mechanical
E. Circulatoric
2. Alcoholic cirrhosis is diagnosed in a patient. Ascyt and edemas on lower
extremities are observed. What blood changes will be found out for a patient?
A. Hypoalbiminemia
B. Hypogamaglobulinemia

C. Hypoalfaglobulinemia
D. Dysproteinemia
E. Hypocholesterinemia
3. In a patient found out the tumor of pancreas head which is accompanied
with disorder of general bilious channel communicating. The contents of what
matter will be multiplied in blood here?
A. Urea
B. Hemoglobin
C. Insulin
D. Bilirubin
E. Adrenalin
4. Patient suffers from the portal cirrhosis, during abdominocentesis 8l. of
liquids are got, that led to weakness and loss of consciousness. What
mechanism of development of this complication?
A. Ischemia as a result of blood redistribution
B. Angiospastic ischemia
C. Embolism of cerebral arteries
D. Thrombosis of cerebral arteries
E. Sharp venous hyperemia of cerebrum
5. Patient N. complaint about dyspeptic disorders, melena, hemorrhoid
bleeding. At an inspection found out expansion of venous vessels on a front
abdominal wall and multiplying the stomach size. What pathology of digestive
system can show up such symptoms?
A. Intestinal autointoxication
B. Ulcer disease
C. Colitis
D. Enteritis
E. Portal hypertension
6. A patient complain about head pain, irritability, rapid fatigueability, pain in
right hypochondrium, itch of skin. At an inspection: yellow coloring of skin
and mycoses, increased liver, pain during palpationi, AP - 80/40 mm Hg, heart
rate 46 /min. In blood: level of direct bilirubin - 34 mcmol/L, level of indirect
- 35,2 mcmol/L, in urine - bilious acids, direct bilirubin, urobilin; the content
of stercobilin in an excrement is diminished. What type of icterus is there?
A. Hemolytic
B. Parenchimatous
C. Subhepatic
D. Obturative
E. Prehepatic
7. A woman works on a production linked with phenilgydrazin (production of
amidopirin), complaints about icteric color of skin and skler, weakness,
fatigueability, multiplying a spleen. What type of icterus take place at patient?

A. Mechanical
B. Hepatic
C. Subhepatic
D. Hemolytic
E. Obturative
8. Disorder of what liver functions of liver at its insufficiency causes
hemorrhagic syndrome?
A. Barrier
B. Antitoxic
C. Excretion of bile
D. Depositing of glycogen
E. Protein synthesis
9. What disease can caused the development of suprahepatic icterus?
A. Viral hepatitis B
B. Cirrhosis of liver
C. Hemolytic disease of new-born
D. Pancreatitis
E. Ulcer disease of stomach and duodenum
10. What diseases can causes the development of hepatic icterus?
A. Gastritis
B. Cholecystitis
C. Pancreatitis
D. Enteritis
E. Viral hepatitis B
11. What diseases can causes the development of subhepatic icterus?
A. Hepatitis
B. Gastritis
C. Pancreatitis
D. Gallstone disease
E. Cirrhosis of liver
12. To the doctor a patient appealed concerning icteric sclera and skin. During
inspection it was not discovered encephalopatia and signs of cholemic and
acholic syndromes. What icterus did develop?
A. Parenchymatous
B. Nuclear
C. Mechanical
D. Subhepatic
E. Hemolytic
13. Arterial hypertension can develop at hepatic insufficiency. What factors
does conduct the increase of arterial pressure in such cases?

A. ldosteron.
B. Adrenalin
C. Renin
D. Noradrenalin
E. Angiotenzin - II.
14. 32A female patient with chronic hepatitis complains of increase sensitivity
to barbiturates which she used previously without any symptoms of
intoxication. The disorder of what hepatic jaundice is responsible for this
state?
A. Bile formation
B. Hemodynamic
C. Metabolic
D. Hemopoetic
E. Phagocytotic
15. A patient with jaundice complains of headache, and insomnia. Objectively:
pulse-54 beats/min. AP- 90/60 mmHg. Coagulation of blood decreases. There
are traces scratching on skin. The action of what component causes these
symptoms?
A. Bilirubin
B. Cholesterol
C. Bile pigment
D. Bile acid
E. Fatty acid
16. Patient D., 48 years, during long time disturb the frequent mycotic feet
diseases and rashes of abscesses on the body. In anamnesis is alcoholism.
What did assist development of disease in this case?
A. Damage of gematoecnhephalic barrier
B. Decrease of liver barrier function
C. Primary immunodeficiency
D. Allergy
E. Chronic intoxication
17. Patient L., 55 years, with the sharp attack of hepatic colic entered to
gastroenterology department. Objectively: temperature- 380, sclers, mucus
and skin of icterus, urine is dark, an excrement is light. Complain is about the
skin itch of skin. What is the reason of icterus for this patient?
A. Obturation of bilious channels
B. Destruction of hepatocytes
C. Increased hemolysis
D. Lipid exchange disorder
E. Protracted reception of products, rich on a carotin

18. The cirrhosis of liver is diagnosed; he is accompanied by ascit and


common hemodynamic disorder. What pathological syndrome does determine
these disorders?
A. Hepatolienal syndrome
B. Hepatocardial
C. Hepatorenal
D. Portal hypertension
E. Hepatointestinal
19. Patient N., 46 years, complaine about absence of appetite, nausea, pain in
right hypochondrium, which increases at adopting rich food, yellow coloring
of scler, darkening of urine. Intravenous injections were conducted one and a
half months ago. In blood: increase of indirect and direct bilirubin. In urine
direct bilirubin and urobilin is determined in great quantity. What mechanism
of indirect bilirubinu absence in urine:
A. Strengthening of indirect bilirubinu inactivation by macrophag
B. Decrease of indirect bilirubinu conjugation in hepatocytes
C. Binding of indirect bilirubin with proteins
D. Indirect bilirubin dont filtered through a kidney membrane
E. Connection of indirect bilirubin by immune complexes
20. A patient 48 years, is in a reanimation after poisoning a tetrachloride
carbon. The state is heavy, patient without consciousness, a pulse is threadlike,
weak filling, heart rate - 40 /min., AP - 75/40 mm Hg. Breathing is periodic
(Biota). In blood: decrease of albumen (16 g/l), direct bilirubin - 155 mcmol/L.
In urine high quantity of ammonium and bilirubin. What caused the
development of the described state for a patient?
A. Cardiogenic shock
B. Respiratory insufficiency
C. Heprtic insufficiency
D. Cardiac insufficiency
E. Kidney insufficiency
21. It is discovered for a patient: yellow coloring of skin and mycoses
membraness, there are traces scratching on skin, depressing reflexes of
tendons and pain sensitiveness. Urine is color of beer, excrement is
acholic. AT is 90/40 mm Hg., heart rate - 52 b/min. In blood increased
cholesterol, direct bilirubin and bilious acids, in urine - bilious acids, direct
bilirubin, and stercobilin is not found. Name a process which caused the
absence of stercobilin in urine and excrement?
A. Stercobilin destroy in an intestine
B. Stercobilin is sucked back in blood
C. Stercobilin destroy in urine
D. There is no bile in intestine and stercobilin does not appear
E. The predecessor of stercobilin din liver

22. In a patient which suffers on gallstone disease, the discoloured fat


excrement appears as a result of obturation of bile ways. Absence of what
component of bile caused the phenomenon of steatorea?
A.Cholesterol
B. Bile acids
C. Bilious pigments
D. Lipid acids
E. Alcaline phosphatas
23. For a woman, 57 years, after the protracted pain attack in right
hypochondrium an icterus appeared. There was suspicion on a sharp gallstone
disease. What index does testify about impassability of bilious channels?
A. Protein fractions of blood plasma
B. General lipid of blood plasma
C. Urinary acid of blood plasma
D. Remaining nitrogen of blood
E. Free and linked bilirubin in plasma.
24. Boy, 15 years, the diagnosis of sharp viral hepatitis is set. Research of what
index must be conducted for confirmation of sharp defeat of hepatic cell?
A. Contents of free and linked bilirubin in plasma
B. Speed of zsidannya of red corpuscles [SHZE].
C. Activity of aminotransferas [ALT and AST] in blood.
D. A level of cholesterol in plasma.
E. Contents of protein fractions of blood plasma
25. Patient, 25 years, the diagnosis of chronic hepatitis is set. Patients
complain about the loss of body mass on 10 kg during 2 months. Objectively:
a skin is dry, peels, pale with a rather yellow tint, petexial hemorrhages on a
skin, gums hemorrhages. Violations of what liver function of represents
promoted hemorrhages?
A. Synthesis of bile pigments
B. Synthesis of glycogen
C. Detoxication
D. Synthesis of protein
E. Depositing
26. After taking a fatty food a patient feels nausea, flaccidity, later the sign of
steathorrhea has appeared, cholesterol in the blood 9,2ml/c. The cause of this
state is the deficiency of:
A. Chylomicrones.
B. Triglyceroides.
C. Bile acids.
D. Fatty acids.

E. Phopholipids
27. Sick 48th years delivered to the clinic with complaints about a weakness,
irritability, sleep disorder. Objectively: skin and sclers are yellow. In blood: an
increase of general bilirubin with prevalation of direct. Acholic excrements.
Urine - dark color. What icterus does take place?
A.Hemolitic
B. Syndrome of Gylbera
C.Parenchimatos
D. Syndrome Kriglera-Nayyara
E. Mechanical
28. For a patient the signs of hepatic insufficiency appeared during treatment
of viral hepatitis B. What changes of blood testify to protein exchange
disorder will be observed in this case?
A. Absolute hyperfibrinogenemia
B. Protein composition of blood is not changed
C. bsolute hypoalbuminemia
D. Absolute hyperalbuminemia
E. Absolute hyperglobulinemia
29. In a patient which appealed to the doctor, there is the yellow coloring of
skin, urine is dark, excrement of darkly yellow. The increase of what matter
concentration will be observed in the blood?
A. Free bilirubin
B. Mesobilirubin
C. Conjugated bilirubin
D. Biliverdin
E. Verdoglobin
30. Patient with liver cirrhosis. What matter, that excreating with urine, can
characterize the state of liver antitoxic function?
A. Urinary acid
B. Hypur acid
S. Ammonium salt
D. Creatinin
E. Amino
*** *** ***

1. A patient with massive burns developed acute renal insufficiency


characterized by a significant and rapid deceleration of glomerular filtration.
What is the mechanism of its development?
A
Reduction of renal blood flow ( )
B
Damage of glomerular filter
C
Reduction of functioning nephron number
D
Rise of pressure of tubular fluid
E
Renal artery embolism

2. A patient has been diagnosed with acute glomerulonephritis that developed


after he had had streptococcal infection. It is most likely that the affection of
basal glomerular membrane is caused by an allergic reaction of the following
type:
A
Immune complex
B
Anaphylactic
C
Cytotoxic
D
Delayed
E
Stimulating
3. A patient with nephrotic syndrome has massive edemata of his face and
limbs. What is the leading pathogenetic mechanism of edemata development?
A
Drop of oncotic blood pressure
B
Increase of vascular permeability
C
Rise of hydrodynamic blood pressure
D
Lymphostasis
E
Increase of lymph outflow
4. A 30 year old woman has face edemata. Examination revealed proteinuria
(5,87 g/l), hypoproteinemia, dysproteinemia, hyperlipidemia. What condition
is the set of these symptoms typical for?
A
Nephrotic syndrome
B
Nephritic syndrome
C
Chronic pyelonephritis
D
Acute renal failure
E
Chronic renal failure
5. Violation of safety rules resulted in calomel intoxication. Two days later the
daily diuresis was 620 ml. A patient experienced headache, vomiting,
convulsions, dyspnea, moist rales in lungs. What pathology is it?
A
Acute renal insufficiency
B
Chronic renal insufficiency
C
Uraemic coma
D
Glomerulonephritis
E
Pyelonephritis
6. Two weeks after lacunar tonsillitis a 20-year-old man started complaining
about general weakness, lower eyelid edemata. After examination the patient
was diagnosed with acute glomerulonephritis. What are the most likely
pathological changes in the urine formula?
A
Proteinuria
B
Cylindruria
C
Presence of fresh erythrocytes
D
Pyuria

Natriuria

7. A patient with a history of chronic glomerulonephritis presents with


azotemia, oliguria, hypo- and isosthenuria, proteinuria. What is the leading
factor in the pathogenesis of these symptoms development under chronic renal
failure?
A
Mass decrease of active nephrons
B
Intensification of glomerular filtration
C
Tubular hyposecretion
D
Disturbed permeability of glomerular membranes
E
Intensification of sodium reabsorption
8. Chronic glomerulonephritis was diagnosed in a 34-year-old patient 3 years
ago. Edema has developed in the last 6 monthes. What caused it?
A
Proteinuria
B
Hyperproduction of vasopressin
C
Disorder of albuminous kidneys function
D
Hyperosmolarity of plasma
E
Hyperaldosteronism
9. In the experiment nephrotoxic Gunea pigs serum was introduced to rabbit.
What kidney disease modeled in this study?
A * Acute diffuse glomerulonephritis
B Nephrotic syndrome
C Acute pyelonephritis
D Chronic renal failure
E Chronic pyelonephritis
10. In patients with chronic renal failure reduced clearance by inulin to 60
ml / min was revealed. What kidney function is broken?
A * glomerular filtration
B tubular secretion
C reabsorption in the proximal nephron
D reabsorption in the distal nephron
E reabsorption in the renal tubules
11. In patients with chronic renal failure were anorexia, dyspepsia, cardiac
arrhythmias, itching of skin. What mechanism of these disorders is the main?
A * Accumulation of products of nitrogen metabolism in the blood
B lipid metabolism
C Changes in carbohydrate metabolism
D renal acidosis
E violations of water-electrolyte metabolism

12. The man was diagnosed acute diffuse glomerulonephritis. From anamnesis
it is known that 18 days before the manifestation of the disease he suffered
from tonsillitis. What mechanism of the renal glomeruli damage is observed in
this case?
A * Immune.
BC nephrotoxic.
Ischemic D.
E medication.
13. In patients with glomerulonephritis revealed: anasarca, BP - 185/105
mmHg, anemia, leukocytosis, hypernitrogenemia, hypoproteinemia. What
index indicate a complication of glomerulonephritis nephrotic syndrome?
A * hypoproteinemia
B Leukocytosis
C Hiperazotemiya
D Hypertension
E Anemia
14. 43-year-old patient was admitted to the nephrology department with
massive edema. Two years had been treated and constantly elevated blood
pressure was noted. Twice was treated with prednisone, with a positive effect.
Urine: the relative density of 1017, protein - 4.0 g/l, erythrocytes - 15-18 in
sight (leached), Leu - 5-7 in sight. What predominantly renal function
compromised patient?
A * Filtration
B Reabsorbtsionnaya
C secretory
D endocrine
E Concentration
15. The patients with chronic bronchiectasis has severe nephropathy with
massive edema. Laboratory studies reveal abundant proteinuria cylindruria, a
significant reduction in the protein content of the serum, hyperlipemia,
hypokalemia and other abnormalities. What is the primary and most
important link in the pathogenesis of edema in this patient?
A * Reduced oncotic pressure of the blood
B Increased hydrostatic blood pressure
C Increasing the pressure of the extracellular fluid
D blockade of lymph
E Increased microvascular permeability
16. The patient one and a half weeks after the streptococcal sore throat
manifested severe edema, increased blood pressure. In urine, hematuria and
moderate proteinuria. In blood antistreptococcal antibodies and reduced

complement components. In microvessels which structures are most likely


localized accumulations of immune complexes led to the development of
nephropathy?
A * glomeruli
B Pyramids
C pelvis
D ureters
E Bladder
17. Masugi caused development of glomerulonephritis in rats as follows: rat
kidney homogenate was introduced to rabbits. A few weeks later the serum of
sensitized rabbits wsa introduced to rats. What type of allergic reaction by
Gell and Coombs underlies the development of glomerulonephritis in rats?
A * Cytotoxic
B anaphylactic
C immunocomplex
D delayed type hypersensitivity
E stimulaning
18. An examination of the patient revealed that the clearance of endogenous
reatinin after collecting 24-hour urine sample, he is 50 ml / min (normal 110-150 ml / min). On reduction function which indicates the presence of an
indicator?
A * glomerular filtration
B tubular reabsorption
C Inretor renal function
D excretion of ions.
E excretion of uric acid.
19. After a previous streptococcal infection in men diagnosed with acute
glomerunephrities. It is most likely that the lesion glomerular basal
membrane is an allergic reaction
A * immunocomplex type
B anaphylactic type
C. Cytotoxic -type
D delayed type
E. Stimulating-type
20. Man '32 for 4 years, suffering from chronic glomerulonephritis.
Hospitalized with signs of anasarca: AT-185/105 mmHg In the blood: Hb-110
g/l, er. - 2.6 * 1012/L Lake. - 9.5 * 109/L, rest nitrogen - 32 mg/L, total protein 50 g / l. What a change from most likely indicates glomerulonephritis with
nephrotic syndrome?
A * hypoproteinemia
B Anemia

C Leukocytosis
D Hypertension
E Hiperazotemiya
1. Shock and signs of acute renal failure (ARF) developed in the patient due
to permanent injury. What is the leading cause of development of ARF in the
case?
A Decreased arterial pressure
B Urine excretion violation
C Increased pressure in the nephron capsule
D Increased pressure in the renal arteries
E Decreased oncotic BP
2. Arterial hypertention is caused by the stenosis of the renal arteries in the
patient. Activation of what system is the main link in the pathogenesys of this
form of hypertension?
A Renin-angiotensin
B Sympathoadrenal
C Parasympathetic
D Kallikrein-kinin
E Hypothalamic-pituitary
3. Chronic glomerulonephritis was diagnosed in a 34-year-old patient 3 years
ago. Edema has developed in the last 6 monthes. What caused it?
A Proteinuria
B Hyperproduction of vasopressin
C Disorder of albuminous kidneys function
D Hyperosmolarity of plasma
E Hyperaldosteronism
4. Two weeks after lacunar tonsillitis a 20-year-old man started complaining
about general weakness, lower eyelid edemata. After examination the patient
was diagnosed with acute glomerulonephritis. What are the most likely
pathological changes in the urine formula?
A Proteinuria
B Cylindruria
C Presence of fresh erythrocytes
D Pyuria
E Natriuria
5. On the 6th day of treatment a patient with acute renal insufficiency
developed polyuria. Diuresis intensification at the beginning of polyuria stage
of acute renal insufficiency is caused by:
A Renewal of filtration in nephrons
B Volume expansion of circulating blood

C Growth of natriuretic factor


D Reduction of aldosteron content in plasma
E Reduction of vasopressin content in plasma
6. A 30 year old woman has face edemata. Examination revealed proteinuria
(5,87 g/l), hypoproteinemia, dysproteinemia, hyperlipidemia. What condition
is the set of these symptoms typical for?
A Nephrotic syndrome
B Nephritic syndrome
C Chronic pyelonephritis
D Acute renal failure
E Chronic renal failure
7. A patient with nephrotic syndrome has massive edemata of his face and
limbs. What is the leading pathogenetic mechanism of edemata development?
A Drop of oncotic blood pressure
B Increase of vascular permeability
C Rise of hydrodynamic blood pressure
D Lymphostasis
E Increase of lymph outflow
8. A driver who got a trauma in a road accident and is shocked has reduction
of daily urinary output down to 300 ml. What is the main pathogenetic factor
of such diuresis change?
A Drop of arterial pressure
B Drop of oncotic blood pressure
C Increased vascular permeability
D Decreased number of functioning glomerules
E Secondary hyperaldosteronism
9. A patient with massive burns developed acute renal insufficiency
characterized by a significant and rapid deceleration of glomerular filtration.
What is the mechanism of its development?
A Reduction of renal blood flow
B Damage of glomerular filter
C Reduction of functioning nephron number
D Rise of pressure of tubular fluid
E Renal artery embolism
10.On the 6th day of treatment a patient with acute renal insufficiency
developed polyuria. Diuresis intensification at the beginning of polyuria stage
of acute renal insufficiency is caused by:
A Renewal of filtration in nephrons
B Volume expansion of circulating blood
C Growth of natriuretic factor

D Reduction of aldosteron content in plasma


E Reduction of vasopressin content in plasma
11.A 30 year old woman has face edemata. Examination revealed proteinuria
(5,87 g/l), hypoproteinemia, dysproteinemia, hyperlipidemia. What condition
is the set of these symptoms typical for?
A Nephrotic syndrome
B Nephritic syndrome
C Chronic pyelonephritis
D Acute renal failure
E Chronic renal failure
12.A patient with nephrotic syndrome has massive edemata of his face and
limbs. What is the leading pathogenetic mechanism of edemata development?
A Drop of oncotic blood pressure
B Increase of vascular permeability
C Rise of hydrodynamic blood pressure
D Lymphostasis
E Increase of lymph outflow
13.Two weeks after lacunar tonsillitis a 20-year-old man started complaining
about general weakness, lower eyelid edemata. After examination the patient
was diagnosed with acute glomerulonephritis. What are the most likely
pathological changes in the urine formula?
A Proteinuria
B Cylindruria
C Presence of fresh erythrocytes
D Pyuria
E Natriuria
14. Patient with a history of chronic glomerulonephritis presents with
azotemia, oliguria, hypo- and isosthenuria, proteinuria. What is the leading
factor in the pathogenesis of these symptoms development under chronic renal
failure?
A Mass decrease of active nephrons
B Intensification of glomerular filtration
C Tubular hyposecretion
D Disturbed permeability of glomerular membranes
E Intensification of sodium reabsorption
Arterial hypertention is caused by the stenosis of the renal arteries in the
patient. Activation of what system is the main link in the pathogenesys of this
form of hypertension?
A Renin-angiotensin
B Sympathoadrenal
C Parasympathetic

D Kallikrein-kinin
E Hypothalamic-pituitary
1. A patient complains about dyspnea provoked by the physical activity.
Clinical examination revealed anaemia and presence of the paraprotein in the
zone of gammaglobulins. To confirm the myeloma diagnosis it is necessary to
determine the following index in the patient's urine:
A.Antitrypsin
B.Ceruloplasmin
C.Bilirubin
D.Haemoglobin
E. Bence Jones protein
2. Two weeks after lacunar tonsillitis a 20-year-old man started complaining
about general weakness, lower eyelid edemata: After examination the patient
was diagnosed with acute glomerulonephritis. What are the most likely
pathological changes in the urine formula?
A. Presence of fresh erythrocytes
B. Natriuria
C. Pyuria
D. Cylindruria
E. Proteinuria
3. A patient with massive burns developed acute renal insufficiency
characterized by a significant and rapid deceleration of glomerular filtration.
What is the mechanism of its development?
A. Damage of glomerular filter
B. Rise of pressure of tubular fluid
C. Reduction of functioning nephron number
D. Reduction of renal blood flow
E. Renal artery embolism
1. For a patient as a result of nephron tubule pathology by Zymnitski test
found out the substantial decrease of kidney ability to the osmotic
concentration of urine. How are these changes named?
A. Proteinuria
B. Hematuria
C. Chsilindruria
D. shypostenuria
E. Leukocyturia
2. For a patient with mieloma protein discovered. What form of proteinuria
does take place for this patient?
A. Renal glomerular.
B. Suprarenal

C. Renal tubular
D. Subrenal tumor
E. Subrenal uretral
3. Patient 30 years, delivered to the clinic with a diagnosis sharp
glomerulonephritis, there was proteinuria. What violations in an organism
caused this phenomenon?
A. Delay of nitrogen exchange products destroying
B. Decrease of oncotic blood pressure
C. Increase of glomerular membrane permeability
D. Increase of hydrostatical pressure on the capillaries wall
E. Decrease of functional nephrons quantity
4. For a patient A., 38 years, with 3 year of lupus disease found out the diffuse
damage of kidney, which is accompanied massive edemas, expressed
proteinuria, hyperlipidemia, dysproteinemia. What is the most reliable
mechanism of proteinuria development in this clinical situation?
A. Nephrons inflammatory damage
B. Tubule ischemic damage
C. Increase the level of blood proteins
D. Nephron autoimmune damage
E. Impression of urine ways
5. Acute glomerulonephritis developed for a patient. What mechanism of his
development?
A. Damaging action of complex IgA or IgD with an antigen
B. Damaging action of complex IgE or IgA with an antigen
C. Damaging action of complex IgG or IgM with an antigen
D. Damaging action of complex IgG or IgA with an antigen
E. Damaging action of complex IgD or IgM with an antigen
6. Patient A, 27 years, delivered in a hospital with the massive gastric bleeding
in a grave condition. AT - 80/60 mm Hg. A patient selects 60 - 80 ml urine for
day, relative specific gravity - 1,028-1,036. In blood plasma increased:
remaining nitrogen, urea, creatinin. What pathogenetic mechanism of day's
diuresis diminishing more credible in this clinical situation?
A. Increase of urine osmotic pressure
B. High level of blood remaining nitrogen
C. Increase of colloid-osmotic blood pressure
D. Decrease of glomerular capillaries hydrostatical pressure
E. An increase of hydrostatical pressure in the Boumen capsule
7. For a patient with chronic diffuse glomerulonephritis chronic insufficiency
of kidney developed. In the terminal stage of insufficiency develops oligo-,
anuria, which predefined:
A. By the ischemia of kidney cortex as vessel spasm result

B. Diminishing of filtrative pressure and filtration


C. Multiplying reabsorbtion of water in distal tubules
D. Disseminated intravascular blood coagulation
E. Decrease the quantity of active nephrons
8. To the rabbit intravenouswas entered nephrotoxic plasma of the Guinean
piggy, which was preliminary immunized by rabbit kidney suspension. What
pathology of kidney is designed thus?
A. Pyelonephritis
B. Glomerulonephritis
C. Uremia
D. Nephrotic syndrome
E. Tubular insufficiency
9. For a patient with a primary nephrotic syndrome the maintenance of
general protein of blood makes 40 g/l. What reason of hypoproteinemia?
A. An exit of protein from vessels to tissues
B. A decrease of protein synthesis in liver
C. Promoted proteolys
D. Proteinuria
E. Disorder of protein suction in an intestine
10. At an inspection of patient is set that clirens of endogenous creatinin after
collection of 24th portion of urine for him makes 50 ml/min (at a norm - 110150 ml/min.). About the decrease of what function does the presence of such
sign testify?
A. Kidney tubule reabsorbtion
B. Kidney incretoric function
C. Export from the organism of ions.
D. Export from the organism of urinary acid.
E. Kidney tubular filtrations
11. For a patient with sharp kidney insufficiency (day's diurez - 50 ml) anuria
developed. What is the main mechanisms lie in basis of its origin?
A. Na reabsorbtion increased
B. Difficulty of urine outflow
C. Disorder of kidney blood circulation
D. Increase of kidney tubular filtrations
E. Water reabsorbtion increased
12. At a pyelonephritis mainly damaged:
A. Kidney tubule
B. Kidney glomerular
C. Interstitial tissue

D. Kidney vascular system


E. Uxtaglomerular vehicle
13. The basic displays of chronic kidney insufficiency are:
A. Metabolic acidosis
B. Intoxication by the products of nitrous exchange;
C. Tissue hypoxia
D. Anemia;
E. Immunodepression
14. At the inherited phosphate kidney diabetes violated:
A. Reabsorbtion of chlorides
B. Reabsorbtion of potassium
C. Ecscretion of sodium
D. Selection of water
E. Reabsorbtion of phosphates
15. Edemas and arterial hypertension appeared for a patient in one and a half
weeks after a heavy streptococcus angina, in urine are hematuria and
moderate proteinuria. In blood are antibodies to antistreptococcus and
decrease of complement components. In what structures microvessels the most
reliable localization of immune complexes which caused the development of
nephropatia?
A. Pyramids
B. Bowls
C. Ureters
D. Urinary bladder
E. Glomerular
16. What main pathogenetic factor in development of edemas at a nephrotic
syndrome?
A. An increase of capillaries hydrostatical pressure
B. An increase of intercellular liquid oncotic pressure
C. Decrease of blood plasma oncotic pressure
D. A decrease of blood plasma osmotic pressure
E. An increase of intercellular liquid osmotic pressure
17. In an experiment, at the design of kidney pathology for an animal,
observed the followings signs: edemas, massive proteinuria, hypoproteinemia,
dysproteinemia, hyperlipemia. What kidney pathology are such signs
characteristic for?
A. Sharp diffuse glomerulonephritis
B. Pyelonephritis
C. Sharp kidney insufficiency.
D. Nephrotic syndrome.

E. Chronic kidney insufficiency


18. For a patient, as a result of sulema poisoning, sharp kidney insufficiency
which includes 4 stages developed: first - initial, second - oliguria, fourth is
convalescence. How is the third stage of sharp kidney insufficiency named?
A. Metabolic
B. Hemodynamic
C. Ischemic
D. Pathochemical
E. Polyuria
19. For a patient, as a result of sulema pathogenic action, sharp kidney
insufficiency developed with the anuria, uremia and kidney necrosis. What is
the localization of kidney necrotic damage in this case?
A. Cerebral matter
B. Ureters
C. Kidney bowls
D. Kidney cups
E. Cortex
20. Syndrome of Fankoni with disorder of sodium ions reabsorbtion, glucose,
amino acid, hydrocarbonats, phosphates found out for a patient. What part of
nephron will be damaged here?
A. Proximal
B. Distal
C. Genle loop
D. Collapsible tubule
E. Glomerular
21. A patient, 19 years, carried angina. In 2 weeks appealed to the doctor with
complaints about oliguria, discoloration of urine (pigwashes of meats). AP 190/100 mm Hg. What is the starting mechanism of arterial hypertension
development at glomerulonephritis?
A. Aldosteron hypersecretion
B. Renin hypersecretion
C. ADG production is increased
D. Kidney glomerular Ischemia
E. Neurogenic component of vascular tone is increased

22. For a patient with a nephrotic syndrome development of edema is linked


with:
A. Anemia
B. Urine selection disorder

C. Vasclar wall permeability disorder


D. Hyperaldosteronism
E. Massive proteinuria
23. A patient entered to the clinic with complaints about a general weakness,
polyuria. It is discovered at an inspection: skin covers are pale, AP is a 180/110
mmHg; blood test : er- 3,21012/l, Hh - 90 g/l ; analysis of urine: specific
gravity - 1,006, albumen -5g/l, leucocytes - 30 in /. What pathology these
symptoms are characteristic for?
A. Kidney amyloidosis
B. Glomerulonephritis
C. Prostatitis
D. Pielonephritis
E. Kidney mieloma
24. Anuria developed, hypernitrogenemia and interstitial lung edema
developed for a patient which was in a shock state. Disorder of what kidney
function is main in development of sharp kidney insufficiency?
A. Reabsorbtion of water increased in tubules
B. Increased of sodium reabsorbtion
C. Decrease of glomerular filtration speed
D. Urinary acid secretion disorder in tubules
E. Potassium reabsorption is decreased
25. What is the mechanism of filtration diminishing in the case of suprarenal
form of sharp renal insufficiency?
A. Damage of glomerular filter.
B. Kidney blood stream decreased
C. Active nephron quantity is diminished.
D. Tubules liquid pressure is increased
E. Decrease of oncotic blood
26. In the blood plasma of patient with urolithiasis there is increased the level
of urinary acid. Contents of remaining nitrogen in blood - 20 mcmole/l. What
change of blood composition found out for a patient?
A. Kidney hypernitrogenemia
B. Kidney hypoazotemia
C. Dysproteinemia
D. Hyperuricemia
E. Hyperacidemia
27. Anoreksia, dyspepsia, disorder of heart rhythm, itch of skin appeared for a
patient symptoms with chronic kidney insufficiency. What main mechanism of
these disorder development?
A. Disorder of lipid exchange

B. Changes of carbohydrate exchange


C. Accumulation of products of nitrous exchange in blood
D. Kidney acidosis
E. Disorder of water-electrolyte exchange
28. What origin of albumen more credible at selective proteinuria 12g/d?
A. ubular
B. Suprarenal.
C. Glomerular.
D. Uretral
E. Subrenal
29. A patient suddenly had pains in the kidney area with an irradiation in a
groin, nausea, vomit, day's diuresis - 50 ml. What is the define the mechanism
of anuria.
A. Disorder of filtration.
B. Increase of reabsorbtion.
C. Exception of nephron parts
D. Reflectory anuria.
E. Obturation of ureter.
30. Persons 72, is suffers on chronic glomerulonephritis. At an inspection:
absence of appetite, vomit, diarrhea, itch of skin, anemia, maintenance of
remaining nitrogen in blood -45 - mmole/l. Indicated signs are caused:
A. By the increase glomerular membrane permeability
B. By the kidney ischemia
C. Deleting of nephron functions
D. Autoimmune damage of glomerular membrane
E. Disorder of concentrating mechanism
31. Patient 32 years, with sharp glomerulonephritis which did not adhere the
limitations of NaCl and water, suddenly lost consciousness, there were cramps.
AP - 220/120 mm. Hg. Pupils are extended. Bradycardia. What complication
did arise up for a patient?
A. Sharp cardiac insufficiency
B. clampsia
C. Hypertensive crisis
D. Kidney comma
E. Epilepsy
32. The frequent protracted angina took place in anamnesis for the patient 25
years. He complains about periodic head pains, rapid fatigueability, edemas of
periorbital area. Found out moderate arterial hypertension, proteinuria,
hypoproteinemia, hyperlipidemia. At kidney ultrasound: a bilateral defeat

with the insignificant increase of kidney parenchyma echogenic. What disease


is most reliable in this case?
A. Sharp glomerulonephritis
B. Chronic pielonephritis
C. Lipoid nephrosis
D. Chronic glomerulonephritis
E. Sharp kidney insufficiency
33. For physically healthy youths soldier after the heavy physical loading in
one-day pedestrian transition on a 50 km in urine found out an albumen (to 1
g/l). What variety of proteinuria did take place?
A. Dehydratation proteinuria
B. Alimentary proteinuria
C. Organic proteinuria
D. Unreal proteinuria
E. Long going (mrsh) proteinuria
34. Unselective unmassive proteinuria is characteristic for?
A. Uretritis
B. Chronic kidney insufficiency
C. Sharp kidney insufficiency
D. Nephritic syndrome.
E. Nephrotic syndrome.
35. How is the terminal stage of kidney insufficiency, which is accompanied
development of metabolic acidosis, azotemia, grey color of skin, itch, smell of
ammonia, by disorder of vital function, named?
A. Sharp kidney insufficiency
B. Tubulopatia
C. Glomerulopatia
D. Uremia
E. Kidney colic
36. Patient, 58 years complaints about growth of AP. At a clinical inspection
found out chronic kidney disease with disorder of renal blood stream.
Activating of what regulator mechanism did become reason of growth of AP
for this patient?
A. Parasympathetic nervous system
B. Sympathetic nervous system
C. Cardio-vascular system
D. Damade of thyroid gland
E. Renin- angiotensin system
37. Woman, 55 years, with kidney insufficiency arterial pressure makes
170/100 mm.Hg. The hyperactivation of what systems caused the proof
increase of arterial pressure?
A. Sympato - adrenal system

B. ypotalamo - hypophysar
C. Renin- angiotensin aldosteron system
D. CNS
E. Kalicrein kinin system
38. Woman 25 years, complain after cooling: pain in the back, disuric displays
(leucocyturia, cylindruria, bacteriuria, day's diuresis without changes). It can
be linked with development of?
A. Glomerulonephritis
B. Sechekamyana illness
C. Radiculitis
D. Adnexsitis
E. Pyelonephritis
39. What quantitative changes of diuresis can result in uremia?
A. Polyuria
B. Dysuria
C. nuria
D. Nicturia
E. All foregoing changes
40. In what case it will be diminishing of glomerular filtration?
A. Pleasure in capillaries - 60 mmHg, oncotic blood pressure - 35 mm Hg,
pressure in capsule 20 mm Hg
B. Pressure in capillaries - 80 mm Hg, oncotic blood pressure - 30 mm Hg,
pressure in capsule - 20 mm Hg
C. Pressure in capillaries - 100 mm Hg, oncotic blood pressure - 30 mm Hg,
pressure in capsule - 20 mm Hg
D. Pressure in capillaries - 80 mm Hg, oncotic pressure - 20 mm Hg, pressure
in capsule 10 mm Hg
E. Pressure in capillaries - 100 mm Hg, onkotic pressure - 20 mm Hg, pressure
in a capsule - 20 mm Hg
41. At research of urine for a patient found out proteinuriya (5 g/l) due to
lowmolecular albumens, hematuria with the lixiviated red corpuscles. For
disorder of what kidney function are such indexes characteristic ?
A. Disorder of tubule secretory function
B. Disorder of glomerular excretory function
C. Disorder of tubular reabsorbtion
D. Disorder of glomerular permeability
E. Extrarenal disorders
42. Reason of anuria in extrarenal of sharp kidney insufficiency is:
A. Necrosis of tubules epithelium
B. Glomerulonephritis
C. Poisoning with heavy metals

D. Hypoproteinemia
E. Obturation of urine ways
43. Kidney insufficiency is characterized anemia. What mechanism of its
development?
A. Hematuriya
B. Hemoglobinuria
C. Erytropoetin production is decreased
D. Hypoproteinemia
E. Albuminuria
44. Kidney diabetes is diagnosed for the patient (renal glycosuria). A diagnosis
is linked:
A. Decrease of glucose reabsorbtion in kidney
B. Diminishing of paratgormon secretion
C. Diminishing of insulin secretion
D. Diminishing of insulin reception
E. Hyperglucemia
45. Patient N., 55 years suffers from chronic glomerulonephrtis during 15
years. What changes of laboratory indexes are most characteristic for the
kidney filtration decrease?
A. Hyperglucemia
B. Hypernitrogenemia
C. Hypoproteinemia
D. Proteinuria
E. Hypo-, isostenuria
46. What transferred indexes are characteristic for a nephrotic syndrome?
A. Proteinuria 400 mg/day
B. Hematuria
C. Proteinuria more than 3,5 g/da
D. Leucocyturia
E. Piuria
47. A girl after the casual consumption of uneatable mushrooms entered to the
reanimation department with the arterial hypotension, anuria,
hyperazotemia, consciousness disorder. What pathology does take place in this
case?
A. Acute glomerulonephritis
B. Acute pyelonephritis
C. Urolithiasis illness
D. Acute kidney insufficiency
E. 48. As a result of accident prevention violation poisoning by sulema took
place. In 2 days daily diuresis made 620 ml. Head pain, vomit, cramps,
appeared for a patient, dysphnoe, in lungs are moist wheezes. What pathology

does take place?


A. Pyelonephritis
B. Chronic kidney insufficiency
C. Glomerulonephritis
D.Acute kidney insufficiency'
E. Uremic coma
49. There were face edemas for a woman 30 years. At an inspection
discovered: hypoproteinemia, dysproteinemia, hyperlipidemia, proteinuria
(5,87 g/l). Such combination of symptoms is characteristic for?
A. Acute kidney insufficiency
B. Nephrotic syndrome
C. Nephritic syndrome
D. Pyelonephritis
E. Chronic kidney insufficiency
50. As a result of the protracted starvation glomerular speed filtration grew on
20%. What is the most reliable mechanism of filtration changes:
A. Increased arterial pressure
B. Decreased blood plasma oncotic pressure
C. Increased coefficient of filtration
D. Increased kidney plasma flow
E. Increased kidney filter permeability
*** *** ***

***

***

1. Examination of a 42 year old patient revealed a tumour of


adenohypophysis. Objectively: the patient's weight is 117 kg, he has moon-like
hyperemic face, red-blue striae of skin distension
on his belly.
Osteoporosis and muscle dystrophy are present. AP is 210/140 mm Hg. What
is the most probable diagnosis?
ACushing's disease
BCushing's syndrome
CConn's disease
DDiabetes mellitus
EEssential hypertension
2. A 50-year-old patient complains of thirst, drinking of a lot of water, marked
polyuria. Blood glucose is 4,8mmol/L, urine glucose and acetone bodies are
absent, urine is colorless, specific gravity is 1,002-1,004. What is the cause of
polyuria?
A Vasopressin insufficiency
Hypothyroidism
Insulin insufficiency
D Aldosteronism
E Thyrotoxicosis

3. To the doctor appealed the man 27 years. On examination it was revealed


an increase the hands, feet and lower jaw. Also observed deformation of joints
(kiphosis), hormonal disorders (impotence, testicular atrophy). Functions
whose cancer affected?
A * anterior part of the pituitary
B adrenal
C pineal body
D The thyroid
E parathyroid
4. Female 26 years, year after a difficult birth with bleeding complain of
general weakness, loss of body weight 18 kg, absence of menstruation.
OBJECTIVE: hypoplasia of the breast. Simmonds disease was diagnosed.
What is the main mechanism of weight loss in women?
A * decrease hormone production of adenohypophysis
B reduction function of sexual glands
C decrease in function of adrenal cortical layer
D hypothyroidism
E hypoparathyroidism
5. The 50-year-old patient after braine infectious disease significantly
increased urine output to 12 liters per day. In analyzing the blood of glucose
was 4.1 mmol / l. What hormone is most likely not enough?
A * antidiuretic
B glucagon
C insulin
D cortisone
E aldosterone
1. Examination of a patient revealed overgrowth of facial bones and soft
tissues, tongue enlargement, wide interdental spaces in the enlarged dental
arch. What changes of the hormonal secretion are the most likely?
A. Hyposecretion of insulin
B. Hyposecretion of thyroxin
C. Hyposecretion of the somatotropic hormone
D. Hypersecretion of insulin
E. Hypersecretion of the somatotropic hormone
2. A 32-year-old patient consulted a doctor about the absence of lactation after
parturition. Such disorder might be explained by the deficit of the following
hormone:
A. Vasopressin
B. Glucagon

C. Thyrocalcitonin
D. Prolactin
E. Somatotropin
3. The height of an adult is 100cm, in proportional constitution and normal
mental development. What hormone insufficiency do the indicated signs
characterize?
A. Growth hormone
B. Thyroxin
C. Antidiuretic
D. Mineralocorticoids
E. Gonadotropic hormone
5. Diabetes insipidus appeared in a patient after cerebral infarction with
impairment of hypothalamus nuclei. What is the cause for increased urination
in this patient?
A. Decrease of reabsorption of sodium
B. Acceleration of glomerular filtration
C. Decrease of water reabsorption
D. Decrease of arterial pressure
E. Hyperglycemia
6. A 10-year-old child has a height of 178 cm and weight of 64 kg. What
endocrine gland disorder does this connected with?
A. Sexual glands
B. Adrenal glands
C. Parathyroid glands
D. Pituitary gland
E. Thyroid gland
7. During an experiment on animals the nervous tracts leading to the crus of
hypophysis were broken. That impaired the entrance of the fallowing
hormones into the blood.
A. Hypophysis honnones
B. Vasopressin and oxytocin
C. Adenohypophysis hormone
D. Thyrotropic hormone
E. Adenocorticotropic hormone
8. A patient aged 20 complains of polydipsia and abundant urination (up to 10
liters for 24 hrs); concentration of glucose in his blood is normal; in his urine
glucose is absent. What hormone absence may cause such clinical picture?
A. Oxytocin
B. Vasopressin
C. Insulin
D. Hydrocortisone

E. Triiodothyronine
9. A patient aged 23 complains of headache, changes of his appearance
(increase of size of feet, fingers, features of the face), hoarseness of the voice,
worsening of the memory. The disease has begun 3 years ago without any
causes. Increase of superciliarry arches, nose, and tongue were found during
physical examination of this patient; the blood sugar was normal; the
urinalysis is out of changes. What is the cause of such condition?
A. Excessive production of corticosteroids
B. Excessive production of somatotropin
C. Insufficiency of aldosterone
D. Insufficiency of glucagon
E. Insufficiency of thyroxin
10. A woman aged 28 addressed to a doctor a month later after delivery with
complaints of decreased breast milk formation. What hormone insufficiency
caused this condition?
A. ACTH
B. Glucagon
C. Insulin
D. Prolactin
E. GH
11. A patient aged 50 complains of thirst, he drinks much water, has marked
polyuria. His blood glucose - 4.8 mmol/L, glucose and acetone bodies are
absent in the urine; urine is colourless, its specific gravity - 1.002-1.004. What
is the cause of polyuria?
A. Insufficiency of ADH
B. Hypothyrosis
C. Insulin insufficiency
D. Aldosteonism
E. Thyrotoxicosis
12. Atrophy of the testis developed in an athlete who systemically used
androgen hormones. This phenomenon is due to the inhibition of secretion of:
A. Gonadotropic hormone
B. Corticoliberine
C. Prolactoliberin
D. Gonadoliberine
E. Testosterone.
13. A patient aged 29 had acute massive blood loss during delivery. Then the
following changes developed: acute loss of weight, atrophy of skeletal muscles,
thinning of skin, decrease of body temperature, hypotension, and

hypoglycemia. What pathology of pituitary gland is the most possible in this


case?
A. Sheehan's disease
B. Parhon's syndrome
C. Diabetes insipidus
D. Adipose-genital dystrophy
E. Pituitary dwarfism
14. A girl aged 5 had the symptoms of premature sexual development, menses,
growth of mammary gland, adipose deposits in the field of pelvis and femur.
The most possible cause of premature sexual development is:
A. Adrenal adenoma
B. Hypopituitarism
C. Polycystic ovary syndrome
D. Hyperplasia of adrenal glands
E. Hormone- active tumor of ovaries
15. A 55-years-old man had an attack of angina pectoris which w as
accompanied by a severe pain behind the breastbone and resulted in anuria.
What mechanism influenced on the appearance of anuria?
A. Excessive somatotropin secretion
B. Excessive vasopressin secretion
C. Decrease of corticotropin secretion
D. Vasopressin deficiency
E. Somatotropin deficiency
16. A boy of 14 years old visited endocrinologist. His mother complains of his
heing behind in physical development and growth. The boy has proportional
constitution, his is 104 cm tall, and his secondary sexual characters are not
marked. The cause of this pathology is hyposecretion of:
A. Gonadotropic hormone
B. Growth hormone
C. All honnoncs of adenohypophvsis
D. ACTH
E. Thyroid stimulating hormone
17. A woman aged 40 complains of general malaise, pain in the heart area,
increase of body weight. At examination of the patient: she is 164 cm height
and 104 kg weight; she has predominant deposits of fat at face, neck,
shoulders, and abdomen; her BP - 165/100 mmHg; content of glucose in her
blood - 7.8 mmol/L. What endocrine gland function is infringed and what
direction at?
A. Pituitary gland, basophilic adenoma, hyperfunction
B. Pituitary gland, eosinophilic adenoma, hyperfunction
C. Sexual glands, liypofunction

D. Cortex of adrenal glands, hypo function


E. Thyroid gland, liypofunction
18. A child aged 4 years behind at physical development, sleeps badly, and has
increased irritability, thirst, and polyuria. Glucose is absent in patient's urine.
Patient has negative reaction to vasopressin injection. What is the mechanism
of revealed disorders?
A. Disturbance of realization of hormone biological activity
B. Impairment of neural regulation of endocrine glands
C. Disorders of biosynthesis and secretion of hormones
D. Disorders of neuroendocrine regulation
E. Disorders of feedback mechanisms
19. Loss of body weight, fatigue, pulling teeth out, and coming hairs out began
in a patient in several months after delivery. She often became ill. At
examination of the patient: subcutaneous adipose tissue is virtually absent; BP
- 90/55 mmHg; body temperature - 36.0 . Biochemical analysis of patient's
blood: glucose content - 3.0 mmol/L. There is increased level of growth
hormone and adrenocorticotropic hormone in her blood. What kind of
pituitary functions is impaired in this patient?
A. Pituitary dwarfism
B. Panhypopituitarism
C. Acromegaly
D. Cushing s disease
E. Diabetes insipidus
20. Polyuria and polydipsia developed in a 40-years-old patient accordingly to
impairment of hypothalamic-pituitary connective way. What hormone
deficiency evokes this changes?
A. Oxytocin
B. Antidiuretic hormone
C. Adrenocorticotropic hormone
D. Growth hormone
E. Thyroid stimulating hormone
21. In a patient excessive development of bones and soft tissues of his face,
enlargement of his tongue, and widened interspaces between teeth in enlarged
teeth arch were found out. What changes of hormone secretion are the most
possible in this patient?
A. Elevated secrction of insulin
B. Increased secretion of thyroxin
C. Increased secretion of grow the hormone
D. Reduced secretion of growth hormone
E. Elevated secrction of vasopressin

22. By X-ray examination of scull bones enlargement of cella turcica, thinned


anterior inclined appendices, and destruction of different places of cella
turcica were revealed. What endocrine gland tumor may lead to such bone
destruction?
A. Thyroid gland
B. Epiphysis
C. Adrenal gland
D. Thymus
E. Pituitary gland
23. A 38-years-old patient complains of thirst (he drinks 8 liters of water
daily), polyuria, loss of weight, and general malaise. He has been sick for 6
months. Urinalysis of the patient shows: special gravity - 1.001, leukocytes - 1
to 2 in field of vision, protein - trace amount. What reason evokes the constant
polyuria in this patient?
A. Reduction of ADH production
B. Injury of renal glomerules
C. Injury of renal tubules
D. Elevation of osmotic pressure of urine
E. Increase of oncotic pressure of urine
24. A child aged 2.5 has retardation in physical development, absence of
appetite, thirst, and polyuria; he sleeps badly. Sugar was not found in the
patient' urine. Which of endocrine pathologies is the reason for disorders of
water-salt metabolism?
A. Hyposecretion of antidiuretic hormone
B. Hypersecretion of antidiuretic hormone
C. Secondary hyperaldosteronism
D. Hyposecretion of growth hormone
E. Hyposecretion of adrenocorticotropic hormone
25. What changes in adult organism may be evoked by hypersecretion of
growth hormone?
A. Development of diabetes mellitus
B. Intensification of lipolysis
C. Intensification of urea production
D. Reduction of transport of amino acids to the cell
E. Increase in content of calcium in blood
26. What hormone increased secretion causes becoming thin at the period of
enhanced growth?
A. Growth hormone
B. Prolactin
C. Glucagon
D. Progesterone

E. Insulin
27. A 25-years-old male patient, who endured influenza, complains of
increased thirst (he drinks 5 to 6 liters of water daily), frequent abundant
urination, and loss of body weight. Patient has dry mucous membranes and
dry skin with reduced turgor. Urinalysis of this patient displays: urine is
colorless, its special gravity - 1.000-1.004, leucocytes and erythrocytes - 1 to 2
in field of vision. What pathology does this patient suffers from?
A. Diabetes insipidus
B. Diabetes mellitus
C. Conn's disease
D. Pheochromocytoma
E. Secondary hyperaldosteronism
28. When prophylactic examining of the first-year pupils in two of them lag in
their height by 2-3 sigma deviations was indicated. No another disorders were
revealed. What hormone deficiency could lead to this pathology?
A. Growth hormone
B. Sexual hormones
C. Insulin
D. Thyroid hormones
E. Glucocorticoids
*** ***
1. After an injury, the patients parathyroid glands were removed. He complaines:
apathy, thirst, a sharp increase in neuromuscular excitability. With metabolic
disorders substance which it is associated:
A * Calcium
B Manganese
C chlor
D molybdenum
E Zinc
2. The patient complains of increased irritability, periodic subfibrilitet. Pulse rate
-120/min. In blood increased hormones T3 and T4. What endocrine pathology most
logical suspect?
A * Hyperthyroidism.
B Adrenal insufficiency.
C Hypoparatireosis
D Hyperparatireosis
E Hypothyroidism.
3. Patient from Carpathians region, suffering from endemic goiter, turned to the
doctor with complaints of gingival pus inflammation and loosening of teeth. Which in
this case is a major factor in the development of periodontitis?

A * Endocrine disorders
B stress influences
C hyposalivation
D swallowing difficulties
E malnutrition
4.
1. A 56 y.o. patient has been suffering from thyreotoxicosis for a long time.
What type of hypoxia can be developed?
A Tissue
Hemic
Circulators
D Respiratory
E Mixed
2. A patient is followed up in an endocrinological dispensary on account of
hyperthyreosis. Weight loss, tachycardia, finger tremor are accompanied by
hypoxia symptoms - headache, fatigue, eye flicker. What mechanism of
thyroid hormones action underlies the development of hypoxia?
A Disjunction of oxydation and phosphorilation
Inhibition of respiratory, ferment synthesis
Competitive inhibition of respiratory ferments
D Intensification of respiratory ferment synthesis
E Specific binding of active centres of respirator; ferments
3. A 46-year-old patient suffering from the diffuse toxic goiter underwent
resection of the thyroid gland. After the surgery the patient presents with
appetite loss, dyspepsia, increased neuromuscular excitement. The body
weight remained unchanged. Body temperature is normal. Which of the
following has caused such a condition in this patient?
A Reduced production of parathormone
Increased production of thyroxin
Increased production of calcitonin
D Increased production of thyroliberin
E Reduced production of thyroxin
4. Periodic renal colics attackes are observed in the woman with primery
hvperparathyroidizm. Ultrasonic examination revealed small stones in the
kidneys. What is the cause of the formation of the stones?
A Hypercalcemia
Hyperphosphatemia
Hypercholesteremia
D Hyperuricemia
E Hyperkalemia

5. Female 53 years old, height 163 cm, body weight 92 kg, even laying fat,
edemata face, slow-moving, apatic. When pressing the foot is a skin dimple.
Disturbances of which gland function takes place?
A * Thyroid.
B pituitary.
C adrenal.
D sexual
E Parathyroid.
6. The patient was diagnosed thyrotoxocosis. In the blood found antithyroid
antibodies. What type of allergic reaction Coombs and Gell observed in the
development of this disease?
A * Stimulating
B anaphylactic
C. Cytotoxic
D immunocomplex
E Delayed type hypersensitivity
7. A woman with primary hyperparathyroidism has periodically repeated attacks
of renal colic. Ultrasound examination shows presence of small stones in her kidneys.
What is the cause of the formation of these stones?
A
B
C
D
E

Hypercalcemia
Hypercholesterolemia
Hyperuricemia
Hyperkalemia
Hyperphosphatemia

8. A female patient with hyperthyroidism has increased body temperature. What is


the mechanism of body temperature elevation?
A
B
C
D
E

Separation of oxidation and oxidative phosphorilation


Increase utilization of glucose by tissues
Intensification of glycogenosis
Intensification of protein catabolism
Elevation of oxidation of fat in liver

9. Female 55 years old, living in the mountains, diagnosed with endemic


goiter. OBJECTIVE: slightly increased fatness, inhibited, apathetic,
enlargement of the thyroid gland. Deficiency of which of the following
elements causes this condition?
A * Iodine
B fluoride
C manganese

D molybdenum
E sodium
11. A patient, who lives in mountain region, was admitted to the hospital with
diagnosis of endemic goiter. What is characteristic for this disease?
A Decreasing of thyroxin and triiodthyronin synthesis
B
C
D
E

Increase of thyroid hormone production


Toxic action of thyroxin and triiodthyronin
Chronic hypoparathyroidism
-

12. The method of indirect calorimetry revealed that the basal metabolic
investigated 40% lower. Violation of any of the endocrine glands can suggest?
A * thyroid gland.
B thymus.
C pancreas.
D epiphysis
E parathyroid
13. The patient found a stable increase in body temperature, tachycardia,
emotional lability, tremor. With the change in hormone production which
linked the emergence of this situation?
A * thyroxine.
B vasopressin.
C testosterone.
D aldosterone.
E insulin.
14. The patient 45 years old during surgery on the thyroid gland
parathyroid accidentally deleted. This has resulted in:
A * tetany
B Increased calcium levels and bone resorption
C Increased calcium, sodium and potassium in the blood
D Decrease in blood pressure
E Increase in blood pressure
A female patient with hyperthyroidism has increased body temperature. What
is the mechanism of body temperature elevation?
A. Separation of oxidation and oxidative phosphorilation
B. Increase utilization of glucose by tissues
C. Intensification of glycogenosis

D. Intensification of protein catabolism


E. Elevation of oxidation of fat in liver
Female 55 years old, living in the mountains, diagnosed with endemic
goiter. Obyektyvno: slightly increased fatness, inhibited, apathetic,
enlargement of the thyroid gland. Deficiency of which of the following
elements causes this condition?
A * Iodine
B fluoride
C manganese
D molybdenum
E sodium
Female 55 years old, living in the mountains, diagnosed with endemic
goiter. OBJECTIVE: slightly increased fatness, inhibited, apathetic,
enlargement of the thyroid gland. Deficiency of which of the following
elements causes this condition?
A * Iodine
B fluoride
C manganese
D molybdenum
E sodium
A patient, who lives in mountain region, was admitted to the hospital with
diagnosis of endemic goiter. What is characteristic for this disease?
A. Decreasing of thyroxin and triiodthyronin synthesis
B. Increase of thyroid hormone production
C. Toxic action of thyroxin and triiodthyronin
D. Chronic hypoparathyroidism
E. The method of indirect calorimetry revealed that the basal metabolic
investigated 40% lower should. Violation of any of the endocrine glands can
suggest?
A * thyroid gland.
B thymus.
C pancreas.
D epiphysis
E parathyroid
The patient found a stable increase in body temperature, tachycardia,
emotional lability, tremor. With the change in hormone production which
linked the emergence of this situation?
A * thyroxine.
B vasopressin.

C testosterone.
D aldosterone.
E insulin.
The patient 45 years old during surgery on the thyroid gland parathyroid
accidentally deleted. This has resulted in:
A * tetany
B Increased calcium levels and bone resorption
C Increased calcium, sodium and potassium in the blood
D Decrease in blood pressure
E Increase in blood pressure
In dog with endocrine disorders were found: a decrease in oxygen
consumption at rest, decreased glucose tolerance, decreased body
temperature. Hormone deficiency which can be found to explain the changes?
A * Thyroxine
B insulin
C somatotropin
D adrenocorticotropic
E gonadotropic
In dog with endocrine disorders were found: a decrease in oxygen
consumption at rest, decreased glucose tolerance, decreased body
temperature. Hormone deficiency which can be found to explain the changes?
A * Thyroxine
B insulin
C somatotropin
D adrenocorticotropic
E gonadotropic
1. Under the influence of harmful ecological factors the normal formation of
lysosomes is inhibited in thyrocytes. What stage of hormone production of
thyroid gland will be impaired?
A. Synthesis of colloid
B. Iodinisation of colloid
C. Synthesis of thyroglobulin
D. Reabsorption of colloid
E. Proteolysis of phagocytosed colloid from follicles.
2. What hormone stimulates inclusion of calcium in osteoblasts of bone tissue
in tooth?
A. Insulin
B. Thyroxin
C. Parathormone
D. Hydrocortisone

E. Calcitonin
3. A patient with thyrotoxicosis has hyperthermia, bulimia, and loss of weight.
What kind of impairment does this connected to?
A. Reaction of fat synthesis
B. Lyses of adenosintriposphoric acid
C. Conjunction oxidation and phosphorylation
D. Reaction of citric acid cycle
E. Reaction of beta-oxidation of fatty acid.
4. A child has a time disturbance of teeth eruption, enamel anomalies, and its
lips and tongue are enlarged. What hormone deficiency causes these changes?
A. Thyroxin
B. Parathormone
C. Thyrocalcitonin
D. Insulin
E. Somatotropin
5. At clinical examination of a woman it was determined increase of basal
metabolism rate by 40%, increased perspiration, tachycardia, and leanness.
What endocrine gland functions are impaired and what direction in?
A. Sexual glands, hypofunction
B. Cortical substance of adrenal glands, hyperfunction
C. Medulla-of adrenal glands, hyperfunction
D. Thyroid gland, hyperfunction
E. Pancreatic gland, hyperfunction
6. A woman with primary hyperparathyroidism has periodically repeated
attacks of renal colic. Ultrasound examination shows presence of small stones
in her kidneys. What is the cause of the formation of these stones?
.Hypercholesterolemia
.Hyperuricemia
.Hyperkalemia
D.Hyperphosphatemia
E.Hypercalcemia
7. The attacks of convulsions appeared in a patient after resection of thyroid
gland. What preparation must be administered in this case?
A. Somatotropin
B. Insulin
C. Thyroxin
D. Prednisolon
E. Parathyroidin

8. A dog with endocrine pathology had decreased oxygen usage in the state of
rest, decreased body temperature, and decrease of glucose tolerance. What
hormone insufficiency may explain the discovered changes?
A. Growth hormone
B. Thyroxin
C. Gonadotropin
D. Adrenocorticotropic hormone
E. Insulin
9. Liquidator of an accident at Chernobilska Power plant began complaining
of increased excitability, nervousness, heartbeat, decrease of body weight,
constant weakness, body tremor, feeling of fever, bad heat endurance. What
gland hyperfunction may be the cause of such state?
A. Thyroid gland
B. Adenohypophysis
C. Adrenal gland
D. Medulla of adrenal gland
E. Parathyroid gland
10. A woman complains of increased irritability, perspiration, weakness, loss
of body weight, tremor of extremities, increased heartbeat rate, and
exophthalmia. What metabolic impairment in the organism accompanies this
disease?
A. Increase of adenosine triphosphoric acid synthesis
B. Decrease of organism sensitivity to hypoxia
C. Weakening of phospholipase activation
D. Increase of basal metabolism
E. Decrease of cholesterol lysis
11. The manifestations of hypoparathyrosis developed in a patient after
strumectomy. What changes in the organism are observed in this case?
A. Hypophosphatemia
B. Resorption of bone tissue
C. Acidosis
D. Hypocalcacmia
E. Decrease of neuromuscular excitability
12. Small height, disproportional development of the body, and insufficient
mental development were found in a boy during examination. What hormone
deficiency caused these changes?
A. Thyroxin
B. Parathormone
C. Thyrocalcitonin
D. Adenocorticotropic hormone
E. Oxytocin

13. With the help of indirect calorimetrical measurement it was determined


that the basal metabolism of the patient was 40% lower than the proper one.
What endocrine gland hypofunction is the cause of described changes?
A. Adrenal glands
B. Thyroid gland
C. Epiphysis
D. Thymus
E. Pancreas
14. Fibrillary muscular tics of arms, legs and face appeared in a woman aged
46 after the operation on thyroid gland. These disorders may be eliminated by
injection of:
A. Parathyroidin
B. Triiodthyronine
C. Thyrotropin
D. Thyroxin
E. Calcitonin.
15. A patient with diffuse toxic goiter has marked exophthalmia. The
appearance of exophthalmia is explained in this pathology by:
A. Antibody circulation against thyroglobulin
B. Presence of immunoglobulins in the blood
C. Production of exophthalmia factor
D. Increase of adrenorecpetor sensitivity to catecholamines
E. Pathogenic action of thyroid prostaglandins.
16. A patient with myxedema came to a doctor. Her face was puffy with poor
facial expression; she had thickened nose and lips. These signs can be
explained by:
A. Free filtration of sodium in glomeruli
B. Accumulation of hydrophilic mucous substances
C. Impairment of sensitivity of volumo- and osmoreceptor
D. Increase of sodium reapsorption in tubules (canaliculi)
E. Increased permeability of capillary walls.
17. Weakness, thirst, sharp increase of neuromuscular excitability with the
development of parathyroid tetany were observed in a dog 1-2 days later after
resection of thyroid glands. What disturbance of electrolyte exchange takes
place in this condition?
A. Hypercalcaemia
B. Hypocalcaemia
C. Hypomagnemia
D. Hypermagnemia
E. Hyponatremia

18. Which of the signs that develops in hyperthyrosis is the most important for
making diagnosis?
A. Tachycardia
B. Subfebrile temperature
C. Increase of basal metabolism
D. Increase excitability
E. Disorder of sleep
19. A patient aged 41 complains of weakness, sweating, fever, tremor of hands,
BP- 160/90 mm Hg. Diffuse toxic goiter was diagnosed (Basedow's diseas).
What is the main mechanism of impairment of the function of cardio-vascular
system in this disease?
A. Decrease of tonus of sympathetic nervous system tonus
B. Increase of tonus of sympathetic nervous system tonus
C. Auto immune reactions
D. Increase of catabolism
E. Hyperthermia
20. Thyrotoxicosis was produced in an animal experimentally. For confirming
this state it is necessary to determine the level of metabolism according to food
taking and phy sical load. What experimental method must be used in this
case?
A. Determination of iodine content in the thyroid gland
B. Determination of concentration of organic iodine in blood
C. Direct calorimetry
D. Determination of oxygen tension in the blood
E. Determination of the content of the gland hormone in the blood
21. A boy aged 9 is in endocrinology department because of increase bone
fragility. What endocrine organ is impaired?
A. Adrenal glands
B. Thymus
C. Thyroid gland
D. Parathyroid gland
E. Epiphysis
22. A child has infringements of teeth enamel and dentin formation because of
reduced content of calcium in his blood. What hormone deficiency may evoke
these changes?
A. Growth hormone
B. Triiodthyronin
C. Parathormone
D. Thyroxin
E. Thyrocalcitonin

23. After the operation on thyroid gland tetany developed in a patient that
manifested in convulsions of striped muscles. What does this complication
may be connected to?
A. Mistaken removal of parathyroid glands
B. Insufficient removal of thyroid tissue
C. Hyperfunction of adrenal glands
D. Hypofunction of adrenal glands
E. Elevated blood content of thyroid-stimulating hormone
24. A female patient with hyperthyroidism has increased body temperature.
What is the mechanism of body temperature elevation?
A. Separation of oxidation and oxidative phosphorilation
B. Increase utilization of glucose by tissues
C. Intensification of glycogenosis
D. Intensification of protein catabolism
E. Elevation of oxidation of fat in liver
25. A woman has fatigue, sleepiness, apathy, worsening of memory, and
edemas. These symptoms appeared in several years after change of dwelling.
"Endemic goiter" is diagnosed in this patient. What substance deficit in water
and food may lead to this disease?
A. Iodine
B. Fluoride
C. Iron
D. Calcium
E. Magnesium
26. A patient, who lives in mountain region, was admitted to the hospital with
diagnosis of endemic goiter. What is characteristic for this disease?
A. Decreasing of thyroxin and triiodthyronin synthesis
B. Increase of thyroid hormone production
C. Toxic action of thyroxin and triiodthyronin
D. Chronic hypoparathyroidism 27. What changes result from separation in oxidation and oxidative
phosphorilation?
A. Elevation in heat production
B. Decrease of free oxidation part
C. Decrease in heat production
D. Enhancement of functional activity of organs
E. Increase in ATP production

28. A 40-years-old patient complains of decreased capability of working,


sleepiness, sensitivity to cold, fragility and coming out of hairs, dryness of
skin, and edema of face and extremities. There is no formation of pit under
the pressing at the anterior surface of patient's calves. Patient has slow tendon
reflexes and bradicardia. What infringement do these changes result from?
A. Hypothyroidism
B. Adrenal insufficiency
C. Hypoparathyroidism
D. Vitamin deficiency
E. Iron deficiency
29. A doctor suspects the hypothyroid goiter in a patient with enlarged thyroid
gland. What sign has a decisive importance for establishing the diagnosis?
A. Decrease of basal metabolism
B. Bradicardia
C. Edemas
D. Arterial hypotension
E. Hypodynamism
30. A 50-years-old female patient, who was operated on thyroid gland for
diffuse toxic goiter, begins complaining that she is spiritless and slow, has fast
fatigability, increased working ability, sleepiness, worsening of memory, and
increasing of body weight At examination the patient has dry skin, edematous
face, and striped nails with broken edges. What is your assumable diagnosis?
A. Myxedema
B. Cushing's disease
C. Acromegaly
D. Obesity
E. Thyrotoxicosis
*** ***
1. To prevent the transplant rejection after organ transplantation it is
required to administer hormonotherapy for the purpose of
immunosuppression. What hormones are used for this purpose?
A Glucocorticoids
Mineralocorticoids
Sexual hormones
D Catecholamines
E Thyroid
2. A girl is diagnosed with adrenogenital syndrome (pseudohermaphroditism).
This pathology was caused by hypersecretion of the following adrenal
hormone:
A Androgen
Estrogen

Aldosterone
D Cortisol
E Adrenalin
3. Examination of a 42 year old patient revealed a tumour of
adenohypophysis. Objectively: the patient's weight is 117 kg, he has moon-like
hyperemic face, red-blue striae of skin distension on his belly. Osteoporosis
and muscle dystrophy are present. AP is 210/140 mm Hg. What is the most
probable diagnosis?
A Cushing's disease
Cushing's syndrome
Conn's disease
D Diabetes mellitus
E Essential hypertension
1. A patient has been given high doses of hydrocortisone for a long time. This
caused atrophy of one of the adrenal cortex zones. Which zone is it?
A. Glomerular and reticular
B. Glomerular
C. D. Reticular
E. Fascial
2. A patient with android-type obesity had been suffering from arterial
hypertension, hyperglycemia, glycosuria for a long time and died from the
cerebral haemorrhage. Pathologic examination revealed pituitary basophil
adenoma, adrenal cortex hyperplasia. What is the most likely diagnosis?
A. Itsenko-Cushing's disease
B. Acromegalia
C. Adiposogenital dystrophy
D. Diabetes mellitus
E. Pituitaiy nanism
3. A careless student suddenly meets his dean. What hormone concentration
will be increased in student's blood more rapidly?
A. Somatotropin
B. Hydrocortisone
C. Corticotropin
D. Thyroliberine
E. Adrenalin
4. A women aged 44 complains of general malaise, pain in the heart area, and
considerable increase of body weight. At examination of this patient following
symptoms are revealed: lunar face, hirsutism, BP 165/100 mmHg, height is

164 cm, weight is 103kg, the adipose deposits mainly on the neck, shoulders,
abdomen. What is the main pathogenic mechanism of the woman's obesity?
A. Decrease of thyroid hormone production
B. Decrease of glycogen production
C. Increase of glucocorticoid production
D. Increase of insulin production
E. Increase of minerelocorticoid production.
5. Adrenalin is used to prolong Novocain action in infiltration anesthesia.
What adrenalin action is this effect connected with?
A. Potentiation of Novocain action at the level of CNS
B. Inhibition of function of nerve ending and conductors
C. Dilatation of vessels
D. Constriction of vessels
E. Inhibition of tissue esterases
6. Persistent hyperglycemia developed in a patient with Cushing's syndrome
under the influence of excessive amount of glucocorticoids. What is the
mechanism of hy perglycemia in this case?
A. Activation of glycogenolysis at the liver
B. Activation of gluconeogenesis
C. Activation of insulinaze of the liver
D. Decrease of hexokinase activation
E. Increase of absorption of glucose in intestine
7. A man aged 38 is in stress state due to industrial conflict. Which of below
mentioned hormones participate in starting stress reaction of the organism?
A. Adrenalin
B. ACTH
C. Glucagon
D. Hydrocortisone
E. Thyroxin
8. While examining a patient a doctor suspected Cushing's syndrome. What
substance determination in patient's blood will prove doctor's supposition?
A. Tocopherol
B. Adrenalin
C. Cholesterol
D. Retinol
E. Cortisol
9. A patient aged 40 was hospitalized with complaints of general malaise,
convulsions of upper and lower extremities, his BP - 160/100 mm Hg. The
results of laboratory examination of him: blood glucose - 6.5 mmoI/L, calcium

- 2 mmol/L, phosphates - 1 mmol/L, sodium-160 mmol/L. Urination-700 ml


for 24 hours. What pathology causes such state?
A. Hyperaldosteronism
B. Hypoaldosteronism
C. Hyperparathyroidism
D. Thyrotoxycosis
E. Rickets
10. Why must a patient, who has taken prednisolone for rheumatoid arthritis
for a long time, avoid contacts with infectious patients?
A. Because of the development of secondary immunodeficiency
B. Because of risk of exacerbation of arthritis
C. Because of risk of thromboembolic complications development
D. Because of the development of lymphopenia
E. Because of the blockade of interferon formation
11. In case of hypercortisolism - Cushing's disease - the following changes in
the organism take place:
A. Development of cachexia
B. Impoverishment of the liver with glycogen
C. Hyperglycemia
D. Hypotension
E. Lymphocytosis
12. A patient admitted to the hospital complains of quick fatigability, loss of
weight, hyperpigmentation of the skin. Her heart sounds are dull. She has
pulse rate - 96 beats per minute and BP - 90/50 Hg. What metabolic
impairments are observed in hypocortisolism?
A. Hyperkalemia
B. Increase of glycogen synthesis
C. Hypernatremia
D. Hyperhydration
E. Hypoglycemia
13. After suffered sepsis a bronze color of the skin typical for Addison's
disease appeared in a woman aged 27. Hyperpigmentation occurs due to the
increased secretion of:
A. Melanocytstimulating hormone
B. Growth hormone
C. Adrenocorticotropic hormone
D. B-lipotropic hormone
E. Thyrotropic hormone
14. A patient with rheumatic arthritis was given hydrocortisone for a long
period of time. He developed hyperglycemia, polyuria, glycosuria, and thirst.

These complications after treatment develop due to the activation of the


process of:
A. Gluconeogenesis
B. Glycogenolysis
C. Glycogenesis
D. Glycolysis
E. Lipolysis
15. During the examination of the patient a doctor found out Cushing's
disease that is characterized by obesity. It is connected with:
A. Excessive use of fats with meal
B. Impairment of ventromedial nuclei of hypothalamus
C. Inhibition of adrenalin synthesis by adrenal glands
D. Production of excessive amount of glycocoticoids
E. Hereditary tendency to hyperlipemia.
16. After physic exertion a patient with pheochromocytoma complains of
tachycardia, increased arterial pressure, and sharp pain in epigastric area.
These attacks may be explained by:
A. Massive release of catecholamines by adrenal glands
B. Release of norepinephrine by sympathetic nerves
C. Activation of hypothalamus vegetative nuclei
D. Increase of thyroid hormone secretion
E. Increased synthesis of adrenocorticotrophic hormone
17. Excessive secretion of a certain hormone was observed in a patient with
pheochromocytoma -a tumor, which appears from the medulla of adrenal
glands. How is this hormone called?
A. Glucagon
B. Insulin
C. Thyroxin
D. Adrenalin
E. Somatotropin
18. Adenoma, which grows from the cells of glomerular zone of adrenal cortex
and is the source of excessive formation of aldosterone was found in a patient.
This resulted in a development of primary hyperaldosteronism or Conn's
disease. What electrolyte exchange does this hormone influence?
A. Ferric ion
B. Calcium ion
C. Magnesium ion
D. Chlorine ion
E. Sodium ion
19. Male patient aged 55 had an increase of pituitary gland dimensions and
hyperplasia of adrenal cortex. At examination of the patient: BP-190/90 mm

Hg, content of blood glucose is 20 mmol/L; there are glucosuria, obesity, and
histurism. What pathology are these changes ty pical for?
A. Barraker-Simmond disease
B. Adipose-genital dystrophy
C. Cushing's syndrome
D. Addison's disease
E. Cushing's disease
20. A 28-years-old patient complains of flaccidity, quick mental and physical
fatigue, and dyspeptic disorders. During the examination following was found
out: positive tests, hypoglycemia, BP - 90/60 mm Hg, hyponatremia, skin
pigmentation. What disease of adrenal glands underlies observed symptoms?
A. Cushing's syndrome
B. Addison's disease
C. Acute insufficiency of adrenal cortex
D. Hypofunction of medullar layer of adrenal glands
E. Conn's syndrome
21. Conn's syndrome was diagnosed in a patient who complained of muscular
weakness, increased urination (at night), and increased arterial blood
pressure. What is typical for this syndrome?
A. Increase of renin, increase of aldosterone, and increase of potassium
B. Decrease of renin, increase of aldosterone, and increase of potassium
C. Decrease of renin, increase of aldosterone, and decrease of potassium
D. Decrease of renin, decrease of aldosterone, and decrease of potassium
E. Increase of renin, decrease of aldosterone, and increase of potassium
22. A female patient aged 44 complains of general malaise, large increase of
body weight, growth of hair on the face, arrest of menses, BP-165/100 mm Hg.
What diagnostic test will help to differentiate Cushing's disease from
Cushing's syndrome?
A. Level of ACTH in blood plasma
B. Level of Cortisol in plasma
C. Excretion of 17-oxyketosteroids with urine
D. X-ray of the skull
E. Number of eosinophils in blood
23. Three months later after delivery female patient C., aged 30, began to
complain of increase of body weight, excessive adipose deposits, mainly in the
area of the neck and face, growth of hair on the upper lip. The increase of size
of cella turcica was determined by X-ray examination. What pathogenic
mechanism underlies determined pathology?
A. Hypersecretion of glucocorticoids
B. Increase of food taking
C. Hyposecretion of thyroxin

D. Development of diabetes mellitus


E. Birth of a child
24. A 51-years-old male patient has been suffering from tuberculosis for 10
years. He is abusing alcohol. At last time, complaints of irritability, fast
fatigability, syncope, muscular weakness, and loss of weight appear in him. At
examination of the patient: hyperpigmentation of his skin; his BP is 90/60
mmHg. What is the most possible reason for disease development in this
patient?
A. Chronic alcohol intoxication
B. Impairment of adrenal gland due to tuberculosis
C. Alcoholic injury of the liver
D. Hypothalamic tumor
E. Thyroid gland impainnent
25. In a patient aged 23 BP rapidly drops down to 70/40 mmHg after surgical
removal of left adrenal gland. Patient has rapid, weak pulse, vomiting, and
cramps; he is covered by cold sweat. What is the most possible cause for this
condition?
A. Hypertrophy of the right adrenal gland
B. Hypofunction of adenohypophisis
C. Atrophy of the right adrenal gland
D. Hyperfunction of adenohypophisis
E. Hypofunction of neurohypophisis
26. A 17-years-old female patient, who has been ill for 6 months, complains of
appearance of mustache and beard, roughness of voice, and absence of
menses. At examination of the patient hirsutism and undeveloped mammal
glands and sexual organs were revealed. What glands impairment has lead to
this pathology?
A. Adrenal glands
B. Thyroid gland
C. Neurohypohpisis
D. Adenohypophisis
E. Sexual glands
27. Hans Sallie, author of doctrine of stress, showed that different stimuli
(heat, cold, pain) always evoke standard non-specific reaction: 1) involution in
thymico- lymphatic system; 2) hemorrhage acute ulcers of stomach and
duodenum. Name the third compound of classical triad.
A. Hypertrophy of adrenal cortex
B. Hypertrophy of beta cells of Langerhans' islets
C. Myocardial infarction
D. Parodontitis
E. Cerebral hemorrhage

28. Arterial hypotension, muscular weakness, and periodic convulsions appear


in the patient with hepatic cirrhosis. Content of sodium is increased and
content of potassium is decreased in patient's blood. What kind of endocrine
disorders underlies this symptoms?
A. Secondary hyperaldosteronism
B. Hypopituitarism
C. Primary hyperaldosteronism
D. Hyperpituitarism
E. Hypoaldosteronism
29. Prolonged intake of mineralocorticoids led to development of muscular
weakness in a patient. What gives rise to these symptoms?
A. Hypokalemia
B. Hyperkalemia
C. Hypernatremia
D. Hyponatremia
E. Hypervolemia
30. Osmotic pressure of man's blood plasma is 350 mosmole/l (standard
pressure is 300 mosmole/l). First of all it will result in high secretion of the
following hormone:
A. Cortisol
B. Aldosteron
C. Natriuretic
D. Vasopressin
E. Adrenocorticotropin
31. Examination of the patient revealed hvperkaliemia and hyponatremia.
Low secretion of wich hormone may cause such changes?
A. Natriuretic
B. Aldosteron
C. Cortisol
D. Parathormone
E. Vasopressin
32. A patient with infectious mononucleosis has been taking glucocorticoids
for two weeks. He has brought into remission, but he fell ill with acute attack
of chronic tonsillitis. What action of glucocorticoids caused this complication?
A. Antitoxic
B. Anti-inflammatory
C. Antishock
D. Antiallergic
E. Immunosuppressive

33. A 45 y.o. woman suffers from Cushing's syndrome- steroid diabetes.


Biochemical examination revealed: hyperglycemia, hypochloremia. Which of
the undermentioned processes is the first to be activated?
A. Glucogenolysis
B. Glucose reabsorption
C. Glucose transport to the cell
D. Gluconeogenesis
E. Glycolysis
34. A 40 y.o. patient cpmplains of intensive hearbeats, sweating, nausea, vision
impairment, arm tremor, hypertension. From his anamnesis: 2 years ago he
was diagnosed with pheohhromocytoma. Hyperproduction of what hormones
causes the given pathology ?
A. ACTH
B. Glucocorticoids
C. Thyroid hormones
D. Catecholamines
E. Aldosterone
35. A 59 year old patient is a plant manager. After the tax inspection of his
plant he felt intense pain behind his breastbone irradiating to his left arm. 15
minutes later his condition came to normal. Wich of the possible mechanisms
of stenocardia development is the leading in this case?
A. Coronary thrombosis
B. Functional heart overload
C. High catecholamine concentration in blood
D. Intravascular aggregation of blood corpuscles
E. Coronary atherosclerosis
36. A concentrated solution of sodium chloride was intravenously injected to
an animal. This caused decreased reabsorption of sodium ions in the renal
tubules. It is the result of the following changes of hormonal secretion:
A. Vasopressin reduction
B. Vasopressin increase
C. Aldosterone increase
D. Aldosterone reduction
E. Reduction of atrial natriuretic factor
*** ***
1. A girl has been diagnosed with adrenogenital syndrome
(pseudohermaphroditism). This pathology is caused by hypersecretion of the
following adrenal hormone:
A. Androgens
B. Glucocorticoids

C. Catecholamines
D. Estrogens
E. Mineralocorticoids
1. A young man aged 17 has the signs of retention of sexual development; his
height is 184 cm; he is asthenic; his secondary sexual characters are weakly
developed. Sex chromatin is absent in this patient's cells. At the age of 4 he
endured epidemic parotitis. What was the cause of hypogonodism?
A. Deficiency of gonadoliberine
B. Deficiency of gonadotropin
C. Cryptorchism
D. Orchitis
E. Chromosomal anomalies (XXY)

B.
C.
D.
E.

2. Pilosis by male type and increase of muscular mass


began appearing in a boy of 5 years old who had
developed previously without declination from the
age norms. At examination of the patient marked
secondary male sexual signs were revealed, but the
size of his testes corresponds to his age. What is the
cause of precocious puberty?
A Androgen producing adrenal tumor
Hormone producing testis tumor
Increase of gonadotropin production
Increase of adrenocorticotropin production
Increase of gonadoliberin production

A.
B.
C.
D.
E.

3. Removal of cancer tumor of the testis in a patient before the period


of sexual maturity resulted in the development of eunochoidism and
was accompanied by the deficiency of the production of:
Androgens
Estrogens
Kinines
Prostaglandins
Cytokines

4. A boy of 14 years old visited endocrinologist. His mother complains


of his being behind in physical development and growth. The boy has
proportional constitution, his is 104 cm tall, and his secondary sexual
characters are not marked. The cause of this pathology is
hyposecretion of:
A. Gonadotropic hormone
B. Growth hormone
C. All hormones of adenohypophysis

D. ACTH
E. Thyroid stimulating hormone

B.
C.
D.
E.

5. When stomatological examining the patient aged 18 the following


symptoms were revealed: woman features of his face, anomalies of
form of crowns in some teeth, catarrhal gingivitis, and resorption of
interdental junctures. Cytological examination of mucous membrane
epithelium discovers that cells contain 1 Barr's body. What
endocrine pathology are these manifestations the most probably
connected to?
A. Hypogonadism
Hypergonadism
Cretinism
Thyrotoxicosis
Chronic hypoparathyroidism

A.
B.
C.
D.
E.

6. What pathological conditions the tumor growing from reticular zone


of adrenal gland may lead to?
Virilization of female organism
Elevation in content of glucocorticoids in blood
Tachycardia
Elevation of content of mineralocorticoids in blood

A.
B.
C.
D.
E.

7. Parents of a 10 y.o. boy consulted a doctor about extension of haircovering, growth of beard and moustache, low voice. Intensified
secretion of which hormone must be assumed?
Of progesterone
Of testosterone
Of oestrogen
Of somatotropin
Of Cortisol

A.
B.
C.
D.
E.

8. A
girl
is
diagnosed
with
adrenogenital
syndrome
(pseudohermaphroditism). This pathology was caused by
hypersecretion of the following adrenal hormone:
Cortisol
Adrenalin
Aldosterone
Estrogen
Androgen
9. Parents of a 10 year old boy consulted a doctor about extension of
hair-covering, growth of beard and moustache, low voice. Intensified
secretion of wich hormone must be assumed?

A.
B.
C.
D.
E.

Of testosterone
Of somatotropin
Of progesterone
Of Cortisol
Of oestrogen

10. In the pubertal period cells of the male sexual glands start producing the
male sexual hormone testosterone that is responsible for formation of the
secondary sexual characters. What cells of the male sexual glands produce this
hormone?
A. Spermatozoa
B. Leidigs cells
C. Sertoli's cells
D. Sustenocytes
E. Sustentacular cells
*** ***
1. After amputating the upper extremity a patient had a bad pain in it. Which
mechanism of the pain feeling formation is more possible in this case?
A. Phantom
B. Reflex
C. Hyposecretion of endorphin
D. Hypersecretion of endorphin
E. Hyposecretion of encephalin
2. After a road accident a patient was diagnosed with a trauma of the
brachium with incomplete rupture of the median nerve. Besides, disorders of
the motor and sense functions, the patient complains of sharp, stinging,
intolerable pain. What kind of pain is it?
A. Somatic.
B. Projective.
C. Reflected.
D. Phantom.
E. Causalgia.
3. A patient, complaining of pain in the region of the left scapula, was
diagnosed with myocardial infarction. What kind of pain is it?
A. Phantom.
B. Visceral.
C. Irradiating (reflected).
D. Early (protopathic).
E.Late (epycritical).
*** ***

1. An experimental rat with extremity paralysis has no tendon and


cutaneous reflexes, muscle tone is decreased, but muscles of the
affected extremity maintain their ability to react with excitation to
the direct action of continious current. What type of paralysis is it?
A Flaccid peripheral
Flaccid central
Spastic peripheral
D Spastic central
E Extrapyramidal
2. A 28 year old man had a gunshot wound of shin that resulted in an
ulcer from the side of the injury. What is the main factor of
neurodystrophy pathogenesis in this case?
A Traumatization of peripheral nerve
Psychical stress
Microcirculation disturbance
D Infection
E Tissue damage

A.
B.
C.
D.
E.

1. Increase of blood pressure and rapid pulse are noticed in a


sportsman at the start before competitions. Influence of which
part of the CNS can above-mentioned changes be explained?
Cortex of hemispheares
Medulla oblongata
Mesencephalon
Diencephalons
Hypothalamus

A.
B.
C.
D.
E.

2. Ptyalism, bradycardia (heart rate 45 beats per minute), miosis


are observed in a man. What is the most possible reason for
such changes?
Increase of sympathetic influence
Increase of parasympathetic influence
Decrease of sympathetic influence
Decrease of parasympathetic influence
Decrease of vegetative influence

3. The reactions, typical for stimulation of parasympathetic


nerves, appeared after irritation of throphogenic zone of
hypothalamus, including preoptic nucleas and anterior
hypothalamic area. What appears in this case?
A. Tachycardia
B. Mydriasis

C. Exophthalmus
D. Decrease of BP
E. Hyperglycemia
4. A patient had hemiplegia after insult. What disorder is
observed in this case?
A.
B.
C.
D.
E.

A.
B.
C.
D.
E.

Taste
Balance
Movement
Vision
Hearing
5. A 28 year old man had a gunshot wound of shin that resulted
in an ulcer from the side of the injury. What is the main factor
of neurodystrophy pathogenesis in this case?
Infection
Tissue damage
Traumatization of peripheral nerve
Microcirculation disturbance
Psychical stress

A.
B.
C.
D.
E.

6. Increase of blood pressure and rapid pulse are noticed in a


sportsman at the start before competitions. Influence of which
part of the CNS can above-mentioned changes be explained?
Cortex of hemispheares
Medulla oblongata
Mesencephalon
Diencephalons
Hypothalamus

A.
B.
C.
D.
E.

7. Ptyalism, bradycardia (heart rate 45 beats per minute), miosis


are observed in a man. What is the most possible reason for
such changes?
Increase of sympathetic influence
Increase of parasympathetic influence
Decrease of sympathetic influence
Decrease of parasympathetic influence
Decrease of vegetative influence

8. The reactions, typical for stimulation of parasympathetic


nerves, appeared after irritation of throphogenic zone of
hypothalamus, including preoptic nucleas and anterior
hypothalamic area. What appears in this case?
A. Tachycardia
B. Mydriasis
C. Exophthalmus

D. Decrease of BP
E. Hyperglycemia

B.
C.
D.
E.

9. In an experiment a part of the brain of an animal has been


removed. As a result, asynergia, atonia, and dysmetria
developed. What part of the brain was removed?
A Mesencephalon.
Frontal part.
Parietal part.
CerebeIlum.
Reticular formation.

A.
B.
C.
D.
E.

10.Patients, suffering from epilepsy, have specific centers in brain


cortex. They function as a pathological determinant. What is the
underlying mechanism of the formation of these centers?
Protective inhibition.
Phenomenon of fallout.
Formation of generator of pathological exaltation.
Overexcitation.
Parabiosis.

B.
C.
D.
E.

11.A patient has trophical changes of skin caused by a cross-cut trauma


of the sciatic nerve. What is the mechanism of the skin lesion?
A Phagocytosis of nerve terminations.
Pain of phantom type.
Stopping of the axoplasmatic current
Destruction of the myelinic membrane.
Damage of Ranve's intercepting.

B.
C.
D.
E.

12.A 68-year-old woman had a stroke which resulted in the absence of


voluntary movements of both upper and lower right extremities. The
tonus of muscles and reflexes is increased. Pathological reflexes are
observed. What kind of paralysis is it?
A Tetraplegic.
Paraplegic.
Peripherie.
Central.
Reflex.

13. After a hypertonic crisis a patient presents with lacking spontaneous


movements in his right arm and leg, muscle tone of these extremities is
increased. What type of motor dysfunction has developed in this case?
A. Peripheral paralysis
B. Central paresis
C. Central paralysis

D. Peripheral paresis
E. Reflectory paresis
*** ***
1. Patient with diabetes didn't get insulin injection in time that caused
hyperglycemic coma (glucose in the blood 50mmol/L). What mechanism is
prevalent in the development of the coma?
AHyperosmia
B Hypokaliemia
CHypoxia
D Hyponatremia
E Acidosis
2. As a result of a trauma a patient has developed traumatic shock that led to
the following disorders: AP is 140/90 mm Hg, Ps is 120 bpm. The patient is
fussy, talkative, pale. Such state relates to the following shock phase:
AErectile
B Latent period
CTerminal
D Torpid
E3. Rats being under stress have muscular hypertonia and high arterial
pressure, high glucose concentration in blood and intensified secretion of
corticotropin and corticosteroids. In what stress phase are these animals?
AAntishock phase
B Exhaustion
CShock phase
D Erectile
E Terminal
4. A 12-year-old teenager has significantly put off weight within 3 months;
glucose concentration rose up to 50 millimole\l. He fell into a coma. What is
the main mechanism of its development?
AHyperosmolar
B Hypoglycemic
C Ketonemic
D Lactacidemic
E Hypoxic

You might also like